Sei sulla pagina 1di 101

LTD Class 1 Petitioners assail the constitutionality of the following provisions of the

IPRA and its Implementing Rules on the ground that they amount to an
I. INTRODUCTION unlawful deprivation of the States ownership over lands of the public domain
as well as minerals and other natural resources therein, in violation of the
A. History of Land Laws regalian doctrine embodied in Section 2, Article XII of the Constitution:
(1) Section 3(a) which defines the extent and coverage of ancestral
Cruz v. Secretary 347 SCRA 128 G.R. No. 135385 December 6, 2000
domains, and Section 3(b) which, in turn, defines ancestral
lands;
RESOLUTION
(2) Section 5, in relation to section 3(a), which provides that ancestral
PER CURIAM:
domains including inalienable public lands, bodies of water,
Petitioners Isagani Cruz and Cesar Europa brought this suit for
mineral and other resources found within ancestral domains
prohibition and mandamus as citizens and taxpayers, assailing the
are private but community property of the indigenous
constitutionality of certain provisions of Republic Act No. 8371 (R.A. 8371),
peoples;
otherwise known as the Indigenous Peoples Rights Act of 1997 (IPRA), and its
(3) Section 6 in relation to section 3(a) and 3(b) which defines the
Implementing Rules and Regulations (Implementing Rules).
composition of ancestral domains and ancestral lands;
In its resolution of September 29, 1998, the Court required respondents
(4) Section 7 which recognizes and enumerates the rights of the
to comment.[if !supportFootnotes][1][endif] In compliance, respondents Chairperson and
indigenous peoples over the ancestral domains;
Commissioners of the National Commission on Indigenous Peoples (NCIP), the
(5) Section 8 which recognizes and enumerates the rights of the
government agency created under the IPRA to implement its provisions, filed
indigenous peoples over the ancestral lands;
on October 13, 1998 their Comment to the Petition, in which they defend the
(6) Section 57 which provides for priority rights of the indigenous
constitutionality of the IPRA and pray that the petition be dismissed for lack of
peoples in the harvesting, extraction, development or
merit.
exploration of minerals and other natural resources within
On October 19, 1998, respondents Secretary of the Department of
the areas claimed to be their ancestral domains, and the
Environment and Natural Resources (DENR) and Secretary of the Department
right to enter into agreements with nonindigenous peoples
of Budget and Management (DBM) filed through the Solicitor General a
for the development and utilization of natural resources
consolidated Comment. The Solicitor General is of the view that the IPRA is
therein for a period not exceeding 25 years, renewable for
partly unconstitutional on the ground that it grants ownership over natural
not more than 25 years; and
resources to indigenous peoples and prays that the petition be granted in
(7) Section 58 which gives the indigenous peoples the responsibility to
part.
maintain, develop, protect and conserve the ancestral
On November 10, 1998, a group of intervenors, composed of Sen. Juan
domains and portions thereof which are found to be
Flavier, one of the authors of the IPRA, Mr. Ponciano Bennagen, a member of
necessary for critical watersheds, mangroves, wildlife
the 1986 Constitutional Commission, and the leaders and members of 112
sanctuaries, wilderness, protected areas, forest cover or
groups of indigenous peoples (Flavier, et. al), filed their Motion for Leave to
reforestation.[if !supportFootnotes][2][endif]
Intervene. They join the NCIP in defending the constitutionality of IPRA and
Petitioners also content that, by providing for an all-encompassing
praying for the dismissal of the petition.
definition of ancestral domains and ancestral lands which might even include
On March 22, 1999, the Commission on Human Rights (CHR) likewise
private lands found within said areas, Sections 3(a) and 3(b) violate the rights
filed a Motion to Intervene and/or to Appear as Amicus Curiae. The CHR
of private landowners.[if !supportFootnotes][3][endif]
asserts that IPRA is an expression of the principle of parens patriae and that
In addition, petitioners question the provisions of the IPRA defining the
the State has the responsibility to protect and guarantee the rights of those
powers and jurisdiction of the NCIP and making customary law applicable to
who are at a serious disadvantage like indigenous peoples. For this reason it
the settlement of disputes involving ancestral domains and ancestral lands on
prays that the petition be dismissed.
the ground that these provisions violate the due process clause of the
On March 23, 1999, another group, composed of the Ikalahan
Constitution.[if !supportFootnotes][4][endif]
Indigenous People and the Haribon Foundation for the Conservation of Natural
These provisions are:
Resources, Inc. (Haribon, et al.), filed a motion to Intervene with attached
(1) sections 51 to 53 and 59 which detail the process of delineation and
Comment-in-Intervention. They agree with the NCIP and Flavier, et al. that
recognition of ancestral domains and which vest on the NCIP
IPRA is consistent with the Constitution and pray that the petition for
the sole authority to delineate ancestral domains and
prohibition and mandamus be dismissed.
ancestral lands;
The motions for intervention of the aforesaid groups and organizations
(2) Section 52[i] which provides that upon certification by the NCIP that
were granted.
a particular area is an ancestral domain and upon notification
Oral arguments were heard on April 13, 1999. Thereafter, the parties
to the following officials, namely, the Secretary of
and intervenors filed their respective memoranda in which they reiterate the
Environment and Natural Resources, Secretary of Interior and
arguments adduced in their earlier pleadings and during the hearing.
1
Local Governments, Secretary of Justice and Commissioner of Implementing the IPRA, and Section 57 of the IPRA which he contends should
the National Development Corporation, the jurisdiction of be interpreted as dealing with the large-scale exploitation of natural resources
said officials over said area terminates; and should be read in conjunction with Section 2, Article XII of the 1987
(3) Section 63 which provides the customary law, traditions and Constitution. On the other hand, Justice Mendoza voted to dismiss the petition
practices of indigenous peoples shall be applied first with solely on the ground that it does not raise a justiciable controversy and
respect to property rights, claims of ownership, hereditary petitioners do not have standing to question the constitutionality of R.A. 8371.
succession and settlement of land disputes, and that any Seven (7) other members of the Court voted to grant the petition.
doubt or ambiguity in the interpretation thereof shall be Justice Panganiban filed a separate opinion expressing the view that Sections
resolved in favor of the indigenous peoples; 3 (a)(b), 5, 6, 7 (a)(b), 8, and related provisions of R.A. 8371 are
(4) Section 65 which states that customary laws and practices shall be unconstitutional. He reserves judgment on the constitutionality of Sections 58,
used to resolve disputes involving indigenous peoples; and 59, 65, and 66 of the law, which he believes must await the filing of specific
(5) Section 66 which vests on the NCIP the jurisdiction over all claims cases by those whose rights may have been violated by the IPRA. Justice Vitug
and disputes involving rights of the indigenous peoples. [if ! also filed a separate opinion expressing the view that Sections 3(a), 7, and 57
supportFootnotes][5][endif]
of R.A. 8371 are unconstitutional. Justices Melo, Pardo, Buena, Gonzaga-Reyes,
Finally, petitioners assail the validity of Rule VII, Part II, Section 1 of the and De Leon join in the separate opinions of Justices Panganiban and Vitug.
NCIP Administrative Order No. 1, series of 1998, which provides that the As the votes were equally divided (7 to 7) and the necessary majority
administrative relationship of the NCIP to the Office of the President is was not obtained, the case was redeliberated upon. However, after
characterized as a lateral but autonomous relationship for purposes of policy redeliberation, the voting remained the same. Accordingly, pursuant to Rule
and program coordination. They contend that said Rule infringes upon the 56, Section 7 of the Rules of Civil Procedure, the petition is DISMISSED.
Presidents power of control over executive departments under Section 17, Attached hereto and made integral parts thereof are the separate
Article VII of the Constitution.[if !supportFootnotes][6][endif] opinions of Justices Puno, Vitug, Kapunan, Mendoza, and Panganiban.
Petitioners pray for the following: SO ORDERED.
(1) A declaration that Sections 3, 5, 6, 7, 8, 52[I], 57, 58, 59, 63, 65 and Davide, Jr., C.J., Bellosillo, Melo, Quisumbing, Pardo, Buena, Gonzaga-
66 and other related provisions of R.A. 8371 are Reyes, Ynares-Santiago, and De Leon, Jr., JJ., concur.
unconstitutional and invalid; Puno, Vitug, Kapunan, Mendoza and Panganiban JJ., see separate opinion
(2) The issuance of a writ of prohibition directing the Chairperson and
Commissioners of the NCIP to cease and desist from
implementing the assailed provisions of R.A. 8371 and its
Implementing Rules;
SEPARATE OPINION
(3) The issuance of a writ of prohibition directing the Secretary of the
PUNO, J.:
Department of Environment and Natural Resources to cease
PRECIS
and desist from implementing Department of Environment
A classic essay on the utility of history was written in 1874
and Natural Resources Circular No. 2, series of 1998;
by Friedrich Nietzsche entitled "On the Uses and Disadvantages of
(4) The issuance of a writ of prohibition directing the Secretary of
History for Life." Expounding on Nietzsche's essay, Judge Richard
Budget and Management to cease and desist from disbursing
Posner[if !supportFootnotes][1][endif] wrote:[if !supportFootnotes][2][endif]
public funds for the implementation of the assailed provisions
"Law is the most historically oriented, or if you like the most
of R.A. 8371; and
backward-looking, the most 'past-dependent,' of the professions. It
(5) The issuance of a writ of mandamus commanding the Secretary of
venerates tradition, precedent, pedigree, ritual, custom, ancient
Environment and Natural Resources to comply with his duty
practices, ancient texts, archaic terminology, maturity, wisdom,
of carrying out the States constitutional mandate to control
seniority, gerontocracy, and interpretation conceived of as a
and supervise the exploration, development, utilization and
method of recovering history. It is suspicious of innovation,
conservation of Philippine natural resources. [if !supportFootnotes][7]
[endif] discontinuities, 'paradigm shifts,' and the energy and brashness of
youth. These ingrained attitudes are obstacles to anyone who
After due deliberation on the petition, the members of the Court voted
wants to re-orient law in a more pragmatic direction. But, by the
as follows:
same token, pragmatic jurisprudence must come to terms
Seven (7) voted to dismiss the petition. Justice Kapunan filed an
with history.
opinion, which the Chief Justice and Justices Bellosillo, Quisumbing, and
When Congress enacted the Indigenous Peoples Rights
Santiago join, sustaining the validity of the challenged provisions of R.A. 8371.
Act (IPRA), it introduced radical concepts into the Philippine legal
Justice Puno also filed a separate opinion sustaining all challenged provisions
system which appear to collide with settled constitutional and jural
of the law with the exception of Section 1, Part II, Rule III of NCIP
precepts on state ownership of land and other natural resources.
Administrative Order No. 1, series of 1998, the Rules and Regulations
The sense and subtleties of this law cannot be appreciated without
2
considering its distinct sociology and the labyrinths of its history. 3, Section 2, Article XII of the 1987
This Opinion attempts to interpret IPRA by discovering its soul Consitution.
shrouded by the mist of our history. After all, the IPRA was enacted (c) The large-scale utilization of natural resources in Section 57 of
by Congress not only to fulfill the constitutional mandate of the IPRA may be harmonized with
protecting the indigenous cultural communities' right to their Paragraphs 1 and 4, Section 2, Article
ancestral land but more importantly, to correct a grave XII of the 1987 Constitution.
historical injustice to our indigenous people. V. The IPRA is a Recognition of Our Active Participation in the
This Opinion discusses the following: International Indigenous Movement.
I. The Development of the Regalian Doctrine in the Philippine Legal DISCUSSION
System. I. THE DEVELOPMENT OF THE REGALIAN DOCTRINE IN THE
A. The Laws of the Indies PHILIPPINE LEGAL SYSTEM.
B. Valenton v. Murciano A. The Laws of the Indies
C. The Public Land Acts and the Torrens System The capacity of the State to own or acquire property is the
D. The Philippine Constitutions state's power of dominium.[if !supportFootnotes][3][endif] This was the
II. The Indigenous Peoples Rights Act (IPRA). foundation for the early Spanish decrees embracing the feudal
A. Indigenous Peoples theory of jura regalia. The "Regalian Doctrine" or jura regalia is a
1. Indigenous Peoples: Their History Western legal concept that was first introduced by the
2. Their Concept of Land Spaniards into the country through the Laws of the Indies
III. The IPRA is a Novel Piece of Legislation. and the Royal Cedulas. The Laws of the Indies, i.e., more
A. Legislative History specifically, Law 14, Title 12, Book 4 of the Novisima Recopilacion
IV. The Provisions of the IPRA Do Not Contravene the Constitution. de Leyes de las Indias, set the policy of the Spanish Crown with
A. Ancestral domains and ancestral lands are the respect to the Philippine Islands in the following manner:
private property of indigenous peoples and do not "We, having acquired full sovereignty over the Indies, and all lands,
constitute part of the land of the public domain. territories, and possessions not heretofore ceded away by our royal
1. The right to ancestral domains and predecessors, or by us, or in our name, still pertaining to the royal
ancestral lands: how acquired crown and patrimony, it is our will that all lands which are held
2. The concept of native title without proper and true deeds of grant be restored to us as they
(a) Cario v. Insular Government belong to us, in order that after reserving before all what to us or to
(b) Indian Title to land our viceroys, audiencias, and governors may seem necessary for
(c) Why the Cario doctrine is unique public squares, ways, pastures, and commons in those places which
3. The option of securing a torrens title to the are peopled, taking into consideration not only their present
ancestral land condition, but also their future and their probable increase, and
B. The right of ownership and possession by the ICCs/IPs to their after distributing to the natives what may be necessary for tillage
ancestral domains is a limited form of ownership and pasturage, confirming them in what they now have and giving
and does not include the right to alienate the same. them more if necessary, all the rest of said lands may remain free
1. The indigenous concept of ownership and customary law and unencumbered for us to dispose of as we may wish.
C. Sections 7 (a), 7 (b) and 57 of the IPRA do not violate the We therefore order and command that all viceroys and presidents
Regalian Doctrine enshrined in Section 2, Article XII of pretorial courts designate at such time as shall to them seem
of the 1987 Constitution. most expedient, a suitable period within which all possessors of
1. The rights of ICCs/IPs over their ancestral domains and lands tracts, farms, plantations, and estates shall exhibit to them and to
2. The right of ICCs/IPs to develop lands and natural resources the court officers appointed by them for this purpose, their title
within the ancestral domains does not deeds thereto. And those who are in possession by virtue of proper
deprive the State of ownership over the deeds and receipts, or by virtue of just prescriptive right shall be
natural resources, control and supervision in protected, and all the rest shall be restored to us to be disposed of
their development and exploitation. at our will."[if !supportFootnotes][4][endif]
(a) Section 1, Part II, Rule III of the Implementing Rules goes beyond The Philippines passed to Spain by virtue of "discovery" and
the parameters of Section 7(a) of the conquest. Consequently, all lands became the exclusive patrimony
law on ownership of ancestral domains and dominion of the Spanish Crown. The Spanish Government took
and is ultra vires. charge of distributing the lands by issuing royal grants and
(b) The small-scale utilization of natural resources in Section 7 (b) concessions to Spaniards, both military and civilian. [if !supportFootnotes][5]
of the IPRA is allowed under Paragraph [endif]
Private land titles could only be acquired from the government

3
either by purchase or by the various modes of land grant from the follows:
Crown.[if !supportFootnotes][6][endif] "In the preamble of this law there is, as is seen, a distinct statement
The Laws of the Indies were followed by the Ley that all those lands belong to the Crown which have not been
Hipotecaria, or the Mortgage Law of 1893.[if !supportFootnotes][7][endif] granted by Philip, or in his name, or by the kings who preceded
The Spanish Mortgage Law provided for the systematic registration him. This statement excludes the idea that there might be
of titles and deeds as well as possessory claims. The law sought to lands not so granted, that did not belong to the king. It
register and tax lands pursuant to the Royal Decree of 1880. The excludes the idea that the king was not still the owner of all
Royal Decree of 1894, or the "Maura Law," was partly an ungranted lands, because some private person had been in the
amendment of the Mortgage Law as well as the Laws of the Indies, adverse occupation of them. By the mandatory part of the law all
as already amended by previous orders and decrees. [if !supportFootnotes][8] the occupants of the public lands are required to produce before
[endif]
This was the last Spanish land law promulgated in the the authorities named, and within a time to be fixed by them, their
Philippines. It required the "adjustment" or registration of all title papers. And those who had good title or showed prescription
agricultural lands, otherwise the lands shall revert to the state. were to be protected in their holdings. It is apparent that it was not
Four years later, by the Treaty of Paris of December 10, the intention of the law that mere possession for a length of time
1898, Spain ceded to the government of the United States all should make the possessors the owners of the land possessed by
rights, interests and claims over the national territory of the them without any action on the part of the authorities." [if !supportFootnotes]
Philippine Islands. In 1903, the United States colonial government, [12][endif]

through the Philippine Commission, passed Act No. 926, the first The preamble stated that all those lands which had not been
Public Land Act. granted by Philip, or in his name, or by the kings who preceded
B. Valenton v. Murciano him, belonged to the Crown. [if !supportFootnotes][13][endif] For those lands
In 1904, under the American regime, this Court decided the granted by the king, the decree provided for a system of
case of Valenton v. Murciano.[if !supportFootnotes][9][endif] assignment of such lands. It also ordered that all possessors of
Valenton resolved the question of which is the better basis agricultural land should exhibit their title deed, otherwise, the land
for ownership of land: long-time occupation or paper title. Plaintiffs would be restored to the Crown.[if !supportFootnotes][14][endif]
had entered into peaceful occupation of the subject land in 1860. The Royal Cedula of October 15, 1754 reinforced the
Defendant's predecessor-in-interest, on the other hand, purchased Recopilacion when it ordered the Crown's principal subdelegate to
the land from the provincial treasurer of Tarlac in 1892. The lower issue a general order directing the publication of the Crown's
court ruled against the plaintiffs on the ground that they had lost all instructions:
rights to the land by not objecting to the administrative sale. "x x x to the end that any and all persons who, since the year 1700,
Plaintiffs appealed the judgment, asserting that their 30-year and up to the date of the promulgation and publication of
adverse possession, as an extraordinary period of prescription in said order, shall have occupied royal lands, whether or not x
the Partidas and the Civil Code, had given them title to the land as x x cultivated or tenanted, may x x x appear and exhibit to
against everyone, including the State; and that the State, not said subdelegates the titles and patents by virtue of which
owning the land, could not validly transmit it. said lands are occupied. x x x. Said subdelegates will at the
The Court, speaking through Justice Willard, decided the case same time warn the parties interested that in case of their
on the basis of "those special laws which from earliest time have failure to present their title deeds within the term
regulated the disposition of the public lands in the colonies." [if ! designated, without a just and valid reason therefor, they will
supportFootnotes][10][endif]
The question posed by the Court was: "Did these be deprived of and evicted from their lands, and they will be
special laws recognize any right of prescription as against the State granted to others."[if !supportFootnotes][15][endif]
as to these lands; and if so, to what extent was it recognized?" On June 25, 1880, the Crown adopted regulations for the
Prior to 1880, the Court said, there were no laws specifically adjustment of lands "wrongfully occupied" by private individuals in
providing for the disposition of land in the Philippines. However, it the Philippine Islands. Valenton construed these regulations
was understood that in the absence of any special law to govern a together with contemporaneous legislative and executive
specific colony, the Laws of the Indies would be followed. Indeed, in interpretations of the law, and concluded that plaintiffs' case fared
the Royal Order of July 5, 1862, it was decreed that until regulations no better under the 1880 decree and other laws which followed it,
on the subject could be prepared, the authorities of the Philippine than it did under the earlier ones. Thus as a general doctrine, the
Islands should follow strictly the Laws of the Indies, the Ordenanza Court stated:
of the Intendentes of 1786, and the Royal Cedula of 1754. [if ! "While the State has always recognized the right of the occupant to
supportFootnotes][11][endif]
a deed if he proves a possession for a sufficient length of time, yet
Quoting the preamble of Law 14, Title 12, Book 4 of the it has always insisted that he must make that proof before
Recopilacion de Leyes de las Indias, the court interpreted it as the proper administrative officers, and obtain from them his

4
deed, and until he did that the State remained the absolute Torrens system of registration formulated by Sir Robert Torrens who patterned
owner."[if !supportFootnotes][16][endif] it after the Merchant Shipping Acts in South Australia. The Torrens system
In conclusion, the Court ruled: "We hold that from 1860 to 1892 requires that the government issue an official certificate of title attesting to
there was no law in force in these Islands by which the plaintiffs the fact that the person named is the owner of the property described therein,
could obtain the ownership of these lands by prescription, without subject to such liens and encumbrances as thereon noted or the law warrants
any action by the State." [if !supportFootnotes][17][endif] Valenton had no rights or reserves.[if !supportFootnotes][26][endif] The certificate of title is indefeasible and
other than those which accrued to mere possession. Murciano, on imprescriptible and all claims to the parcel of land are quieted upon issuance
the other hand, was deemed to be the owner of the land by virtue of said certificate. This system highly facilitates land conveyance and
of the grant by the provincial secretary. In effect, Valenton upheld negotiation.[if !supportFootnotes][27][endif]
the Spanish concept of state ownership of public land. D. The Philippine Constitutions
As a fitting observation, the Court added that "[t]he policy The Regalian doctrine was enshrined in the 1935
pursued by the Spanish Government from earliest times, Constitution. One of the fixed and dominating objectives of the
requiring settlers on the public lands to obtain title deeds 1935 Constitutional Convention was the nationalization and
therefor from the State, has been continued by the conservation of the natural resources of the country. [if !supportFootnotes][28]
American Government in Act No. 926."[if !supportFootnotes][18][endif] [endif]
There was an overwhelming sentiment in the
C. The Public Land Acts and the Torrens System Convention in favor of the principle of state ownership of
Act No. 926, the first Public Land Act, was passed in pursuance of the natural resources and the adoption of the Regalian
provisions of the the Philippine Bill of 1902. The law governed the disposition doctrine.[if !supportFootnotes][29][endif] State ownership of natural resources
of lands of the public domain. It prescribed rules and regulations for the was seen as a necessary starting point to secure recognition of the
homesteading, selling, and leasing of portions of the public domain of the state's power to control their disposition, exploitation, development,
Philippine Islands, and prescribed the terms and conditions to enable persons or utilization.[if !supportFootnotes][30][endif] The delegates to the Constitutional
to perfect their titles to public lands in the Islands. It also provided for the Convention very well knew that the concept of State ownership of
"issuance of patents to certain native settlers upon public lands," for the land and natural resources was introduced by the Spaniards,
establishment of town sites and sale of lots therein, for the completion of however, they were not certain whether it was continued and
imperfect titles, and for the cancellation or confirmation of Spanish applied by the Americans. To remove all doubts, the Convention
concessions and grants in the Islands." In short, the Public Land Act operated approved the provision in the Constitution affirming the Regalian
on the assumption that title to public lands in the Philippine Islands remained doctrine.[if !supportFootnotes][31][endif]
in the government;[if !supportFootnotes][19][endif] and that the government's title to Thus, the 1935 Constitution, in Section 1 of Article XIII on
public land sprung from the Treaty of Paris and other subsequent treaties "Conservation and Utilization of Natural Resources," reads as
between Spain and the United States.[if !supportFootnotes][20][endif] The term "public follows:
land" referred to all lands of the public domain whose title still remained in the "Sec. 1. All agricultural, timber, and mineral lands of the
government and are thrown open to private appropriation and settlement, [if ! public domain, waters, minerals, coal, petroleum, and other
supportFootnotes][21][endif]
and excluded the patrimonial property of the government mineral oils, all forces of potential energy, and other
and the friar lands.[if !supportFootnotes][22][endif] natural resources of the Philippines belong to the State,
Act No. 926 was superseded in 1919 by Act 2874, the second Public and their disposition, exploitation, development, or
Land Act. This new law was passed under the Jones Law. It was more utilization shall be limited to citizens of the Philippines, or
comprehensive in scope but limited the exploitation of agricultural lands to to corporations or associations at least sixty per centum of
Filipinos and Americans and citizens of other countries which gave Filipinos the capital of which is owned by such citizens, subject to
the same privileges.[if !supportFootnotes][23][endif] After the passage of the 1935 any existing right, grant, lease, or concession at the time of
Constitution, Act 2874 was amended in 1936 by Commonwealth Act No. the inauguration of the Government established under this
141. Commonwealth Act No. 141 remains the present Public Land Law and it Constitution. Natural resources, with the exception of public
is essentially the same as Act 2874. The main difference between the two agricultural land, shall not be alienated, and no license,
relates to the transitory provisions on the rights of American citizens and concession, or lease for the exploitation, development, or utilization
corporations during the Commonwealth period at par with Filipino citizens and of any of the natural resources shall be granted for a period
corporations.[if !supportFootnotes][24][endif] exceeding twenty-five years, except as to water rights for irrigation,
Grants of public land were brought under the operation of the water supply, fisheries, or industrial uses other than the
Torrens system under Act 496, or the Land Registration Law of 1903. development of water power, in which cases beneficial use may be
Enacted by the Philippine Commission, Act 496 placed all public and private the measure and the limit of the grant."
lands in the Philippines under the Torrens system. The law is said to be almost The 1973 Constitution reiterated the Regalian doctrine in Section 8,
a verbatim copy of the Massachussetts Land Registration Act of 1898, [if ! Article XIV on the "National Economy and the Patrimony of the Nation," to wit:
supportFootnotes][25][endif]
which, in turn, followed the principles and procedure of the "Sec. 8. All lands of the public domain, waters, minerals,

5
coal, petroleum and other mineral oils, all forces of and defines the extent of these lands and domains. The
potential energy, fisheries, wildlife, and other natural ownership given is the indigenous concept of ownership
resources of the Philippines belong to the State. With the under customary law which traces its origin to native title.
exception of agricultural, industrial or commercial, Other rights are also granted the ICCs/IPs, and these are:
residential, and resettlement lands of the public domain, - the right to develop lands and natural resources;
natural resources shall not be alienated, and no license, - the right to stay in the territories;
concession, or lease for the exploration, development, - the right in case of displacement;
exploitation, or utilization of any of the natural resources - the right to safe and clean air and water;
shall be granted for a period exceeding twenty-five years, - the right to claim parts of reservations;
renewable for not more than twenty-five years, except as to - the right to resolve conflict;[if !supportFootnotes][32][endif]
water rights for irrigation, water supply, fisheries, or industrial uses - the right to ancestral lands which include
other than the development of water power, in which cases a. the right to transfer land/property to/among members of the
beneficial use may be the measure and the limit of the grant." same ICCs/IPs, subject to customary laws and traditions of
The 1987 Constitution reaffirmed the Regalian doctrine in the community concerned;
Section 2 of Article XII on "National Economy and Patrimony," to b. the right to redemption for a period not exceeding 15 years from
wit: date of transfer, if the transfer is to a non-member of the
"Sec. 2. All lands of the public domain, waters, minerals, ICC/IP and is tainted by vitiated consent of the ICC/IP, or if the
coal, petroleum, and other mineral oils, all forces of transfer is for an unconscionable consideration. [if !supportFootnotes]
potential energy, fisheries, forests or timber, wildlife, flora [33][endif]

and fauna, and other natural resources are owned by the Within their ancestral domains and ancestral lands, the
State. With the exception of agricultural lands, all other ICCs/IPs are given the right to self-governance and empowerment,
natural resources shall not be alienated. The exploration, [if !supportFootnotes][34][endif]
social justice and human rights,[if !supportFootnotes][35]
development and utilization of natural resources shall be [endif]
the right to preserve and protect their culture, traditions,
under the full control and supervision of the State. The institutions and community intellectual rights, and the right to
State may directly undertake such activities or it may enter develop their own sciences and technologies.[if !supportFootnotes][36][endif]
into co-production, joint venture, or production-sharing To carry out the policies of the Act, the law created the
agreements with Filipino citizens, or corporations or National Commission on Indigenous Peoples (NCIP). The NCIP is an
associations at least sixty per centum of whose capital is independent agency under the Office of the President and is
owned by such citizens. Such agreements may be for a period composed of seven (7) Commissioners belonging to ICCs/IPs from
not exceeding twenty-five years, renewable for not more than each of the ethnographic areas-- Region I and the Cordilleras;
twenty-five years, and under such terms and conditions as may be Region II; the rest of Luzon; Island groups including Mindoro,
provided by law. In cases of water rights for irrigation, water supply, Palawan, Romblon, Panay and the rest of the Visayas; Northern and
fisheries, or industrial uses other than the development of water Western Mindanao; Southern and Eastern Mindanao; and Central
power, beneficial use may be the measure and limit of the grant. Mindanao.[if !supportFootnotes][37][endif] The NCIP took over the functions of
x x x." the Office for Northern Cultural Communities and the Office for
Simply stated, all lands of the public domain as well as Southern Cultural Communities created by former President
all natural resources enumerated therein, whether on public or Corazon Aquino which were merged under a revitalized structure. [if !
private land, belong to the State. It is this concept of State supportFootnotes][38][endif]

ownership that petitioners claim is being violated by the Disputes involving ICCs/IPs are to be resolved under customary laws
IPRA. and practices. When still unresolved, the matter may be brought to the
II. THE INDIGENOUS PEOPLES RIGHTS ACT. NCIP, which is granted quasi-judicial powers. [if !supportFootnotes][39][endif] The NCIP's
Republic Act No. 8371 is entitled "An Act to Recognize, Protect and Promote decisions may be appealed to the Court of Appeals by a petition for review.
the Rights of Indigenous Cultural Communities/ Indigenous Peoples, Creating a Any person who violates any of the provisions of the Act such
National Commission on Indigenous Peoples, Establishing Implementing as, but not limited to, unauthorized and/or unlawful intrusion upon
Mechanisms, Appropriating Funds Therefor, and for Other Purposes." It is ancestral lands and domains shall be punished in accordance with
simply known as "The Indigenous Peoples Rights Act of 1997" or the customary laws or imprisoned from 9 months to 12 years and/or
IPRA. fined from P100,000.00 to P500,000.00 and obliged to pay
The IPRA recognizes the existence of the indigenous cultural damages.[if !supportFootnotes][40][endif]
communities or indigenous peoples (ICCs/IPs) as a distinct sector A. Indigenous Peoples
in Philippine society. It grants these people the ownership and The IPRA is a law dealing with a specific group of people, i.e.,
possession of their ancestral domains and ancestral lands, the Indigenous Cultural Communities (ICCs) or the Indigenous

6
Peoples (IPs). The term "ICCs" is used in the 1987 Constitution while Rizal, Quezon; Alangan or Mangyan, Batangan, Buid or Buhid,
that of "IPs" is the contemporary international language in the Hanunuo and Iraya of Oriental and Occidental Mindoro; Tadyawan of
International Labor Organization (ILO) Convention 169 [if !supportFootnotes] Occidental Mindoro; Cuyonon, Palawanon, Tagbanua and Tao't bato
[41][endif]
and the United Nations (UN) Draft Declaration on the Rights of Palawan.
of Indigenous Peoples.[if !supportFootnotes][42][endif] 4. In Region V-- Aeta of Camarines Norte and Camarines Sur; Aeta-
ICCs/IPs are defined by the IPRA as: Abiyan, Isarog, and Kabihug of Camarines Norte; Agta, and Mayon
"Sec. 3 [h]. Indigenous Cultural Communities/ Indigenous Peoples-- of Camarines Sur; Itom of Albay, Cimaron of Sorsogon; and the
refer to a group of people or homogeneous societies identified by Pullon of Masbate and Camarines Sur.
self-ascription and ascription by others, who have continuously 5. In Region VI-- Ati of Negros Occidental, Iloilo and Antique, Capiz;
lived as organized community on communally bounded and defined the Magahat of Negros Occidental; the Corolano and Sulod.
territory, and who have, under claims of ownership since time 6. In Region VII-- Magahat of Negros Oriental and Eskaya of Bohol.
immemorial, occupied, possessed and utilized such territories, 7. In Region IX-- the Badjao numbering about 192,000 in Tawi-Tawi,
sharing common bonds of language, customs, traditions and other Zamboanga del Sur; the Kalibugan of Basilan, the Samal, Subanon
distinctive cultural traits, or who have, through resistance to and Yakat.
political, social and cultural inroads of colonization, non-indigenous 8. Region X-- Numbering 1.6 million in Region X alone, the IPs are:
religions and cultures, became historically differentiated from the the Banwaon, Bukidnon, Matigsalog, Talaanding of Bukidnon; the
majority of Filipinos. ICCs/IPs shall likewise include peoples who are Camiguin of Camiguin Island; the Higa-unon of Agusan del Norte,
regarded as indigenous on account of their descent from the Agusan del Sur, Bukidnon and Misamis Occidental; the Tigwahanon
populations which inhabited the country, at the time of conquest or of Agusan del Sur, Misamis Oriental and and Misamis Occidental,
colonization, or at the time of inroads of non-indigenous religions the Manobo of the Agusan provinces, and the Umayamnon of
and cultures, or the establishment of present state boundaries, who Agusan and Bukidnon.
retain some or all of their own social, economic, cultural and 9. In Region XI-- There are about 1,774,065 IPs in Region XI. They
political institutions, but who may have been displaced from their are tribes of the Dibabaon, Mansaka of Davao del Norte; B'laan,
traditional domains or who may have resettled outside their Kalagan, Langilad, T'boli and Talaingod of Davao del Sur;
ancestral domains." Mamamanua of Surigao del Sur; Mandaya of the Surigao provinces
Indigenous Cultural Communities or Indigenous Peoples refer to a and Davao Oriental; Manobo Blit of South Cotabato; the
group of people or homogeneous societies who have continuously Mangguangon of Davao and South Cotabato; Matigsalog of Davao
lived as an organized community on communally bounded and del Norte and Del Sur; Tagakaolo, Tasaday and Ubo of South
defined territory. These groups of people have actually occupied, possessed Cotabato; and Bagobo of Davao del sur and South Cotabato.
and utilized their territories under claim of ownership since time immemorial. 10. In Region XII-- Ilianen, Tiruray, Maguindanao, Maranao, Tausug,
They share common bonds of language, customs, traditions and other Yakan/Samal, and Iranon.[if !supportFootnotes][43][endif]
distinctive cultural traits, or, they, by their resistance to political, social and How these indigenous peoples came to live in the Philippines goes
cultural inroads of colonization, non-indigenous religions and cultures, became back to as early as 25,000 to 30,000 B.C.
historically differentiated from the Filipino majority. ICCs/IPs also include Before the time of Western contact, the Philippine archipelago was
descendants of ICCs/IPs who inhabited the country at the time of conquest or peopled largely by the Negritos, Indonesians and Malays. [if !supportFootnotes][44][endif]
colonization, who retain some or all of their own social, economic, cultural and The strains from these groups eventually gave rise to common cultural
political institutions but who may have been displaced from their traditional features which became the dominant influence in ethnic reformulation in the
territories or who may have resettled outside their ancestral domains. archipelago. Influences from the Chinese and Indian civilizations in the third or
1. Indigenous Peoples: Their History fourth millenium B.C. augmented these ethnic strains. Chinese economic and
Presently, Philippine indigenous peoples inhabit the interiors socio-cultural influences came by way of Chinese porcelain, silk and traders.
and mountains of Luzon, Mindanao, Mindoro, Negros, Samar, Leyte, Indian influence found their way into the religious-cultural aspect of pre-
and the Palawan and Sulu group of islands. They are composed of colonial society.[if !supportFootnotes][45][endif]
110 tribes and are as follows: The ancient Filipinos settled beside bodies of water.
1. In the Cordillera Autonomous Region-- Kankaney, Ibaloi, Bontoc, Hunting and food gathering became supplementary activities as
Tinggian or Itneg, Ifugao, Kalinga, Yapayao, Aeta or Agta or Pugot, reliance on them was reduced by fishing and the cultivation of the
and Bago of Ilocos Norte and Pangasinan; Ibanag of Isabela, soil.[if !supportFootnotes][46][endif] From the hinterland, coastal, and riverine
Cagayan; Ilongot of Quirino and Nueva Vizcaya; Gaddang of communities, our ancestors evolved an essentially homogeneous
Quirino, Nueva Vizcaya, Itawis of Cagayan; Ivatan of Batanes, Aeta culture, a basically common way of life where nature was a
of Cagayan, Quirino and Isabela. primary factor. Community life throughout the archipelago was
2. In Region III-- Aetas. influenced by, and responded to, common ecology. The generally
3. In Region IV-- Dumagats of Aurora, Rizal; Remontado of Aurora, benign tropical climate and the largely uniform flora and fauna

7
favored similarities, not differences.[if !supportFootnotes][47][endif] Life was This ancient communalism was practiced in accordance with the concept of
essentially subsistence but not harsh.[if !supportFootnotes][48][endif] mutual sharing of resources so that no individual, regardless of status, was
The early Filipinos had a culture that was basically Malayan without sustenance. Ownership of land was non-existent or
in structure and form. They had languages that traced their origin unimportant and the right of usufruct was what regulated the
to the Austronesian parent-stock and used them not only as media development of lands.[if !supportFootnotes][59][endif] Marine resources and fishing
of daily communication but also as vehicles for the expression of grounds were likewise free to all. Coastal communities depended for their
their literary moods.[if !supportFootnotes][49][endif] They fashioned concepts economic welfare on the kind of fishing sharing concept similar to those in
and beliefs about the world that they could not see, but which they land communities.[if !supportFootnotes][60][endif] Recognized leaders, such as the
sensed to be part of their lives. [if !supportFootnotes][50][endif] They had their chieftains and elders, by virtue of their positions of importance, enjoyed some
own religion and religious beliefs. They believed in the immortality economic privileges and benefits. But their rights, related to either land and
of the soul and life after death. Their rituals were based on beliefs sea, were subject to their responsibility to protect the communities from
in a ranking deity whom they called Bathalang Maykapal, and a danger and to provide them with the leadership and means of survival. [if !
host of other deities, in the environmental spirits and in soul spirits. supportFootnotes][61][endif]

The early Filipinos adored the sun, the moon, the animals and birds, Sometime in the 13th century, Islam was introduced to the
for they seemed to consider the objects of Nature as something to archipelago in Maguindanao. The Sultanate of Sulu was established and
be respected. They venerated almost any object that was close to claimed jurisdiction over territorial areas represented today by Tawi-tawi, Sulu,
their daily life, indicating the importance of the relationship Palawan, Basilan and Zamboanga. Four ethnic groups were within this
between man and the object of nature.[if !supportFootnotes][51][endif] jurisdiction: Sama, Tausug, Yakan and Subanon. [if !supportFootnotes][62][endif] The
The unit of government was the "barangay," a term that Sultanate of Maguindanao spread out from Cotabato toward Maranao territory,
derived its meaning from the Malay word "balangay," meaning, a now Lanao del Norte and Lanao del Sur.[if !supportFootnotes][63][endif]
boat, which transported them to these shores. [if !supportFootnotes][52][endif] The Muslim societies evolved an Asiatic form of feudalism where land
The barangay was basically a family-based community and was still held in common but was private in use. This is clearly indicated
consisted of thirty to one hundred families. Each barangay was in the Muslim Code of Luwaran. The Code contains a provision on the lease of
different and ruled by a chieftain called a "dato." It was the cultivated lands. It, however, has no provision for the acquisition, transfer,
chieftain's duty to rule and govern his subjects and promote their cession or sale of land.[if !supportFootnotes][64][endif]
welfare and interests. A chieftain had wide powers for he exercised The societies encountered by Magellan and Legaspi
all the functions of government. He was the executive, legislator therefore were primitive economies where most production was
and judge and was the supreme commander in time of war. [if ! geared to the use of the producers and to the fulfillment of kinship
supportFootnotes][53][endif]
obligations. They were not economies geared to exchange and
Laws were either customary or written. Customary laws were handed profit.[if !supportFootnotes][65][endif] Moreover, the family basis of barangay
down orally from generation to generation and constituted the bulk membership as well as of leadership and governance worked to
of the laws of the barangay. They were preserved in songs and chants and splinter the population of the islands into numerous small and
in the memory of the elder persons in the community. [if !supportFootnotes][54][endif] The separate communities.[if !supportFootnotes][66][endif]
written laws were those that the chieftain and his elders promulgated from When the Spaniards settled permanently in the Philippines in 1565,
time to time as the necessity arose. [if !supportFootnotes][55][endif] The oldest known they found the Filipinos living in barangay settlements scattered
written body of laws was the Maragtas Code by Datu Sumakwel at about 1250 along water routes and river banks. One of the first tasks imposed on the
A.D. Other old codes are the Muslim Code of Luwaran and the Principal Code missionaries and the encomenderos was to collect all scattered Filipinos
of Sulu.[if !supportFootnotes][56][endif] Whether customary or written, the laws dealt with together in a reduccion.[if !supportFootnotes][67][endif] As early as 1551, the Spanish
various subjects, such as inheritance, divorce, usury, loans, partnership, crime government assumed an unvarying solicitous attitude towards the natives. [if !
and punishment, property rights, family relations and adoption. Whenever supportFootnotes][68][endif]
The Spaniards regarded it a sacred "duty to conscience and
disputes arose, these were decided peacefully through a court composed by humanity to civilize these less fortunate people living in the obscurity of
the chieftain as "judge" and the barangay elders as "jury." Conflicts arising ignorance" and to accord them the "moral and material advantages" of
between subjects of different barangays were resolved by arbitration in which community life and the "protection and vigilance afforded them by the same
a board composed of elders from neutral barangays acted as arbiters. [if ! laws."[if !supportFootnotes][69][endif]
supportFootnotes][57][endif]
The Spanish missionaries were ordered to establish pueblos
Baranganic society had a distinguishing feature: the absence of where the church and convent would be constructed. All the new
private property in land. The chiefs merely administered the lands in the Christian converts were required to construct their houses around
name of the barangay. The social order was an extension of the family with the church and the unbaptized were invited to do the same. [if !
chiefs embodying the higher unity of the community. Each individual, supportFootnotes][70][endif]
With the reduccion, the Spaniards attempted to
therefore, participated in the community ownership of the soil and the "tame" the reluctant Filipinos through Christian indoctrination using
instruments of production as a member of the barangay. [if !supportFootnotes][58][endif] the convento/casa real/plaza complex as focal point. The reduccion,

8
to the Spaniards, was a "civilizing" device to make the Filipinos law- now living in peace and contentment, surrounded by civilization to which they
abiding citizens of the Spanish Crown, and in the long run, to make are unable or unwilling to conform. Such tribal government should, however,
them ultimately adopt Hispanic culture and civilization. [if !supportFootnotes] be subjected to wise and firm regulation; and, without undue or petty
[71][endif]
interference, constant and active effort should be exercised to prevent
All lands lost by the old barangays in the process of pueblo barbarous practices and introduce civilized customs." [if !supportFootnotes][80][endif]
organization as well as all lands not assigned to them and the Placed in an alternative of either letting the natives alone or guiding
pueblos, were now declared to be crown lands or realengas, them in the path of civilization, the American government chose "to
belonging to the Spanish king. It was from the realengas that land adopt the latter measure as one more in accord with humanity and
grants were made to non-Filipinos.[if !supportFootnotes][72][endif] with the national conscience."[if !supportFootnotes][81][endif]
The abrogation of the Filipinos' ancestral rights in land and the The Americans classified the Filipinos into two: the
introduction of the concept of public domain were the most Christian Filipinos and the non-Christian Filipinos. The term
immediate fundamental results of Spanish colonial theory and law. [if ! "non-Christian" referred not to religious belief, but to a geographical
supportFootnotes][73][endif]
The concept that the Spanish king was the owner of area, and more directly, "to natives of the Philippine Islands of a low
everything of value in the Indies or colonies was imposed on the grade of civilization, usually living in tribal relationship apart from
natives, and the natives were stripped of their ancestral rights to settled communities."[if !supportFootnotes][82][endif]
land.[if !supportFootnotes][74][endif] Like the Spaniards, the Americans pursued a policy of assimilation. In
Increasing their foothold in the Philippines, the Spanish 1903, they passed Act No. 253 creating the Bureau of Non-Christian
colonialists, civil and religious, classified the Filipinos according to Tribes (BNCT). Under the Department of the Interior, the BNCT's primary
their religious practices and beliefs, and divided them into three task was to conduct ethnographic research among unhispanized Filipinos,
types . First were the Indios, the Christianized Filipinos, who including those in Muslim Mindanao, with a "special view to determining the
generally came from the lowland populations. Second, were the most practicable means for bringing about their advancement in civilization
Moros or the Muslim communities, and third, were the infieles or and prosperity." The BNCT was modeled after the bureau dealing with
the indigenous communities.[if !supportFootnotes][75][endif] American Indians. The agency took a keen anthropological interest in
The Indio was a product of the advent of Spanish culture. Philippine cultural minorities and produced a wealth of valuable materials
This class was favored by the Spaniards and was allowed certain about them.[if !supportFootnotes][83][endif]
status although below the Spaniards. The Moros and infieles were The 1935 Constitution did not carry any policy on the non-Christian
regarded as the lowest classes.[if !supportFootnotes][76][endif] Filipinos. The raging issue then was the conservation of the national
The Moros and infieles resisted Spanish rule and Christianity. The patrimony for the Filipinos.
Moros were driven from Manila and the Visayas to Mindanao; while the In 1957, the Philippine Congress passed R.A. No. 1888, an "Act to
infieles, to the hinterlands. The Spaniards did not pursue them into the effectuate in a more rapid and complete manner the economic, social, moral
deep interior. The upland societies were naturally outside the immediate and political advancement of the non-Christian Filipinos or national cultural
concern of Spanish interest, and the cliffs and forests of the hinterlands were minorities and to render real, complete, and permanent the integration of all
difficult and inaccessible, allowing the infieles, in effect, relative security.[if ! said national cultural minorities into the body politic, creating the
supportFootnotes][77][endif]
Thus, the infieles, which were peripheral to colonial Commission on National Integration charged with said functions." The law
administration, were not only able to preserve their own culture but also called for a policy of integration of indigenous peoples into the Philippine
thwarted the Christianization process, separating themselves from the newly mainstream and for this purpose created the Commission on National
evolved Christian community.[if !supportFootnotes][78][endif] Their own political, economic Integration (CNI).[if !supportFootnotes][84][endif] The CNI was given, more or less, the
and social systems were kept constantly alive and vibrant. same task as the BNCT during the American regime. The post-
The pro-Christian or pro-Indio attitude of colonialism brought independence policy of integration was like the colonial policy of
about a generally mutual feeling of suspicion, fear, and hostility assimilation understood in the context of a guardian-ward
between the Christians on the one hand and the non-Christians on relationship.[if !supportFootnotes][85][endif]
the other. Colonialism tended to divide and rule an otherwise The policy of assimilation and integration did not yield the
culturally and historically related populace through a colonial desired result. Like the Spaniards and Americans, government
system that exploited both the virtues and vices of the Filipinos. [if ! attempts at integration met with fierce resistance. Since
supportFootnotes][79][endif]
World War II, a tidal wave of Christian settlers from the lowlands of
President McKinley, in his instructions to the Philippine Commission Luzon and the Visayas swamped the highlands and wide open
of April 7, 1900, addressed the existence of the infieles: spaces in Mindanao.[if !supportFootnotes][86][endif] Knowledge by the
"In dealing with the uncivilized tribes of the Islands, the Commission settlers of the Public Land Acts and the Torrens system
should adopt the same course followed by Congress in permitting the resulted in the titling of several ancestral lands in the
tribes of our North American Indians to maintain their tribal settlers' names. With government initiative and
organization and government, and under which many of those tribes are participation, this titling displaced several indigenous

9
peoples from their lands. Worse, these peoples were also provisions which insure the right of tribal Filipinos to
displaced by projects undertaken by the national government in the preserve their way of life.[if !supportFootnotes][96][endif] This Constitution
name of national development.[if !supportFootnotes][87][endif] goes further than the 1973 Constitution by expressly
It was in the 1973 Constitution that the State adopted the guaranteeing the rights of tribal Filipinos to their ancestral
following provision: domains and ancestral lands. By recognizing their right to
"The State shall consider the customs, traditions, beliefs, and their ancestral lands and domains, the State has effectively
interests of national cultural communities in the formulation and upheld their right to live in a culture distinctly their own.
implementation of State policies."[if !supportFootnotes][88][endif] 2. Their Concept of Land
For the first time in Philippine history, the "non-Christian tribes" or Indigenous peoples share distinctive traits that set them apart from the
the "cultural minorities" were addressed by the highest law of the Filipino mainstream. They are non-Christians. They live in less accessible,
Republic, and they were referred to as "cultural communities." More marginal, mostly upland areas. They have a system of self-government not
importantly this time, their "uncivilized" culture was given some recognition dependent upon the laws of the central administration of the Republic of the
and their "customs, traditions, beliefs and interests" were to be considered by Philippines. They follow ways of life and customs that are perceived as
the State in the formulation and implementation of State policies. President different from those of the rest of the population.[if !supportFootnotes][97][endif] The kind
Marcos abolished the CNI and transferred its functions to the Presidential of response the indigenous peoples chose to deal with colonial threat worked
Adviser on National Minorities (PANAMIN). The PANAMIN was tasked to well to their advantage by making it difficult for Western concepts and religion
integrate the ethnic groups that sought full integration into the larger to erode their customs and traditions. The "infieles societies" which had
community, and at the same time "protect the rights of those who wish to become peripheral to colonial administration, represented, from a cultural
preserve their original lifeways beside the larger community."[if !supportFootnotes][89] perspective, a much older base of archipelagic culture. The political systems
[endif]
In short, while still adopting the integration policy, the decree were still structured on the patriarchal and kinship oriented arrangement of
recognized the right of tribal Filipinos to preserve their way of life. [if ! power and authority. The economic activities were governed by the concepts
supportFootnotes][90][endif]
of an ancient communalism and mutual help. The social structure which
In 1974, President Marcos promulgated P.D. No. 410, otherwise known as emphasized division of labor and distinction of functions, not status, was
the Ancestral Lands Decree. The decree provided for the issuance of land maintained. The cultural styles and forms of life portraying the varieties of
occupancy certificates to members of the national cultural communities who social courtesies and ecological adjustments were kept constantly vibrant. [if !
were given up to 1984 to register their claims. [if !supportFootnotes][91][endif] In 1979, the supportFootnotes][98][endif]

Commission on the Settlement of Land Problems was created under E.O. Land is the central element of the indigenous peoples' existence.
No. 561 which provided a mechanism for the expeditious resolution of land There is no traditional concept of permanent, individual, land ownership.
problems involving small settlers, landowners, and tribal Filipinos. [if !supportFootnotes] Among the Igorots, ownership of land more accurately applies to the tribal
[92][endif]
right to use the land or to territorial control. The people are the secondary
Despite the promulgation of these laws, from 1974 to the owners or stewards of the land and that if a member of the tribe ceases to
early 1980's, some 100,000 Kalingas and Bontoks of the Cordillera work, he loses his claim of ownership, and the land reverts to the beings of
region were displaced by the Chico River dam project of the the spirit world who are its true and primary owners. Under the concept of
National Power Corporation (NPC). The Manobos of Bukidnon saw "trusteeship," the right to possess the land does not only belong to the
their land bulldozed by the Bukidnon Sugar Industries Company present generation but the future ones as well.[if !supportFootnotes][99][endif]
(BUSCO). In Agusan del Sur, the National Development Company Customary law on land rests on the traditional belief that no one owns the
was authorized by law in 1979 to take approximately 40,550 land except the gods and spirits, and that those who work the land are its
hectares of land that later became the NDC-Guthrie plantation in mere stewards.[if !supportFootnotes][100][endif] Customary law has a strong
Agusan del Sur. Most of the land was possessed by the Agusan preference for communal ownership, which could either be ownership by
natives.[if !supportFootnotes][93][endif] Timber concessions, water projects, a group of individuals or families who are related by blood or by marriage, [if !
plantations, mining, and cattle ranching and other projects of the supportFootnotes][101][endif]
or ownership by residents of the same locality who may not
national government led not only to the eviction of the indigenous be related by blood or marriage. The system of communal ownership under
peoples from their land but also to the reduction and destruction of customary laws draws its meaning from the subsistence and highly
their natural environment.[if !supportFootnotes][94][endif] collectivized mode of economic production. The Kalingas, for instance, who
The Aquino government signified a total shift from the policy of are engaged in team occupation like hunting, foraging for forest products, and
integration to one of preservation. Invoking her powers under the swidden farming found it natural that forest areas, swidden farms, orchards,
Freedom Constitution, President Aquino created the Office of Muslim pasture and burial grounds should be communally-owned. [if !supportFootnotes][102][endif]
Affairs, Office for Northern Cultural Communities and the Office for For the Kalingas, everybody has a common right to a common economic base.
Southern Cultural Communities all under the Office of the President. Thus, as a rule, rights and obligations to the land are shared in common.
[if !supportFootnotes][95][endif]
Although highly bent on communal ownership, customary law on land
The 1987 Constitution carries at least six (6) also sanctions individual ownership. The residential lots and terrace rice

10
farms are governed by a limited system of individual ownership. It is for easy titling or grant of lands to migrant homesteaders within the
limited because while the individual owner has the right to use and dispose of traditional areas of the ICCs."[if !supportFootnotes][109][endif]
the property, he does not possess all the rights of an exclusive and full owner Senator Flavier further declared:
as defined under our Civil Code. [if !supportFootnotes][103][endif] Under Kalinga customary "The IPs are the offsprings and heirs of the peoples who have first
law, the alienation of individually-owned land is strongly discouraged except in inhabited and cared for the land long before any central
marriage and succession and except to meet sudden financial needs due to government was established. Their ancestors had territories over
sickness, death in the family, or loss of crops. [if !supportFootnotes][104][endif] Moreover, which they ruled themselves and related with other tribes. These
and to be alienated should first be offered to a clan-member before any territories- the land- include people, their dwelling, the mountains,
village-member can purchase it, and in no case may land be sold to a non- the water, the air, plants, forest and the animals. This is their
member of the ili.[if !supportFootnotes][105][endif] environment in its totality. Their existence as indigenous peoples is
Land titles do not exist in the indigenous peoples' economic and manifested in their own lives through political, economic, socio-
social system. The concept of individual land ownership under the cultural and spiritual practices. The IPs culture is the living and
civil law is alien to them. Inherently colonial in origin, our national irrefutable proof to this.
land laws and governmental policies frown upon indigenous claims to Their survival depends on securing or acquiring land rights;
ancestral lands. Communal ownership is looked upon as inferior, if asserting their rights to it; and depending on it. Otherwise, IPs shall
not inexistent.[if !supportFootnotes][106][endif] cease to exist as distinct peoples."[if !supportFootnotes][110][endif]
III. THE IPRA IS A NOVEL PIECE OF LEGISLATION. To recognize the rights of the indigenous peoples effectively,
A. The Legislative History of the IPRA Senator Flavier proposed a bill based on two postulates: (1) the
It was to address the centuries-old neglect of the Philippine concept of native title; and (2) the principle of parens patriae.
indigenous peoples that the Tenth Congress of the Philippines, by their joint According to Senator Flavier, "[w]hile our legal tradition
efforts, passed and approved R.A. No. 8371, the Indigenous Peoples subscribes to the Regalian Doctrine reinstated in Section 2, Article
Rights Act (IPRA) of 1997. The law was a consolidation of two Bills-- Senate XII of the 1987 Constitution," our "decisional laws" and
Bill No. 1728 and House Bill No. 9125. jurisprudence passed by the State have "made exception to the
Principally sponsored by Senator Juan M. Flavier,[if ! doctrine." This exception was first laid down in the case of Cario
supportFootnotes][107][endif]
Senate Bill No. 1728 was a consolidation of four v. Insular Government where:
proposed measures referred to the Committees on Cultural "x x x the court has recognized long occupancy of land by an
Communities, Environment and Natural Resources, Ways and indigenous member of the cultural communities as one of
Means, as well as Finance. It adopted almost en toto the private ownership, which, in legal concept, is termed "native
comprehensive version of Senate Bill Nos. 1476 and 1486 which title." This ruling has not been overturned. In fact, it was
was a result of six regional consultations and one national affirmed in subsequent cases."[if !supportFootnotes][111][endif]
consultation with indigenous peoples nationwide.[if ! Following Cario, the State passed Act No. 926, Act No. 2874, C.A.
supportFootnotes][108][endif]
At the Second Regular Session of the Tenth No. 141, P.D. 705, P.D. 410, P.D. 1529, R.A. 6734 (the Organic Act for
Congress, Senator Flavier, in his sponsorship speech, gave a the Autonomous Region of Muslim Mindanao). These laws, explicitly
background on the situation of indigenous peoples in the or implicitly, and liberally or restrictively, recognized "native title"
Philippines, to wit: or "private right" and the existence of ancestral lands and domains.
"The Indigenous Cultural Communities, including the Bangsa Moro, Despite the passage of these laws, however, Senator Flavier
have long suffered from the dominance and neglect of government continued:
controlled by the majority. Massive migration of their Christian "x x x the executive department of government since the American
brothers to their homeland shrunk their territory and many of the occupation has not implemented the policy. In fact, it was
tribal Filipinos were pushed to the hinterlands. Resisting the more honored in its breach than in its observance, its wanton
intrusion, dispossessed of their ancestral land and with the massive disregard shown during the period unto the Commonwealth
exploitation of their natural resources by the elite among the and the early years of the Philippine Republic when
migrant population, they became marginalized. And the government organized and supported massive resettlement
government has been an indispensable party to this insidious of the people to the land of the ICCs."
conspiracy against the Indigenous Cultural Communities (ICCs). It Senate Bill No. 1728 seeks to genuinely recognize the IPs
organized and supported the resettlement of people to their right to own and possess their ancestral land. The bill was prepared
ancestral land, which was massive during the Commonwealth and also under the principle of parens patriae inherent in the supreme
early years of the Philippine Republic. Pursuant to the Regalian power of the State and deeply embedded in Philippine legal
Doctrine first introduced to our system by Spain through the Royal tradition. This principle mandates that persons suffering from
Decree of 13 February 1894 or the Maura Law, the government serious disadvantage or handicap, which places them in a position
passed laws to legitimize the wholesale landgrabbing and provide of actual inequality in their relation or transaction with others, are

11
entitled to the protection of the State. economic, social and cultural welfare. It shall include ancestral
Senate Bill No. 1728 was passed on Third Reading by twenty-one (21) lands, forests, pasture, residential, agricultural, and other lands
Senators voting in favor and none against, with no abstention. [if ! individually owned whether alienable and disposable or otherwise,
supportFootnotes][112][endif]
hunting grounds, burial grounds, worship areas, bodies of water,
House Bill No. 9125 was sponsored by Rep. Zapata, Chairman of the mineral and other natural resources, and lands which may no
Committee on Cultural Communities. It was originally authored and longer be exclusively occupied by ICCs/IPs but from which they
subsequently presented and defended on the floor by Rep. Gregorio traditionally had access to for their subsistence and traditional
Andolana of North Cotabato.[if !supportFootnotes][113][endif] activities, particularly the home ranges of ICCs/IPs who are still
Rep. Andolana's sponsorhip speech reads as follows: nomadic and/or shifting cultivators;
"This Representation, as early as in the 8th Congress, filed a bill of b) Ancestral Lands.-- Subject to Section 56 hereof, refers to land
similar implications that would promote, recognize the rights of occupied, possessed and utilized by individuals, families and clans
indigenous cultural communities within the framework of national who are members of the ICCs/IPs since time immemorial, by
unity and development. themselves or through their predecessors-in-interest, under claims
Apart from this, Mr. Speaker, is our obligation, the government's of individual or traditional group ownership, continuously, to the
obligation to assure and ascertain that these rights shall be well- present except when interrupted by war, force majeure or
preserved and the cultural traditions as well as the indigenous laws displacement by force, deceit, stealth, or as a consequence of
that remained long before this Republic was established shall be government projects and other voluntary dealings entered into by
preserved and promoted. There is a need, Mr. Speaker, to look into government and private individuals/corporations, including, but not
these matters seriously and early approval of the substitute bill limited to, residential lots, rice terraces or paddies, private forests,
shall bring into reality the aspirations, the hope and the dreams of swidden farms and tree lots."
more than 12 million Filipinos that they be considered in the Ancestral domains are all areas belonging to ICCs/IPs held under a claim of
mainstream of the Philippine society as we fashion for the year ownership, occupied or possessed by ICCs/IPs by themselves or through their
2000." [if !supportFootnotes][114][endif] ancestors, communally or individually since time immemorial, continuously
Rep. Andolana stressed that H.B. No. 9125 is based on the until the present, except when interrupted by war, force majeure or
policy of preservation as mandated in the Constitution. He also displacement by force, deceit, stealth or as a consequence of government
emphasized that the rights of IPs to their land was enunciated in projects or any other voluntary dealings with government and/or private
Cario v. Insular Government which recognized the fact that they individuals or corporations. Ancestral domains comprise lands, inland
had vested rights prior to the establishment of the Spanish and waters, coastal areas, and natural resources therein and includes
American regimes.[if !supportFootnotes][115][endif] ancestral lands, forests, pasture, residential, agricultural, and other
After exhaustive interpellation, House Bill No. 9125, and its lands individually owned whether alienable or not, hunting grounds,
corresponding amendments, was approved on Second Reading with burial grounds, worship areas, bodies of water, mineral and other
no objections. natural resources. They also include lands which may no longer be
IV. THE PROVISIONS OF THE IPRA DO NOT CONTRAVENE THE exclusively occupied by ICCs/IPs but from which they traditionally had access
CONSTITUTION. to for their subsistence and traditional activities, particularly the home ranges
A. Ancestral Domains and Ancestral Lands are the of ICCs/IPs who are still nomadic and/or shifting cultivators. [if !supportFootnotes][116][endif]
Private Property of Indigenous Peoples and Do Not Ancestral lands are lands held by the ICCs/IPs under the same conditions as
Constitute Part of the Land of the Public Domain. ancestral domains except that these are limited to lands and that these lands
The IPRA grants to ICCs/IPs a distinct kind of ownership over are not merely occupied and possessed but are also utilized by the ICCs/IPs
ancestral domains and ancestral lands. Ancestral lands are not the same under claims of individual or traditional group ownership. These lands include
as ancestral domains. These are defined in Section 3 [a] and [b] of the but are not limited to residential lots, rice terraces or paddies, private forests,
Indigenous Peoples Right Act, viz: swidden farms and tree lots.[if !supportFootnotes][117][endif]
"Sec. 3 a) Ancestral Domains. -- Subject to Section 56 hereof, The procedures for claiming ancestral domains and lands are
refer to all areas generally belonging to ICCs/IPs comprising lands, similar to the procedures embodied in Department Administrative
inland waters, coastal areas, and natural resources therein, held Order (DAO) No. 2, series of 1993, signed by then Secretary of the
under a claim of ownership, occupied or possessed by ICCs/IPs by Department of Environment and Natural Resources (DENR) Angel
themselves or through their ancestors, communally or individually Alcala.[if !supportFootnotes][118][endif] DAO No. 2 allowed the delineation of
since time immemorial, continuously to the present except when ancestral domains by special task forces and ensured the issuance
interrupted by war, force majeure or displacement by force, deceit, of Certificates of Ancestral Land Claims (CALC's) and Certificates of
stealth or as a consequence of government projects or any other Ancestral Domain Claims (CADC's) to IPs.
voluntary dealings entered into by government and private The identification and delineation of these ancestral domains
individuals/corporations, and which are necessary to ensure their and lands is a power conferred by the IPRA on the National

12
Commission on Indigenous Peoples (NCIP). [if !supportFootnotes][119][endif] The and domains. The IPRA categorically declares ancestral lands and
guiding principle in identification and delineation is self-delineation. domains held by native title as never to have been public land.
[if !supportFootnotes][120][endif]
This means that the ICCs/IPs have a decisive Domains and lands held under native title are, therefore,
role in determining the boundaries of their domains and in all the indisputably presumed to have never been public lands and are
activities pertinent thereto.[if !supportFootnotes][121][endif] private.
The procedure for the delineation and recognition of (a) Cario v. Insular Government[if !supportFootnotes][129][endif]
ancestral domains is set forth in Sections 51 and 52 of the IPRA. The concept of native title in the IPRA was taken from the
The identification, delineation and certification of ancestral lands 1909 case of Cario v. Insular Government.[if !supportFootnotes][130][endif]
is in Section 53 of said law. Cario firmly established a concept of private land title that existed
Upon due application and compliance with the procedure irrespective of any royal grant from the State.
provided under the law and upon finding by the NCIP that the In 1903, Don Mateo Cario, an Ibaloi, sought to register with
application is meritorious, the NCIP shall issue a Certificate of the land registration court 146 hectares of land in Baguio
Ancestral Domain Title (CADT) in the name of the community Municipality, Benguet Province. He claimed that this land had been
concerned.[if !supportFootnotes][122][endif] The allocation of lands within the possessed and occupied by his ancestors since time immemorial;
ancestral domain to any individual or indigenous corporate that his grandfather built fences around the property for the holding
(family or clan) claimants is left to the ICCs/IPs concerned to decide of cattle and that his father cultivated some parts of the land. Cario
in accordance with customs and traditions. [if !supportFootnotes][123][endif] With inherited the land in accordance with Igorot custom. He tried to
respect to ancestral lands outside the ancestral domain, the have the land adjusted under the Spanish land laws, but no
NCIP issues a Certificate of Ancestral Land Title (CALT). [if !supportFootnotes] document issued from the Spanish Crown. [if !supportFootnotes][131][endif] In
[124][endif]
1901, Cario obtained a possessory title to the land under the
CADT's and CALT's issued under the IPRA shall be registered Spanish Mortgage Law.[if !supportFootnotes][132][endif] The North American
by the NCIP before the Register of Deeds in the place where the colonial government, however, ignored his possessory title and built
property is situated.[if !supportFootnotes][125][endif] a public road on the land prompting him to seek a Torrens title to
(1) Right to Ancestral Domains and Ancestral Lands: How his property in the land registration court. While his petition was
Acquired pending, a U.S. military reservation [if !supportFootnotes][133][endif] was
The rights of the ICCs/IPs to their ancestral domains and proclaimed over his land and, shortly thereafter, a military
ancestral lands may be acquired in two modes: (1) by native title detachment was detailed on the property with orders to keep cattle
over both ancestral lands and domains; or (2) by torrens and trespassers, including Cario, off the land.[if !supportFootnotes][134][endif]
title under the Public Land Act and the Land Registration In 1904, the land registration court granted Cario's
Act with respect to ancestral lands only. application for absolute ownership to the land. Both the
(2) The Concept of Native Title Government of the Philippine Islands and the U.S. Government
Native title is defined as: appealed to the C.F.I. of Benguet which reversed the land
"Sec. 3 [l]. Native Title-- refers to pre-conquest rights to lands and registration court and dismissed Cario's application. The Philippine
domains which, as far back as memory reaches, have been held Supreme Court[if !supportFootnotes][135][endif] affirmed the C.F.I. by applying
under a claim of private ownership by ICCs/IPs, have never been the Valenton ruling. Cario took the case to the U.S. Supreme Court.
public lands and are thus indisputably presumed to have been [if !supportFootnotes][136][endif]
On one hand, the Philippine government
held that way since before the Spanish Conquest." [if !supportFootnotes][126] invoked the Regalian doctrine and contended that Cario failed to
[endif]
comply with the provisions of the Royal Decree of June 25, 1880,
Native title refers to ICCs/IPs' preconquest rights to lands and domains held which required registration of land claims within a limited period of
under a claim of private ownership as far back as memory reaches. These time. Cario, on the other, asserted that he was the absolute owner
lands are deemed never to have been public lands and are indisputably of the land jure gentium, and that the land never formed part of the
presumed to have been held that way since before the Spanish Conquest. The public domain.
rights of ICCs/IPs to their ancestral domains (which also include ancestral In a unanimous decision written by Justice Oliver Wendell
lands) by virtue of native title shall be recognized and respected. [if !supportFootnotes] Holmes, the U.S. Supreme Court held:
[127][endif]
Formal recognition, when solicited by ICCs/IPs concerned, shall be "It is true that Spain, in its earlier decrees, embodied the universal
embodied in a Certificate of Ancestral Domain Title (CADT), which shall feudal theory that all lands were held from the Crown, and perhaps
recognize the title of the concerned ICCs/IPs over the territories identified and the general attitude of conquering nations toward people not
delineated.[if !supportFootnotes][128][endif] recognized as entitled to the treatment accorded to those in the
Like a torrens title, a CADT is evidence of private ownership same zone of civilization with themselves. It is true, also, that in
of land by native title. Native title, however, is a right of private legal theory, sovereignty is absolute, and that, as against foreign
ownership peculiarly granted to ICCs/IPs over their ancestral lands nations, the United States may assert, as Spain asserted, absolute

13
power. But it does not follow that, as against the inhabitants of the The court thus laid down the presumption of a certain title held (1) as far
Philippines, the United States asserts that Spain had such power. back as testimony or memory went, and (2) under a claim of private
When theory is left on one side, sovereignty is a question of ownership. Land held by this title is presumed to "never have been public
strength, and may vary in degree. How far a new sovereign shall land."
insist upon the theoretical relation of the subjects to the head in the Against this presumption, the U.S. Supreme Court analyzed the Spanish
past, and how far it shall recognize actual facts, are matters for it to decrees upheld in the 1904 decision of Valenton v. Murciano. The U.S.
decide."[if !supportFootnotes][137][endif] Supreme Court found no proof that the Spanish decrees did not honor native
The U.S. Supreme Court noted that it need not accept Spanish title. On the contrary, the decrees discussed in Valenton appeared to
doctrines. The choice was with the new colonizer. Ultimately, the matter had recognize that the natives owned some land, irrespective of any royal grant.
to be decided under U.S. law. The Regalian doctrine declared in the preamble of the Recopilacion was all
The Cario decision largely rested on the North American "theory and discourse" and it was observed that titles were admitted to exist
constitutionalist's concept of "due process" as well as the pronounced policy beyond the powers of the Crown, viz:
"to do justice to the natives."[if !supportFootnotes][138][endif] It was based on the strong "If the applicant's case is to be tried by the law of Spain, we do not
mandate extended to the Islands via the Philippine Bill of 1902 that "No law discover such clear proof that it was bad by that law as to satisfy us
shall be enacted in said islands which shall deprive any person of life, liberty, that he does not own the land. To begin with, the older decrees and
or property without due process of law, or deny to any person therein the laws cited by the counsel for the plaintiff in error seem to indicate
equal protection of the laws." The court declared: pretty clearly that the natives were recognized as owning some
"The acquisition of the Philippines was not like the settlement of the white lands, irrespective of any royal grant. In other words, Spain did not
race in the United States. Whatever consideration may have been shown to assume to convert all the native inhabitants of the Philippines into trespassers
the North American Indians, the dominant purpose of the whites in America or even into tenants at will. For instance, Book 4, title 12, Law 14 of the the
was to occupy land. It is obvious that, however stated, the reason for our Recopilacion de Leyes de las Indias, cited for a contrary conclusion in Valenton
taking over the Philippines was different. No one, we suppose, would deny v. Murciano, 3 Philippine 537, while it commands viceroys and others, when it
that, so far as consistent with paramount necessities, our first object in the seems proper, to call for the exhibition of grants, directs them to confirm
internal administration of the islands is to do justice to the natives, not to those who hold by good grants or justa prescripcion. It is true that it begins
exploit their country for private gain. By the Organic Act of July 1, 1902, by the characteristic assertion of feudal overlordship and the origin
chapter 1369, section 12 (32 Statutes at Large, 691), all the property and of all titles in the King or his predecessors. That was theory and
rights acquired there by the United States are to be administered 'for the discourse. The fact was that titles were admitted to exist that owed
benefit of the inhabitants thereof.' It is reasonable to suppose that the attitude nothing to the powers of Spain beyond this recognition in their
thus assumed by the United States with regard to what was unquestionably its books." (Emphasis supplied).[if !supportFootnotes][141][endif]
own is also its attitude in deciding what it will claim for its own. The same The court further stated that the Spanish "adjustment" proceedings
statute made a bill of rights, embodying the safeguards of the Constitution, never held sway over unconquered territories. The wording of the Spanish
and, like the Constitution, extends those safeguards to all. It provides that 'no laws were not framed in a manner as to convey to the natives that failure to
law shall be enacted in said islands which shall deprive any person of life, register what to them has always been their own would mean loss of such
liberty, or property without due process of law, or deny to any person therein land. The registration requirement was "not to confer title, but simply to
the equal protection of the laws.' In the light of the declaration that we have establish it;" it was "not calculated to convey to the mind of an Igorot chief the
quoted from section 12, it is hard to believe that the United States was ready notion that ancient family possessions were in danger, if he had read every
to declare in the next breath that "any person" did not embrace the word of it."
inhabitants of Benguet, or that it meant by "property" only that which had By recognizing this kind of title, the court clearly repudiated the
become such by ceremonies of which presumably a large part of the doctrine of Valenton. It was frank enough, however, to admit the possibility
inhabitants never had heard, and that it proposed to treat as public land what that the applicant might have been deprived of his land under Spanish law
they, by native custom and by long association,-- of the profoundest factors in because of the inherent ambiguity of the decrees and concomitantly, the
human thought,-- regarded as their own."[if !supportFootnotes][139][endif] various interpretations which may be given them. But precisely because of
The Court went further: the ambiguity and of the strong "due process mandate" of the
"[E]very presumption is and ought to be against the government in a case like Constitution, the court validated this kind of title. [if !supportFootnotes][142][endif]
the present. It might, perhaps, be proper and sufficient to say that This title was sufficient, even without government administrative action, and
when, as far back as testimony or memory goes, the land has been entitled the holder to a Torrens certificate. Justice Holmes explained:
held by individuals under a claim of private ownership, it will be "It will be perceived that the rights of the applicant under the Spanish law
presumed to have been held in the same way from before the present a problem not without difficulties for courts of a legal tradition. We
Spanish conquest, and never to have been public land. Certainly in a have deemed it proper on that account to notice the possible effect of the
case like this, if there is doubt or ambiguity in the Spanish law, we ought to change of sovereignty and the act of Congress establishing the fundamental
give the applicant the benefit of the doubt."[if !supportFootnotes][140][endif] principles now to be observed. Upon a consideration of the whole case we are

14
of the opinion that law and justice require that the applicant should be protecting the public forests in which they roamed. [if !supportFootnotes][151][endif]
granted what he seeks, and should not be deprived of what, by the practice Speaking through Justice Malcolm, the court said:
and belief of those among whom he lived, was his property, through a refined "Reference was made in the President's instructions to the Commission to the
interpretation of an almost forgotten law of Spain." [if !supportFootnotes][143][endif] policy adopted by the United States for the Indian Tribes. The methods
Thus, the court ruled in favor of Cario and ordered the registration of followed by the Government of the Philippine Islands in its dealings with the
the 148 hectares in Baguio Municipality in his name.[if !supportFootnotes][144][endif] so-called non-Christian people is said, on argument, to be practically identical
Examining Cario closer, the U.S. Supreme Court did not categorically with that followed by the United States Government in its dealings with the
refer to the title it upheld as "native title." It simply said: Indian tribes. Valuable lessons, it is insisted, can be derived by an
"The Province of Benguet was inhabited by a tribe that the Solicitor- investigation of the American-Indian policy.
General, in his argument, characterized as a savage tribe that never From the beginning of the United States, and even before, the Indians have
was brought under the civil or military government of the Spanish been treated as "in a state of pupilage." The recognized relation between the
Crown. It seems probable, if not certain, that the Spanish officials Government of the United States and the Indians may be described as that of
would not have granted to anyone in that province the registration to guardian and ward. It is for the Congress to determine when and how the
which formerly the plaintiff was entitled by the Spanish Laws, and guardianship shall be terminated. The Indians are always subject to the
which would have made his title beyond question good. Whatever may plenary authority of the United States.[if !supportFootnotes][152][endif]
have been the technical position of Spain it does not follow that, in the view of x x x.
the United States, he had lost all rights and was a mere trespasser when the As to the second point, the facts in the Standing Bear case and the Rubi case
present government seized his land. The argument to that effect seems to are not exactly identical. But even admitting similarity of facts, yet it is known
amount to a denial of native titles through an important part of the Island of to all that Indian reservations do exist in the United States, that Indians have
Luzon, at least, for the want of ceremonies which the Spaniards would not been taken from different parts of the country and placed on these
have permitted and had not the power to enforce."[if !supportFootnotes][145][endif] reservations, without any previous consultation as to their own wishes, and
This is the only instance when Justice Holmes used the term "native title" in that, when once so located, they have been made to remain on the
the entire length of the Cario decision. It is observed that the widespread use reservation for their own good and for the general good of the country. If any
of the term "native title" may be traced to Professor Owen James Lynch, Jr., a lesson can be drawn from the Indian policy of the United States, it is that the
Visiting Professor at the University of the Philippines College of Law from the determination of this policy is for the legislative and executive branches of the
Yale University Law School. In 1982, Prof. Lynch published an article in the government and that when once so decided upon, the courts should not
Philippine Law Journal entitled Native Title, Private Right and Tribal interfere to upset a carefully planned governmental system. Perhaps, just as
Land Law.[if !supportFootnotes][146][endif] This article was made after Professor Lynch many forceful reasons exist for the segregation of the Manguianes in Mindoro
visited over thirty tribal communities throughout the country and studied the as existed for the segregation of the different Indian tribes in the United
origin and development of Philippine land laws. [if !supportFootnotes][147][endif] He States."[if !supportFootnotes][153][endif]
discussed Cario extensively and used the term "native title" to refer to Cario's Rubi applied the concept of Indian land grants or reservations in the
title as discussed and upheld by the U.S. Supreme Court in said case. Philippines. An Indian reservation is a part of the public domain set apart by
(b) Indian Title proper authority for the use and occupation of a tribe or tribes of Indians. [if !
In a footnote in the same article, Professor Lynch stated that the supportFootnotes][154][endif]
It may be set apart by an act of Congress, by treaty, or by
concept of "native title" as defined by Justice Holmes in Cario "is conceptually executive order, but it cannot be established by custom and prescription. [if !
similar to "aboriginal title" of the American Indians. [if !supportFootnotes][148][endif] This is supportFootnotes][155][endif]

not surprising, according to Prof. Lynch, considering that during the American Indian title to land, however, is not limited to land grants or
regime, government policy towards ICCs/IPs was consistently made in reservations. It also covers the "aboriginal right of possession or
reference to native Americans.[if !supportFootnotes][149][endif] This was clearly occupancy."[if !supportFootnotes][156][endif] The aboriginal right of possession depends
demonstrated in the case of Rubi v. Provincial Board of Mindoro.[if ! on the actual occupancy of the lands in question by the tribe or nation as their
supportFootnotes][150][endif]
ancestral home, in the sense that such lands constitute definable territory
In Rubi, the Provincial Board of Mindoro adopted a Resolution occupied exclusively by the particular tribe or nation. [if !supportFootnotes][157][endif] It is a
authorizing the provincial governor to remove the Mangyans from their right which exists apart from any treaty, statute, or other governmental
domains and place them in a permanent reservation in Sitio Tigbao, Lake action, although in numerous instances treaties have been negotiated with
Naujan. Any Mangyan who refused to comply was to be imprisoned. Rubi and Indian tribes, recognizing their aboriginal possession and delimiting their
some Mangyans, including one who was imprisoned for trying to escape from occupancy rights or settling and adjusting their boundaries. [if !supportFootnotes][158]
the reservation, filed for habeas corpus claiming deprivation of liberty under [endif]

the Board Resolution. This Court denied the petition on the ground of police American jurisprudence recognizes the Indians' or native Americans'
power. It upheld government policy promoting the idea that a permanent rights to land they have held and occupied before the "discovery" of
settlement was the only successful method for educating the Mangyans, the Americas by the Europeans. The earliest definitive statement by
introducing civilized customs, improving their health and morals, and the U.S. Supreme Court on the nature of aboriginal title was made in

15
1823 in Johnson & Graham's Lessee v. M'Intosh.[if !supportFootnotes][159][endif] "It has never been contended that the Indian title amounted to nothing. Their
In Johnson, the plaintiffs claimed the land in question under two (2) right of possession has never been questioned. The claim of
grants made by the chiefs of two (2) Indian tribes. The U.S. Supreme Court government extends to the complete ultimate title, charged with this
refused to recognize this conveyance, the plaintiffs being private persons. The right of possession, and to the exclusive power of acquiring that
only conveyance that was recognized was that made by the Indians to the right."[if !supportFootnotes][162][endif]
government of the European discoverer. Speaking for the court, Chief Justice It has been said that the history of America, from its discovery to the
Marshall pointed out that the potentates of the old world believed that they present day, proves the universal recognition of this principle. [if !supportFootnotes][163]
had made ample compensation to the inhabitants of the new world by [endif]

bestowing civilization and Christianity upon them; but in addition, said the The Johnson doctrine was a compromise. It protected Indian rights and
court, they found it necessary, in order to avoid conflicting settlements and their native lands without having to invalidate conveyances made by the
consequent war, to establish the principle that discovery gives title to the government to many U.S. citizens.[if !supportFootnotes][164][endif]
government by whose subjects, or by whose authority, the discovery Johnson was reiterated in the case of Worcester v. Georgia.[if !
was made, against all other European governments, which title might supportFootnotes][165][endif]
In this case, the State of Georgia enacted a law requiring all
be consummated by possession.[if !supportFootnotes][160][endif] The exclusion of all white persons residing within the Cherokee nation to obtain a license or permit
other Europeans gave to the nation making the discovery the sole right of from the Governor of Georgia; and any violation of the law was deemed a high
acquiring the soil from the natives and establishing settlements upon it. As misdemeanor. The plaintiffs, who were white missionaries, did not obtain said
regards the natives, the court further stated that: license and were thus charged with a violation of the Act.
"Those relations which were to exist between the discoverer and the natives The U.S. Supreme Court declared the Act as unconstitutional for
were to be regulated by themselves. The rights thus acquired being exclusive, interfering with the treaties established between the United States and the
no other power could interpose between them. Cherokee nation as well as the Acts of Congress regulating intercourse with
In the establishment of these relations, the rights of the original inhabitants them. It characterized the relationship between the United States government
were, in no instance, entirely disregarded; but were necessarily, to a and the Indians as:
considerable extent, impaired. They were admitted to be the rightful "The Indian nations were, from their situation, necessarily dependent on some
occupants of the soil, with a legal as well as just claim to retain foreign potentate for the supply of their essential wants, and for their
possession of it, and to use it according to their own discretion; but protection from lawless and injurious intrusions into their country. That power
their rights to complete sovereignty, as independent nations, were necessarily was naturally termed their protector. They had been arranged under the
diminished, and their power to dispose of the soil at their own will, to protection of Great Britain; but the extinguishment of the British power in their
whomsoever they pleased, was denied by the fundamental principle that neighborhood, and the establishment of that of the United States in its place,
discovery gave exclusive title to those who made it. led naturally to the declaration, on the part of the Cherokees, that they were
While the different nations of Europe respected the right of the under the protection of the United States, and of no other power. They
natives as occupants, they asserted the ultimate dominion to be in assumed the relation with the United States which had before subsisted with
themselves; and claimed and exercised, as a consequence of this Great Britain.
ultimate dominion, a power to grant the soil, while yet in possession This relation was that of a nation claiming and receiving the protection of one
of the natives. These grants have been understood by all to convey a more powerful, not that of individuals abandoning their national character,
title to the grantees, subject only to the Indian right of occupancy." [if ! and submitting as subjects to the laws of a master."[if !supportFootnotes][166][endif]
supportFootnotes][161][endif]
It was the policy of the U.S. government to treat the Indians as nations
Thus, the discoverer of new territory was deemed to have obtained the with distinct territorial boundaries and recognize their right of occupancy over
exclusive right to acquire Indian land and extinguish Indian titles. Only to all the lands within their domains. Thus:
the discoverer-- whether to England, France, Spain or Holland-- did this right "From the commencement of our government Congress has passed acts to
belong and not to any other nation or private person. The mere acquisition of regulate trade and intercourse with the Indians; which treat them as nations,
the right nonetheless did not extinguish Indian claims to land. Rather, until the respect their rights, and manifest a firm purpose to afford that protection
discoverer, by purchase or conquest, exercised its right, the concerned Indians which treaties stipulate. All these acts, and especially that of 1802, which is
were recognized as the "rightful occupants of the soil, with a legal as well as still in force, manifestly consider the several Indian nations as distinct
just claim to retain possession of it." Grants made by the discoverer to her political communities, having territorial boundaries, within which
subjects of lands occupied by the Indians were held to convey a title to the their authority is exclusive, and having a right to all the lands within
grantees, subject only to the Indian right of occupancy. Once the discoverer those boundaries, which is not only acknowledged, but guaranteed
purchased the land from the Indians or conquered them, it was only then that by the United States.
the discoverer gained an absolute title unrestricted by Indian rights. x x x.
The court concluded, in essence, that a grant of Indian lands by Indians "The Indian nations had always been considered as distinct,
could not convey a title paramount to the title of the United States itself to independent political communities, retaining their original natural
other parties, saying: rights, as the undisputed possessors of the soil from time

16
immemorial, with the single exception of that imposed by irresistible power, disposal under general laws. [if !supportFootnotes][178][endif] Indian land which has been
which excluded them from intercourse with any other European potentate abandoned is deemed to fall into the public domain. [if !supportFootnotes][179][endif] On
than the first discoverer of the coast of the particular region claimed: and this the other hand, an Indian reservation is a part of the public domain set apart
was a restriction which those European potentates imposed on themselves, as for the use and occupation of a tribe of Indians. [if !supportFootnotes][180][endif] Once set
well as on the Indians. The very term "nation," so generally applied to them, apart by proper authority, the reservation ceases to be public land, and until
means "a people distinct from others." x x x.[if !supportFootnotes][167][endif] the Indian title is extinguished, no one but Congress can initiate any
The Cherokee nation, then, is a distinct community, occupying its own preferential right on, or restrict the nation's power to dispose of, them. [if !
territory, with boundaries accurately described, in which the laws of Georgia supportFootnotes][181][endif]

can have no force, and which the citizens of Georgia have no right to enter but The American judiciary struggled for more than 200 years with the
with the assent of the Cherokees themselves or in conformity with treaties and ancestral land claims of indigenous Americans.[if !supportFootnotes][182][endif] And
with the acts of Congress. The whole intercourse between the United States two things are clear. First, aboriginal title is recognized. Second, indigenous
and this nation is, by our Constitution and laws, vested in the government of property systems are also recognized. From a legal point of view, certain
the United States."[if !supportFootnotes][168][endif] benefits can be drawn from a comparison of Philippine IPs to native
The discovery of the American continent gave title to the government of Americans.[if !supportFootnotes][183][endif] Despite the similarities between native title
the discoverer as against all other European governments. Designated as the and aboriginal title, however, there are at present some misgivings on
naked fee,[if !supportFootnotes][169][endif] this title was to be consummated by possession whether jurisprudence on American Indians may be cited authoritatively in the
and was subject to the Indian title of occupancy. The discoverer acknowledged Philippines. The U.S. recognizes the possessory rights of the Indians over their
the Indians' legal and just claim to retain possession of the land, the Indians land; title to the land, however, is deemed to have passed to the U.S. as
being the original inhabitants of the land. The discoverer nonetheless asserted successor of the discoverer. The aboriginal title of ownership is not specifically
the exclusive right to acquire the Indians' land-- either by purchase, recognized as ownership by action authorized by Congress. [if !supportFootnotes][184][endif]
"defensive" conquest, or cession-- and in so doing, extinguish the Indian title. The protection of aboriginal title merely guards against encroachment by
Only the discoverer could extinguish Indian title because it alone asserted persons other than the Federal Government. [if !supportFootnotes][185][endif] Although
ultimate dominion in itself. Thus, while the different nations of Europe there are criticisms against the refusal to recognize the native Americans'
respected the rights of the natives as occupants, they all asserted the ownership of these lands,[if !supportFootnotes][186][endif] the power of the State to
ultimate dominion and title to be in themselves.[if !supportFootnotes][170][endif] extinguish these titles has remained firmly entrenched. [if !supportFootnotes][187][endif]
As early as the 19th century, it became accepted doctrine that Under the IPRA, the Philippine State is not barred form asserting
although fee title to the lands occupied by the Indians when the sovereignty over the ancestral domains and ancestral lands. [if !supportFootnotes][188]
colonists arrived became vested in the sovereign-- first the [endif]
The IPRA, however, is still in its infancy and any similarities between its
discovering European nation and later the original 13 States and the application in the Philippines vis--vis American Jurisprudence on aboriginal title
United States-- a right of occupancy in the Indian tribes was will depend on the peculiar facts of each case.
nevertheless recognized. The Federal Government continued the policy of (c) Why the Cario doctrine is unique
respecting the Indian right of occupancy, sometimes called Indian title, which In the Philippines, the concept of native title first upheld in Cario and
it accorded the protection of complete ownership. [if !supportFootnotes][171][endif] But this enshrined in the IPRA grants ownership, albeit in limited form, of the land to
aboriginal Indian interest simply constitutes "permission" from the whites to the ICCs/IPs. Native title presumes that the land is private and was never
occupy the land, and means mere possession not specifically recognized as public. Cario is the only case that specifically and categorically
ownership by Congress.[if !supportFootnotes][172][endif] It is clear that this right of recognizes native title. The long line of cases citing Cario did not
occupancy based upon aboriginal possession is not a property right. [if ! touch on native title and the private character of ancestral domains
supportFootnotes][173][endif]
It is vulnerable to affirmative action by the federal and lands. Cario was cited by the succeeding cases to support the
government who, as sovereign, possessed exclusive power to extinguish the concept of acquisitive prescription under the Public Land Act which is
right of occupancy at will.[if !supportFootnotes][174][endif] Thus, aboriginal title is not a different matter altogether. Under the Public Land Act, land sought to be
the same as legal title. Aboriginal title rests on actual, exclusive and registered must be public agricultural land. When the conditions specified
continuous use and occupancy for a long time. [if !supportFootnotes][175][endif] It entails in Section 48 [b] of the Public Land Act are complied with, the possessor of the
that land owned by Indian title must be used within the tribe, subject to its land is deemed to have acquired, by operation of law, a right to a grant of the
laws and customs, and cannot be sold to another sovereign government nor to land.[if !supportFootnotes][189][endif] The land ceases to be part of the public domain, [if !
any citizen.[if !supportFootnotes][176][endif] Such title as Indians have to possess and supportFootnotes][190][endif]
ipso jure,[if !supportFootnotes][191][endif] and is converted to private
occupy land is in the tribe, and not in the individual Indian; the right of property by the mere lapse or completion of the prescribed statutory period.
individual Indians to share in the tribal property usually depends upon tribal It was only in the case of Oh Cho v. Director of Lands [if !supportFootnotes][192]
membership, the property of the tribe generally being held in communal [endif]
that the court declared that the rule that all lands that were not acquired
ownership.[if !supportFootnotes][177][endif] from the government, either by purchase or grant, belong to the public
As a rule, Indian lands are not included in the term "public lands," which domain has an exception. This exception would be any land that should have
is ordinarily used to designate such lands as are subject to sale or other been in the possession of an occupant and of his predecessors-in-interest

17
since time immemorial. It is this kind of possession that would justify the ancestral lands are classified as alienable and disposable agricultural lands of
presumption that the land had never been part of the public domain or that it the public domain, provided, they are agricultural in character and are
had been private property even before the Spanish conquest. [if !supportFootnotes][193] actually used for agricultural, residential, pasture and tree farming purposes.
[endif]
Oh Cho, however, was decided under the provisions of the Public Land These lands shall be classified as public agricultural lands regardless of
Act and Cario was cited to support the applicant's claim of acquisitive whether they have a slope of 18% or more.
prescription under the said Act. The classification of ancestral land as public agricultural land is in
All these years, Cario had been quoted out of context simply to justify compliance with the requirements of the Public Land Act and the Land
long, continuous, open and adverse possession in the concept of owner of Registration Act. C.A. 141, the Public Land Act, deals specifically with lands of
public agricultural land. It is this long, continuous, open and adverse the public domain.[if !supportFootnotes][198][endif] Its provisions apply to those lands
possession in the concept of owner of thirty years both for ordinary citizens [if ! "declared open to disposition or concession" x x x "which have not been
supportFootnotes][194][endif]
and members of the national cultural minorities [if !supportFootnotes] reserved for public or quasi-public purposes, nor appropriated by the
[195][endif]
that converts the land from public into private and entitles the Government, nor in any manner become private property, nor those on which
registrant to a torrens certificate of title. a private right authorized and recognized by this Act or any other valid law x x
(3) The Option of Securing a Torrens Title to the Ancestral Land x or which having been reserved or appropriated, have ceased to be so." [if !
Indicates that the Land is Private. supportFootnotes][199][endif]
Act 496, the Land Registration Act, allows registration only
The private character of ancestral lands and domains as laid down in of private lands and public agricultural lands. Since ancestral domains and
the IPRA is further strengthened by the option given to individual ICCs/IPs lands are private, if the ICC/IP wants to avail of the benefits of C.A.
over their individually-owned ancestral lands. For purposes of registration 141 and Act 496, the IPRA itself converts his ancestral land,
under the Public Land Act and the Land Registration Act, the IPRA regardless of whether the land has a slope of eighteen per cent
expressly converts ancestral land into public agricultural land which (18%) or over,[if !supportFootnotes][200][endif] from private to public agricultural
may be disposed of by the State. The necessary implication is that land for proper disposition.
ancestral land is private. It, however, has to be first converted to The option to register land under the Public Land Act and the Land
public agricultural land simply for registration purposes. To wit: Registration Act has nonetheless a limited period. This option must be
"Sec. 12. Option to Secure Certificate of Title Under Commonwealth Act 141, exercised within twenty (20) years from October 29, 1997, the date of
as amended, or the Land Registration Act 496-- Individual members of cultural approval of the IPRA.
communities, with respect to their individually-owned ancestral lands who, by Thus, ancestral lands and ancestral domains are not part of the lands
themselves or through their predecessors-in-interest, have been in continuous of the public domain. They are private and belong to the ICCs/IPs.
possession and occupation of the same in the concept of owner since time Section 3 of Article XII on National Economy and Patrimony of the 1987
immemorial or for a period of not less than thirty (30) years immediately Constitution classifies lands of the public domain into four categories: (a)
preceding the approval of this Act and uncontested by the members of the agricultural, (b) forest or timber, (c) mineral lands, and (d) national parks.
same ICCs/IPs shall have the option to secure title to their ancestral lands Section 5 of the same Article XII mentions ancestral lands and ancestral
under the provisions of Commonwealth Act 141, as amended, or the Land domains but it does not classify them under any of the said four categories. To
Registration Act 496. classify them as public lands under any one of the four classes will
For this purpose, said individually-owned ancestral lands, which are render the entire IPRA law a nullity. The spirit of the IPRA lies in the
agricultural in character and actually used for agricultural, residential, pasture, distinct concept of ancestral domains and ancestral lands. The IPRA addresses
and tree farming purposes, including those with a slope of eighteen percent the major problem of the ICCs/IPs which is loss of land. Land and space are of
(18%) or more, are hereby classified as alienable and disposable agricultural vital concern in terms of sheer survival of the ICCs/IPs. [if !supportFootnotes][201][endif]
lands. The 1987 Constitution mandates the State to "protect the rights of
The option granted under this section shall be exercised within twenty (20) indigenous cultural communities to their ancestral lands" and that
years from the approval of this Act."[if !supportFootnotes][196][endif] "Congress provide for the applicability of customary laws x x x in
ICCs/IPs are given the option to secure a torrens certificate of title over their determining the ownership and extent of ancestral domain." [if !
individually-owned ancestral lands. This option is limited to ancestral lands supportFootnotes][202][endif]
It is the recognition of the ICCs/IPs distinct rights of
only, not domains, and such lands must be individually, not communally, ownership over their ancestral domains and lands that breathes life
owned. into this constitutional mandate.
Ancestral lands that are owned by individual members of ICCs/IPs who, by B. The right of ownership and possession by the ICCs/IPs of their
themselves or through their predecessors-in-interest, have been in continuous ancestral domains is a limited form of ownership and does not
possession and occupation of the same in the concept of owner since time include the right to alienate the same.
immemorial[if !supportFootnotes][197][endif] or for a period of not less than 30 years, which Registration under the Public Land Act and Land Registration Act
claims are uncontested by the members of the same ICCs/IPs, may be recognizes the concept of ownership under the civil law. This ownership is
registered under C.A. 141, otherwise known as the Public Land Act, or Act 496, based on adverse possession for a specified period, and harkens to Section 44
the Land Registration Act. For purposes of registration, the individually-owned of the Public Land Act on administrative legalization (free patent) of imperfect

18
or incomplete titles and Section 48 (b) and (c) of the same Act on the judicial This concept is based on Roman Law which the Spaniards introduced to the
confirmation of imperfect or incomplete titles. Thus: Philippines through the Civil Code of 1889. Ownership, under Roman Law, may
"Sec. 44. Any natural-born citizen of the Philippines who is not the owner of be exercised over things or rights. It primarily includes the right of the owner
more than twenty-four hectares and who since July fourth, 1926 or prior to enjoy and dispose of the thing owned. And the right to enjoy and dispose of
thereto, has continuously occupied and cultivated, either by himself or the thing includes the right to receive from the thing what it produces, [if !
through his predecessors-in-interest, a tract or tracts of agricultural public supportFootnotes][205][endif]
the right to consume the thing by its use, [if !supportFootnotes][206][endif]
lands subject to disposition, or who shall have paid the real estate tax thereon the right to alienate, encumber, transform or even destroy the thing owned, [if !
while the same has not been occupied by any person shall be entitled, under supportFootnotes][207][endif]
and the right to exclude from the possession of the thing
the provisions of this chapter, to have a free patent issued to him for such owned by any other person to whom the owner has not transmitted such
tract or tracts of such land not to exceed twenty-four hectares. thing.[if !supportFootnotes][208][endif]
A member of the national cultural minorities who has continuously 1. The Indigenous Concept of Ownership and Customary Law.
occupied and cultivated, either by himself or through his Ownership of ancestral domains by native title does not entitle the
predecessors-in-interest, a tract or tracts of land, whether disposable ICC/IP to a torrens title but to a Certificate of Ancestral Domain Title (CADT).
or not since July 4, 1955, shall be entitled to the right granted in the The CADT formally recognizes the indigenous concept of ownership of the
preceding paragraph of this section: Provided, That at the time he ICCs/IPs over their ancestral domain. Thus:
files his free patent application he is not the owner of any real "Sec. 5. Indigenous concept of ownership.- Indigenous concept of ownership
property secured or disposable under the provision of the Public sustains the view that ancestral domains and all resources found therein shall
Land Law.[if !supportFootnotes][203][endif] serve as the material bases of their cultural integrity. The indigenous concept
x x x. of ownership generally holds that ancestral domains are the ICCs/IPs private
"Sec. 48. The following described citizens of the Philippines, occupying lands but community property which belongs to all generations and therefore
of the public domain or claiming to own any such lands or an interest therein, cannot be sold, disposed or destroyed. It likewise covers sustainable
but whose titles have not been perfected or completed, may apply to the traditional resource rights."
Court of First Instance of the province where the land is located for The right of ownership and possession of the ICCs/IPs to their
confirmation of their claims and the issuance of a certificate of title therefor, ancestral domains is held under the indigenous concept of
under the Land Registration Act, to wit: ownership. This concept maintains the view that ancestral domains
(a) [perfection of Spanish titles] xxx. are the ICCs/IPs private but community property. It is private simply
(b) Those who by themselves or through their predecessors-in-interest have because it is not part of the public domain. But its private character
been in open, continuous, exclusive, and notorious possession and occupation ends there. The ancestral domain is owned in common by the ICCs/IPs
of agricultural lands of the public domain, under a bona fide claim of and not by one particular person. The IPRA itself provides that areas
acquisition or ownership, for at least thirty years immediately preceding the within the ancestral domains, whether delineated or not, are presumed to be
filing of the application for confirmation of title except when prevented by war communally held.[if !supportFootnotes][209][endif] These communal rights, however,
or force majeure. These shall be conclusively presumed to have performed all are not exactly the same as co-ownership rights under the Civil Code.
the conditions essential to a Government grant and shall be entitled to a [if !supportFootnotes][210][endif]
Co-ownership gives any co-owner the right to demand
certificate of title under the provisions of this Chapter. partition of the property held in common. The Civil Code expressly provides
(c) Members of the national cultural minorities who by themselves or that "[n]o co-owner shall be obliged to remain in the co-ownership." Each co-
through their predecessors-in-interest have been in open, owner may demand at any time the partition of the thing in common, insofar
continuous, exclusive and notorious possession and occupation of as his share is concerned. [if !supportFootnotes][211][endif] To allow such a right over
lands of the public domain suitable to agriculture, whether ancestral domains may be destructive not only of customary law of the
disposable or not, under a bona fide claim of ownership for at least community but of the very community itself.[if !supportFootnotes][212][endif]
30 years shall be entitled to the rights granted in sub-section (b) Communal rights over land are not the same as corporate rights over
hereof."[if !supportFootnotes][204][endif] real property, much less corporate condominium rights. A corporation
Registration under the foregoing provisions presumes that the land was can exist only for a maximum of fifty (50) years subject to an extension of
originally public agricultural land but because of adverse possession since July another fifty years in any single instance. [if !supportFootnotes][213][endif] Every
4, 1955 (free patent) or at least thirty years (judicial confirmation), the land stockholder has the right to disassociate himself from the corporation. [if !
has become private. Open, adverse, public and continuous possession is supportFootnotes][214][endif]
Moreover, the corporation itself may be dissolved voluntarily
sufficient, provided, the possessor makes proper application therefor. The or involuntarily.[if !supportFootnotes][215][endif]
possession has to be confirmed judicially or administratively after which a Communal rights to the land are held not only by the present
torrens title is issued. possessors of the land but extends to all generations of the ICCs/IPs,
A torrens title recognizes the owner whose name appears in the past, present and future, to the domain. This is the reason why the
certificate as entitled to all the rights of ownership under the civil law. The ancestral domain must be kept within the ICCs/IPs themselves. The domain
Civil Code of the Philippines defines ownership in Articles 427, 428 and 429. cannot be transferred, sold or conveyed to other persons. It belongs to the

19
ICCs/IPs as a community. b) Right to Develop Lands and Natural Resources.-- Subject to Section 56
Ancestral lands are also held under the indigenous concept of hereof, the right to develop, control and use lands and territories
ownership. The lands are communal. These lands, however, may be traditionally occupied, owned, or used; to manage and conserve
transferred subject to the following limitations: (a) only to the members of the natural resources within the territories and uphold the
same ICCs/IPs; (b) in accord with customary laws and traditions; and (c) responsibilities for future generations; to benefit and share the
subject to the right of redemption of the ICCs/IPs for a period of 15 years if the profits from allocation and utilization of the natural resources found
land was transferred to a non-member of the ICCs/IPs. therein; the right to negotiate the terms and conditions for the
Following the constitutional mandate that "customary law govern exploration of natural resources in the areas for the purpose of
property rights or relations in determining the ownership and extent of ensuring ecological, environmental protection and the conservation
ancestral domains,"[if !supportFootnotes][216][endif] the IPRA, by legislative fiat, measures, pursuant to national and customary laws; the right to an
introduces a new concept of ownership. This is a concept that has informed and intelligent participation in the formulation and implementation
long existed under customary law.[if !supportFootnotes][217][endif] of any project, government or private, that will affect or impact upon the
Custom, from which customary law is derived, is also recognized ancestral domains and to receive just and fair compensation for any damages
under the Civil Code as a source of law. [if !supportFootnotes][218][endif] Some articles which they may sustain as a result of the project; and the right to effective
of the Civil Code expressly provide that custom should be applied in cases measures by the government to prevent any interference with, alienation and
where no codal provision is applicable. [if !supportFootnotes][219][endif] In other words, in encroachment upon these rights;"
the absence of any applicable provision in the Civil Code, custom, when duly c) Right to Stay in the Territories.-- The right to stay in the territory and not to
proven, can define rights and liabilities.[if !supportFootnotes][220][endif] be removed therefrom. No ICCs/IPs will be relocated without their free and
Customary law is a primary, not secondary, source of rights under the IPRA prior informed consent, nor through any means other than eminent domain. x
and uniquely applies to ICCs/IPs. Its recognition does not depend on the x x;
absence of a specific provision in the civil law. The indigenous concept d) Right in Case of Displacement.-- In case displacement occurs as a result of
of ownership under customary law is specifically acknowledged and natural catastrophes, the State shall endeavor to resettle the displaced
recognized, and coexists with the civil law concept and the laws on land titling ICCs/IPs in suitable areas where they can have temporary life support
and land registration.[if !supportFootnotes][221][endif] systems: x x x;
To be sure, the indigenous concept of ownership exists even e) Right to Regulate the Entry of Migrants.-- Right to regulate the entry of
without a paper title. The CADT is merely a "formal recognition" of native migrant settlers and organizations into their domains;
title. This is clear from Section 11 of the IPRA, to wit: f) Right to Safe and Clean Air and Water.--For this purpose, the ICCs/IPs shall
"Sec. 11. Recognition of Ancestral Domain Rights.-- The rights of ICCs/IPs to have access to integrated systems for the management of their inland waters
their ancestral domains by virtue of Native Title shall be recognized and and air space;
respected. Formal recognition, when solicited by ICCs/IPs concerned shall be g) Right to Claim Parts of Reservations.-- The right to claim parts of the
embodied in a Certificate of Ancestral Domain Title, which shall recognize the ancestral domains which have been reserved for various purposes, except
title of the concerned ICCs/IPs over the territories identified and delineated." those reserved and intended for common and public welfare and service;
The moral import of ancestral domain, native land or being native is h) Right to Resolve Conflict.-- Right to resolve land conflicts in accordance with
"belongingness" to the land, being people of the land-- by sheer force of customary laws of the area where the land is located, and only in default
having sprung from the land since time beyond recall, and the faithful nurture thereof shall the complaints be submitted to amicable settlement and to the
of the land by the sweat of one's brow. This is fidelity of usufructuary relation Courts of Justice whenever necessary."
to the land-- the possession of stewardship through perduring, intimate tillage, Section 8 provides for the rights over ancestral lands:
and the mutuality of blessings between man and land; from man, care for "Sec. 8. Rights to Ancestral Lands.-- The right of ownership and possession of
land; from the land, sustenance for man.[if !supportFootnotes][222][endif] the ICCs/IPs to their ancestral lands shall be recognized and protected.
C. Sections 7 (a), 7 (b) and 57 of the IPRA Do Not Violate the Regalian a) Right to transfer land/property.-- Such right shall include the right to
Doctrine Enshrined in Section 2, Article XII of the 1987 Constitution. transfer land or property rights to/among members of the same ICCs/IPs,
1. The Rights of ICCs/IPs Over Their Ancestral Domains and Lands subject to customary laws and traditions of the community concerned.
The IPRA grants the ICCs/IPs several rights over their ancestral domains b) Right to Redemption.-- In cases where it is shown that the transfer of
and ancestral lands. Section 7 provides for the rights over ancestral domains: land/property rights by virtue of any agreement or devise, to a non-member of
"Sec. 7. Rights to Ancestral Domains.-- The rights of ownership and possession the concerned ICCs/IPs is tainted by the vitiated consent of the ICCs/IPs, or is
of ICCs/IPs to their ancestral domains shall be recognized and protected. Such transferred for an unconscionable consideration or price, the transferor ICC/IP
rights include: shall have the right to redeem the same within a period not exceeding fifteen
a) Right of Ownership.- The right to claim ownership over lands, bodies of (15) years from the date of transfer."
water traditionally and actually occupied by ICCs/IPs, sacred places, Section 7 (a) defines the ICCs/IPs the right of ownership over their
traditional hunting and fishing grounds, and all improvements made ancestral domains which covers (a) lands, (b) bodies of water traditionally
by them at any time within the domains; and actually occupied by the ICCs/IPs, (c) sacred places, (d) traditional hunting

20
and fishing grounds, and (e) all improvements made by them at any time supportFootnotes][223][endif]

within the domains. The right of ownership includes the following rights: All lands of the public domain and all natural resources-- waters,
(1) the right to develop lands and natural resources; (b) the right to stay in the minerals, coal, petroleum, and other mineral oils, all forces of potential
territories; (c) the right to resettlement in case of displacement; (d) the right energy, fisheries, forests or timber, wildlife, flora and fauna, and other natural
to regulate the entry of migrants; (e) the right to safe and clean air and water; resources-- are owned by the State. The Constitution provides that in the
(f) the right to claim parts of the ancestral domains as reservations; and (g) exploration, development and utilization of these natural resources, the State
the right to resolve conflict in accordance with customary laws. exercises full control and supervision, and may undertake the same in four (4)
Section 8 governs their rights to ancestral lands. Unlike ownership over modes:
the ancestral domains, Section 8 gives the ICCs/IPs also the right to transfer 1. The State may directly undertake such activities; or
the land or property rights to members of the same ICCs/IPs or non-members 2. The State may enter into co-production, joint venture or production-
thereof. This is in keeping with the option given to ICCs/IPs to secure a torrens sharing agreements with Filipino citizens or qualified
title over the ancestral lands, but not to domains. corporations;
2. The Right of ICCs/IPs to Develop Lands and Natural Resources Within 3. Congress may, by law, allow small-scale utilization of natural
the Ancestral Domains Does Not Deprive the State of Ownership Over the resources by Filipino citizens;
Natural Resources and Control and Supervision in their Development and 4. For the large-scale exploration, development and utilization of
Exploitation. minerals, petroleum and other mineral oils, the President
The Regalian doctrine on the ownership, management and utilization of may enter into agreements with foreign-owned corporations
natural resources is declared in Section 2, Article XII of the 1987 involving technical or financial assistance.
Constitution, viz: As owner of the natural resources, the State is accorded primary
"Sec. 2. All lands of the public domain, waters, minerals, coal, power and responsibility in the exploration, development and
petroleum, and other mineral oils, all forces of potential energy, utilization of these natural resources. The State may directly undertake
fisheries, forests or timber, wildlife, flora and fauna, and other the exploitation and development by itself, or, it may allow participation by
natural resources are owned by the State. With the exception of the private sector through co-production,[if !supportFootnotes][224][endif] joint venture,[if !
agricultural lands, all other natural resources shall not be alienated. The supportFootnotes][225][endif]
or production-sharing agreements. [if !supportFootnotes][226][endif] These
exploration, development, and utilization of natural resources shall agreements may be for a period of 25 years, renewable for another 25 years.
be under the full control and supervision of the State. The State may The State, through Congress, may allow the small-scale utilization of natural
directly undertake such activities, or, it may enter into co-production, resources by Filipino citizens. For the large-scale exploration of these
joint venture, or production-sharing agreements with Filipino resources, specifically minerals, petroleum and other mineral oils, the State,
citizens, or corporations or associations at least sixty per centum of through the President, may enter into technical and financial assistance
whose capital is owned by such citizens. Such agreements may be for a agreements with foreign-owned corporations.
period not exceeding twenty-five years, renewable for not more than twenty- Under the Philippine Mining Act of 1995, (R.A. 7942) and the People's
five years, and under such terms and conditions as may be provided by law. In Small-Scale Mining Act of 1991 (R.A. 7076) the three types of agreements, i.e.,
cases of water rights for irrigation, water supply, fisheries, water supply, co-production, joint venture or production-sharing, may apply to both large-
fisheries, or industrial uses other than the development of water power, scale[if !supportFootnotes][227][endif] and small-scale mining.[if !supportFootnotes][228][endif] "Small-
beneficial use may be the measure and limit of the grant. scale mining" refers to "mining activities which rely heavily on manual labor
The State shall protect the nation's marine wealth in its archipelagic waters, using simple implements and methods and do not use explosives or heavy
territorial sea, and exclusive economic zone, and reserve its use and mining equipment."[if !supportFootnotes][229][endif]
enjoyment exclusively to Filipino citizens. Examining the IPRA, there is nothing in the law that grants to the
The Congress may, by law, allow small-scale utilization of natural ICCs/IPs ownership over the natural resources within their ancestral
resources by Filipino citizens, as well as cooperative fish farming, with domains. The right of ICCs/IPs in their ancestral domains includes
priority to subsistence fishermen and fishworkers in rivers, lakes, bays, and ownership, but this "ownership" is expressly defined and limited in
lagoons. Section 7 (a) as:
The President may enter into agreements with foreign-owned corporations "Sec. 7. a) Right of ownership-- The right to claim ownership over lands,
involving either technical or financial assistance for large-scale exploration, bodies of water traditionally and actually occupied by ICCs/IPs, sacred places,
development, and utilization of minerals, petroleum, and other traditional hunting and fishing grounds, and all improvements made by them
mineral oils according to the general terms and conditions provided by law, at any time within the domains;"
based on real contributions to the economic growth and general welfare of the The ICCs/IPs are given the right to claim ownership over "lands, bodies of
country. In such agreements, the state shall promote the development and water traditionally and actually occupied by ICCs/IPs, sacred places, traditional
use of local scientific and technical resources. hunting and fishing grounds, and all improvements made by them at any time
The President shall notify the Congress of every contract entered into in within the domains." It will be noted that this enumeration does not mention
accordance with this provision, within thirty days from its execution." [if ! bodies of water not occupied by the ICCs/IPs, minerals, coal, wildlife, flora and

21
fauna in the traditional hunting grounds, fish in the traditional fishing grounds, and share the profits from allocation and utilization of the natural resources
forests or timber in the sacred places, etc. and all other natural resources found therein; the right to negotiate the terms and conditions for the
found within the ancestral domains. Indeed, the right of ownership under exploration of natural resources in the areas for the purpose of ensuring
Section 7 (a) does not cover "waters, minerals, coal, petroleum and ecological, environmental protection and the conservation measures, pursuant
other mineral oils, all forces of potential energy, fisheries, forests or to national and customary laws; the right to an informed and intelligent
timber, wildlife, flora and fauna and all other natural resources" participation in the formulation and implementation of any project,
enumerated in Section 2, Article XII of the 1987 Constitution as government or private, that will affect or impact upon the ancestral domains
belonging to the State. and to receive just and fair compensation for any damages which they may
The non-inclusion of ownership by the ICCs/IPs over the natural sustain as a result of the project; and the right to effective measures by the
resources in Section 7(a) complies with the Regalian doctrine. government to prevent any interference with, alienation and encroachment
(a) Section 1, Part II, Rule III of the Implementing Rules Goes Beyond upon these rights;"
the Parameters of Sec. 7 (a) of the IPRA And is Unconstitutional. The right to develop lands and natural resources under Section 7 (b) of
The Rules Implementing the IPRA [if !supportFootnotes][230][endif] in Section 1, Part the IPRA enumerates the following rights:
II, Rule III reads: a) the right to develop, control and use lands and territories traditionally
"Section 1. Rights of Ownership. ICCs/IPs have rights of ownership over lands, occupied;
waters, and natural resources and all improvements made by them at any b) the right to manage and conserve natural resources within the territories
time within the ancestral domains/ lands. These rights shall include, but not and uphold the responsibilities for future generations;
limited to, the right over the fruits, the right to possess, the right to use, right c) the right to benefit and share the profits from the allocation and utilization
to consume, right to exclude and right to recover ownership, and the rights or of the natural resources found therein;
interests over land and natural resources. The right to recover shall be d) the right to negotiate the terms and conditions for the exploration of
particularly applied to lands lost through fraud or any form or vitiated consent natural resources for the purpose of ensuring ecological, environmental
or transferred for an unconscionable price." protection and the conservation measures, pursuant to national and
Section 1 of the Implementing Rules gives the ICCs/IPs rights of ownership customary laws;
over "lands, waters and natural resources." The term "natural resources" is e) the right to an informed and intelligent participation in the formulation and
not one of those expressly mentioned in Section 7 (a) of the law. Our implementation of any project, government or private, that will affect or
Constitution and jurisprudence clearly declare that the right to claim impact upon the ancestral domains and to receive just and fair compensation
ownership over land does not necessarily include the right to claim ownership for any damages which they may sustain as a result of the project;
over the natural resources found on or under the land. [if !supportFootnotes][231][endif] The f) the right to effective measures by the government to prevent any
IPRA itself makes a distinction between land and natural resources. interference with, alienation and encroachment upon these rights. [if !supportFootnotes]
Section 7 (a) speaks of the right of ownership only over the land [233][endif]

within the ancestral domain. It is Sections 7 (b) and 57 of the law Ownership over the natural resources in the ancestral domains
that speak of natural resources, and these provisions, as shall be remains with the State and the ICCs/IPs are merely granted the right
discussed later, do not give the ICCs/IPs the right of ownership over to "manage and conserve" them for future generations, "benefit and
these resources. share" the profits from their allocation and utilization, and "negotiate
The constitutionality of Section 1, Part II, Rule III of the Implementing the terms and conditions for their exploration" for the purpose of
Rules was not specifically and categorically challenged by petitioners. "ensuring ecological and environmental protection and conservation
Petitioners actually assail the constitutionality of the Implementing Rules in measures." It must be noted that the right to negotiate the terms and
general.[if !supportFootnotes][232][endif] Nevertheless, to avoid any confusion in the conditions over the natural resources covers only their exploration which must
implementation of the law, it is necessary to declare that the inclusion of be for the purpose of ensuring ecological and environmental protection of, and
"natural resources" in Section 1, Part II, Rule III of the Implementing Rules conservation measures in the ancestral domain. It does not extend to the
goes beyond the parameters of Section 7 (b) of the law and is contrary to exploitation and development of natural resources.
Section 2, Article XII of the 1987 Constitution. Simply stated, the ICCs/IPs' rights over the natural resources take the
(b) The Small-Scale Utilization of Natural Resources In Sec. 7 (b) of form of management or stewardship. For the ICCs/IPs may use these
the IPRA Is Allowed Under Paragraph 3, Section 2 of Article XII of the resources and share in the profits of their utilization or negotiate the terms for
Constitution. their exploration. At the same time, however, the ICCs/IPs must ensure that
Ownership over natural resources remain with the State and the IPRA in the natural resources within their ancestral domains are conserved for future
Section 7 (b) merely grants the ICCs/IPs the right to manage them, viz: generations and that the "utilization" of these resources must not harm the
"Sec. 7 (b) Right to Develop Lands and Natural Resources.-- Subject to Section ecology and environment pursuant to national and customary laws. [if !
56 hereof, right to develop, control and use lands and territories traditionally supportFootnotes][234][endif]

occupied, owned, or used; to manage and conserve natural resources within The limited rights of "management and use" in Section 7 (b) must be
the territories and uphold the responsibilities for future generations; to benefit taken to contemplate small-scale utilization of natural resources as

22
distinguished from large-scale. Small-scale utilization of natural Section 57 of IPRA, the State, as owner of these natural resources,
resources is expressly allowed in the third paragraph of Section 2, may directly undertake the development and exploitation of the
Article XII of the Constitution "in recognition of the plight of forest natural resources by itself, or in the alternative, it may recognize the
dwellers, gold panners, marginal fishermen and others similarly situated who priority rights of the ICCs/IPs as owners of the land on which the
exploit our natural resources for their daily sustenance and survival." [if ! natural resources are found by entering into a co-production, joint
supportFootnotes][235][endif]
Section 7 (b) also expressly mandates the ICCs/IPs to venture, or production-sharing agreement with them. The State may
manage and conserve these resources and ensure environmental and likewise enter into any of said agreements with a non-member of the
ecological protection within the domains, which duties, by their very nature, ICCs/IPs, whether natural or juridical, or enter into agreements with
necessarily reject utilization in a large-scale. foreign-owned corporations involving either technical or financial
(c) The Large-Scale Utilization of Natural Resources In Section 57 of assistance for the large-scale exploration, development and
the IPRA Is Allowed Under Paragraphs 1 and 4, Section 2, Article XII utilization of minerals, petroleum, and other mineral oils, or allow
of the 1987 Constitution. such non-member to participate in its agreement with the ICCs/IPs. If
Section 57 of the IPRA provides: the State decides to enter into an agreement with a non-ICC/IP member, the
"Sec. 57. Natural Resources within Ancestral Domains.-- The ICCs/IPs shall National Commission on Indigenous Peoples (NCIP) shall ensure that the rights
have priority rights in the harvesting, extraction, development or of the ICCs/IPs under the agreement shall be protected. The agreement shall
exploitation of any natural resources within the ancestral domains. A be for a period of 25 years, renewable for another 25 years.
non-member of the ICCs/IPs concerned may be allowed to take part in the To reiterate, in the large-scale utilization of natural resources within the
development and utilization of the natural resources for a period of not ancestral domains, the State, as owner of these resources, has four (4)
exceeding twenty-five (25) years renewable for not more than twenty-five (25) options: (1) it may, of and by itself, directly undertake the development and
years: Provided, That a formal and written agreement is entered into with the exploitation of the natural resources; or (2) it may recognize the priority rights
ICCs/IPs concerned or that the community, pursuant to its own decision- of the ICCs/IPs by entering into an agreement with them for such development
making process, has agreed to allow such operation: Provided finally, That the and exploitation; or (3) it may enter into an agreement with a non-member of
NCIP may exercise visitorial powers and take appropriate action to safeguard the ICCs/IPs, whether natural or juridical, local or foreign; or (4) it may allow
the rights of the ICCs/IPs under the same contract." such non-member to participate in the agreement with the ICCs/IPs.
Section 57 speaks of the "harvesting, extraction, development or The rights granted by the IPRA to the ICCs/IPs over the natural
exploitation of natural resources within ancestral domains" and "gives the resources in their ancestral domains merely gives the ICCs/IPs, as
ICCs/IPs 'priority rights' therein." The terms "harvesting, extraction, owners and occupants of the land on which the resources are found,
development or exploitation" of any natural resources within the the right to the small-scale utilization of these resources, and at the
ancestral domains obviously refer to large-scale utilization. It is same time, a priority in their large-scale development and
utilization not merely for subsistence but for commercial or other extensive exploitation. Section 57 does not mandate the State to automatically
use that require technology other than manual labor. [if !supportFootnotes][236][endif] The give priority to the ICCs/IPs. The State has several options and it is
law recognizes the probability of requiring a non-member of the ICCs/IPs to within its discretion to choose which option to pursue. Moreover, there
participate in the development and utilization of the natural resources and is nothing in the law that gives the ICCs/IPs the right to solely undertake the
thereby allows such participation for a period of not more than 25 years, large-scale development of the natural resources within their domains. The
renewable for another 25 years. This may be done on condition that a formal ICCs/IPs must undertake such endeavour always under State supervision or
written agreement be entered into by the non-member and members of the control. This indicates that the State does not lose control and ownership over
ICCs/IPs. the resources even in their exploitation. Sections 7 (b) and 57 of the law
Section 57 of the IPRA does not give the ICCs/IPs the right to "manage simply give due respect to the ICCs/IPs who, as actual occupants of the land
and conserve" the natural resources. Instead, the law only grants the ICCs/IPs where the natural resources lie, have traditionally utilized these resources for
"priority rights" in the development or exploitation thereof. Priority means their subsistence and survival.
giving preference. Having priority rights over the natural resources does not Neither is the State stripped of ownership and control of the natural
necessarily mean ownership rights. The grant of priority rights implies that resources by the following provision:
there is a superior entity that owns these resources and this entity has the "Section 59. Certification Precondition.-- All departments and other
power to grant preferential rights over the resources to whosoever itself governmental agencies shall henceforth be strictly enjoined from issuing,
chooses. renewing or granting any concession, license or lease, or entering into any
Section 57 is not a repudiation of the Regalian doctrine. Rather, it is an production-sharing agreement. without prior certification from the NCIP that
affirmation of the said doctrine that all natural resources found within the the area affected does not overlap with any ancestral domain. Such
ancestral domains belong to the State. It incorporates by implication the certification shall only be issued after a field-based investigation is conducted
Regalian doctrine, hence, requires that the provision be read in the light of by the Ancestral Domains Office of the area concerned: Provided, That no
Section 2, Article XII of the 1987 Constitution. Interpreting Section 2, certification shall be issued by the NCIP without the free and prior informed
Article XII of the 1987 Constitution [if !supportFootnotes][237][endif] in relation to and written consent of the ICCs/IPs concerned: Provided, further, That no

23
department, government agency or government-owned or -controlled agenda.[if !supportFootnotes][242][endif]
corporation may issue new concession, license, lease, or production sharing International institutions and bodies have realized the necessity of
agreement while there is a pending application for a CADT: Provided, finally, applying policies, programs and specific rules concerning IPs in some nations.
That the ICCs/IPs shall have the right to stop or suspend, in accordance with The World Bank, for example, first adopted a policy on IPs as a result of the
this Act, any project that has not satisfied the requirement of this consultation dismal experience of projects in Latin America. [if !supportFootnotes][243][endif] The World
process." Bank now seeks to apply its current policy on IPs to some of its projects in
Concessions, licenses, lease or production-sharing agreements for the Asia. This policy has provided an influential model for the projects of the Asian
exploitation of natural resources shall not be issued, renewed or granted by all Development Bank.[if !supportFootnotes][244][endif]
departments and government agencies without prior certification from the The 1987 Philippine Constitution formally recognizes the existence of
NCIP that the area subject of the agreement does not overlap with any ICCs/IPs and declares as a State policy the promotion of their rights within the
ancestral domain. The NCIP certification shall be issued only after a field- framework of national unity and development. [if !supportFootnotes][245][endif] The IPRA
based investigation shall have been conducted and the free and prior amalgamates the Philippine category of ICCs with the international category of
informed written consent of the ICCs/IPs obtained. Non-compliance with the IPs,[if !supportFootnotes][246][endif] and is heavily influenced by both the International
consultation requirement gives the ICCs/IPs the right to stop or suspend any Labor Organization (ILO) Convention 169 and the United Nations (UN) Draft
project granted by any department or government agency. Declaration on the Rights of Indigenous Peoples.[if !supportFootnotes][247][endif]
As its subtitle suggests, this provision requires as a precondition for the ILO Convention No. 169 is entitled the "Convention Concerning
issuance of any concession, license or agreement over natural resources, that Indigenous and Tribal Peoples in Independent Countries" [if !supportFootnotes][248][endif]
a certification be issued by the NCIP that the area subject of the agreement and was adopted on June 27, 1989. It is based on the Universal Declaration of
does not lie within any ancestral domain. The provision does not vest the NCIP Human Rights, the International Covenant on Economic, Social and Cultural
with power over the other agencies of the State as to determine whether to Rights, the International Covenant on Civil and Political Rights, and many
grant or deny any concession or license or agreement. It merely gives the other international instruments on the prevention of discrimination. [if !
NCIP the authority to ensure that the ICCs/IPs have been informed of the supportFootnotes][249][endif]
ILO Convention No. 169 revised the "Convention Concerning
agreement and that their consent thereto has been obtained. Note that the the Protection and Integration of Indigenous and Other Tribal and Semi-Tribal
certification applies to agreements over natural resources that do not Populations in Independent Countries" (ILO No. 107) passed on June 26, 1957.
necessarily lie within the ancestral domains. For those that are found within Developments in international law made it appropriate to adopt new
the said domains, Sections 7(b) and 57 of the IPRA apply. international standards on indigenous peoples "with a view to removing the
V. THE IPRA IS A RECOGNITION OF OUR ACTIVE PARTICIPATION IN THE assimilationist orientation of the earlier standards," and recognizing the
INDIGENOUS INTERNATIONAL MOVEMENT. aspirations of these peoples to exercise control over their own institutions,
The indigenous movement can be seen as the heir to a history of anti- ways of life and economic development."[if !supportFootnotes][250][endif]
imperialism stretching back to prehistoric times. The movement received a CONCLUSION
massive impetus during the 1960's from two sources. First, the decolonization The struggle of the Filipinos throughout colonial history had been
of Asia and Africa brought into the limelight the possibility of peoples plagued by ethnic and religious differences. These differences were carried
controlling their own destinies. Second, the right of self-determination was over and magnified by the Philippine government through the imposition of a
enshrined in the UN Declaration on Human Rights. [if !supportFootnotes][238][endif] The rise national legal order that is mostly foreign in origin or derivation. [if !supportFootnotes]
of the civil rights movement and anti-racism brought to the attention of North [251][endif]
Largely unpopulist, the present legal system has resulted in the
American Indians, Aborigines in Australia, and Maori in New Zealand the alienation of a large sector of society, specifically, the indigenous peoples. The
possibility of fighting for fundamental rights and freedoms. histories and cultures of the indigenes are relevant to the evolution of
In 1974 and 1975, international indigenous organizations were founded, Philippine culture and are vital to the understanding of contemporary
[if !supportFootnotes][239][endif]
and during the 1980's, indigenous affairs were on the problems.[if !supportFootnotes][252][endif] It is through the IPRA that an attempt was made
international agenda. The people of the Philippine Cordillera were the first by our legislators to understand Filipino society not in terms of myths and
Asians to take part in the international indigenous movement. It was the biases but through common experiences in the course of history. The
Cordillera People's Alliance that carried out successful campaigns against the Philippines became a democracy a centennial ago and the decolonization
building of the Chico River Dam in 1981-82 and they have since become one process still continues. If the evolution of the Filipino people into a democratic
of the best-organized indigenous bodies in the world. [if !supportFootnotes][240][endif] society is to truly proceed democratically, i.e., if the Filipinos as a whole are to
Presently, there is a growing concern for indigenous rights in the participate fully in the task of continuing democratization, [if !supportFootnotes][253][endif]
international scene. This came as a result of the increased publicity focused it is this Court's duty to acknowledge the presence of indigenous and
on the continuing disrespect for indigenous human rights and the destruction customary laws in the country and affirm their co-existence with the land laws
of the indigenous peoples' environment, together with the national in our national legal system.
governments' inability to deal with the situation. [if !supportFootnotes][241][endif] With the foregoing disquisitions, I vote to uphold the constitutionality of
Indigenous rights came as a result of both human rights and environmental the Indigenous Peoples Rights Act of 1997.
protection, and have become a part of today's priorities for the international

24
SEPARATE OPINION has been guided by a working definition in the 1986 Report of UN Special
KAPUNAN, J.: Rapporteur Martinez Cobo:[if !supportFootnotes][5][endif]
You ask if we own the land. . . How can you own that which will outlive you? Indigenous communities, peoples and nations are those which, having a
Only the race own the land because only the race lives forever. To claim a historical continuity with pre-invasion and pre-colonial societies that
piece of land is a birthright of every man. The lowly animals claim their place; developed on their territories, consider themselves distinct from other
how much more man? Man is born to live. Apu Kabunian, lord of us all, gave us sections of the societies now prevailing in those territories, or parts of them.
life and placed us in the world to live human lives. And where shall we obtain They form at present non-dominant sections of society and are determined to
life? From the land. To work (the land) is an obligation, not merely a right. In preserve, develop and transmit to future generations their ancestral
tilling the land, you possess it. And so land is a grace that must be nurtured. territories, and their ethnic identity, as the basis of their continued existence
To enrich it and make it fructify is the eternal exhortation of Apu Kabunian to as peoples, in accordance with their own cultural patterns, social institutions
all his children. Land is sacred. Land is beloved. From its womb springs life. and legal systems.
- Macli-ing Dulag, Chieftain of the Kalinga Tribe (quoted in Ponciano L. This historical continuity may consist of the continuation, for an extended
Bennagen, "Tribal Filipinos" in Indigenous View of Land and period reaching into the present, of one or more of the following factors:
the Environment, ed. Shelton H. Davis, the World Bank (a) Occupation of ancestral lands, or at least of part of them;
Discussion Papers, No. 188, pp. 71-72.) (b) Common ancestry with the original occupants of these lands;
It is established doctrine that a statute should be construed whenever (c) Culture in general, or in specific manifestations (such as religion,
possible in harmony with, rather than in violation of, the Constitution. [if ! living under a tribal system, membership of an indigenous
supportFootnotes][1][endif]
The presumption is that the legislature intended to enact a community, dress, means of livelihood, life-style, etc.);
valid, sensible and just law and one which operates no further than may be (d) Language (whether used as the only language, as mother-tongue, as
necessary to effectuate the specific purpose of the law. [if !supportFootnotes][2][endif] the habitual means of communication at home or in the
The challenged provisions of the Indigenous Peoples Rights Act (IPRA) family, or as the main, preferred, habitual, general or normal
must be construed in view of such presumption of constitutionality. Further, language);
the interpretation of these provisions should take into account the purpose of (e) Residence in certain parts of the country; or in certain regions of the
the law, which is to give life to the constitutional mandate that the rights of world;
the indigenous peoples be recognized and protected. (f) Other relevant facts.[if !supportFootnotes][6][endif]
The struggle of our indigenous peoples to reclaim their ancestral lands In Philippine constitutional law, the term indigenous peoples pertains to
and domains and therefore, their heritage, is not unique. It is one that they those groups of Filipinos who have retained a high degree of continuity from
share with the red-skinned Indians of the United States, with the aborigines of pre-Conquest culture.[if !supportFootnotes][7][endif] Philippine legal history, however, has
Australia, the Maori of New Zealand and the Sazmi of Sweden, to name a few. not been kind to the indigenous peoples, characterized them as uncivilized, [if !
Happily, the nations in which these indigenous peoples live all have enacted supportFootnotes][8][endif]
backward people,[if !supportFootnotes][9][endif] with barbarous practices[if
measures in an attempt to heal an oppressive past by the promise of a !supportFootnotes][10][endif]
and a low order of intelligence.[if !supportFootnotes][11][endif]
progressive future. Thus has the international community realized the Drawing inspiration from both our fundamental law and international
injustices that have been perpetrated upon the indigenous peoples. This law, IPRA now employs the politically-correct conjunctive term indigenous
sentiment among the family of nations is expressed in a number of peoples/indigenous cultural communities as follows:
documents, the most recent and most comprehensive of which is the Draft Sec. 3. Definition of Terms.-- For purposes of this Act, the following terms shall
United Nations Declaration on the Rights of Indigenous Peoples which was mean:
adopted by the UN Sub-Commission on Prevention of Discrimination and xxx
Protection of Minorities by its resolution on August 26, 1994. Among the rights (h) Indigenous peoples/Indigenous cultural communities. - refer to a group of
recognized by the UN Draft is the restitution of lands, territories and even the people or homogenous societies identified by self-ascription and ascription by
resources which the indigenous peoples have traditionally owned or otherwise others, who have continuously lived as organized community on communally
occupied or used, and which have been confiscated, occupied, used or bounded and defined territory, and who have, under claims of ownership since
damaged without the free and informed consent of the indigenous peoples. time immemorial, occupied, possessed and utilized such territories, sharing
A Historical Backdrop on the Indigenous Peoples common bonds of language, customs, traditions, and other distinctive cultural
The term indigenous traces its origin to the Old Latin word indu, traits, or who have, through resistance to political, social and cultural inroads
meaning within. In the sense the term has come to be used, it is nearer in of colonization, non-indigenous religions and cultures, became historically
meaning to the Latin word indigenus, which means native.[if !supportFootnotes][3][endif] differentiated from the majority of Filipinos. Indigenous peoples shall likewise
Indigenous refers to that which originated or has been produced naturally in a include peoples who are regarded as indigenous on account of their descent
particular land, and has not been introduced from the outside. [if !supportFootnotes][4] from the populations which inhabited the country at the time of conquest or
[endif]
In international law, the definition of what constitutes indigenous peoples colonization, or at the time of inroads of non-indigenous religions and cultures,
attains some degree of controversy. No definition of the term indigenous or the establishment of present State boundaries, who retain some or all of
peoples has been adopted by the United Nations (UN), although UN practice their own social, economic, cultural and political institutions, but who may

25
have been displaced from their traditional domains or who may have resettled inequities by equitably diffusing wealth and political power for the common
outside their ancestral domains x x x. good.
Long before the Spaniards set foot in these islands, the indigenous To this end, the State shall regulate the acquisition, ownership, use and
peoples were already plowing our soil and hunting in our forests. The Filipinos disposition of property and its increments.[if !supportFootnotes][19][endif]
of Aeta and Malay stock, who were the original inhabitants of our archipelago, Sec. 6. The State shall apply the principles of agrarian reform or stewardship,
were, at that time, practicing a native culture. From the time the Spaniards whenever applicable in accordance with law, in the disposition and utilization
arrived up to the early part of the American regime,[if !supportFootnotes][12][endif] these of other natural resources, including lands of the public domain under lease or
native inhabitants resisted foreign invasion, relentlessly fighting for their concession, subject to prior rights, homestead rights of small settlers, and the
lands. Today, from the remote uplands of Northern Luzon, to Palawan, Mindoro rights of indigenous communities to their ancestral lands. [if !supportFootnotes][20][endif]
and Mindanao, the indigenous peoples continue to live on and cultivate their Sec. 17. The State shall recognize, respect, and protect the rights of
ancestral lands, the lands of their forefathers. indigenous cultural communities to preserve and develop their cultures,
Though Filipinos today are essentially of the same stock as the traditions, and institutions. It shall consider these rights in the formulation of
indigenous peoples, our national culture exhibits only the last vestiges of this national plans and policies.[if !supportFootnotes][21][endif]
native culture. Centuries of colonial rule and neocolonial domination have Sec. 12. The Congress may create a consultative body to advise the President
created a discernible distinction between the cultural majority and the group on policies affecting indigenous cultural communities, the majority of the
of cultural minorities.[if !supportFootnotes][13][endif] The extant Philippine national culture members of which shall come from such communities. [if !supportFootnotes][22][endif]
is the culture of the majority; its indigenous roots were replaced by foreign IPRA was enacted precisely to implement the foregoing constitutional
cultural elements that are decidedly pronounced, if not dominant. [if !supportFootnotes] provisions. It provides, among others, that the State shall recognize and
[14][endif]
While the culture of the majority reoriented itself to Western influence, promote the rights of indigenous peoples within the framework of national
the culture of the minorities has retained its essentially native character. unity and development, protect their rights over the ancestral lands and
One of every six Filipinos is a member of an indigenous cultural ancestral domains and recognize the applicability of customary laws
community. Around twelve million Filipinos are members of the one hundred governing property rights or relations in determining the ownership and extent
and ten or so indigenous cultural communities, [if !supportFootnotes][15][endif] accounting of the ancestral domains.[if !supportFootnotes][23][endif] Moreover, IPRA enumerates the
for more than seventeen per centum of the estimated seventy million civil and political rights of the indigenous peoples; [if !supportFootnotes][24][endif] spells
Filipinos[if !supportFootnotes][16][endif] in our country. Sadly, the indigenous peoples are out their social and cultural rights; [if !supportFootnotes][25][endif] acknowledges a general
one of the poorest sectors of Philippine society. The incidence of poverty and concept of indigenous property right and recognizes title thereto; [if !supportFootnotes]
malnutrition among them is significantly higher than the national average. [26][endif]
and creates the NCIP as an independent agency under the Office of the
The indigenous peoples are also among the most powerless. Perhaps because President.[if !supportFootnotes][27][endif]
of their inability to speak the language of law and power, they have been Preliminary Issues
relegated to the fringes of society. They have little, if any, voice in national A. The petition presents an actual controversy.
politics and enjoy the least protection from economic exploitation. The time-tested standards for the exercise of judicial review are: (1) the
The Constitutional Policies on Indigenous Peoples existence of an appropriate case; (2) an interest personal and substantial by
The framers of the 1987 Constitution, looking back to the long the party raising the constitutional question; (3) the plea that the function be
destitution of our less fortunate brothers, fittingly saw the historic opportunity exercised at the earliest opportunity; and (4) the necessity that the
to actualize the ideals of people empowerment and social justice, and to reach constitutional question be passed upon in order to decide the case. [if !
out particularly to the marginalized sectors of society, including the supportFootnotes][28][endif]

indigenous peoples. They incorporated in the fundamental law several Courts can only decide actual controversies, not hypothetical questions
provisions recognizing and protecting the rights and interests of the or cases.[if !supportFootnotes][29][endif] The threshold issue, therefore, is whether an
indigenous peoples, to wit: appropriate case exists for the exercise of judicial review in the present case.
Sec. 22. The State recognizes and promotes the rights of indigenous peoples An actual case or controversy means an existing case or controversy
within the framework of national unity and development. [if !supportFootnotes][17][endif] which is both ripe for resolution and susceptible of judicial determination, and
Sec. 5. The State, subject to the provisions of this Constitution and national that which is not conjectural or anticipatory, [if !supportFootnotes][30][endif] or that which
development policies and programs, shall protect the rights of indigenous seeks to resolve hypothetical or feigned constitutional problems. [if !supportFootnotes]
cultural communities to their ancestral lands to ensure their economic, social, [31][endif]
A petition raising a constitutional question does not present an actual
and cultural well-being. controversy, unless it alleges a legal right or power. Moreover, it must show
The Congress may provide for the applicability of customary laws governing that a conflict of rights exists, for inherent in the term controversy is the
property rights and relations in determining the ownership and extent of presence of opposing views or contentions. [if !supportFootnotes][32][endif] Otherwise, the
ancestral domains.[if !supportFootnotes][18][endif] Court will be forced to resolve issues which remain unfocused because they
Sec. 1. The Congress shall give the highest priority to the enactment of lack such concreteness provided when a question emerges precisely framed
measures that protect and enhance the right of all the people to human from a clash of adversary arguments exploring every aspect of a multi-faceted
dignity, reduce social, economic and political inequalities, and remove cultural situation embracing conflicting and demanding interests. [if !supportFootnotes][33][endif]

26
The controversy must also be justiciable; that is, it must be susceptible of same injury. Each elector stands on the same basis with reference to
judicial determination.[if !supportFootnotes][34][endif] maintaining a petition whether or not the relief sought by the relator should
In the case at bar, there exists a live controversy involving a clash of be granted.[if !supportFootnotes][43][endif]
legal rights. A law has been enacted, and the Implementing Rules and In Taada v. Tuvera,[if !supportFootnotes][44][endif] the Court enforced the public
Regulations approved. Money has been appropriated and the government right to due process and to be informed of matters of public concern.
agencies concerned have been directed to implement the statute. It cannot be In Garcia vs. Board of Investments, [if !supportFootnotes][45][endif] the Court upheld
successfully maintained that we should await the adverse consequences of the public right to be heard or consulted on matters of national concern.
the law in order to consider the controversy actual and ripe for judicial In Oposa v. Factoran,[if !supportFootnotes][46][endif] the Court recognized the public
resolution. It is precisely the contention of the petitioners that the law, on its right of citizens to a balanced and healthful ecology which, for the first time in
face, constitutes an unconstitutional abdication of State ownership over lands our nations constitutional history, is solemnly incorporated in the fundamental
of the public domain and other natural resources. Moreover, when the State law.[if !supportFootnotes][47][endif] Mr. Justice (now Chief Justice) Hilario G. Davide, Jr.,
machinery is set into motion to implement an alleged unconstitutional statute, delivering the opinion of the Court, stated that:
this Court possesses sufficient authority to resolve and prevent imminent Such a right belongs to a different category of rights altogether for it
injury and violation of the constitutional process. concerns nothing less than self-preservation and self-perpetuation-aptly and
B. Petitioners, as citizens and taxpayers, have the requisite standing to raise fittingly stressed by petitioners-the advancement of which may even be said
the constitutional questions herein. to predate all governments and constitutions. As a matter of fact, these
In addition to the existence of an actual case or controversy, a person basic rights need not even be written in the Constitution for they are
who assails the validity of a statute must have a personal and substantial assumed to exist from the inception of humankind.[if !supportFootnotes][48][endif]
interest in the case, such that, he has sustained, or will sustain, a direct injury Petitioners, as citizens, possess the public right to ensure that the
as a result of its enforcement.[if !supportFootnotes][35][endif] Evidently, the rights asserted national patrimony is not alienated and diminished in violation of the
by petitioners as citizens and taxpayers are held in common by all the Constitution. Since the government, as the guardian of the national
citizens, the violation of which may result only in a generalized grievance. [if ! patrimony, holds it for the benefit of all Filipinos without distinction as to
supportFootnotes][36][endif]
Yet, in a sense, all citizens and taxpayers suits are efforts to ethnicity, it follows that a citizen has sufficient interest to maintain a suit to
air generalized grievances about the conduct of government and the ensure that any grant of concessions covering the national economy and
allocation of power.[if !supportFootnotes][37][endif] patrimony strictly complies with constitutional requirements. Thus, the
In several cases, the Court has adopted a liberal attitude with regard to preservation of the integrity and inviolability of the national patrimony is a
standing.[if !supportFootnotes][38][endif] The proper party requirement is considered as proper subject of a citizens suit.
merely procedural,[if !supportFootnotes][39][endif] and the Court has ample discretion with In addition, petitioners, as taxpayers, possess the right to restrain
regard thereto.[if !supportFootnotes][40][endif] As early as 1910, the Court in the case of officials from wasting public funds through the enforcement of an
Severino vs. Governor General [if !supportFootnotes][41][endif] held: unconstitutional statute. It is well-settled that a taxpayer has the right to
x x x [W]hen the relief is sought merely for the protection of private rights, the enjoin public officials from wasting public funds through the implementation of
relator must show some personal or special interest in the subject matter, an unconstitutional statute,[if !supportFootnotes][49][endif] and by necessity, he may assail
since he is regarded as the real party in interest and his right must clearly the validity of a statute appropriating public funds. [if !supportFootnotes][50][endif] The
appear. Upon the other hand, when the question is one of public right taxpayer has paid his taxes and contributed to the public coffers and, thus,
and the object of the mandamus is to procure the enforcement of a public may inquire into the manner by which the proceeds of his taxes are spent. The
duty, the people are regarded as the real party in interest, and the expenditure by an official of the State for the purpose of administering an
relator at whose instigation the proceedings are instituted need not invalid law constitutes a misapplication of such funds. [if !supportFootnotes][51][endif]
show that he has any legal or special interest in the result, it being The IPRA appropriates funds as indicated in its title: "An Act to
sufficient to show that he is a citizen and as such interested in the Recognize, Protect and Promote the Rights of Indigenous Cultural
execution of the laws.[if !supportFootnotes][42][endif] Communities/Indigenous Peoples, Creating the National Commission on
This Court has recognized that a public right, or that which belongs to Indigenous Peoples, Establishing Implementing Mechanisms, Appropriating
the people at large, may also be the subject of an actual case or controversy. Funds Therefor, and for Other Purposes." In the same manner, Section 79
In Severino, we ruled that a private citizen may enforce a public right in behalf authorizes for the expenditure of public funds by providing that the amount
of other citizens. We opined therein that: necessary to finance [its] initial implementation shall be charged against the
[T]he right which [petitioner] seeks to enforce is not greater or different from current year's appropriation for the Office for Northern Cultural Communities
that of any other qualified elector in the municipality of Silay. It is also true (the ONCC) and the Office for Southern Cultural Communities (the OSCC), [if !
that the injury which he would suffer in case he fails to obtain the relief sought supportFootnotes][52][endif]
which were merged as organic offices of the NCIP. [if !
would not be greater or different from that of the other electors; but he is supportFootnotes][53][endif]
Thus, the IPRA is a valid subject of a taxpayers suit.
seeking to enforce a public right as distinguished from a private right. C. The petition for prohibition and mandamus is not an improper remedy.
The real party in interest is the public, or the qualified electors of the Prohibition is an extraordinary writ directed against any tribunal,
town of Silay. Each elector has the same right and would suffer the corporation, board, officer or person, whether exercising judicial, quasi-judicial

27
or ministerial functions, ordering said entity or person to desist from further are special and important reasons therefor, clearly and specifically
proceedings when said proceedings are without or in excess of said entitys or set out in the petition. This is established policy. It is a policy necessary to
persons jurisdiction, or are accompanied with grave abuse of discretion, and prevent inordinate demands upon the Courts time and attention which are
there is no appeal or any other plain, speedy and adequate remedy in the better devoted to those matters within its exclusive jurisdiction, and to
ordinary course of law.[if !supportFootnotes][54][endif] Mandamus, on the other hand, is an prevent further over-crowding of the Courts docket x x x. [if !supportFootnotes][59][endif]
extraordinary writ commanding a tribunal, corporation, board, officer or (Emphasis supplied.)
person, immediately or at some other specified time, to do the act required to IPRA aims to rectify the historical injustice inflicted upon indigenous
be done, when said entity or person unlawfully neglects the performance of an peoples. Its impact upon the lives not only of the indigenous peoples but also
act which the law specifically enjoins as a duty resulting from an office, trust upon the lives of all Filipinos cannot be denied. The resolution of this case by
or station, or when said entity or person unlawfully excludes another from the the Court at the earliest opportunity is necessary if the aims of the law are to
use and enjoyment of a right or office to which such other is entitled, and be achieved. This reason is compelling enough to allow petitioners invocation
there is no other plain, speedy and adequate remedy in the ordinary course of of this Courts jurisdiction in the first instance.
law.[if !supportFootnotes][55][endif] Substantive Issues
In this case, the petitioners pray that respondents be restrained from Primary Issue
implementing the challenged provisions of the IPRA and its Implementing The issue of prime concern raised by petitioners and the Solicitor
Rules and the assailed DENR Circular No. 2, series of 1998, and that the same General revolves around the constitutionality of certain provisions of IPRA,
officials be enjoined from disbursing public funds for the implementation of specifically Sections 3(a), 3(b), 5, 6, 7, 8, 57, 58 and 59. These provisions
the said law and rules. They further ask that the Secretary of the DENR be allegedly violate Section 2, Article XII of the Constitution, which states:
compelled to perform his duty to control and supervise the activities Sec. 2. All lands of the public domain, waters, minerals, coal, petroleum, and
pertaining to natural resources. other mineral oils, all forces of potential energy, fisheries, forests or timber,
Prohibition will lie to restrain the public officials concerned from wildlife, flora and fauna, and other natural resources are owned by the State.
implementing the questioned provisions of the IPRA and from disbursing funds With the exception of agricultural lands, all other natural resources shall not
in connection therewith if the law is found to be unconstitutional. Likewise, be alienated. The exploration, development, and utilization of natural
mandamus will lie to compel the Secretary of the DENR to perform his duty to resources shall be under the full control and supervision of the State. The
control and supervise the exploration, development, utilization and State may directly undertake such activities, or it may enter into co-
conservation of the countrys natural resources. Consequently, the petition for production, joint venture, or production-sharing agreements with Filipino
prohibition and mandamus is not an improper remedy for the relief sought. citizens, or corporations or associations at least sixty per centum of whose
D. Notwithstanding the failure of petitioners to observe the hierarchy of capital is owned by such citizens. Such agreements may be for a period not
courts, the Court assumes jurisdiction over the petition in view of the exceeding twenty-five years, renewable for not more than twenty-five years,
importance of the issues raised therein. and under such terms and conditions as may be provided by law. In cases of
Between two courts of concurrent original jurisdiction, it is the lower water rights for irrigation, water supply, fisheries, or industrial uses other than
court that should initially pass upon the issues of a case. That way, as a the development of water power, beneficial use may be the measure and limit
particular case goes through the hierarchy of courts, it is shorn of all but the of the grant.
important legal issues or those of first impression, which are the proper The State shall protect the nations marine wealth in its archipelagic waters,
subject of attention of the appellate court. This is a procedural rule borne of territorial sea, and exclusive economic zone, and reserve its use and
experience and adopted to improve the administration of justice. enjoyment exclusively to Filipino citizens.
This Court has consistently enjoined litigants to respect the hierarchy of The Congress, may, by law, allow small-scale utilization of natural resources
courts. Although this Court has concurrent jurisdiction with the Regional Trial by Filipino citizens, as well as cooperative fish farming, with priority to
Courts and the Court of Appeals to issue writs of certiorari, prohibition, subsistence fishermen and fishworkers in rivers, lakes, bays and lagoons.
mandamus, quo warranto, habeas corpus and injunction,[if !supportFootnotes][56][endif] The President may enter into agreements with foreign-owned corporations
such concurrence does not give a party unrestricted freedom of choice of involving either technical or financial assistance for large-scale exploration,
court forum. The resort to this Courts primary jurisdiction to issue said writs development and utilization of minerals, petroleum, and other mineral oils
shall be allowed only where the redress desired cannot be obtained in the according to the general terms and conditions provided by law, based on real
appropriate courts or where exceptional and compelling circumstances justify contributions to the economic growth and general welfare of the country. In
such invocation.[if !supportFootnotes][57][endif] We held in People v. Cuaresma[if ! such agreements, the State shall promote the development and use of local
supportFootnotes][58][endif]
that: scientific and technical resources.
A becoming regard for judicial hierarchy most certainly indicates that petitions The President shall notify the Congress of every contract entered into in
for the issuance of extraordinary writs against first level (inferior) courts accordance with this provision, within thirty days from its execution.
should be filed with the Regional Trial Court, and those against the latter, with Under IPRA, indigenous peoples may obtain the recognition of their right
the Court of Appeals. A direct invocation of the Supreme Courts original of ownership[if !supportFootnotes][60][endif] over ancestral lands and ancestral domains by
jurisdiction to issue these writs should be allowed only where there virtue of native title.[if !supportFootnotes][61][endif] The term ancestral lands under the

28
statute refers to lands occupied by individuals, families and clans who are of the public domain and, thus, cannot be extended to other lands of the
members of indigenous cultural communities, including residential lots, rice public domain such as forest or timber, mineral lands, and national parks.
terraces or paddies, private forests, swidden farms and tree lots. These lands Fourth, the Solicitor General asserts that even assuming that native title over
are required to have been occupied, possessed and utilized by them or ancestral lands and ancestral domains existed by virtue of the Cario doctrine,
through their ancestors since time immemorial, continuously to the present. [if ! such native title was extinguished upon the ratification of the 1935
supportFootnotes][62][endif]
On the other hand, ancestral domains is defined as areas Constitution.
generally belonging to indigenous cultural communities, including ancestral Fifth, petitioners admit that Congress is mandated under Section 5, Article XII
lands, forests, pasture, residential and agricultural lands, hunting grounds, of the Constitution to protect that rights of indigenous peoples to their
worship areas, and lands no longer occupied exclusively by indigenous ancestral lands and ancestral domains. However, they contend that the
cultural communities but to which they had traditional access, particularly the mandate is subject to Section 2, Article XII and the theory of jura regalia
home ranges of indigenous cultural communities who are still nomadic or embodied therein. According to petitioners, the recognition and protection
shifting cultivators. Ancestral domains also include inland waters, coastal under R.A. 8371 of the right of ownership over ancestral lands and ancestral
areas and natural resources therein. [if !supportFootnotes][63][endif] Again, the same are domains is far in excess of the legislative power and constitutional mandate of
required to have been held under a claim of ownership, occupied or possessed Congress.
by ICCs/IPs, by themselves or through their ancestors, communally or Finally, on the premise that ancestral lands and ancestral domains are owned
individually since time immemorial, continuously to the present. [if !supportFootnotes] by the State, petitioners posit that R.A. 8371 violates Section 2, Article XII of
[64][endif]
Under Section 56, property rights within the ancestral domains already the Constitution which prohibits the alienation of non-agricultural lands of the
existing and/or vested upon effectivity of said law shall be recognized and public domain and other natural resources.
respected. I am not persuaded by these contentions.
Ownership is the crux of the issue of whether the provisions of IPRA Undue reliance by petitioners and the Solicitor General on the theory of
pertaining to ancestral lands, ancestral domains, and natural resources are jura regalia is understandable. Not only is the theory well recognized in our
unconstitutional. The fundamental question is, who, between the State and legal system; it has been regarded, almost with reverence, as the immutable
the indigenous peoples, are the rightful owners of these properties? postulate of Philippine land law. It has been incorporated into our fundamental
It bears stressing that a statute should be construed in harmony with, law and has been recognized by the Court.[if !supportFootnotes][67][endif]
and not in violation, of the fundamental law. [if !supportFootnotes][65][endif] The reason is Generally, under the concept of jura regalia, private title to land must
that the legislature, in enacting a statute, is assumed to have acted within its be traced to some grant, express or implied, from the Spanish Crown or its
authority and adhered to the constitutional limitations. Accordingly, courts successors, the American Colonial government, and thereafter, the Philippine
should presume that it was the intention of the legislature to enact a valid, Republic. The belief that the Spanish Crown is the origin of all land titles in the
sensible, and just law and one which operates no further than may be Philippines has persisted because title to land must emanate from some
necessary to effectuate the specific purpose of the law. [if !supportFootnotes][66][endif] source for it cannot issue forth from nowhere.[if !supportFootnotes][68][endif]
A. The provisions of IPRA recognizing the ownership of indigenous peoples In its broad sense, the term jura regalia refers to royal rights,[if !
over the ancestral lands and ancestral domains are not unconstitutional. supportFootnotes][69][endif]
or those rights which the King has by virtue of his
In support of their theory that ancestral lands and ancestral domains prerogatives.[if !supportFootnotes][70][endif] In Spanish law, it refers to a right which the
are part of the public domain and, thus, owned by the State, pursuant to sovereign has over anything in which a subject has a right of property or
Section 2, Article XII of the Constitution, petitioners and the Solicitor General propriedad.[if !supportFootnotes][71][endif] These were rights enjoyed during feudal times
advance the following arguments: by the king as the sovereign.
First, according to petitioners, the King of Spain under international law The theory of the feudal system was that title to all lands was originally
acquired exclusive dominion over the Philippines by virtue of discovery and held by the King, and while the use of lands was granted out to others who
conquest. They contend that the Spanish King under the theory of jura regalia, were permitted to hold them under certain conditions, the King theoretically
which was introduced into Philippine law upon Spanish conquest in 1521, retained the title.[if !supportFootnotes][72][endif] By fiction of law, the King was regarded
acquired title to all the lands in the archipelago. as the original proprietor of all lands, and the true and only source of title, and
Second, petitioners and the Solicitor General submit that ancestral lands and from him all lands were held. [if !supportFootnotes][73][endif] The theory of jura regalia was
ancestral domains are owned by the State. They invoke the theory of jura therefore nothing more than a natural fruit of conquest.[if !supportFootnotes][74][endif]
regalia which imputes to the State the ownership of all lands and makes the The Regalian theory, however, does not negate native title to lands held
State the original source of all private titles. They argue that the Philippine in private ownership since time immemorial. In the landmark case of Cario vs.
State, as successor to Spain and the United States, is the source of any Insular Government[if !supportFootnotes][75][endif] the United States Supreme Court,
asserted right of ownership in land. reversing the decision[if !supportFootnotes][76][endif]of the pre-war Philippine Supreme
Third, petitioners and the Solicitor General concede that the Cario doctrine Court, made the following pronouncement:
exists. However, petitioners maintain that the doctrine merely states that title x x x Every presumption is and ought to be taken against the Government in a
to lands of the public domain may be acquired by prescription. The Solicitor case like the present. It might, perhaps, be proper and sufficient to say that
General, for his part, argues that the doctrine applies only to alienable lands when, as far back as testimony or memory goes, the land has been

29
held by individuals under a claim of private ownership, it will be successors-in-interest, the United States and the Philippine Government.
presumed to have been held in the same way from before the There has been no transfer of title from the State as the land has been
Spanish conquest, and never to have been public land. x x x.[if ! regarded as private in character as far back as memory goes. In contrast,
supportFootnotes][77][endif]
(Emphasis supplied.) ownership of land by acquisitive prescription against the State involves a
The above ruling institutionalized the recognition of the existence of conversion of the character of the property from alienable public land to
native title to land, or ownership of land by Filipinos by virtue of possession private land, which presupposes a transfer of title from the State to a private
under a claim of ownership since time immemorial and independent of any person. Since native title assumes that the property covered by it is private
grant from the Spanish Crown, as an exception to the theory of jura regalia. land and is deemed never to have been part of the public domain, the Solicitor
In Cario, an Igorot by the name of Mateo Cario applied for registration in Generals thesis that native title under Cario applies only to lands of the public
his name of an ancestral land located in Benguet. The applicant established domain is erroneous. Consequently, the classification of lands of the public
that he and his ancestors had lived on the land, had cultivated it, and had domain into agricultural, forest or timber, mineral lands, and national parks
used it as far they could remember. He also proved that they had all been under the Constitution[if !supportFootnotes][82][endif] is irrelevant to the application of the
recognized as owners, the land having been passed on by inheritance Cario doctrine because the Regalian doctrine which vests in the State
according to native custom. However, neither he nor his ancestors had any ownership of lands of the public domain does not cover ancestral lands and
document of title from the Spanish Crown. The government opposed the ancestral domains.
application for registration, invoking the theory of jura regalia. On appeal, the Legal history supports the Cario doctrine.
United States Supreme Court held that the applicant was entitled to the When Spain acquired sovereignty over the Philippines by virtue of its
registration of his native title to their ancestral land. discovery and occupation thereof in the 16th century and the Treaty of
Cario was decided by the U.S. Supreme Court in 1909, at a time when Tordesillas of 1494 which it entered into with Portugal, [if !supportFootnotes][83][endif] the
decisions of the U.S. Court were binding as precedent in our jurisdiction. [if ! continents of Asia, the Americas and Africa were considered as terra nullius
supportFootnotes][78][endif]
We applied the Cario doctrine in the 1946 case of Oh Cho vs. although already populated by other peoples. [if !supportFootnotes][84][endif] The discovery
Director of Lands,[if !supportFootnotes][79][endif] where we stated that [a]ll lands that and occupation by the European States, who were then considered as the only
were not acquired from the Government either by purchase or by grant, members of the international community of civilized nations, of lands in the
belong to the public domain, but [a]n exception to the rule would be any land said continents were deemed sufficient to create title under international law. [if
that should have been in the possession of an occupant and of his !supportFootnotes][85][endif]

predecessors in interest since time immemorial, for such possession would Although Spain was deemed to have acquired sovereignty over the
justify the presumption that the land had never been part of the public domain Philippines, this did not mean that it acquired title to all lands in the
or that it had been private property even before the Spanish conquest. [if ! archipelago. By virtue of the colonial laws of Spain, the Spanish Crown was
supportFootnotes][80][endif]
considered to have acquired dominion only over the unoccupied and
Petitioners however aver that the U.S. Supreme Courts ruling in Cario unclaimed portions of our islands.[if !supportFootnotes][86][endif]
was premised on the fact that the applicant had complied with the requisites In sending the first expedition to the Philippines, Spain did not intend to
of acquisitive prescription, having established that he and his predecessors-in- deprive the natives of their property. Miguel Lopez de Legazpi was under
interest had been in possession of the property since time immemorial. In instruction of the Spanish King to do no harm to the natives and to their
effect, petitioners suggest that title to the ancestral land applied for by Cario property. In this regard, an authority on the early Spanish colonial period in
was transferred from the State, as original owner, to Cario by virtue of the Philippines wrote:
prescription. They conclude that the doctrine cannot be the basis for The government of [the King of Spain] Philip II regarded the Philippines as a
decreeing by mere legislative fiatthat ownership of vast tracts of land belongs challenging opportunity to avoid a repetition of the sanguinary conquests of
to [indigenous peoples] without judicial confirmation. [if !supportFootnotes][81][endif] Mexico and Peru. In his written instructions for the Adelantado Legazpi, who
The Solicitor General, for his part, claims that the Cario doctrine applies commanded the expedition, Philip II envisaged a bloodless pacification of the
only to alienable lands of the public domain and, as such, cannot be extended archipelago. This extraordinary document could have been lifted almost
to other lands of the public domain such as forest or timber, mineral lands, verbatim from the lectures of the Dominican theologian, Francisco de Vitoria,
and national parks. delivered in the University of Salamanca. The King instructed Legazpi to
There is no merit in these contentions. inform the natives that the Spaniards had come to do no harm to their persons
A proper reading of Cario would show that the doctrine enunciated or to their property. The Spaniards intended to live among them in peace and
therein applies only to lands which have always been considered as in friendship and "to explain to them the law of Jesus Christ by which they will
private, and not to lands of the public domain, whether alienable or be saved." Although the Spanish expedition could defend themselves if
otherwise. A distinction must be made between ownership of land under attacked, the royal instructions admonished the commander to commit no
native title and ownership by acquisitive prescription against the State. aggressive act which might arouse native hostility.[if !supportFootnotes][87][endif]
Ownership by virtue of native title presupposes that the land has been held by Spanish colonial laws recognized and respected Filipino landholdings
its possessor and his predecessors-in-interest in the concept of an owner since including native land occupancy.[if !supportFootnotes][88][endif] Thus, the Recopilacin de
time immemorial. The land is not acquired from the State, that is, Spain or its Leyes de las Indias expressly conferred ownership of lands already held by the

30
natives.[if !supportFootnotes][89][endif] The royal decrees of 1880 and 1894 did not domains. While the Solicitor General admits that such a theory would
extinguish native title to land in the Philippines. The earlier royal decree, necessarily impair vested rights, he reasons out that even vested rights of
dated June 25, 1880, provided that all those in unlawful possession of royal ownership over ancestral lands and ancestral domains are not absolute and
lands must legalize their possession by means of adjustment proceedings, [if ! may be impaired by the legitimate exercise of police power.
supportFootnotes][90][endif]
and within the period specified. The later royal decree, dated I cannot agree. The text of the provision of the 1935 Constitution
February 13, 1894, otherwise known as the Maura Law, declared that titles invoked by the Solicitor General, while embodying the theory of jura regalia, is
that were capable of adjustment under the royal decree of 1880, but for which too clear for any misunderstanding. It simply declares that all agricultural,
adjustment was not sought, were forfeited. Despite the harsh wording of the timber, and mineral lands of the public domain, waters, minerals, coal,
Maura Law, it was held in the case of Cario that the royal decree of 1894 petroleum, and other mineral oils, all forces of potential energy, and other
should not be construed as confiscation of title, but merely as the withdrawal natural resources of the Philippines belong to the State. [if !supportFootnotes][99][endif]
of the privilege of registering such title.[if !supportFootnotes][91][endif] Nowhere does it state that certain lands which are absolutely necessary for
Neither was native title disturbed by the Spanish cession of the social welfare and existence, including those which are not part of the public
Philippines to the United States, contrary to petitioners assertion that the US domain, shall thereafter be owned by the State. If there is any room for
merely succeeded to the rights of Spain, including the latters rights over lands constitutional construction, the provision should be interpreted in favor of the
of the public domain.[if !supportFootnotes][92][endif] Under the Treaty of Paris of December preservation, rather than impairment or extinguishment, of vested rights.
10, 1898, the cession of the Philippines did not impair any right to property Stated otherwise, Section 1, Article XII of the 1935 Constitution cannot be
existing at the time.[if !supportFootnotes][93][endif] During the American colonial regime, construed to mean that vested right which had existed then were
native title to land was respected, even protected. The Philippine Bill of 1902 extinguished and that the landowners were divested of their lands, all in the
provided that property and rights acquired by the US through cession from guise of wrest[ing] control of those portions of the natural resources [which
Spain were to be administered for the benefit of the Filipinos. [if !supportFootnotes][94] the State] deems absolutely necessary for social welfare and existence. On
[endif]
In obvious adherence to libertarian principles, McKinleys Instructions, as the contrary, said Section restated the fundamental rule against the
well as the Philippine Bill of 1902, contained a bill of rights embodying the diminution of existing rights by expressly providing that the ownership of
safeguards of the US Constitution. One of these rights, which served as an lands of the public domain and other natural resources by the State is subject
inviolable rule upon every division and branch of the American colonial to any existing right, grant, lease, or concessions. The existing rights that
government in the Philippines, [if !supportFootnotes][95][endif] was that no person shall be were intended to be protected must, perforce, include the right of
deprived of life, liberty, or property without due process of law. [if !supportFootnotes][96] ownership by indigenous peoples over their ancestral lands and domains.
[endif]
These vested rights safeguarded by the Philippine Bill of 1902 were in The words of the law should be given their ordinary or usual meaning, [if !
turn expressly protected by the due process clause of the 1935 Constitution. supportFootnotes][100][endif]
and the term existing rights cannot be assigned an unduly
Resultantly, property rights of the indigenous peoples over their ancestral restrictive definition.
lands and ancestral domains were firmly established in law. Petitioners concede that Congress is mandated under Section 5, Article
Nonetheless, the Solicitor General takes the view that the vested rights XII of the 1987 Constitution [if !supportFootnotes][101][endif]to protect the rights of
of indigenous peoples to their ancestral lands and domains were abated by indigenous peoples to their ancestral lands and ancestral domains.
the direct act by the sovereign Filipino people of ratifying the 1935 Nonetheless, they contend that the recognition and protection under IPRA of
Constitution.[if !supportFootnotes][97][endif] He advances the following arguments: the right of ownership of indigenous peoples over ancestral lands and
The Sovereign, which is the source of all rights including ownership, has the ancestral domains are far in excess of the legislative power and constitutional
power to restructure the consolidation of rights inherent in ownership in the mandate of the Congress, [if !supportFootnotes][102][endif] since such recognition and
State. Through the mandate of the Constitutions that have been adopted, the protection amount to the alienation of lands of the public domain, which is
State has wrested control of those portions of the natural resources it deems proscribed under Section 2, Article XII of the Constitution.
absolutely necessary for social welfare and existence. It has been held that Section 5, Article XII of the Constitution expresses the sovereign intent
the State may impair vested rights through a legitimate exercise of police to protect the rights of indigenous peoples to their ancestral lands. In its
power. general and ordinary sense, the term right refers to any legally enforceable
Vested rights do not prohibit the Sovereign from performing acts not only claim.[if !supportFootnotes][103][endif] It is a power, privilege, faculty or demand inherent
essential to but determinative of social welfare and existence. To allow in one person and incident upon another. [if !supportFootnotes][104][endif] When used in
otherwise is to invite havoc in the established social system. x x x relation to property, "right" includes any interest in or title to an object, or any
Time-immemorial possession does not create private ownership in cases of just and legal claim to hold, use and enjoy it.[if !supportFootnotes][105][endif] Said provision
natural resources that have been found from generation to generation to be in the Constitution cannot, by any reasonable construction, be interpreted to
critical to the survival of the Sovereign and its agent, the State. [if !supportFootnotes][98] exclude the protection of the right of ownership over such ancestral lands.
[endif]
For this reason, Congress cannot be said to have exceeded its constitutional
Stated simply, the Solicitor Generals argument is that the State, as the mandate and power in enacting the provisions of IPRA, specifically Sections
source of all titles to land, had the power to re-vest in itself, through the 1935 7(a) and 8, which recognize the right of ownership of the indigenous peoples
Constitution, title to all lands, including ancestral lands and ancestral over ancestral lands.

31
The second paragraph of Section 5, Article XII also grants Congress the MR. BENNAGEN. The concept of customary laws is that it is considered
power to provide for the applicability of customary laws governing property as ownership by private individuals, clans and even communities.
rights or relations in determining the ownership and extent of ancestral MR. SUAREZ. So, there will be two aspects to this situation. This means that
domains. In light of this provision, does Congress have the power to decide the State will set aside the ancestral domain and there is a separate law for
whether ancestral domains shall be private property or part of the public that. Within the ancestral domain it could accept more specific ownership in
domain? Also, does Congress have the power to determine whether the extent terms of individuals within the ancestral lands.
of ancestral domains shall include the natural resources found therein? MR. BENNAGEN. Individuals and groups within the ancestral domain. [if !
It is readily apparent from the constitutional records that the framers of supportFootnotes][107][endif]
(Emphasis supplied.)
the Constitution did not intend Congress to decide whether ancestral domains It cannot be correctly argued that, because the framers of the
shall be public or private property. Rather, they acknowledged that ancestral Constitution never expressly mentioned Cario in their deliberations, they did
domains shall be treated as private property, and that customary laws shall not intend to adopt the concept of native title to land, or that they were
merely determine whether such private ownership is by the entire indigenous unaware of native title as an exception to the theory of jura regalia.[if !
cultural community, or by individuals, families, or clans within the community. supportFootnotes][108][endif]
The framers of the Constitution, as well as the people
The discussion below between Messrs. Regalado and Bennagen and Mr. Chief adopting it, were presumed to be aware of the prevailing judicial doctrines
Justice Davide, then members of the 1986 Constitutional Commission, is concerning the subject of constitutional provisions, and courts should take
instructive: these doctrines into consideration in construing the Constitution. [if !supportFootnotes]
MR. REGALADO. Thank you, Madame President. May I seek some clarifications [109][endif]

from either Commissioner Bennagen or Commissioner Davide regarding this Having thus recognized that ancestral domains under the Constitution
phrase CONGRESS SHALL PROVIDE FOR THE APPLICABILITY OF CUSTOMARY are considered as private property of indigenous peoples, the IPRA, by
LAWS GOVERNING PROPERTY RIGHTS OR RELATIONS in determining the affirming or acknowledging such ownership through its various provisions,
ownership and extent of the ancestral domain, because ordinarily it is the law merely abides by the constitutional mandate and does not suffer any vice of
on ownership and the extent thereof which determine the property rights or unconstitutionality.
relations arising therefrom. On the other hand, in this proposed amendment Petitioners interpret the phrase subject to the provisions of this
the phraseology is that it is the property rights or relations which shall be used Constitution and national development policies and programs in Section 5,
as the basis in determining the ownership and extent of the ancestral domain. Article XII of the Constitution to mean as subject to the provision of Section 2,
I assume there must be a certain difference in the customary laws and our Article XII of the Constitution, which vests in the State ownership of all lands of
regular civil laws on property. the public domain, mineral lands and other natural resources. Following this
MR. DAVIDE. That is exactly the reason, Madam President, why we will leave it interpretation, petitioners maintain that ancestral lands and ancestral
to Congress to make the necessary exception to the general law on property domains are the property of the State.
relations. This proposition is untenable. Indeed, Section 2, Article XII reiterates the
MR. REGALADO. I was thinking if Commissioner Bennagen could give us an declarations made in the 1935 and 1973 Constitutions on the state policy of
example of such a customary law wherein it is the property rights and conservation and nationalization of lands of the public domain and natural
relations that determine the ownership and the extent of that ownership, resources, and is of paramount importance to our national economy and
unlike the basic fundamental rule that it is the ownership and the extent of patrimony. A close perusal of the records of the 1986 Constitutional
ownership which determine the property rights and relations arising therefrom Commission reveals that the framers of the Constitution inserted the phrase
and consequent thereto. Perhaps, these customary laws may have a different subject to the provisions of this Constitution mainly to prevent the impairment
provision or thrust so that we could make the corresponding suggestions also of Torrens titles and other prior rights in the determination of what constitutes
by way of an amendment. ancestral lands and ancestral domains, to wit:
MR. DAVIDE. That is exactly my own perception. MR. NATIVIDAD. Just one question. I want to clear this section protecting
MR. BENNAGEN. Let me put it this way. ancestral lands. How does this affect the Torrens title and other prior rights?
There is a range of customary laws governing certain types of ownership. MR. BENNAGEN. I think that was also discussed in the committee hearings and
There would be ownership based on individuals, on clan or lineage, or we did say that in cases where due process is clearly established in terms of
on community. And the thinking expressed in the consultation is that this prior rights, these two have to be respected.
should be codified and should be recognized in relation to existing national MR. NATIVIDAD. The other point is: How vast is this ancestral land? Is it true
laws. That is essentially the concept. [if !supportFootnotes][106][endif] (Emphasis supplied.) that parts of Baguio City are considered as ancestral lands?
The intention to treat ancestral domains as private property is also MR. BENNAGEN. They could be regarded as such. If the Commissioner still
apparent from the following exchange between Messrs. Suarez and Bennagen: recalls, in one of the publications that I provided the Commissioners, the parts
MR. SUAREZ. When we speak of customary laws governing property rights or could be considered as ancestral domain in relation to the whole population of
relations in determining the ownership and extent of the ancestral domain, Cordillera but not in relation to certain individuals or certain groups.
are we thinking in terms of the tribal ownership or community ownership or of MR. NATIVIDAD. The Commissioner means that the whole Baguio City is
private ownership within the ancestral lands or ancestral domain? considered as ancestral land?

32
MR. BENNAGEN. Yes, in the sense that it belongs to Cordillera or in the same BISHOP BACANI. In Commissioner Davides formulation of the first sentence, he
manner that Filipinos can speak of the Philippine archipelago as ancestral says: The State, SUBJECT TO THE provisions of this Constitution AND
land, but not in terms of the right of a particular person or particular group to NATIONAL DEVELOPMENT POLICIES AND PROGRAMS shall guarantee the rights
exploit, utilize, or sell it. of cultural or tribal communities to their ancestral lands to insure their
MR. NATIVIDAD. But is clear that the prior rights will be respected. economic, social and cultural well-being. There are at least two concepts here
MR. BENNAGEN. Definitely. [if !supportFootnotes][110][endif] which receive different weights very often. They are the concepts of national
Thus, the phrase subject to the provisions of this Constitution was development policies and programs, and the rights of cultural or tribal
intended by the framers of the Constitution as a reiteration of the communities to their ancestral lands, et cetera. I would like to ask: When the
constitutional guarantee that no person shall be deprived of property without Commissioner proposed this amendment, which was the controlling concept? I
due process of law. ask this because sometimes the rights of cultural minorities are precisely
There is another reason why Section 5 of Article XII mandating the transgressed in the interest of national development policies and programs.
protection of rights of the indigenous peoples to their ancestral lands cannot Hence, I would like to know which is the controlling concept here. Is it the
be construed as subject to Section 2 of the same Article ascribing ownership rights of indigenous peoples to their ancestral lands or is it national
of all public lands to the State. The Constitution must be construed as a whole. development policies and programs.
It is a rule that when construction is proper, the whole Constitution is MR. DAVIDE. It is not really a question of which is primary or which is
examined in order to determine the meaning of any provision. That more paramount. The concept introduced here is really the balancing
construction should be used which would give effect to the entire instrument. [if of interests. That is what we seek to attain. We have to balance the interests
!supportFootnotes][111][endif]
taking into account the specific needs and the specific interests also of these
Thus, the provisions of the Constitution on State ownership of public cultural communities in like manner that we did so in the autonomous regions.
lands, mineral lands and other natural resources should be read together with [if !supportFootnotes][115][endif]
(Emphasis supplied.)
the other provisions thereof which firmly recognize the rights of the B. The provisions of R.A. 8371 do not infringe upon the States ownership over
indigenous peoples. These, as set forth hereinbefore, [if !supportFootnotes][112][endif] the natural resources within the ancestral domains.
include: Section 22, Article II, providing that the State recognizes and Petitioners posit that IPRA deprives the State of its ownership over
promotes the rights of indigenous peoples within the framework of national mineral lands of the public domain and other natural resources, [if !supportFootnotes]
unity and development; Section 5, Article XII, calling for the protection of [116][endif]
as well as the States full control and supervision over the exploration,
the rights of indigenous cultural communities to their ancestral lands to development and utilization of natural resources.[if !supportFootnotes][117][endif]
ensure their economic, social, and cultural well-being, and for the applicability Specifically, petitioners and the Solicitor General assail Sections 3 (a), [if !
of customary laws governing property rights and relations in determining the supportFootnotes][118][endif]
5,[if !supportFootnotes][119][endif] and 7[if !supportFootnotes][120][endif] of IPRA as
ownership and extent of ancestral domains; Section 1, Article XIII, directing violative of Section 2, Article XII of the Constitution which states, in part, that
the removal or reduction of social, economic, political and cultural inequities [a]ll lands of the public domain, waters, minerals, coal, petroleum, and other
and inequalities by equitably diffusing wealth and political power for the mineral oils, all forces of potential energy, fisheries, forests or timber, wildlife,
common good; Section 6, Article XIII, directing the application of the flora and fauna, and other natural resources are owned by the State. [if !
principles of agrarian reform or stewardship in the disposition and utilization of supportFootnotes][121][endif]
They would have the Court declare as unconstitutional
other natural resources, subject to prior rights, homestead rights of small Section 3(a) of IPRA because the inclusion of natural resources in the
settlers, and the rights of indigenous communities to their ancestral lands; definition of ancestral domains purportedly results in the abdication of State
Section 17, Article XIV, decreeing that the State shall recognize, respect, ownership over these resources.
and protect the rights of indigenous cultural communities to preserve and I am not convinced.
develop their cultures, traditions, and institutions; and Section 12, Article Section 3(a) merely defines the coverage of ancestral domains, and
XVI, authorizing the Congress to create a consultative body to advise the describes the extent, limit and composition of ancestral domains by setting
President on policies affecting indigenous cultural communities. forth the standards and guidelines in determining whether a particular area is
Again, as articulated in the Constitution, the first goal of the national to be considered as part of and within the ancestral domains. In other words,
economy is the more equitable distribution of opportunities, income, Section 3(a) serves only as a yardstick which points out what properties are
and wealth.[if !supportFootnotes][113][endif] Equity is given prominence as the first within the ancestral domains. It does not confer or recognize any right of
objective of national economic development.[if !supportFootnotes][114][endif] The framers ownership over the natural resources to the indigenous peoples. Its purpose is
of the Constitution did not, by the phrase subject to the provisions of this definitional and not declarative of a right or title.
Constitution and national development policies and programs, intend to The specification of what areas belong to the ancestral domains is, to
establish a hierarchy of constitutional norms. As explained by then our mind, important to ensure that no unnecessary encroachment on private
Commissioner (now Chief Justice) Hilario G. Davide, Jr., it was not their properties outside the ancestral domains will result during the delineation
objective to make certain interests primary or paramount, or to create process. The mere fact that Section 3(a) defines ancestral domains to include
absolute limitations or outright prohibitions; rather, the idea is towards the the natural resources found therein does not ipso facto convert the character
balancing of interests: of such natural resources as private property of the indigenous peoples.

33
Similarly, Section 5 in relation to Section 3(a) cannot be construed as a source CHAIRMAN FLAVIER. Okay, thank you. So we now move to Section 8, there is a
of ownership rights of indigenous people over the natural resources simply Senate version you do not have and if you agree we will adopt that. [if !
because it recognizes ancestral domains as their private but community supportFootnotes][127][endif]
(Emphasis supplied.)
property. Further, Section 7 makes no mention of any right of ownership of the
The phrase private but community property is merely descriptive of the indigenous peoples over the natural resources. In fact, Section 7(a) merely
indigenous peoples concept of ownership as distinguished from that provided recognizes the right to claim ownership over lands, bodies of water
in the Civil Code. In Civil Law, ownership is the independent and general traditionally and actually occupied by indigenous peoples, sacred places,
power of a person over a thing for purposes recognized by law and within the traditional hunting and fishing grounds, and all improvements made by them
limits established thereby.[if !supportFootnotes][122][endif] The civil law concept of at any time within the domains. Neither does Section 7(b), which enumerates
ownership has the following attributes: jus utendi or the right to receive from certain rights of the indigenous peoples over the natural resources found
the thing that which it produces, jus abutendi or the right to consume the within their ancestral domains, contain any recognition of ownership vis-a-vis
thing by its use, jus disponendi or the power to alienate, encumber, transform the natural resources.
and even destroy that which is owned and jus vidicandi or the right to exclude What is evident is that the IPRA protects the indigenous peoples rights
other persons from the possession the thing owned. [if !supportFootnotes][123][endif] In and welfare in relation to the natural resources found within their ancestral
contrast, the indigenous peoples concept of ownership emphasizes the domains,[if !supportFootnotes][128][endif] including the preservation of the ecological
importance of communal or group ownership. By virtue of the communal balance therein and the need to ensure that the indigenous peoples will not
character of ownership, the property held in common cannot be sold, disposed be unduly displaced when State-approved activities involving the natural
or destroyed[if !supportFootnotes][124][endif] because it was meant to benefit the whole resources located therein are undertaken.
indigenous community and not merely the individual member. [if !supportFootnotes][125] Finally, the concept of native title to natural resources, unlike native
[endif]
title to land, has not been recognized in the Philippines. NCIP and Flavier, et
That IPRA is not intended to bestow ownership over natural resources to al. invoke the case of Reavies v. Fianza[if !supportFootnotes][129][endif] in support of their
the indigenous peoples is also clear from the deliberations of the bicameral thesis that native title to natural resources has been upheld in this jurisdiction.
conference committee on Section 7 which recites the rights of indigenous [if !supportFootnotes][130][endif]
They insist that it is possible for rights over natural
peoples over their ancestral domains, to wit: resources to vest on a private (as opposed to a public) holder if these were
CHAIRMAN FLAVIER. Accepted. Section 8 [if !supportFootnotes][126][endif] rights to ancestral held prior to the 1935 Constitution. [if !supportFootnotes][131][endif] However, a judicious
domain, this is where we transferred the other provision but here itself examination of Reavies reveals that, contrary to the position of NCIP and
--- Flavier, et al., the Court did not recognize native title to natural resources.
HON. DOMINGUEZ. Mr. Chairman, if I maybe allowed to make a very short Rather, it merely upheld the right of the indigenous peoples to claim
Statement. Earlier, Mr. Chairman, we have decided to remove the ownership of minerals under the Philippine Bill of 1902.
provisions on natural resources because we all agree that that While as previously discussed, native title to land or private ownership
belongs to the State. Now, the plight or the rights of those by Filipinos of land by virtue of time immemorial possession in the concept of
indigenous communities living in forest and areas where it could be an owner was acknowledged and recognized as far back during the Spanish
exploited by mining, by dams, so can we not also provide a provision to colonization of the Philippines, there was no similar favorable treatment as
give little protection or either rights for them to be consulted before any regards natural resources. The unique value of natural resources has been
mining areas should be done in their areas, any logging done in their acknowledged by the State and is the underlying reason for its consistent
areas or any dam construction because this has been disturbing our assertion of ownership and control over said natural resources from the
people especially in the Cordilleras. So, if there could be, if our lawyers Spanish regime up to the present. [if !supportFootnotes][132][endif] Natural resources,
or the secretariat could just propose a provision for incorporation here especially minerals, were considered by Spain as an abundant source of
so that maybe the right to consultation and the right to be revenue to finance its battles in wars against other nations. Hence, Spain, by
compensated when there are damages within their ancestral lands. asserting its ownership over minerals wherever these may be found, whether
CHAIRMAN FLAVIER. Yes, very well taken but to the best of my recollection in public or private lands, recognized the separability of title over lands and
both are already considered in subsequent sections which we are now that over minerals which may be found therein. [if !supportFootnotes][133][endif]
looking for. On the other hand, the United States viewed natural resources as a
HON. DOMINGUEZ. Thank you. source of wealth for its nationals. As the owner of natural resources over the
CHAIRMAN FLAVIER. First of all there is a line that gives priority use for the Philippines after the latters cession from Spain, the United States saw it fit to
indigenous people where they are. Number two, in terms of the mines allow both Filipino and American citizens to explore and exploit minerals in
there is a need for prior consultation of source which is here already. So, public lands, and to grant patents to private mineral lands. A person who
anyway it is on the record that you want to make sure that the acquired ownership over a parcel of private mineral land pursuant to the laws
secretariat takes note of those two issues and my assurance is that it is then prevailing could exclude other persons, even the State, from exploiting
already there and I will make sure that they cross check. minerals within his property.[if !supportFootnotes][134][endif] Although the United States
HON. ADAMAT. I second that, Mr. Chairman. made a distinction between minerals found in public lands and those found in

34
private lands, title in these minerals was in all cases sourced from the State. obtained.[if !supportFootnotes][140][endif] In return, the indigenous peoples are given the
The framers of the 1935 Constitution found it necessary to maintain the States responsibility to maintain, develop, protect and conserve the ancestral
ownership over natural resources to insure their conservation for future domains or portions thereof which are found to be necessary for critical
generations of Filipinos, to prevent foreign control of the country through watersheds, mangroves, wildlife sanctuaries, wilderness, protected areas,
economic domination; and to avoid situations whereby the Philippines would forest cover, or reforestation.[if !supportFootnotes][141][endif]
become a source of international conflicts, thereby posing danger to its The Solicitor General argues that these provisions deny the State an
internal security and independence.[if !supportFootnotes][135][endif] active and dominant role in the utilization of our countrys natural resources.
The declaration of State ownership and control over minerals and other Petitioners, on the other hand, allege that under the Constitution the
natural resources in the 1935 Constitution was reiterated in both the 1973 [if ! exploration, development and utilization of natural resources may only be
supportFootnotes][136][endif]
and 1987 Constitutions.[if !supportFootnotes][137][endif] undertaken by the State, either directly or indirectly through co-production,
Having ruled that the natural resources which may be found within the joint venture, or production-sharing agreements. [if !supportFootnotes][142][endif] To
ancestral domains belong to the State, the Court deems it necessary to clarify petitioners, no other method is allowed by the Constitution. They likewise
that the jurisdiction of the NCIP with respect to ancestral domains under submit that by vesting ownership of ancestral lands and ancestral domains in
Section 52 [i] of IPRA extends only to the lands and not to the natural the indigenous peoples, IPRA necessarily gives them control over the use and
resources therein. enjoyment of such natural resources, to the prejudice of the State. [if !
Section 52[i] provides: supportFootnotes][143][endif]

Turnover of Areas Within Ancestral Domains Managed by Other Government Section 2, Article XII of the Constitution provides in paragraph 1 thereof
Agencies. -- The Chairperson of the NCIP shall certify that the area covered is that the exploration, development and utilization of natural resources must be
an ancestral domain. The secretaries of the Department of Agrarian Reform, under the full control and supervision of the State, which may directly
Department of Environment and Natural Resources, Department of Interior undertake such activities or enter into co-production, joint venture, or
and Local Government, and Department of Justice, the Commissioner of the production-sharing agreements. This provision, however, should not be read in
National Development Corporation, and any other government agency isolation to avoid a mistaken interpretation that any and all forms of utilization
claiming jurisdiction over the area shall be notified thereof. Such notification of natural resources other than the foregoing are prohibited. The Constitution
shall terminate any legal basis for the jurisdiction previously claimed. must be regarded as consistent with itself throughout. [if !supportFootnotes][144][endif] No
Undoubtedly, certain areas that are claimed as ancestral domains may constitutional provision is to be separated from all the others, or to be
still be under the administration of other agencies of the Government, such as considered alone, all provisions bearing upon a particular subject are to be
the Department of Agrarian Reform, with respect to agricultural lands, and the brought into view and to be so interpreted as to effectuate the great purposes
Department of Environment and Natural Resources with respect to timber, of the fundamental law.[if !supportFootnotes][145][endif]
forest and mineral lands. Upon the certification of these areas as ancestral In addition to the means of exploration, development and utilization of
domain following the procedure outlined in Sections 51 to 53 of the IPRA, the countrys natural resources stated in paragraph 1, Section 2 of Article XII,
jurisdiction of the government agency or agencies concerned over lands the Constitution itself states in the third paragraph of the same section that
forming part thereof ceases. Nevertheless, the jurisdiction of government Congress may, by law, allow small-scale utilization of natural resources
agencies over the natural resources within the ancestral domains does not by its citizens.[if !supportFootnotes][146][endif] Further, Section 6, Article XIII, directs the
terminate by such certification because said agencies are mandated under State, in the disposition and utilization of natural resources, to apply the
existing laws to administer the natural resources for the State, which is the principles of agrarian reform or stewardship. [if !supportFootnotes][147][endif] Similarly,
owner thereof. To construe Section 52[i] as divesting the State, through the Section 7, Article XIII mandates the State to protect the rights of subsistence
government agencies concerned, of jurisdiction over the natural resources fishermen to the preferential use of marine and fishing resources.[if !
within the ancestral domains would be inconsistent with the established supportFootnotes][148][endif]
Clearly, Section 2, Article XII, when interpreted in view of
doctrine that all natural resources are owned by the State. the pro-Filipino, pro-poor philosophy of our fundamental law, and in harmony
C. The provisions of IPRA pertaining to the utilization of natural resources are with the other provisions of the Constitution rather as a sequestered
not unconstitutional. pronouncement,[if !supportFootnotes][149][endif] cannot be construed as a prohibition
The IPRA provides that indigenous peoples shall have the right to against any and all forms of utilization of natural resources without the States
manage and conserve the natural resources found on the ancestral domains, direct participation.
to benefit from and share in the profits from the allocation and utilization of Through the imposition of certain requirements and conditions for the
these resources, and to negotiate the terms and conditions for the exploration exploration, development and utilization of the natural resources under
of such natural resources.[if !supportFootnotes][138][endif] The statute also grants them existing laws,[if !supportFootnotes][150][endif] the State retains full control over such
priority rights in the harvesting, extraction, development or exploitation of any activities, whether done on small-scale basis[if !supportFootnotes][151][endif] or otherwise.
natural resources within the ancestral domains. [if !supportFootnotes][139][endif] Before the The rights given to the indigenous peoples regarding the exploitation of
NCIP can issue a certification for the renewal, or grant of any concession, natural resources under Sections 7(b) and 57 of IPRA amplify what has been
license or lease, or for the perfection of any production-sharing agreement the granted to them under existing laws, such as the Small-Scale Mining Act of
prior informed written consent of the indigenous peoples concerned must be 1991 (R.A. 7076) and the Philippine Mining Act of 1995 (R.A. 7942). R.A. 7076

35
expressly provides that should an ancestral land be declared as a peoples the State from undertaking activities, or entering into co-production, joint
small-scale mining area, the members of the indigenous peoples living within venture or production-sharing agreements with private entities, to utilize the
said area shall be given priority in the awarding of small-scale mining natural resources which may be located within the ancestral domains. There is
contracts.[if !supportFootnotes][152][endif] R.A. 7942 declares that no ancestral land no intention, as between the State and the indigenous peoples, to create a
shall be opened for mining operations without the prior consent of hierarchy of values; rather, the object is to balance the interests of the State
the indigenous cultural community concerned[if !supportFootnotes][153][endif] and in for national development and those of the indigenous peoples.
the event that the members of such indigenous cultural community give their Neither does the grant of priority rights to the indigenous peoples
consent to mining operations within their ancestral land, royalties shall be exclude non-indigenous peoples from undertaking the same activities within
paid to them by the parties to the mining to the contract.[if !supportFootnotes][154][endif] the ancestral domains upon authority granted by the proper governmental
In any case, a careful reading of Section 7(b) would reveal that the agency. To do so would unduly limit the ownership rights of the State over the
rights given to the indigenous peoples are duly circumscribed. These rights natural resources.
are limited only to the following: to manage and conserve natural resources To be sure, the act of the State of giving preferential right to a particular
within territories and uphold it for future generations; to benefit and share sector in the utilization of natural resources is nothing new. As previously
the profits from allocation and utilization of the natural resources found mentioned, Section 7, Article XIII of the Constitution mandates the protection
therein; to negotiate the terms and conditions for the exploration of by the State of the rights of subsistence fishermen, especially of local
natural resources in the areas for the purpose of ensuring ecological, communities, to the preferential use of communal marine and fishing
environmental protection and the conservation measures, pursuant to national resources, both inland and offshore.
and customary laws; to an informed and intelligent participation in the Section 57 further recognizes the possibility that the exploration and
formulation and implementation of any project, government or private, that exploitation of natural resources within the ancestral domains may disrupt the
will affect or impact upon the ancestral domains and to receive just and fair natural environment as well as the traditional activities of the indigenous
compensation for any damages which they may sustain as a result of the peoples therein. Hence, the need for the prior informed consent of the
project, and the right to effective measures by the government to prevent indigenous peoples before any search for or utilization of the natural resources
any interference with, alienation and encroachment of these rights. within their ancestral domains is undertaken.
It must be noted that the right to negotiate terms and conditions In a situation where the State intends to directly or indirectly undertake
granted under Section 7(b) pertains only to the exploration of natural such activities, IPRA requires that the prior informed consent of the indigenous
resources. The term exploration refers only to the search or prospecting of peoples be obtained. The State must, as a matter of policy and law, consult
mineral resources, or any other means for the purpose of determining the the indigenous peoples in accordance with the intent of the framers of the
existence and the feasibility of mining them for profit. [if !supportFootnotes][155][endif] The Constitution that national development policies and programs should involve a
exploration, which is merely a preliminary activity, cannot be equated with the systematic consultation to balance local needs as well as national plans. As
entire process of exploration, development and utilization of natural resources may be gathered from the discussion of the framers of the Constitution on this
which under the Constitution belong to the State. point, the national plan presumably takes into account the requirements of the
Section 57, on the other hand, grants the indigenous peoples priority region after thorough consultation. [if !supportFootnotes][156][endif] To this end, IPRA grants
rights in the utilization of natural resources and not absolute ownership to the indigenous peoples the right to an informed and intelligent participation
thereof. Priority rights does not mean exclusive rights. What is granted is in the formulation and implementation of any project, government or private,
merely the right of preference or first consideration in the award of privileges and the right not to be removed therefrom without their free and prior
provided by existing laws and regulations, with due regard to the needs and informed consent.[if !supportFootnotes][157][endif] As to non-members, the prior informed
welfare of indigenous peoples living in the area. consent takes the form of a formal and written agreement between the
There is nothing in the assailed law which implies an automatic or indigenous peoples and non-members under the proviso in Section 57 in case
mechanical character in the grant of concessions. Nor does the law negate the the State enters into a co-production, joint venture, or production-sharing
exercise of sound discretion by government entities. Several factors still have agreement with Filipino citizens, or corporations. This requirement is not
to be considered. For example, the extent and nature of utilization and the peculiar to IPRA. Existing laws and regulations such as the Philippine
consequent impact on the environment and on the indigenous peoples way of Environmental Policy,[if !supportFootnotes][158][endif] the Environmental Impact System, [if !
life are important considerations. Moreover, the indigenous peoples must supportFootnotes][159][endif]
the Local Government Code[if !supportFootnotes][160][endif] and the
show that they live in the area and that they are in the best position to Philippine Mining Act of 1995[if !supportFootnotes][161][endif]already require increased
undertake the required utilization. consultation and participation of stakeholders, such as indigenous peoples, in
It must be emphasized that the grant of said priority rights to the planning of activities with significant environment impact.
indigenous peoples is not a blanket authority to disregard pertinent laws and The requirement in Section 59 that prior written informed consent of the
regulations. The utilization of said natural resources is always subject to indigenous peoples must be procured before the NCIP can issue a certification
compliance by the indigenous peoples with existing laws, such as R.A. 7076 for the issuance, renewal, or grant of any concession, license or lease, or to
and R.A. 7942 since it is not they but the State, which owns these resources. the perfection of any production-sharing agreement, must be interpreted, not
It also bears stressing that the grant of priority rights does not preclude as a grant of the power to control the exploration, development and utilization

36
of natural resources, but merely the imposition of an additional requirement second, posting and publication; third, investigation and inspection by the
for such concession or agreement. The clear intent of the law is to protect the ADO; fourth, delineation; lastly, evaluation by the NCIP upon submission of a
rights and interests of the indigenous peoples which may be adversely report by the ADO.[if !supportFootnotes][166][endif] Hence, we cannot sustain the
affected by the operation of such entities or licensees. arguments of the petitioners that the law affords no protection to those who
Corollary Issues are not indigenous peoples.
A. IPRA does not violate the Due Process clause. Neither do the questioned sections of IPRA on the composition and
The first corollary issue raised by petitioners is whether IPRA violates powers and jurisdiction of the NCIP [if !supportFootnotes][167][endif] and the application of
Section 1, Article III of the Constitution, which provides that no person shall be customary law,[if !supportFootnotes][168][endif] violate the due process clause of the
deprived of life, liberty, or property without due process of law, nor shall any Constitution.
person be deprived the equal protection of the laws. Petitioners point out that IPRA provides that the NCIP shall be composed
Petitioners maintain that the broad definition of ancestral lands and exclusively of members of indigenous peoples, [if !supportFootnotes][169][endif] and that
ancestral domains under Section 3(a) and 3(b) of IPRA includes private lands. the NCIP shall have jurisdiction over all claims and disputes involving
They argue that the inclusion of private lands in the ancestral lands and indigenous peoples,[if !supportFootnotes][170][endif] including even disputes between a
ancestral domains violates the due process clause. [if !supportFootnotes][162][endif] member of such communities and one who is not a member, as well as over
Petitioners contention is erroneous. disputes in the delineation of ancestral domains. [if !supportFootnotes][171][endif]
Sections 3(a) and 3(b) expressly provide that the definition of ancestral Petitioners clarify that they do not claim that the members of the NCIP are
lands and ancestral domains are subject to Section 56, which reads: incapable of being fair and impartial judges. They merely contend that the
Sec. 56. Existing Property Rights Regimes. Property rights within the NCIP will not appear to be impartial, because a party who is not a member of
ancestral domains already existing and/or vested upon effectivity of this Act, an indigenous cultural community who must defend his case against [one who
shall be recognized and protected. is] before judges who are all members of [indigenous peoples] cannot but
Petitioners, however, contend that Section 56 aims to protect only the harbor a suspicion that they do not have the cold neutrality of an impartial
vested rights of indigenous peoples, but not those who are not members of judge.[if !supportFootnotes][172][endif]
such communities. Following their interpretation, IPRA, under Section 56, In addition, petitioners claim that IPRA prescribes that customary laws
recognizes the rights of indigenous peoples to their ancestral lands and shall be applied first in disputes involving property, succession and land, [if !
ancestral domains, subject to the vested rights of the same supportFootnotes][173][endif]
and that such laws shall likewise be used in disputes
communities to such ancestral lands and ancestral domains. Such involving indigenous peoples.[if !supportFootnotes][174][endif] They assert that [w]hen the
interpretation is obviously incorrect. dispute involves a member of an [indigenous cultural community and another
The property rights referred to in Section 56 belong to those acquired who is not], a resolution of such a dispute based on customary laws. . . would
by individuals, whether indigenous or non-indigenous peoples. Said provision clearly be a denial of due process. . . [because those who are not indigenous
makes no distinction as to the ethnic origins of the ownership of these peoples] do not know what these customary laws are.[if !supportFootnotes][175][endif]
property rights. The IPRA thus recognizes and respects vested rights Petitioners concerns are unfounded. The fact that the NCIP is composed
regardless of whether they pertain to indigenous or non-indigenous peoples. of members of the indigenous peoples does not mean that it (the NCIP) is
Where the law does not distinguish, the courts should not distinguish. [if ! incapable, or will appear to be so incapable, of delivering justice to the non-
supportFootnotes][163][endif]
What IPRA only requires is that these property rights already indigenous peoples. A persons possession of the trait of impartiality desirable
exist and/or vested upon its effectivity. of a judge has nothing to do with his or her ethnic roots. In this wise, the
Further, by the enactment of IPRA, Congress did not purport to annul indigenous peoples are as capable of rendering justice as the non-indigenous
any and all Torrens titles within areas claimed as ancestral lands or ancestral peoples for, certainly, the latter have no monopoly of the concept of justice.
domains. The statute imposes strict procedural requirements for the proper In any case, there are sufficient checks in the law against any abuse by
delineation of ancestral lands and ancestral domains as safeguards against the NCIP of its quasi-judicial powers. Section 67 states that the decision of the
the fraudulent deprivation of any landowner of his land, whether or not he is NCIP shall be appealable to the Court of Appeals by petition for review. The
member of an indigenous cultural community. In all proceedings for regular remedies under our rules of procedure are likewise available to any
delineation of ancestral lands and ancestral domains, the Director of Lands party aggrieved by the decision of the NCIP.
shall appear to represent the interest of the Republic of the Philippines. [if ! Anent the use of customary laws in determining the ownership and
supportFootnotes][164][endif]
With regard to ancestral domains, the following procedure extent of ancestral domains, suffice it to say that such is allowed under
is mandatory: first, petition by an indigenous cultural community, or motu paragraph 2, Section 5 of Article XII of the Constitution. Said provision states,
proprio by the NCIP; second, investigation and census by the Ancestral The Congress may provide for the applicability of customary laws governing
domains Office ("ADO") of the NCIP; third, preliminary report by the ADO; property rights and relations in determining the ownership and extent of the
fourth, posting and publication; and lastly, evaluation by the NCIP upon ancestral domains. Notably, the use of customary laws under IPRA is not
submission of the final report of the ADO. [if !supportFootnotes][165][endif] With regard to absolute, for the law speaks merely of primacy of use.[if !supportFootnotes][176][endif]
ancestral lands, unless such lands are within an ancestral domain, the The IPRA prescribes the application of such customary laws where these
statute imposes the following procedural requirements: first, application; present a workable solution acceptable to the parties, who are members of

37
the same indigenous group. This interpretation is supported by Section 1, Rule is characterized as a lateral but autonomous relationship for
IX of the Implementing Rules which states: purposes of policy and program coordination. This relationship shall be
RULE IX. JURISDICTION AND PROCEDURES FOR ENFORCEMENT OF RIGHTS carried out through a system of periodic reporting. Matters of day-to-day
Section 1. Primacy of Customary Law. All conflicts related to ancestral domains administration or all those pertaining to internal operations shall be left to the
and lands, involving ICCs/IPs, such as but not limited to conflicting claims and discretion of the Chairperson of the Commission, as the Chief Executive
boundary disputes, shall be resolved by the concerned parties through the Officer.
application of customary laws in the area where the disputed ancestral Petitioners asseverate that the aforecited rule infringes upon the power
domain or land is located. of control of the President over the NCIP by characterizing the relationship of
All conflicts related to the ancestral domains or lands where one of the NCIP to the Office of the President as lateral but autonomous...for
the parties is a non-ICC/IP or where the dispute could not be resolved purposes of policy and program coordination.
through customary law shall be heard and adjudicated in accordance Although both Section 40 of the IPRA and Section 1, Part II, Rule VII of
with the Rules on Pleadings, Practice and Procedures Before the NCIP the Implementing Rules characterize the NCIP as an independent agency
to be adopted hereafter. (Emphasis supplied.) under the Office of the President, such characterization does not remove said
The application of customary law is limited to disputes concerning body from the Presidents control and supervision.
property rights or relations in determining the ownership and extent The NCIP has been designated under IPRA as the primary government
of the ancestral domains,[if !supportFootnotes][177][endif] where all the parties involved agency responsible for the formulation and implementation of policies, plans
are members of indigenous peoples, [if !supportFootnotes][178][endif] specifically, of the and programs to promote and protect the rights and well being of the
same indigenous group. It therefore follows that when one of the parties to a indigenous peoples and the recognition of their ancestral domain as well as
dispute is a non-member of an indigenous group, or when the indigenous their rights thereto.[if !supportFootnotes][182][endif] It has been granted administrative, [if !
peoples involved belong to different groups, the application of customary law supportFootnotes][183][endif]
quasi-legislative[if !supportFootnotes][184][endif] and quasi-judicial
is not required. powers [if !supportFootnotes][185][endif]
to carry out its mandate. The diverse nature of the
Like any other law, the objective of IPRA in prescribing the primacy of NCIPs functions renders it impossible to place said agency entirely under the
customary law in disputes concerning ancestral lands and domains where all control of only one branch of government and this, apparently, is the reason
parties involved are indigenous peoples is justice. The utilization of customary for its characterization by Congress as an independent agency. An
laws is in line with the constitutional policy of recognizing the application independent agency is defined as an administrative body independent of the
thereof through legislation passed by Congress. executive branch or one not subject to a superior head of department, as
Furthermore, the recognition and use of customary law is not a novel distinguished from a subordinate agency or an administrative body whose
idea in this jurisdiction. Under the Civil Code, use of customary law is action is subject to administrative review or revision. [if !supportFootnotes][186][endif]
sanctioned, as long as it is proved as a fact according to the rules of evidence, That Congress did not intend to place the NCIP under the control of the
[if !supportFootnotes][179][endif]
and it is not contrary to law, public order or public policy. [if ! President in all instances is evident in the IPRA itself, which provides that the
supportFootnotes][180][endif]
Moreover, the Local Government Code of 1991 calls for the decisions of the NCIP in the exercise of its quasi-judicial functions shall be
recognition and application of customary laws to the resolution of issues appealable to the Court of Appeals, [if !supportFootnotes][187][endif] like those of the
involving members of indigenous peoples. This law admits the operation of National Labor Relations Commission (NLRC) and the Securities and Exchange
customary laws in the settling of disputes if such are ordinarily used in Commission (SEC). Nevertheless, the NCIP, although independent to a certain
barangays where majority of the inhabitants are members of indigenous degree, was placed by Congress under the office of the President and, as
peoples.[if !supportFootnotes][181][endif] such, is still subject to the Presidents power of control and supervision granted
B. Section 1, Part II, Rule VII of the Implementing Rules of IPRA does not under Section 17, Article VII of the Constitution [if !supportFootnotes][188][endif] with
infringe upon the Presidents power of control over the Executive Department. respect to its performance of administrative functions, such as the following:
The second corollary issue is whether the Implementing Rules of IPRA (1) the NCIP must secure the Presidents approval in obtaining loans to finance
violate Section 17, Article VII of the Constitution, which provides that: its projects;[if !supportFootnotes][189][endif] (2) it must obtain the Presidents approval for
The President shall have control of all the executive departments, bureaus, any negotiation for funds and for the acceptance of gifts and/or properties in
and offices. He shall ensure that the laws be faithfully executed. whatever from and from whatever source;[if !supportFootnotes][190][endif] (3) the NCIP
The assailed provision of the Implementing Rules provides: shall submit annual reports of its operations and achievements to the
Rule VII. The National Commission on Indigenous Peoples (NCIP) President, and advise the latter on all matters relating to the indigenous
xxx peoples;[if !supportFootnotes][191][endif] and (4) it shall exercise such other powers as may
Part II: NCIP as an Independent Agency Under the Office of the President be directed by the President.[if !supportFootnotes][192][endif] The President is also given
Section 1. The NCIP is the primary agency of government for the formulation the power to appoint the Commissioners of the NCIP [if !supportFootnotes][193][endif] as
and implementation of policies, plans and programs to recognize, promote well as to remove them from office for cause motu proprio or upon the
and protect the rights and well-being of indigenous peoples. It shall be an recommendation of any indigenous community. [if !supportFootnotes][194][endif]
independent agency under the Office of the President. As such, the To recapitulate:
administrative relationship of the NCIP to the Office of the President (1) The provisions of the IPRA (specifically Sections 3, paragraphs (a)

38
and (b), 5, 6, 7, and 8) affirming the ownership by the indigenous peoples of Republic of the Philippines
their ancestral lands and domains by virtue of native title do not diminish the SUPREME COURT
States ownership of lands of the public domain, because said ancestral lands Manila
and domains are considered as private land, and never to have been part of EN BANC
the public domain, following the doctrine laid down in Cario vs. Insular G.R. No. 5246 September 16, 1910
Government;[if !supportFootnotes][195][endif] MANUELA GREY ALBA, ET AL., petitioners-appellants,
(2) The constitutional provision vesting ownership over minerals, vs.
mineral lands and other natural resources in the State is not violated by ANACLETO R. DE LA CRUZ, objector-appellee.
Sections 3, 5, 7, 56, 57, 58 and 59 of the IPRA which grant certain rights to the Ramon Salinas, for appellants. Aniceto G. Reyes, for appellee.
indigenous peoples over the natural resources found within the ancestral TRENT, J.:
domains, e.g., to benefit from and share in the profits from the allocation and These petitioners, Manuela, Jose, Juan, and Francisco, surnamed Grey y Alba,
utilization of the same, as well as priority rights in the harvesting, extraction, are the only heirs of Doa Segunda Alba Clemente and Honorato Grey,
development or exploitation thereof. The State retains full control over the deceased. Remedios Grey y Alba, a sister of the petitioners, was married on
exploration, development and utilization of natural resources even with the the 21st day of March, 1903, to Vicente Reyes and died on the 13th of July,
grant of said rights to the indigenous peoples, through the imposition of 1905, without leaving any heirs except her husband. The four petitioners, as
requirements and conditions for the utilization of natural resources under coowners, sought to have registered the following-described property:
existing laws, such as the Small-Scale Mining Act of 1991 [if !supportFootnotes][196] A parcel of land situated in the barrio of Talampas, municipality of Baliuag,
[endif]
and the Philippine Mining Act of 1995. [if !supportFootnotes][197][endif] Moreover, the Province of Bulacan, upon which are situated three houses and one camarin of
rights granted to indigenous peoples for the utilization of natural resources light material, having a superficial area of 52 hectares, 51 ares, and 22
within their ancestral domains merely amplify what has been earlier granted centares; bounded on the north by the highway (calzada) of Talampas and the
to them under the aforesaid laws; lands of Rita Ruiz Mateo; on the east by the lands of the said Rita Ruiz Mateo,
(3) While the IPRA recognizes the rights of indigenous peoples with Hermenegildo Prado, Policarpo de Jesus, and a stream called Sapang Buslut;
regard to their ancestral lands and domains, it also protects the vested rights on the south by the same stream and the lands of the capellania; and on the
of persons, whether indigenous or non-indigenous peoples, who may have west by the stream called Sapang Buslut, and the lands of Vicente de la Cruz,
acquired rights of ownership lands or rights to explore and exploit natural Jose Camacho and Domingo Ruiz Mateo.
resources within the ancestral lands and domains;[if !supportFootnotes][198][endif] This parcel of agricultural land is used for the raising of rice and sugar cane
(4) The Due Process Clause of the Constitution is not violated by the and is assessed at $1,000 United States currency. The petition, which was filed
provisions (Sections 40, 51-54, 62, 63, 65 and 66) of the IPRA which, among on the 18th of December, 1906, was accompanied by a plan and technical
others, establish the composition of the NCIP, and prescribe the application of description of the above-described parcel of land.
customary law in certain disputes involving indigenous peoples. The fact the After hearing the proofs presented, the court entered, on the 12th of February,
NCIP is composed wholly of indigenous peoples does not mean that it is 1908, a decree in accordance with the provisions of paragraph 6 of section 54
incapable of being impartial. Moreover, the use of customary laws is of Act No. 926, directing that the land described in the petitioner be registered
sanctioned by paragraph 2, Section 5 of Article XII of the Constitution; and in the names of the four petitioners, as coowners, subject to the usufructuary
(5) The provision of the Implementing Rules characterizing the NCIP as right of Vicente Reyes, widower of Remedios Grey.
an independent agency under the Office of the President does not infringe On the 16th of June, 1908, Anacleto Ratilla de la Cruz filed a motion in the
upon the Presidents power of control under Section 17, Article VII of the Court of Land Registration asking for a revision of the case, including the
Constitution, since said provision as well as Section 40 of the IPRA expressly decision, upon the ground that he is the absolute owner of the two parcels of
places the NCIP under the Office of the President, and therefore under the land which are described in said motion, and which, according to his
Presidents control and supervision with respect to its administrative functions. allegations, are included in the lands decreed to the petitioners. He alleged
However, insofar as the decisions of the NCIP in the exercise of its quasi- that the decree of February 12, 1908, was obtained maliciously and
judicial powers are concerned, the same are reviewable by the Court of fraudulently by the petitioners, thereby depriving him of said two parcels of
Appeals, like those of the NLRC and the SEC. land. He further alleged that he was the absolute owner of the two parcels of
In view of the foregoing, I vote to DISMISS the petition. land, having inherited them from his father, Baldomero R. de la Cruz, who had
a state grant for the same. He therefore asked, under the provisions of section
Owen Lynch, Native Title, Private Right and Tribal Land Law; and Introductory 38 of the Land Registration Act (No. 496), a revision of the case, and that the
Survey 57 Philippine Law Journal 268 (1982) (Separate) said decree be modified so as to exclude the two parcels of land described in
said motion. The Land Court upon this motion reopened the case, and after
hearing the additional evidence presented by both parties, rendered, on the
23rd of November, 1908, its decision modifying the former decree by
B. The Torrent System excluding from the same the two parcels of land claimed by Anacleto Ratilla
ALBA v. DELA CRUZ G.R. No. 5246 September 16, 1910 J. Trent de la Cruz. From this decision and judgment the petitioners appealed and now

39
insist, first, that the trial court erred in reopening the case and modifying its petitioners and he father of the appellee the land is not described. In the
decree dated the 12th of February, 1908, for the reason that said decree was rental contract between Jose Grey, one of the petitioners, and Estanislao R. de
not obtained by means of fraud; and, second, that the court erred in holding la Cruz, brother of the appellee, the two small parcels of land in question are
that the two parcels of land described in the appellee's motion are not their included, according to the description given therein. This was found to be true
property. by the court below, but the said court held that as this contract was made by
It was agreed by counsel that the two small parcels now in dispute forma part Estanislao R. de la Cruz it was not binding upon Anacleto R. de la Cruz, the
of the land described in the petition and were included in the decree of appellee.
February 12, 1908, and that the petitioners are the owners of the remainder of The two small parcels of land in question were purchased by the parents of
the land described in the said decree. the petitioners in 1864, as is evidenced by the public document of purchase
The petitioners inherited this land from their parents, who acquired the same, and sale of that year. The same two parcels of land are included in the state
including the two small parcels in question, by purchase, as is evidenced by a grant issued in favor of Baldomero Ratilla de la Cruz in 1895. This grant was
public document dated the 26th of November, 1864, duly executed before obtained after the death of the petitioners' parents and while they were
Francisco Iriarte, alcalde mayor and judge of the Court of First Instance of the minors. So it is clear that the petitioners honestly believed that the appellee
Province of Bulacan. was occupying the said parcels as their lessee at the time they presented
Baldomero R. de la Cruz, father of the appellee, obtained in march, 1895, a their application for registration. They did not act in bad faith, nor with any
state grant for several parcels of land, including the two parcels in question. fraudulent intent, when they omitted to include in their application the name
This grant was duly inscribed in the old register of property in Bulacan on the of the appellee as one of the occupants of the land. They believed that it was
6th of April of the same year. not necessary nor required that they include in their application the names of
It is admitted that at the time the appellants presented their petition in this their tenants. Under these circumstances, did the court below commit an error
case the appellee was occupying the two parcels of land now in question. It is in reopening this case in June, 1908, after its decree had been entered in
also admitted that the name of the appellee does not appear in the said February of the same year?
petition as an occupant of the said two parcels. The petitioners insist that the The application for the registration is to be in writing, signed and sworn to by
appellee was occupying these parcels as their tenant and for this reason they the applicant, or by some person duly authorized in his behalf. It is to contain
did not include his name in their petition, as an occupant, while the appellee an accurate description of the land. It shall contain the name in full and the
contends that he was occupying the said parcels as the absolute owner under address of the applicant, and also the names and addresses of all occupants
the estate grant by inheritance. of land and of all adjoining owners, if known; and, if not known, it shall state
The court below held that the failure on the part of the petitioners to include what search has been made to find them. In the form of notice given by
the name of the appellee in their petition, as an occupant of these two parcels statute, which shall be sworn to, the applicant is required to state and set
of land, was a violation of section 21 of Act No. 496, and that this constituted forth clearly all mortgages or encumbrances affecting said land, if any, the
fraud within the meaning of section 38 of said Land Registration Act. The trial rights and interests, legal or equitable, in the possession, remainder,
court further held that the grant from the estate should prevail over the public reversion, or expectancy of all persons, with their names in full, together with
document of purchase of 1864. their place of residence and post office addresses. Upon receipt of the
The mother of the petitioners died on November 15, 1881; their father died application the clerk shall cause notice of the filling to be published twice in
prior to that time. Manuela, the oldest of the petitioners, was about six years the Official Gazette. This published notice shall be directed to all persons
of age when their mother died. So these children were minors when the father appearing to have an interest in the land sought to be registered and to the
of the appellee obtained the estate grant. adjoining owners, and also "to all whom it may concern." In addition to the
On the 13th of June, 1882, Jose Grey, uncle and representative of the notice in the Official Gazette the Land Court shall, within seven days after said
petitioners, who were then minors, rented the land owned by the petitioners' publication, cause a copy of the notice, in Spanish, to be mailed by the clerk
deceased parents to one Irineo Jose for a period of three years. On the 23d of to every person named in the application whose address is known; to cause a
March, 1895, the said Jose Grey, as the representative of the petitioners, duly attested copy of the notice, in Spanish, to be posted in a conspicuous
rented the same land for a period of six years to Baldomero R. de la Cruz, place on every parcel of land included in the application, and in a conspicuous
father of the appellee. This rental contract was duly executed in writing. This place on the chief municipal building of the town in which the land is situated.
land was cultivated during these six years by Baldomero R. de la Cruz and his The court may also cause other or further notice of the application to be given
children, one of whom is the appellee. On the 14th of December, 1905, Jose in such manner and to such persons as it may deem proper. The certificate of
Grey, for himself and the other petitioners, rented the same land to Estanislao the clerk that he has served the notice as directed by the court by publication
R. de la Cruz for a period of two years. Estanislao de la Cruz on entering into or mailing shall be conclusive proof of such service. Within the time allowed in
this rental contract with Jose Grey did so for himself and his brothers, one of the notices, if no person appears and answers, the court may at once, upon
whom is the appellee. While the appellee admits that his father and brother motion of the applicant, no reason to the contrary appearing, order a general
entered into these rental contracts and did, in fact, cultivate the petitioners' default. By the description in the published notice "to all whom it may
land, nevertheless he insists that the two small parcels in question were not concern," and by express provisions of law "all the word are made parties
included in these contracts. In the rental contract between the uncle of the defendant and shall be concluded by the default an order." If the court, after

40
hearing, finds that the applicant has title, as stated in his application, a decree certificates of title, it has been held in some cases that the "fraud" there
or registration shall be entered. mentioned means actual or moral fraud, not merely constructive or legal
Every decree of registration shall bind the land and quiet title thereto, subject fraud. In other cases "fraud" has been said to include constructive, legal, and
only to the exceptions stated in the following section. It shall be conclusive every kind of fraud. In other cases, against, knowledge of other persons' right,
upon and against all persons, including the Insular Government, and all the and the deliberate acquisition of registered title in the face of such knowledge,
branches thereof, whether mentioned by name in the application, notice, or has been held to be "fraud" which rendered voidable the certificates of title so
citation, or included in the general description "to all whom it may concern." obtained; and voluntary ignorance is, for this purpose, the same as
Such decree shall not be opened by reason of the absence, infancy, or other knowledge. But in none of these three classes of cases was there absent the
disability of any person affected thereby, nor by any proceedings in any court element of intention to deprive another of just rights, which constitutes the
for reversing judgments or decrees; subject, however, to the right of any essential characteristics of actual as distinguished from legal-fraud. (Id., p.
person deprived of land or of any estate or interest therein by decree of 835, and cases cited in notes Nos. 85, 86, 87, 88, and 89 at bottom of pages
registration obtained by fraud to file in the Court of Land Registration a 835 and 836.)
petition for review within one year. . . . (Sec. 38 of Act No. 496.) By "fraud" is meant actual fraud-dishonesty of some sort. (Judgment of Privy
The appellee is not included in any of the exceptions named in section 38 Council in Assets Co. vs. Mere Roihi, and Assets Co. vs. Panapa Waihopi,
referred to above. decided in March, 1905, cited by Hogg in his Supplementary Addendum to his
It will be seen that the applicant is required to mention not only the work on Australian Torrens System, supra.) The same meaning should be
outstanding interest which he admits but also all claims of interest, though given to the word "fraud" used in section 38 of our statutes (Act No. 496).
denied by him. By express provision of law the world are made parties The question as to whether any particular transaction shows fraud, within the
defendant by the description in the notice "to all whom it may concern." meaning of the word as used in our statutes, will in each case be a question of
Although the appellee, occupying the two small parcels of land in question fact. We will not attempt to say what acts would constitutes this kind of fraud
under the circumstances as we have set forth, was not served with notice, he in other cases. This must be determined from the fact an circumstances in
was made a party defendant by publication; and the entering of a decree on each particular case. The only question we are called upon to determine, and
the 12th of February, 1908, must be held to be conclusive against all persons, have determined, is whether or not, under the facts and circumstances in this
including the appellee, whether his (appellee's) name is mentioned in the case, the petitioners did obtain the decree of February 12, 1908, by means of
application, notice, or citation. fraud.
The said decree of February 12, 1908, should not have been opened on It might be urged that the appellee has been deprived of his property without
account of the absence, infancy, or other disability of any person affected due process of law, in violation of section 5 of the Act of Congress of July 1,
thereby, and could have been opened only on the ground that the said decree 1902, known as the Philippine Bill," which provides "that no law shall be
had been obtained by fraud. That decree was not obtained by fraud on the enacted in the said Islands which shall deprive any person of life, liberty, or
part of the applicants, inasmuch as they honestly believed that the appellee property without due process of law."
was occupying these two small parcels of land as their tenant. One of the The Land Registration Act requires that all occupants be named in the petition
petitioner went upon the premises with the surveyor when the original plan and given notice by registered mail. This did not do the appellee any good, as
was made. he was not notified; but he was made a party defendant, as we have said, by
Proof of constructive fraud is not sufficient to authorize the Court of Land means of the publication "to all whom it may concern." If this section of the
Registration to reopen a case and modify its decree. Specific, intentional acts Act is to be upheld this must be declared to be due process of law.
to deceive and deprive anther of his right, or in some manner injure him, must Before examining the validity of this part of the Act it might be well to note
be alleged and proved; that is, there must be actual or positive fraud as the history and purpose of what is known as the "Torrens Land Registration
distinguished from constructive fraud. System." This system was introduced in South Australia by Sir Robert Torrens
The question as to the meaning of the word "fraud" in the Australian statutes in 1857 and was there worked out in its practicable form.
has been frequently raised. Two distinctions have been noted by the The main principle of registration is to make registered titles indefeasible. As
Australian courts; the first is the distinction between the meaning of the word we have said, upon the presentation in the Court of Land Registration of an
"fraud" in the sections relating to the conclusive effect of certificates of title, application for the registration of the title to lands, under this system, the
and its meaning in the sections relating to the protection of bona fide theory of the law is that all occupants, adjoining owners, adverse claimants,
purchasers from registered proprietors. The second is the distinction between and other interested persons are notified of the proceedings, and have have a
"legal," "equitable," or "constructive" fraud, and "actual" or "moral" fraud. In right to appear in opposition to such application. In other words, the
none of the groups of the sections of the Australian statutes relating to the proceeding is against the whole word. This system was evidently considered
conclusive effect of certificates of title, and in which fraud is referred to, is by the Legislature to be a public project when it passed Act No. 496. The
there any express indication of the meaning of "fraud," with the sole exception interest of the community at large was considered to be preferred to that of
of that of the South Australian group. (Hogg on Australian Torrens System, p. private individuals.
834.) At the close of this nineteenth century, all civilized nations are coming to
With regard to decisions on the sections relating to the conclusive effect of registration of title to land, because immovable property is becoming more

41
and more a matter of commercial dealing, and there can be no trade without People vs. Simon, 176 Ill., 165; Tyler vs. Judges, 175 Mass., 71.)
security. (Dumas's Lectures, p. 23.) Act No. 496 of the Philippine Commission, known as the "Land Registration
The registered proprietor will no longer have reasons to fear that he may Act," was copied substantially from the Massachussetts law of 1898.
evicted because his vendor had, unknown to him, already sold the and to a The Illinois and Massachusetts statutes were upheld by the supreme courts of
third person. . . The registered proprietor may feel himself protected against those States.
any defect in his vendor's title. (Id., p. 21.) It is not enough to show a procedure to be unconstitutional to say that we
The following summary of benefits of the system of registration of titles, made never heard of it before. (Tyler vs. Judges, supra; Hurtado vs. California, 110 U.
by Sir Robert Torrens, has been fully justified in its use: S., 516.)
First. It has substituted security for insecurity. Looked at either from the point of view of history or of the necessary
Second. It has reduced the costs of conveyances from pounds to shillings, and requirements of justice, a proceeding in rem dealing with a tangible res may
the time occupied from months to days. be instituted and carried to judgment without personal service upon claimants
Third. It has exchanged brevity and clearness for obscurity and verbiage. within the State or notice by name to those outside of it, and not encounter
Fourth. It has so simplified ordinary dealings that he who has mastered the any provision of either constitution. Jurisdiction is secured by the power of the
"three R's" can transact his own conveyancing. court over the res. As we have said, such a proceeding would be impossible,
Fifth. It affords protection against fraud. were this not so, for it hardly would do to make a distinction between the
Sixth. It has restored to their just value many estates held under good holding constitutional rights of claimants who were known and those who were not
titles, but depreciated in consequence of some blur or technical defect, and known to the plaintiff, when the proceeding is to bar all. (Tyler vs. Judges,
has barred the reoccurrence of any similar faults. (Sheldon on Land supra.)
Registration, pp. 75, 76.) This same doctrine is annunciated in Pennoyer vs. Neff (95 U. S., 714); The
The boldest effort to grapple with the problem of simplification of title to land Mary (9 Cranch, 126); Mankin vs. Chandler (2 Brock., 125); Brown vs. Levee
was made by Mr. (afterwards Sir Robert) Torrens, a layman, in South Australia Commission (50 Miss., 468); 2 Freeman, Judgments, 4th ed., secs. 606, 611.
in 1857. . . . In the Torrens system title by registration takes the place of "title If the technical object of the suit is to establish a claim against some particular
by deeds" of the system under the "general" law. A sale of land, for example, person, with a judgment which generally, in theory at least, binds his body, or
is effected by a registered transfer, upon which a certificate of title is issued. to bar some individual claim or objection, so that only certain persons are
The certificate is guaranteed by statute, and, with certain exceptions, entitled to be heard in defense, the action is in personam, although it may
constitutes indefeasible title to the land mentioned therein. Under the old concern the right to or possession of a tangible thing. If, on the other hand,
system the same sale would be effected by a conveyance, depending for its the object is to bar indifferently all who might be minded to make an objection
validity, apart from intrinsic flaws, on the correctness of a long series of prior of any sort against the right sought to be established, and if anyone in the
deeds, wills, etc. . . . The object of the Torrens system, them, is to do away world has a right to be heard on the strenght of alleging facts which, if true,
with the delay, uncertainty, and expense of the old conveyancing system. show an inconsistent interest, the proceeding is in rem. (Tyler vs. Judges,
(Duffy & Eagleson on The Transfer of Land Act, 1890, pp. 2, 3, 5, 7.) supra.)
By "Torrens" system generally are meant those systems of registration of In the case of Hamilton vs. Brown (161 U. S., 256) a judgment of escheat was
transactions with interest in land whose declared object . . . is, under held conclusive upon persons notified by advertisement to all persons
governmental authority, to establish and certify to the ownership of an interested. In this jurisdiction, by the provisions of the Code of Civil Procedure,
absolute and indefeasible title to realty, and to simplify its transfer. (Hogg on Act No. 190, a decree allowing or disallowing a will binds everybody, although
Australian Torrens system, supra, pp. 1, 2.) the only notice of the proceedings given is by general notice to all persons
Compensation for errors from assurance funds is provided in all countries in interested.
which the Torrens system has been enacted. Cases of error no doubt will The supreme court Massachusetts, in the case of Tyler vs. Judges (supra), did
always occur. The percentage of errors, as compared with the number of not rest its judgment as to the conclusive effect of the decree upon the ground
registered dealings in Australia, is very small. In New South Wales there were, that the State has absolute power to determine the persons to whom a man's
in 1889, 209, 894 registered dealings, the average risk of error being only 2 property shall go at his death, but upon the characteristics of a proceeding in
cents for each dealing. In Queensland the risk of error was only 1 cents, the rem. So we conclude that the proceedings had in the case at bar, under all the
number of registered dealings being 233,309. In Tasmania and in Western facts and circumstances, especially the absolute lack on the part of the
Australia not a cent was paid for compensation for errors during the whole petitioners of any dishonest intent to deprive the appellee of any right, or in
time of operation, (Dumas's Lectures, supra, p. 96.) This system has been any way injure him, constitute due process of law.
adopted in various countries of the civilized world, including some of the As to whether or not the appellee can succesfully maintain an action under
States of the American Union, and practical experience has demonstrated that the provisions of sections 101 and 102 of the Land Registration Act (secs.
it has been successful as a public project. 2365, 2366, Compilation) we do not decide.
The validity of some of the provisions of the statutes adopting the Torrens For these reasons we are of the opinion, and so hold, that the judgment
system has been the subject of judicial decision in the courts of the United appealed from should be, and the same is hereby reversed and judgment
States. (People vs. Chase, 165 Ill., 527; State vs. Guilbert, 56 Ohio St., 575; entered in favor of the petitioners in conformity with the decree of the lower

42
court of February 12, 1908, without special ruling as to costs. It is so ordered. opposed the registration of that part of the lot on which the wall was situate
Arellano, C.J., Torres, Johnson and Moreland, JJ., concur. they had lost it, even though it had been theretofore registered in their name.
Granting that theory to be correct one, and granting even that the wall and
LEGARDA v. SALEEBY G.R. No. L-8936 October 2, 1915 J. Johnson the land occupied by it, in fact, belonged to the defendant and his
predecessors, then the same theory should be applied to the defendant
Republic of the Philippines himself. Applying that theory to him, he had already lost whatever right he
SUPREME COURT had therein, by permitting the plaintiffs to have the same registered in their
Manila name, more than six years before. Having thus lost hid right, may he be
EN BANC permitted to regain it by simply including it in a petition for registration? The
G.R. No. L-8936 October 2, 1915 plaintiffs having secured the registration of their lot, including the wall, were
CONSUELO LEGARDA, with her husband MAURO PRIETO, plaintiffs- they obliged to constantly be on the alert and to watch all the proceedings in
appellants, the land court to see that some one else was not having all, or a portion of the
vs. same, registered? If that question is to be answered in the affirmative, then
N.M. SALEEBY, defendant-appellee. the whole scheme and purpose of the torrens system of land registration must
Singson, Ledesma and Lim for appellants.D.R. Williams for appellee. fail. The real purpose of that system is to quiet title to land; to put a stop
forever to any question of the legality of the title, except claims which were
JOHNSON, J.: noted at the time of registration, in the certificate, or which may arise
From the record the following facts appear: subsequent thereto. That being the purpose of the law, it would seem that
First. That the plaintiffs and the defendant occupy, as owners, adjoining lots in once a title is registered the owner may rest secure, without the necessity of
the district of Ermita in the city of Manila. waiting in the portals of the court, or sitting in the "mirador de su casa," to
Second. That there exists and has existed a number of years a stone wall avoid the possibility of losing his land. Of course, it can not be denied that the
between the said lots. Said wall is located on the lot of the plaintiffs. proceeding for the registration of land under the torrens system is judicial
Third. That the plaintiffs, on the 2d day of March, 1906, presented a petition in (Escueta vs. .Director of Lands, 16 Phil. Rep., 482). It is clothed with all the
the Court of Land Registration for the registration of their lot. After a forms of an action and the result is final and binding upon all the world. It is an
consideration of said petition the court, on the 25th day of October, 1906, action in rem. (Escueta vs. Director of Lands (supra); Grey Alba vs. De la Cruz,
decreed that the title of the plaintiffs should be registered and issued to them 17 Phil. rep., 49 Roxas vs. Enriquez, 29 Phil. Rep., 31; Tyler vs. Judges, 175
the original certificate provided for under the torrens system. Said registration Mass., 51 American Land Co. vs. Zeiss, 219 U.S., 47.)
and certificate included the wall. While the proceeding is judicial, it involves more in its consequences than
Fourth. Later the predecessor of the defendant presented a petition in the does an ordinary action. All the world are parties, including the government.
Court of Land Registration for the registration of the lot now occupied by him. After the registration is complete and final and there exists no fraud, there are
On the 25th day of March, 1912, the court decreed the registration of said title no innocent third parties who may claim an interest. The rights of all the world
and issued the original certificate provided for under the torrens system. The are foreclosed by the decree of registration. The government itself assumes
description of the lot given in the petition of the defendant also included said the burden of giving notice to all parties. To permit persons who are parties in
wall. the registration proceeding (and they are all the world) to again litigate the
Fifth. Several months later (the 13th day of December, 1912) the plaintiffs same questions, and to again cast doubt upon the validity of the registered
discovered that the wall which had been included in the certificate granted to title, would destroy the very purpose and intent of the law. The registration,
them had also been included in the certificate granted to the defendant .They under the torrens system, does not give the owner any better title than he
immediately presented a petition in the Court of Land Registration for an had. If he does not already have a perfect title, he can not have it registered.
adjustment and correction of the error committed by including said wall in the Fee simple titles only may be registered. The certificate of registration
registered title of each of said parties. The lower court however, without accumulates in open document a precise and correct statement of the exact
notice to the defendant, denied said petition upon the theory that, during the status of the fee held by its owner. The certificate, in the absence of fraud, is
pendency of the petition for the registration of the defendant's land, they the evidence of title and shows exactly the real interest of its owner. The title
failed to make any objection to the registration of said lot, including the wall, once registered, with very few exceptions, should not thereafter be impugned,
in the name of the defendant. altered, changed, modified, enlarged, or diminished, except in some direct
Sixth. That the land occupied by t he wall is registered in the name of each of proceeding permitted by law. Otherwise all security in registered titles would
the owners of the adjoining lots. The wall is not a joint wall. be lost. A registered title can not be altered, modified, enlarged, or diminished
Under these facts, who is the owner of the wall and the land occupied by it? in a collateral proceeding and not even by a direct proceeding, after the lapse
The decision of the lower court is based upon the theory that the action for of the period prescribed by law.
the registration of the lot of the defendant was a judicial proceeding and that For the difficulty involved in the present case the Act (No. 496) providing for
the judgment or decree was binding upon all parties who did not appear and the registration of titles under the torrens system affords us no remedy. There
oppose it. In other words, by reason of the fact that the plaintiffs had not is no provision in said Act giving the parties relief under conditions like the

43
present. There is nothing in the Act which indicates who should be the owner any person deprived of land or of any estate or interest therein by decree of
of land which has been registered in the name of two different persons. registration obtained by fraud to file in the Court of Land Registration a
The rule, we think, is well settled that the decree ordering the registration of a petition for review within one year after entry of the decree (of registration),
particular parcel of land is a bar to future litigation over the same between the provided no innocent purchaser for value has acquired an interest.
same parties .In view of the fact that all the world are parties, it must follow It will be noted, from said section, that the "decree of registration" shall not be
that future litigation over the title is forever barred; there can be no persons opened, for any reason, in any court, except for fraud, and not even for fraud,
who are not parties to the action. This, we think, is the rule, except as to rights after the lapse of one year. If then the decree of registration can not be
which are noted in the certificate or which arise subsequently, and with opened for any reason, except for fraud, in a direct proceeding for that
certain other exceptions which need not be dismissed at present. A title once purpose, may such decree be opened or set aside in a collateral proceeding
registered can not be defeated, even by an adverse, open, and notorious by including a portion of the land in a subsequent certificate or decree of
possession. Registered title under the torrens system can not be defeated by registration? We do not believe the law contemplated that a person could be
prescription (section 46, Act No. 496). The title, once registered, is notice to deprived of his registered title in that way.
the world. All persons must take notice. No one can plead ignorance of the We have in this jurisdiction a general statutory provision which governs the
registration. right of the ownership of land when the same is registered in the ordinary
The question, who is the owner of land registered in the name of two different registry in the name of two persons. Article 1473 of the Civil Code provides,
persons, has been presented to the courts in other jurisdictions. In some among other things, that when one piece of real property had been sold to
jurisdictions, where the "torrens" system has been adopted, the difficulty has two different persons it shall belong to the person acquiring it, who first
been settled by express statutory provision. In others it has been settled by inscribes it in the registry. This rule, of course, presupposes that each of the
the courts. Hogg, in his excellent discussion of the "Australian Torrens vendees or purchasers has acquired title to the land. The real ownership in
System," at page 823, says: "The general rule is that in the case of two such a case depends upon priority of registration. While we do not now decide
certificates of title, purporting to include the same land, the earlier in date that the general provisions of the Civil Code are applicable to the Land
prevails, whether the land comprised in the latter certificate be wholly, or only Registration Act, even though we see no objection thereto, yet we think, in the
in part, comprised in the earlier certificate. (Oelkers vs. Merry, 2 Q.S.C.R., 193; absence of other express provisions, they should have a persuasive influence
Miller vs. Davy, 7 N.Z.R., 155; Lloyd vs. Myfield, 7 A.L.T. (V.) 48; Stevens vs. in adopting a rule for governing the effect of a double registration under said
Williams, 12 V.L. R., 152; Register of Titles, vs. Esperance Land Co., 1 W.A.R., Act. Adopting the rule which we believe to be more in consonance with the
118.)" Hogg adds however that, "if it can be very clearly ascertained by the purposes and the real intent of the torrens system, we are of the opinion and
ordinary rules of construction relating to written documents, that the inclusion so decree that in case land has been registered under the Land Registration
of the land in the certificate of title of prior date is a mistake, the mistake may Act in the name of two different persons, the earlier in date shall prevail.
be rectified by holding the latter of the two certificates of title to be In reaching the above conclusion, we have not overlooked the forceful
conclusive." (See Hogg on the "Australian torrens System," supra, and cases argument of the appellee. He says, among other things; "When Prieto et al.
cited. See also the excellent work of Niblack in his "Analysis of the Torrens were served with notice of the application of Teus (the predecessor of the
System," page 99.) Niblack, in discussing the general question, said: "Where defendant) they became defendants in a proceeding wherein he, Teus, was
two certificates purport to include the same land the earlier in date prevails. ... seeking to foreclose their right, and that of orders, to the parcel of land
In successive registrations, where more than one certificate is issued in described in his application. Through their failure to appear and contest his
respect of a particular estate or interest in land, the person claiming under the right thereto, and the subsequent entry of a default judgment against them,
prior certificates is entitled to the estate or interest; and that person is they became irrevocably bound by the decree adjudicating such land to Teus.
deemed to hold under the prior certificate who is the holder of, or whose claim They had their day in court and can not set up their own omission as ground
is derived directly or indirectly from the person who was the holder of the for impugning the validity of a judgment duly entered by a court of competent
earliest certificate issued in respect thereof. While the acts in this country do jurisdiction. To decide otherwise would be to hold that lands with torrens titles
not expressly cover the case of the issue of two certificates for the same land, are above the law and beyond the jurisdiction of the courts".
they provide that a registered owner shall hold the title, and the effect of this As was said above, the primary and fundamental purpose of the torrens
undoubtedly is that where two certificates purport to include the same system is to quiet title. If the holder of a certificate cannot rest secure in this
registered land, the holder of the earlier one continues to hold the title" (p. registered title then the purpose of the law is defeated. If those dealing with
237). registered land cannot rely upon the certificate, then nothing has been gained
Section 38 of Act No. 496, provides that; "It (the decree of registration) shall by the registration and the expense incurred thereby has been in vain. If the
be conclusive upon and against all persons, including the Insular Government holder may lose a strip of his registered land by the method adopted in the
and all the branches thereof, whether mentioned by name in the application, present case, he may lose it all. Suppose within the six years which elapsed
notice, or citation, or included in the general description "To all whom it may after the plaintiff had secured their title, they had mortgaged or sold their
concern." Such decree shall not be opened by reason of the absence, infancy, right, what would be the position or right of the mortgagee or vendee? That
or other disability of any person affected thereby, nor by any proceeding in mistakes are bound to occur cannot be denied, and sometimes the damage
any court for reversing judgments or decrees; subject, however, to the right of done thereby is irreparable. It is the duty of the courts to adjust the rights of

44
the parties under such circumstances so as to minimize such damages, taking or interest of the owner of the first original certificate, his heirs, assigns, or
into consideration al of the conditions and the diligence of the respective vendee? The first original certificate is recorded in the public registry. It is
parties to avoid them. In the present case, the appellee was the first negligent never issued until it is recorded. The record notice to all the world. All persons
(granting that he was the real owner, and if he was not the real owner he can are charged with the knowledge of what it contains. All persons dealing with
not complain) in not opposing the registration in the name of the appellants. the land so recorded, or any portion of it, must be charged with notice of
He was a party-defendant in an action for the registration of the lot in whatever it contains. The purchaser is charged with notice of every fact shown
question, in the name of the appellants, in 1906. "Through his failure to by the record and is presumed to know every fact which the record
appear and to oppose such registration, and the subsequent entry of a default discloses .This rule is so well established that it is scarcely necessary to cite
judgment against him, he became irrevocably bound by the decree authorities in its support (Northwestern National Bank vs. Freeman, 171 U.S.,
adjudicating such land to the appellants. He had his day in court and should 620, 629; Delvin on Real Estate, sections 710, 710 [a]).
not be permitted to set up his own omissions as the ground for impugning the When a conveyance has been properly recorded such record is constructive
validity of a judgment duly entered by a court of competent jurisdiction." notice of its contents and all interests, legal and equitable, included therein.
Granting that he was the owner of the land upon which the wall is located, his (Grandin vs. Anderson, 15 Ohio State, 286, 289; Orvis vs. Newell, 17 Conn.,
failure to oppose the registration of the same in the name of the appellants, in 97; Buchanan vs. Intentional Bank, 78 Ill., 500; Youngs vs. Wilson, 27 N.Y., 351;
the absence of fraud, forever closes his mouth against impugning the validity McCabe vs. Grey, 20 Cal., 509; Montefiore vs. Browne, 7 House of Lords Cases,
of that judgment. There is no more reason why the doctrine invoked by the 341.)
appellee should be applied to the appellants than to him. Under the rule of notice, it is presumed that the purchaser has examined
We have decided, in case of double registration under the Land Registration every instrument of record affecting the title. Such presumption is irrebutable.
Act, that the owner of the earliest certificate is the owner of the land. That is He is charged with notice of every fact shown by the record and is presumed
the rule between original parties. May this rule be applied to successive to know every fact which an examination of the record would have disclosed.
vendees of the owners of such certificates? Suppose that one or the other of This presumption cannot be overcome by proof of innocence or good faith.
the parties, before the error is discovered, transfers his original certificate to Otherwise the very purpose and object of the law requiring a record would be
an "innocent purchaser." The general rule is that the vendee of land has no destroyed. Such presumption cannot be defeated by proof of want of
greater right, title, or interest than his vendor; that he acquires the right which knowledge of what the record contains any more than one may be permitted
his vendor had, only. Under that rule the vendee of the earlier certificate to show that he was ignorant of the provisions of the law. The rule that all
would be the owner as against the vendee of the owner of the later certificate. persons must take notice of the facts which the public record contains is a rule
We find statutory provisions which, upon first reading, seem to cast some of law. The rule must be absolute. Any variation would lead to endless
doubt upon the rule that the vendee acquires the interest of the vendor only. confusion and useless litigation.
Sections 38, 55, and 112 of Act No. 496 indicate that the vendee may acquire While there is no statutory provision in force here requiring that original deeds
rights and be protected against defenses which the vendor would not. Said of conveyance of real property be recorded, yet there is a rule requiring
sections speak of available rights in favor of third parties which are cut off by mortgages to be recorded. (Arts. 1875 and 606 of the Civil Code.) The record
virtue of the sale of the land to an "innocent purchaser." That is to say, of a mortgage is indispensable to its validity. (Art .1875.) In the face of that
persons who had had a right or interest in land wrongfully included in an statute would the courts allow a mortgage to be valid which had not been
original certificate would be unable to enforce such rights against an recorded, upon the plea of ignorance of the statutory provision, when third
"innocent purchaser," by virtue of the provisions of said sections. In the parties were interested? May a purchaser of land, subsequent to the recorded
present case Teus had his land, including the wall, registered in his name. He mortgage, plead ignorance of its existence, and by reason of such ignorance
subsequently sold the same to the appellee. Is the appellee an "innocent have the land released from such lien? Could a purchaser of land, after the
purchaser," as that phrase is used in said sections? May those who have been recorded mortgage, be relieved from the mortgage lien by the plea that he
deprived of their land by reason of a mistake in the original certificate in favor was a bona fide purchaser? May there be a bona fide purchaser of said land,
of Teus be deprived of their right to the same, by virtue of the sale by him to bona fide in the sense that he had no knowledge of the existence of the
the appellee? Suppose the appellants had sold their lot, including the wall, to mortgage? We believe the rule that all persons must take notice of what the
an "innocent purchaser," would such purchaser be included in the phrase public record contains in just as obligatory upon all persons as the rule that all
"innocent purchaser," as the same is used in said sections? Under these men must know the law; that no one can plead ignorance of the law. The fact
examples there would be two innocent purchasers of the same land, is said that all men know the law is contrary to the presumption. The conduct of men,
sections are to be applied .Which of the two innocent purchasers, if they are at times, shows clearly that they do not know the law. The rule, however, is
both to be regarded as innocent purchasers, should be protected under the mandatory and obligatory, notwithstanding. It would be just as logical to allow
provisions of said sections? These questions indicate the difficulty with which the defense of ignorance of the existence and contents of a public record.
we are met in giving meaning and effect to the phrase "innocent purchaser," In view, therefore, of the foregoing rules of law, may the purchaser of land
in said sections. from the owner of the second original certificate be an "innocent purchaser,"
May the purchaser of land which has been included in a "second original when a part or all of such land had theretofore been registered in the name of
certificate" ever be regarded as an "innocent purchaser," as against the rights another, not the vendor? We are of the opinion that said sections 38, 55, and

45
112 should not be applied to such purchasers. We do not believe that the It would be seen to a just and equitable rule, when two persons have acquired
phrase "innocent purchaser should be applied to such a purchaser. He cannot equal rights in the same thing, to hold that the one who acquired it first and
be regarded as an "innocent purchaser" because of the facts contained in the who has complied with all the requirements of the law should be protected.
record of the first original certificate. The rule should not be applied to the In view of our conclusions, above stated, the judgment of the lower court
purchaser of a parcel of land the vendor of which is not the owner of the should be and is hereby revoked. The record is hereby returned to the court
original certificate, or his successors. He, in nonsense, can be an "innocent now having and exercising the jurisdiction heretofore exercised by the land
purchaser" of the portion of the land included in another earlier original court, with direction to make such orders and decrees in the premises as may
certificate. The rule of notice of what the record contains precludes the idea of correct the error heretofore made in including the land in the second original
innocence. By reason of the prior registry there cannot be an innocent certificate issued in favor of the predecessor of the appellee, as well as in all
purchaser of land included in a prior original certificate and in a name other other duplicate certificates issued.
than that of the vendor, or his successors. In order to minimize the difficulties Without any findings as to costs, it is so ordered.
we think this is the safe rule to establish. We believe the phrase "innocent Arellano, C.J., Torrens, and Araullo, JJ., concur.
purchaser," used in said sections, should be limited only to cases where
unregistered land has been wrongfully included in a certificate under the CAPITAL SUBDIVISION V. PROV. Of Negros Occidental G.R. No. L-16257
torrens system. When land is once brought under the torrens system, the Jan 31, 1963
record of the original certificate and all subsequent transfers thereof is notice
to all the world. That being the rule, could Teus even regarded as the holder in Republic of the Philippines
good fifth of that part of the land included in his certificate of the appellants? SUPREME COURT
We think not. Suppose, for example, that Teus had never had his lot registered Manila
under the torrens system. Suppose he had sold his lot to the appellee and had EN BANC
included in his deed of transfer the very strip of land now in question. Could G.R. No. L-16257 January 31, 1963
his vendee be regarded as an "innocent purchaser" of said strip? Would his CAPITOL SUBDIVISION, INC., plaintiff-appellant,
vendee be an "innocent purchaser" of said strip? Certainly not. The record of vs.
the original certificate of the appellants precludes the possibility. Has the PROVINCE OF NEGROS OCCIDENTAL, defendant-appellee.
appellee gained any right by reason of the registration of the strip of land in San Juan, Africa & Benedicto for plaintiff-appellant.Eduardo P. Arboleda and
the name of his vendor? Applying the rule of notice resulting from the record Jesus S. Rodriguez for defendant-appellee.
of the title of the appellants, the question must be answered in the negative. CONCEPCION, J.:
We are of the opinion that these rules are more in harmony with the purpose Plaintiff, Capitol Subdivision, Inc., seeks to recover from defendant, the
of Act No. 496 than the rule contended for by the appellee. We believe that Province of Negros Occidental, the possession of Lot 378 of the cadastral
the purchaser from the owner of the later certificate, and his successors, survey of Bacolod, Negros Occidental, and a reasonable compensation for the
should be required to resort to his vendor for damages, in case of a mistake use and occupation of said lot by the defendant from November 8, 1935, in
like the present, rather than to molest the holder of the first certificate who addition to attorney's fees and costs. On June 28, 1951, the Court of First
has been guilty of no negligence. The holder of the first original certificate and Instance of Negros Occidental rendered judgment for the plaintiff. On appeal
his successors should be permitted to rest secure in their title, against one taken by the defendant, this judgment was, however, set aside by the
who had acquired rights in conflict therewith and who had full and complete Supreme Court (see G.R. No. L-6204, decided on July 31, 1956), which,
knowledge of their rights. The purchaser of land included in the second likewise, ordered the case remanded to the lower court "for further trial", after
original certificate, by reason of the facts contained in the public record and which another decision was rendered by said court of first instance dismissing
the knowledge with which he is charged and by reason of his negligence, plaintiff's complaint and ordering plaintiff to execute a deed conveying Lot 378
should suffer the loss, if any, resulting from such purchase, rather than he who to the defendant. The case is, once again, before us, this time on appeal by
has obtained the first certificate and who was innocent of any act of the plaintiff, the subject matter of litigation being worth more than P200,000,
negligence. exclusive of interest and costs.
The foregoing decision does not solve, nor pretend to solve, all the difficulties The main facts are not in dispute. Said Lot 378 is part of Hacienda
resulting from double registration under the torrens system and the Mandalagan, consisting of Lots 378, 405, 407, 410, 1205, 1452 and 1641 of
subsequent transfer of the land. Neither do we now attempt to decide the the aforementioned cadastral survey, with an aggregate area of over 502
effect of the former registration in the ordinary registry upon the registration hectares, originally registered in the name of Agustin Amenabar and Pilar
under the torrens system. We are inclined to the view, without deciding it, that Amenabar. Lot 378 has an area of 22,783 sq. meters, more or less, and was
the record under the torrens system, supersede all other registries. If that covered by Original Certificate of Title No. 1776 (Exhibit 4), issued on August
view is correct then it will be sufficient, in dealing with land registered and 25, 1916, in the name of the Amenabars. On November 30, 1920, the latter
recorded alone. Once land is registered and recorded under the torrens sold the aforementioned hacienda to Jose Benares (also referred to in some
system, that record alone can be examined for the purpose of ascertaining the documents as Jose Benares Montelibano) for the sum of P300,000, payable
real status of the title to the land. installments, as set forth in the deed of sale, Exhibit 21. On February 8, 1924,
46
said Original Certificate of Title No. 1776 was cancelled and Jose Benares lot publicly, adversely, notoriously and continuously as owner thereof; that,
obtained, in lieu thereof, Transfer Certificate of Title No. 6295 in his name. "for some reason or other and for cause beyond comprehension of the
Meanwhile, or on March 12, 1921, the Hacienda, including Lot 378, had been defendant title thereto was never transferred in the name of said defendant";
mortgaged by Jose Benares to the Bacolod-Murcia Milling Co. for the sum of that said lot had been placed in defendant's name for assessment purposes
P27,991.74 (Exhibit Y-2). On December 6, 1926, Jose Benares again under Tax Declaration No. 16269 (dated December 31, 1937); and that
mortgaged the Hacienda, including said Lot 378, on the Philippine National plaintiff had acted in bad faith in purchasing said lot from the Bank in 1935,
Bank, subject to the first mortgage held by the Bacolod-Murcia Milling Co. for plaintiff knew then that the provincial hospital was where it is up to the
(Exhibit Y-1). These transactions were duly recorded in the office of the present, and did not declare said lot in its name for assessment purposes until
Register of Deeds of Negros Occidental and annotated on the corresponding 1950, aside from the fact that Alfredo Montelibano, the controlling
certificate of title, including said Transfer Certificate of Title No. 6295, covering stockholder, president and general manager of plaintiff corporation, was the
Lot 378. first City Mayor of Bacolod which contributed to the support, operation and
The mortgage in favor of the Bank was subsequently foreclosed, in pursuance maintenance of said hospital. In an amended answer, dated November 8,
of a decision of the Court of First Instance of Negros Occidental dated 1950, defendant alleged, also, that the aforementioned expropriation case
September 29, 1931 (Exhibit U-1), and the Bank acquired the Hacienda, was "amicably settled as between the parties herein, in the sense that the ...
including Lot 378, as purchaser at the foreclosure sale. Accordingly, said Province of Negros Occidental would pay ... and did in fact pay to Jose Benares
Transfer Certificate of Title No. 6295 was cancelled and, in its stead, transfer the assessed value of Lot 378 ... and whatever consideration pertaining to
Certificate of Title No. 17166 0151 which, owing to its subsequent loss, had said lot in excess of its assessed value which was paid by the Province would
to be reconstituted as Transfer Certificate of Title No. RT-1371 in the name be donated and was in fact donated by said ... Jose Benares in favor of the
of the Bank, was issued on March 14, 1934 (Exhibit P). Soon, later, or on Province purposely for hospital site".
November 8, 1935, the Bank agreed to sell the Hacienda to Carlos P. Benares, The main question for determination in this case is whether or not defendant
son of Jose Banares, for the sum of P400,000, payable in annual installments, herein had acquired Lot 378 in the aforementioned expropriation proceedings.
subject to the condition that, until full payment thereof, title would remain in This decision appealed from in effect decided this question in the affirmative
the Bank (Exhibit R). Thereafter, Carlos P. Benares transferred his rights, under and declared that plaintiff merely holds it in trust for the defendant, in view of
this contract with the Bank, to plaintiff herein, which completed the payment which it ordered the former to convey said lot to the latter. This conclusion is
of the installments due to the Bank in 1949. Hence, on September 29, 1949, predicated, substantially, upon the following premises, namely that case No.
the Bank executed the corresponding deed of absolute sale to the plaintiff 3041 of the Court of First Instance of Negros Occidental for the expropriation
(Exhibit Q) and Transfer Certificate of Title No. 1798, covering 378 was issued, of the hospital site of said province, was actually commenced on January 26,
in lieu of Transfer Certificate of Title No. 17166 (or reconstituted Transfer 1924; that, among the lands sought to be expropriated in said case was Lot
Certificate of Title RT-1371), in plaintiff's name (Exhibit O). 377 of the aforementioned cadastral survey, belonging to one Anacleta
Wherefore, the parties respectfully pray that the foregoing stipulation of facts Agsam, who sold it, on July 10, 1926, to the defendant (Exhibit BB), in whose
be admitted and approved by this Honorable Court, without prejudice to the favor the corresponding transfer certificate of title (Exhibit BB-2) was issued
parties adducing other evidence to prove their case not covered by this on July 12, 1926; that, according the testimony of Jose Benares, the
stipulation of facts. 1wph1.t expropriation of Lot 378 was settled amicably upon payment to him of the
At this juncture, it should be noted that, despite the acquisition of the sum of P12,000; and that defendant's failure to secure the corresponding
Hacienda in 1934 by the Bank, the latter did not take possession of the transfer certificate of title to Lot 378 was due to "the mistaken notion or belief
property for Jose Benares claimed to be entitled to retain it under an alleged that said lot forms part of Lot No. 405-B" in the plan (Exhibit X.).
right of lease. For this reason, the deed of promise to sell, executed by the The testimony of Jose Benares does not deserve, however, full faith and
Bank in favor of Carlos P. Benares, contained a caveat emptor stipulation. credence, because:
When, upon the execution of the deed of absolute sale (Exhibit Q) by the 1. Jose Benares appears to be strongly biased and prejudiced against the
Bank, on September 29, 1949, plaintiff took steps to take possession the plaintiff and its president, for the former believes that the latter had
Hacienda, it was discovered that Lot 378 was the land occupied by the "manipulated" to exclude him from plaintiff corporation, and there have been
Provincial Hospital of Negros Occidental. Immediately, thereafter, or on four (4) litigations between Jose Benares and plaintiff, all of which have been
October 4, 1949, plaintiff made representations with the proper officials to finally decided against the former;
clarify the status of said occupation and, not being satisfied with the 2. The testimony of Jose Benares is extremely contradictory. Thus: (a) he
explanations given by said officials, it brought the present action on June 10, testified to having been paid P12,000 by the Government, although, at the
1950. rate of P1,000 a hectare at which, he would have us believe, he agreed to sell
In its answer dated June 24, 1950, defendant maintained that it had acquired Lot 378; he should have received less than P3,000 for its 22,783 sq. meters;
Lot 378 in the year; 1924-1925, through expropriation proceedings; that (b) he claimed to have received said sum of P12,000.00 "in the year 1924 or
immediately after the commencement of said proceedings in 1924, it took 1925", about "2 or 3 days" after the Government had taken possession of the
possession of said lot and began the construction thereon of the provincial land, and to have sent the money next day to Pilar Amenabar, but the latter
hospital, which was completed in 1926; that since then it had occupied said acknowledged to have received the said sum of P12,000 on November 7,

47
1928; ... It is evident that there were no further proceedings in connection with the
3. Said testimony was contradicted by that of defendant's witness Jose Marco, expropriation case and the chances are that the case was dismissed. The
former deputy clerk of court of Negros Occidental, for: (a) Jose Benares Court had to examine carefully and minutely every single piece of evidence
asserted that there was a written compromise agreement between him and adduced by both parties in order to arrive at the correct solution of the
the Government, whereas Marco averred that agreement was merely oral; and mystery. The Court believes that the failure of the government to secure the
(b) Marco stated that Benares had agreed to accept, as compensation for Lot corresponding transfer of title to Lot 378 lies in the mistaken notion or belief
378, the assessed value thereof, which was P430, and to donate to the that said lot forms a part of Lot 405-B. This conclusion was arrived at after
Government the difference between this sum and the true value of the examining closely the plan, Exhibit X. The plan shows that while all the
property, but Benares affirmed that he was first offered P300 per hectare, subdivided lots were properly identified by lot numbers, that particular portion
which he rejected, and that he later demanded P1,000 a hectare, which the at the lower corner of the plan encircled with red pencil, marked Exhibit X-1, is
Government agreed to pay, although, subsequently, he said that Rafael not labelled with the corresponding lot number and that portion is precisely lot
Alunan and Mariano Yulo had prevailed upon him to accept P1,000 per No. 378, now in question, where the hospital building was constructed. This
hectare; plan was prepared for the government on May 12, 1927 by public land
4. Jose Benares was, also, contradicted by defendant's witness Ildefonso surveyor, Mr. Formento, embracing lots covering over 22 hectares for the
Coscolluela, the provincial treasurer of Negros Occidental at the time of the Capitol and hospital sites. The fact that this particular portion was not labelled
expropriation, who positively assured the Court that the expropriation case with the corresponding lot number might have misled the authorities to
"was not yet terminated" and that "negotiations were still pending" for the believe that it formed a part of lot 405-B, which adjoins it, although separated
acquisition of Lot 378 by the Government when he retired from the service in by the creek. This lack of reasonable explanation why the government failed
1934. to secure the corresponding certificate of title to lot 378, when there is
Upon the other hand, several circumstances strongly indicate that no sufficient proof that Jose Benares was paid and he executed the deed of sale
compromise agreement for the acquisition of the land by the Government had in favor of the government.
been reached and that the expropriation had not been consummated. For Although said decision appears to have been prepared with the
instance: conscientiousness and moral courage that account for the well earned
1. The only entries in the docket relative to the expropriation case refer to its reputation and prestige of the Philippine judiciary, we find ourselves unable to
filing and the publication in the newspaper of the corresponding notices concur in the foregoing view. To begin with, there is no evidence, and
(Exhibit 1);. defendant has not even tried to prove, that the expropriation case had ever
2. The registration of the deed of sale of Lot 377 by Anacleta Agsam to the been dismissed insofar as Lot 378 is concerned. Hence, the lower court merely
Government, followed by the cancellation of the certificate of title in her name speculated about the "chances that the (expropriation) case was dismissed."
and the issuance, in lieu thereof, of another title in the name of the Province, By the way, the contrary was intimated by defendant's witness, Ildefonso
when contrasted with the absence of a similar deed of assignment and of a Coscolluela, for he testified that the expropriation case was still pending in
transfer certificate title in favor of the Province as regards Lot 378, strongly 1934, when he ceased to be the provincial treasurer, and the record before us
suggest that no such assignment or agreement with respect to Lot 378 had suggests that since the Province took possession of the land in 1924 or 1925
been made or reached;. and completed the construction of the hospital in 1926, there were no further
3. The property was mortgaged to the Bacolod-Murcia Milling Co. since March proceedings in said case..
12, 1921, and this mortgage, duly registered and annotated, inter alia, on With respect to the plan, Exhibit X, there is, likewise, no evidence whatsoever
Transfer Certificate of Title No. 1776, in the name of Jose Benares, was not that the authorities had been "misled ... to believe" that the portion at the
cancelled until September 28, 1935. Moreover, Lot 378 could not have been lower corner of said plan which was enclosed, during the trial, within a
expropriated without the intervention of the Milling Co. Yet, the latter was not circle in red pencil, and marked as Exhibit X-1 formed part of Lot 405-B,
made a party in the expropriation proceedings; which had been expropriated by the Province of Negros Occidental. In fact,
4. On December 26, 1926, Jose Benares constituted second mortgage in favor said portion, Exhibit X-1, is not part of the land covered by the plan Exhibit X.
of the Bank, which would not have accepted the mortgage had Lot 378 not A close examination of the latter shows that the boundaries of said portion are
belonged then to the mortgagor. Neither could said lot have been not delimited on the plan. More important still, on the right hand side of
expropriated subsequently thereto without the Bank's knowledge and Exhibit X, the following appears in bold letters: "Subdivision & Consolidation
participation. What is more, in the deed executed by the Bank, on November PLAN of Lots Nos. 400, 401, 403,405, 406, 407 and 410 Bacolod Cadastre as
8, 1935 (Exhibit R), promising to sell the Hacienda Mandalagan to Carlos surveyed for the Provincial Government of Bacolod, Negros Occidental (Capitol
Benares, it was explicitly stated that portions of Lots 405, 407 and 410, site)". The absence of Lot 378 from said enumeration and the explicit
forming part of said Hacienda and designated as Lots 405-A, 407-A; 407-B and statement in Exhibit X to the effect that it refers to the "Capitol Site", negates
410-A, had been expropriated by the Provincial Government of Negros the possibility of its being mistaken by any body, much less by government
Occidental, thus indicating, by necessary implication, that Lot 378 had not engineers, as including the hospital site, and, hence, said Lot 378. Lastly, the
been expropriated. very evidence for the defendant herein, specially the assessor's field sheets
The decision appealed from says: and declarations of real property for tax purposes (Exhibits 9, 10, 11, 12 and

48
13) show that the Government had always regarded Lot 378, not Lot 405, as "Upon the other hand, the main purpose of the Torrens System is to avoid
part of the Provincial Hospital Site. In any event, said possibility of mistake, if possible conflicts of title in and to real estate, and to facilitate transactions
any, which would be remote, cannot suffice to warrant in the face of relative thereto giving the public the right to rely upon the face of Torrens
documentary evidence to the contrary the conclusion that Lot 378 has certificate of title and to dispense with the of inquiring further, except when
already been acquired by the Government. the party concerned has actual knowledge of facts and circumstances that
How about the P12,000 received by Jose Benares from the Government and should impel a reasonably cautious man to make such further inquiry (Tiburcio
applied by him to the payment of his debt to Pilar Amenabar? Said amount vs. PHHC, L-13479, October 31, 1959; Revilla vs. Galindez, G.R. No. L-19940,
could not possibly be the price of Lot 378, for, at the rate of P1,000 a hectare March 30, 1960; Manacop, Jr. vs. Cansino, G.R. No. L-13791, February 27,
allegedly agreed therefor, its price could not have exceeded P3,000.00. In this 1961). In the case at bar plaintiff had no such actual knowledge, it being an
connection, it should be noted that, aside from the expropriation proceedings established fact that he was not aware until 1949 that the land on which the
for the hospital site, another expropriation case for the Capitol site, affecting provincial hospital stood was Lot 378. Furthermore, since the year 1921, or
another property of Jose Benares, appears to have been instituted in the Court before the expropriation case for the hospital site had begun, said lot was
of First Instance of Negros Occidental. Jose Benares may have mistaken the mortgaged to the Bacolod-Murcia Milling Co., and the mortgage, duly
payment for his land included in the Capitol site, as one intended for Lot 378, registered, as well as annotated on the corresponding certificate of title, was
which was affected by the hospital site. And this possibility may amount to a not cancelled until September 28, 1935. Prior to this date, or on December 26,
probability when we consider that he erroneously believed that there had 1926, Lot 378 was subjected to a second mortgage in favor of the Bank, which
been only one expropriation case, instead of two cases, against him, and that acquired title thereto, thru foreclosure proceedings, in 1934. When the Bank
Lot 378, was not included in the mortgage constituted by him in favor of the agreed on November 8, 1935, to sell the property to Carlos P. Benares and the
Philippine National Bank. Evidently, he did not have, at least, an accurate latter, subsequently conveyed his rights to plaintiff herein, as well as when the
recollection of the events or transactions affecting his properties, and, hence, bank executed the deed of absolute sale in plaintiff's favor on September 20,
his testimony may not be relied upon. 1949, the title to the property was in the name of the Bank. Considering that
Thus, the evidence on record is far from sufficient to establish the alleged sugar centrals as well as banks are known to have an array of experienced
acquisition by the defendant of Lot 378, which must be held, therefore, to be and competent lawyers, it cannot be said that plaintiff was not justified in
the exclusive property of plaintiff herein. assuming that said institutions had scrutinized the background of Lot 378 and
The lower court entertained no doubts about the veracity of the testimony of were satisfied that the same belonged to the mortgagor when said mortgages
plaintiff's president to the effect that he did not know until 1949 that the land were constituted, and to the Bank when said deed of sale was executed. In
on which the Provincial Hospital Building stands is Lot 378. Yet, it held that short, we find that plaintiff herein is a purchaser in good faith and for value..
plaintiff was "not a purchaser in good faith for having constructive knowledge As regards the compensation that, as such, it may collect from the defendant,
of defendant's possession of the property at the time it was bought by the we are of the opinion, and so hold, that, since the latter's right to expropriate
plaintiff", because Carlos P. Benares whose right to buy the Hacienda Lot 378 is not contested, and is seemingly conceded, the plaintiff may
Mandalagan from the Bank was acquired by plaintiff "is a part owner of the demand what is due by reason of the expropriation of said lot. In short,
Capitol Subdivision and holds a responsible position therein"; because the plaintiff is entitled to recover from the defendant the fair and full equivalent to
hospital was already constructed in Lot 378 since 1926 and the lot was Lot 378, as of the time when possession thereof was actually taken by the
declared in the name of the Government" and "when plaintiff bought the lot in defendant, plus consequential damages including attorney's fees from
1935 the purchaser should have inquired as to its location and improvement"; which consequential damages the consequential benefits, if any, should be
because "it took the plaintiff 14 years to sleep over the supposed rights to deducted, with interests, at the legal rate, on the aggregate sum due to the
take possession of lot No. 378"; and because "of the overwhelming fact that plaintiff, from and after the date of said actual taking. The case should be
lot No. 378 was erroneously or inadvertently included by the deeds of sale remanded, therefore, to the lower court for the reception of evidence on the
(Exhibits Q & R) executed by the Philippine National Bank in favor of the date of said actual taking and the amount of compensation collectible from
plaintiff subdivision and that same lot was occupied by the defendant the defendant, and the rendition, thereafter, of the corresponding decision
government for the provincial hospital for the last 34 years, as owner thereof". thereon..
As above stated, however, and the lower court conceded, plaintiff's president WHEREFORE, the decision appealed from is hereby reversed and the records
did not know until 1949 that lot 378 was the very land occupied by the remanded to the lower court for further proceedings, as above stated, with
provincial hospital. Moreover, there is a total absence of evidence that this costs against the defendant. It is so ordered..
fact was known to Carlos P. Benares before 1949. Neither may such knowledge Bengzon, C.J., Padilla, Bautista Angelo, Labrador, Reyes, J.B.L., Barrera,
be deduced from the circumstances that he is a son of its former owner, Jose Paredes, Dizon and Regala, JJ., concur.Makalintal, J., took no part.
Benares, for even the latter appears not to be well-posted on the status of his
properties. Indeed, Jose Benares did not apparently know that there were two Separate Opinion of J. Puno in Cruz v. Secretary
(2) expropriation proceedings effecting said properties: that the P12,000
received by him from the Government was not meant for Lot 378; and that II. Constitutional Provisions
this lot was one of the properties mortgaged by him to the Bank. A. Regalian Doctrine
49
TANALEGA, respondents.
Sec. 2, Art. XII, 1987 Consti The Solicitor General for petitioners.
Section 2. All lands of the public domain, waters, minerals, coal, petroleum, Benjamin M. Reyes for private respondent.
and other mineral oils, all forces of potential energy, fisheries, forests or
timber, wildlife, flora and fauna, and other natural resources are owned by the DE CASTRO, J.:
State. With the exception of agricultural lands, all other natural resources shall The two (2) above-entitled petitions for review on certiorari of the decisions
not be alienated. The exploration, development, and utilization of natural dated October 9, 1972 and October 16, 1972 issued by the CFI of Bataan,
resources shall be under the full control and supervision of the State. The Branch I, in LRC No. N-210, and in LRC No. N-206, respectively, involve a
State may directly undertake such activities, or it may enter into co- common issue. For convenience, they are hereby decided jointly.
production, joint venture, or production-sharing agreements with Filipino G.R. No. L-35778:
citizens, or corporations or associations at least sixty per centum of whose On May 4, 1972, respondent Luisito Martinez filed with the lower court an
capital is owned by such citizens. Such agreements may be for a period not application for registration of title under Act 496 of one (1) parcel of land,
exceeding twenty-five years, renewable for not more than twenty-five years, situated in the Municipality of Mariveles, Bataan, containing an area of
and under such terms and conditions as may be provided by law. In cases of 323,093 square meters, more or less.
water rights for irrigation, water supply fisheries, or industrial uses other than On July 7, 1972 the lower court issued an order of general default except as to
the development of water power, beneficial use may be the measure and limit the Republic of the Philippines and the Province of Bataan.
of the grant. On July 24, 1972, the Republic of the Philippines filed with the lower court an
The State shall protect the nations marine wealth in its archipelagic waters, opposition to the application stating that the parcel of land applied for is a
territorial sea, and exclusive economic zone, and reserve its use and portion of the public domain belonging to the Republic, not subject to private
enjoyment exclusively to Filipino citizens. appropriation.
The Congress may, by law, allow small-scale utilization of natural resources by On September 16, 1972, the lower court issued an order reading:
Filipino citizens, as well as cooperative fish farming, with priority to Considering the testimony of the Provincial Forester Leonides B. Rodriguez
subsistence fishermen and fishworkers in rivers, lakes, bays, and lagoons. during the hearing of August 8, 1972 that this land, subject matter of this
The President may enter into agreements with foreign-owned corporations application, was a subject of cadastral proceeding and that this land was
involving either technical or financial assistance for large-scale exploration, assigned as Lot No. 626 (Tsn, August 3, 1972, page 41), this case is ordered
development, and utilization of minerals, petroleum, and other mineral oils re-opened and the Land Registration Commissioner is directed to submit his
according to the general terms and conditions provided by law, based on real report and/or comment as to whether this lot is covered by the Mariveles
contributions to the economic growth and general welfare of the country. In Cadastre within five (5) days from receipt hereof.
such agreements, the State shall promote the development and use of local xxx xxx xxx
scientific and technical resources. On October 5, 1972, the Commissioner of Land Registration submitted to the
The President shall notify the Congress of every contract entered into in lower court a report stating.
accordance with this provision, within thirty days from its execution. That the parcel of land applied for registration in the above-entitled case is
entirely inside Lot No. 626 of the Cadastral Survey of Mariveles, Province of
Carino v. Insular Gov. (Separate) Bataan, Cad. Case no. 19, LTC Cad. Record No. 1097.
xxx xxx xxx
Republic v. Vera, 120 SCRA 210 (1983) Records show that in the hearing of this case in the lower court, applicant
Republic of the Philippines Luisito Martinez, 62 years old, testified that he is the owner of the land applied
SUPREME COURT for, having inherited the same from his parents, consisting of 32 hectares,
Manila more or less; that he started possessing the land in 1938; that about 8
SECOND DIVISION hectares of the land is planted to palay, and there are about 42 mango trees;
G.R. No. L-35778 January 27, 1983 that kamoteng kahoy is also planted thereon; that he declared the land for
REPUBLIC OF THE PHILIPPINES, and THE DIRECTOR OF LANDS, taxation purposes only in 1969 because all the records were lost during the
petitioners, war, and that possession was continuous, open, undisturbed and in the
vs. concept of owner.
HON. ABRAHAM P. VERA, Judge, CFI, Bataan, Branch I, and LUISITO Another witness, Antonio Reyes, 67 years old, testified that he is the overseer
MARTINEZ, respondents. of Luisito Martinez; that the area of his land is 32 hectares, more or less; that
G.R. No. L-35779 January 27, l983 since 1938, applicant has possessed this land; that eight (8) hectares of land
REPUBLIC OF THE PHILIPPINES, and THE DIRECTOR OF LANDS, is devoted to palay, and his son Manuel Reyes and Silvestre Garcia are the
petitioners, ones tilling the land, and the harvest is shared alike between applicant, on one
vs. hand, and Manuel Reyes and Silvestre Garcia, on the other; that eighteen (18)
HON. ABRAHAM P. VERA, judge, CFI, Bataan, Branch I, and THELMA hectares, more or less, is planted to vegetables.

50
While another witness, Silvestre Garcia, 60 years old, testified that he worked submitted documentary proof in support of the opposition filed by the
on the land of the applicant since 1932 which is 32 hectares, more or less; provincial fiscal's office in this case.
that said Luisito Martinez inherited the land from his parents; that he plants At the hearing of this case in the lower court, applicant Thelma Tanalega, 27
palay only on four (4) hectares; that there are 42 mango trees on the land, years old, testified that she had possessed the land "openly, adversely,
G.R. No. L-35779: notoriously and in the concept of owner since February 2, 1970 when the said
On March 21, 1972, respondent Thelma Tanalega filed an application for land was sold to her by Elisa Llamas who allegedly possessed this land" in the
registration under Act No. 496 in the Court of First Instance of Bataan, same manner since 1935; that the applicant had paid for the taxes of the land
docketed as Land Registration Case No. N-206, L.R.C. Rec. No. N-41884, of two for the years 1970-1972.
(2) parcels of land located in the barrio of Camaya, municipality of Mariveles, Another witness, Miguel Ocampo, 57 years old, testified that his parents were
province of Bataan, containing an area of 443,297 square meters, more or the ones working on the land before 1935 and due to the illness of his parents,
less, and 378,506 square meters, more or less, respectively, and more on their request to owner Elisa Llamas, he became overseer up to 1970 when
particularly described and Identified as portions of Lot 626, Mariveles the same was sold to applicant; that 16 hectares of these lands were planted
Cadastre, covered by Plans (LRC) SWO-13430 and (LRC) SWO-13431, to palay while others were devoted to pasture land and planting vegetables.
respectively. Witness Agapito del Rosario, 50 years old, who testified that since childhood,
On March 21, 1972, the corresponding notice of initial hearing was duly issued he had known Elisa Llamas to be the owner of the land applied for; that she
by the Commissioner of Land Registration. was the one managing the planting and improving of the land; that he used to
On March 21, 1972, the lower court ordered the Bureau of Lands to submit a see Leopoldo de Guzman and another one also named Agapito del Rosario
report within ten (10) days if the land subject of the application has been worked on the 16 hectares portion of the land; that Elisa Llamas informed him
issued patents or is the subject of any pending application for the issuance of that in 1970 she sold the land to Thelma Tanalega.
patents. Likewise, the lower court directed the Commissioner of Land At the hearing on August 24, 1972, Fiscal Arsenio Guzman who is appearing
Registration to submit within the same period his report if the land applied for for the government, submitted a certification dated July 3, 1972 of Leonides B.
has been issued a title or is the subject of a pending decree. Rodriguez, District Forester of Balanga, Bataan (Exhibit 3) which states "that
On May 23, 1972, the Chief Surveyor of the Land Registration Commission the tract of land situated at Barrio Camaya, Mariveles, Bataan containing an
filed a report in the lower court, stating that the parcels of land applied for approximate area of EIGHTY TWO HECTARES more or less, as shown and
registration "do not appear to have been passed upon and approved by the described in the attached photostat copy of Plans in two sheets, as surveyed
Director of Lands as required by Section 1858 of the Revised Administrative for Thelma Tanalega, et al., was found to be within the Alienable and
Code." Later, on July 24, 1972, the Chief Surveyor of the Land Registration Disposable Block, Project 4-B, Mariveles, Bataan, certified by the Director of
Commission filed in the lower court another report or manifestation stating Forestry as such on February 16, 1972."
"that Plans (LRC) SWO-13430 and 13431, LRC Case No. N-206, LRC Record No. The applicant did not present as witness her predecessor-in-interest, Elisa
N-41884, when plotted on the Municipal Index Map on file in the Commission Llamas, to testify on the alleged possession of the land. The applicant also
does not appear to overlap with any previously titled property under Act 496; failed to present Guillermo Ramirez, who was hired by her as overseer and her
that the plan and records of said Land Registration application will be alleged tenants. Not a single tenant was presented as witness to prove that
subjected to further examination as soon as the decision to be rendered by the applicant had possessed the land as owners.
this Honorable Court is received in this Commission to determine whether or In both cases, the Court of First Instance of Bataan in two separate decisions,
not a patent or title has in the meantime been issued in order to avoid dated October 9, 1972 and October 16, 1972, confirmed the titles to subject
duplication or overlapping of titles." parcels of land and adjudicated them in favor of applicants Luisito Martinez
At the hearing on June 21, 1972, on motion of the applicant's counsel, the and Thelma Tanalega, now respondents herein.
lower court issued an Order of General Default against all persons, with the In the instant petitions for review the Republic of the Philippines, through the
exception of the Director of Lands and the Director of Forestry, represented by Solicitor General, argued that Lot 626, Mariveles Cadastre was declared public
the Office of the provincial fiscal, and the oppositor Eliseo Martinez land by the decision of the Cadastral Court dated October 11, 1937 and such
represented by Atty. Angelino Banzon, who were directed to file their being the case, the lower court is without jurisdiction over the subject matter
respective oppositions, of the application for voluntary registration under Act 496. Petitioner likewise
On July 7, 1972, the provincial fiscal filed his opposition in behalf of the stressed that the lands in question can no longer be subject to registration by
Directors of Lands and of Forestry, alleging that the parcels of land applied for voluntary proceedings, for they have already been subjected to compulsory
are portions of the public domain belonging to the Republic of the Philippines, registration proceedings under the Cadastral Act.
not subject to private appropriation. The petitions are meritorious and reversal of the questioned decisions is in
Thereafter, the case was tried. The applicant, Thelma Tanalega (respondent order.
herein), testified in her behalf, and presented two (2) witnesses, namely, It is noteworthy that as per the report of the Commissioner of Land
Miguel Ocampo, 57 years old, and Agapito del Rosario, 50 years old, as well as Registration, 1 the land subject matter of the instant proceedings "is entirely
her documentary evidence in support of her application for registration. On inside Lot No. 626 of the Cadastral Survey of Mariveles, Province of Bataan,
the other hand,. Fiscal Arsenio Roman appeared for the government, and Cad. Case No. 19, LRC Cad. Record No. 1097"; that some portions of Lot No.

51
626 were decreed and titles were issued therefor; and that "portion declared expropriated and sub-divided in favor of new amortizing-owner-beneficiaries.
Public Land as per decision dated October 11, 1937." The submission of the plan is a statutory requirement of mandatory character
In a cadastral proceedings any person claiming any interest in any part of the and unless the plan and its technical description are duly approved by the
lands object of the petition is required by Section 9 of Act No. 2259 to file an Director of Lands, the same are not of much value. 4
answer on or before the return day or within such further time as may be WHEREFORE, the decisions dated October 9,1972 and October 16, 1972 of the
allowed by the court, giving the details required by law, such as: (1) Age of the Court of First Instance of Bataan, Branch I should be, as they are hereby
claimant; (2) Cadastral number of lot or lots claimed, or the block and lot reversed. Without pronouncement as to costs.
numbers, as the case may be; (3) Name of the barrio and municipality, SO ORDERED.
township or settlement in which the lots are situated; (4) Names of the owners Makasiar, Aquino, Concepcion, Jr., Guerrero and Escolin JJ., concur.
of adjoining lots; (5) If claimant is in possession of the lots claims and can Abad Santos, J., concurs in the result.
show no express grant of the land by the Government to him or to his Bureau of Forestry v. Court of Appeals, 153 SCRA 351 (1987)
predecessors-in-interest, the answer need state the length of time property Republic of the Philippines
was held in possession and the manner it was acquired, giving the length of SUPREME COURT
time, as far as known, during which his predecessors, if any, held possession; Manila
(6) If claimant is not in possession or occupation of the land, the answer shall FIRST DIVISION
set forth the interest claimed by him and the time and manner of its G.R. No. L-37995 August 31, 1987
acquisition; (7) If the lots have been assessed for taxation, their last assessed BUREAU OF FORESTRY, BUREAU OF LANDS and PHILIPPINE FISHERIES
value; and (8) Encumbrance, if any, affecting the lots and the names of COMMISSION, petitioners,
adverse claimants as far as known. In the absence of successful claimants, the vs.
property is declared public land. COURT OF APPEALS and FILOMENO GALLO, respondents.
In the instant cases, private respondents apparently either did not file their
answers in the aforesaid cadastral proceedings or failed to substantiate their PARAS, J.:
claims over the portions they were then occupying, otherwise, titles over the Before Us is a petition for review on certiorari, which seeks to annul and set
portions subject of their respective claims would have been issued to them. aside the Decision 1 (promulgated on April 11, 1973) of the respondent court
The Cadastral Court must have declared the lands in question public lands, in CA-G.R. No. 38163-R, affirming the decision 2 (dated April 6, 1966) of the
and its decision had already become final and conclusive. then Court of First Instance of Iloilo in Land Registration Case No. N-506,
Respondents are now barred by prior judgment to assert their rights over the G.L.R.O. Record No. N-20783 entitled "Filomeno Gallo, Applicant vs. Bureau of
subject land, under the doctrine of res judicata. A cadastral proceeding is one Forestry, Bureau of Lands, and Philippine Fisheries Commission, oppositors. "
in rem and binds the whole world. Under this doctrine, parties are precluded The dispositive portion of the trial court's decision reads as follows:
from re-litigating the same issues already determined by final judgment. 2 WHEREFORE, the court Orders the registration of Lots Nos. 2, 3, and 4 and the
Even granting that respondents can still petition for judicial confirmation of bigger portion of Lot No. 1 after excluding the portion Identified as Lot 1-A
imperfect title over the lands subject matter of the instant cases, the same together with the improvements thereon in the name of Filomeno Gallo, of
must necessarily fail. It is to be noted that in the instant cases evidence for legal age, widower, Filipino citizen, and resident of 155 Fuentes Street, Iloilo
the respondents themselves tend to show that only portions of the entire area City, Philippines. Lots Nos. 1, 2 and 3 are subject to the road right-of-way of 15
applied for are cultivated. A mere casual cultivation of portions of the land by meters wide which is presently known as Sto. Rosario Rizal Montpiller
the claimant does not constitute possession under claim of ownership. In that provincial Road and Buenavista-Daraga provincial Road they being properties
sense, possession is not exclusive and notorious so as to give rise to a of the Province of Iloilo and should be registered in the name of said province.
presumptive grant from the State. The possession of public land however long The oppositions of the Director of Lands, Director of Forestry and the
the period thereof may have extended, never confers title thereto upon the Philippine Fisheries Commission are dismissed. Lot 1-A with an area of 2.6864
possessor because the statute of limitations with regard to public land does hectares which is enclosed in red pencil and is found inside Lot No. 1 in the
not operate against the State, unless the occupant can prove possession and plan Exhibit is hereby declared public land. After the decision has become final
occupation of the same under claim of ownership for the required number of let the corresponding decree be issued.
years to constitute a grant from the State. 3 Applicants, therefore, have failed SO ORDERED. (p. 38, Joint Record on Appeal Annex "A." p. 25, Rollo)
to submit convincing proof actual, peaceful and adverse possession in the This appeal also seeks to annul and set aside respondent court's resolution
concept of owners of the entire area in question during the period required by dated December 14, 1973 denying for lack of merit, herein petitioners' motion
law. for reconsideration.
Apart from the foregoing, the survey plans submitted by petitioners were not The basic issue which petitioners raise in this appeal is
approved by the Director of Lands but by the Land Registration Commission. Whether or not the classification of lands of the public domain by the
The Land Registration Commission has no authority to approve original survey Executive Branch of the Government into agricultural, forest or mineral can be
plans in this particular case. Section 34-A of R.A. No. 6389 relied upon by changed or varied by the court depending upon the evidence adduced before
respondents applies only to lands subject of tenancy relation which are it. (p. 9, Brief for the Petitioners, p. 105, Rollo)
52
The antecedent facts of the case are as follows: NOT CONVINCINGLY SHOWN THAT THE REMOTE PREDECESSOR-IN-INTEREST
On July 11, 1961, four (4) parcels of land situated in Buenavista, Iloilo POSSESSED THE LAND IN QUESTION SINCE TIME IMMEMORIAL. (pp. 9 & 20,
described in Plan Psu-150727, containing an approximate area of 30.5943 Brief for the Petitioners, p. 105, Rollo)
hectares were the subject of an application for registration by Mercedes Diago Out of the 30.5943 hectares applied for registration under the Torrens System,
who alleged among others that she herself occupied said parcels of land 11.1863 hectares are coconut lands and admittedly within the disposable
having bought them from the testate estate of the late Jose Ma. Nava who, in portion of the public domain. These are more particularly Identified as parcels
his lifetime, had bought the lands in turn from Canuto Gustilo on June 21, "B," B-1", "B-2" and "B-3" of the sketch plan Exh. "1-A." The rest, consisting of
1934. The Director of Lands opposed said application on the ground that 19.4080 hectares and Identified as parcels A, A-1, A-2 and A-3 of the same
neither the applicant nor her predecessors-in-interest have sufficient title over plan Exh. "1-A," is now the center of controversy of the present appeal.
the lands applied for, which could be registered under the Torrens systems, Petitioners contend that respondent court completely ignored the undisputed
and that they have never been in open, continuous and exclusive possession facts that 1) the controverted area is within Timberland Block "B," L.C. Project
of the said lands for at least 30 years prior to the filing of the application. The No. 38, L.C. Map No. 1971 of Buenavista, Iloilo and that 2) the certification of
Director of Forestry on the other hand anchored his opposition principally on February 18, 1956 of the then Director of Forestry to the effect that the area in
the ground that certain specific portions of the lands subject matter of the question is needed for forest purposes. Respondent court in affirming the
application, with an area of approximately 194,080 square meters are decision of the Iloilo trial court ruled that although the controverted portion of
mangrove swamps and are within Timberland Block "B " L.C. Project No. 38, 19.4080 hectares are mangrove and nipa swamps within Timberland Block
L.C. Map No. 1971 of Buenavista, Iloilo. "B," L.C. Project No. 38, same cannot be considered part of the public forest
On June 30, 1965, respondent Filomeno Gallo, having purchased the subject not susceptible of private ownership since petitioners failed to submit
parcels of land from Mercedes Diago on April 27, 1965, moved to be convincing proof that these lands are more valuable for forestry than for
substituted in place of the latter, attaching to his motion an Amended agricultural purposes, and the presumption is that these are agricultural lands.
Application for Registration of Title substantially reproducing the allegations in Respondent court based its conclusion upon the premise that whether or not a
the application of Mercedes Diago. Petitioner Philippine Fisheries Commission controverted parcel of land is forest land, is a question of fact which should be
also moved on August 30, 1965 to be substituted in place of petitioner Bureau settled by competent proofs, and if such a question be an issue in a land
of Forestry as oppositor over a portion of the land sought to be registered, registration proceeding, it is incumbent upon the Director of Forestry to
supervision and control of said portion having been transferred from the submit to the court convincing proofs that the land in dispute is not more
Bureau of Forestry to the Philippine Fisheries Commission. valuable for agriculture than for forest purposes. It is the position of
On April 6, 1966, the trial court rendered its decision ordering the registration respondent that respondent court did "not hesitate to apply this presumption
of the four (4) parcels of land in the name of respondent Filomeno Gallo after with full force particularly where, as in the case at bar, the lands applied for
excluding a portion Identified as Lot "1-A" which is the site of the municipal have been possessed and cultivated by the applicant and his predecessors-in-
hall of Buenavista town, and subjecting Lots Nos. 1, 2 and 3 to the road-of-way interest for a long number of years without the government taking any
of 15 meters width. positive step to dislodge the occupants from their holdings which have passed
Petitioners appealed from said decision to the respondent Court of Appeals from one to another by inheritance or by purchase." (p. 9, Brief for private
assigning the following errors in their brief: respondents) Otherwise stated, it is Our impression that private respondents
THE TRIAL COURT ERRED IN ORDERING THE REGISTRATION OF THE SUBJECT claim the rule of prescription against the government.
LAND WHICH CONSISTS OF TIMBERLAND, FORESHORELAND AND LAND Such contentions of private respondents do not hold water. Admittedly the
BELONGING TO THE PUBLIC DOMAIN HENCE UNREGISTERABLE. controversial area is within a timberland block as classification of the
THE TRIAL COURT ERRED IN HOLDING THAT THE POSSESSION OF THE municipality and certified to by the Director of Forestry on February 18, 1956
APPLICANT-APPELLEE AND HIS PREDECESSORS-IN-INTEREST HAD BEEN as lands needed for forest purposes and hence they are portions of the public
PEACEFUL, OPEN, CONTINUOUS, UNINTERRUPTED AND ADVERSE TO domain which cannot be the subject of registration proceedings. Clearly
CLAIMANTS AND IN THE CONCEPT OF OWNER. (p. 6, Brief for the Petitioners, therefore the land is public land and there is no need for the Director of
p. 105, Rollo) Forestry to submit to the court convincing proofs that the land in dispute is not
Respondent court affirmed said decision and denied a motion for more valuable for agriculture than for forest purposes, as there was no
reconsideration of the same hence the present petition with two (2) assigned question of whether the land is forest land or not. Be it remembered that said
errors, basically the same issues raised with the respondent court: forest land had been declared and certified as such by the Director of the
RESPONDENT COURT ERRED IN NOT HOLDING THAT THE DETERMINATION OF Bureau of Forestry on February 18, 1956, several years before the original
WHETHER A PUBLIC LAND IS AGRICULTURAL OR STILL A FOREST LAND RESTS applicant of the lands for registration Mercedes Diago, filed it on July 11, 1961.
EXCLUSIVELY UPON THE DIRECTOR OF FORESTRY (NOW DIRECTOR OF FOREST In the case of Government of the Philippine Islands vs. Abella, 49 Phil. 49,
DEVELOPMENT), THE SECRETARY OF NATURAL RESOURCES) AND THE cited by private respondents themselves in their brief, We held
PRESIDENT OF THE PHILIPPINES. Following the decision of Ankon vs. Government of the Philippine Islands (40
RESPONDENT COURT ERRED IN NOT HOLDING THAT THE LAND IS PRESUMED Phil. 10), it is again held, that whether a particular parcel of land is more
TO BELONG TO THE PUBLIC DOMAIN AND PRIVATE RESPONDENT HEREIN HAS valuable for forestry purposes than for agricultural purposes, or vice versa, is

53
a fact which must be established during the trial of the case. Whether the SECOND DIVISION
particular land is agricultural, forestry or mineral is a question to be settled in G.R. No. L-50464 January 29, 1990
each particular case unless the Bureau of Forestry has, under the authority SUNBEAM CONVENIENCE FOODS INC., CORAL BEACH DEVELOPMENT
conferred upon it by law, prior to the intervention of private interest, set aside CORP., and the REGISTER OF DEEDS OF BATAAN, petitioners,
said land for forestry or mineral resources. (Italics for emphasis) vs.
We also held in the case of Republic vs. Animas, 56 SCRA 499, 503 that- HON. COURT OF APPEALS and THE REPUBLIC OF THE PHILIPPINES,
... As a general rule, timber or forest lands are not alienable or disposable respondents.
under either the Constitution of 1935 or the Constitution of 1973. Filoteo T. Banzon for petitioners.
... It is the Bureau of Forestry that has jurisdiction and authority over the SARMIENTO, J.:
demarcation, protection, management, reproduction, occupancy and use of all In this petition for review on certiorari, Convenience Foods Corporation
public forests and forest reservations and over the granting of licenses for the (hereafter simply SUNBEAM) and Coral Beach Development Corporation
taking of products therefrom, including stone and earth (Section 1816 of the (hereafter simply CORAL BEACH) bring to our attention the decision rendered
Revised Administrative Code). That the area in question is a forest or timber by the Court of Appeals in "Republic of the Philippines v. Hon. Pedro T.
land is clearly established by the certification made by the Bureau of Forest Santiago, et al.," disposing as follows:
Development that it is within the portion of the area which was reverted to the
category of forest land, approved by the President on March 7, 1958. WHEREFORE, the writ prayed for is granted. The order of the respondent judge
As provided for under Sec. 6 of Commonwealth Act No. 141, which was lifted dated October 7, 1977, dismissing Civil Case No. 4062 is set aside, and
from Act No. 2874, the classification or reclassification of public lands into respondent judge is ordered to require private respondents to file their answer
alienable or disposable, mineral or forest lands is now a prerogative of the to the complaint in said Civil Case No. 4062 and thereafter to proceed with the
Executive Department of the government and not of the courts. With these trial of the case on the merits and to render judgment thereon.'
rules, there should be no more room for doubt that it is not the court which The following facts stated by the respondent Court in its decision and restated
determines the classification of lands of the public domain into agricultural, by the petitioners in their petition are accurate:
forest or mineral but the Executive Branch of the Government, through the (a) On April 29, 1963, the Director of Lands caused the issuance of a Sales
Office of the President. Hence, it was grave error and/or abuse of discretion for Patent in favor of defendant Sunbeam Convenience Foods, Inc., over the
the respondent court to ignore the uncontroverted facts that (1) the disputed parcels of land both situated in Mariveles, Bataan and more particularly
area is within a timberland block and (2) as certified to by the then Director of described and bounded as follows:
Forestry, the area is needed for forest purposes. Lot 1-Sgs-2409 (area 3,113,695 sq. m )
Furthermore, private respondents Cannot claim to have obtained their title by Lot 2-Sgs-2409 area 1,401,855 sq. m
prescription inasmuch as the application filed by them necessarily implied an (b) On May 3, 1963, the aforesaid Sales Patent was registered with the
admission that the portions applied for are part of the public domain which defendant Register of Deeds of Bataan who in turn issued Original Certificate
cannot be acquired by prescription, unless the law expressly permits it. It is a of Title No. Sp-24 in favor of defendant Sunbeam Convenience Foods, Inc., for
rule of law that possession of forest lands, however long, cannot ripen into the two parcels of land above-described;
private ownership (Director of Forestry vs. Munoz, 23 SCRA 1184). (c) Subsequently, Original Certificate of Title No. Sp-24 was cancelled and in
WHEREFORE, in the light of the foregoing, the assailed decision is hereby SET lieu thereof, Transfer Certificate of Title No. T-12421 was issued over Lot 1,
ASIDE, and a new one is hereby rendered, declaring that: Sgs-2409, while Transfer Certificate of Title No. 12422 was issued over Lot 2,
1) Parcels "B," "B-1," "B-2 and "B-3" of the sketch plan Exhibit "1-A" consisting Sgs-2409, both in favor of defendant Coral Beach Development Corporation I
of 11.1863 hectares of coconut land and admittedly within the disposable (d) On May 11, 1976, the Solicitor General in the name of the Republic of the
portion of the public domain are hereby ordered registered in the name of the Philippines instituted before the Court of First Instance of Bataan, an action for
applicant Filomeno Gallo and/or his successors-in-interest as provided for by reversion docketed as Civil Case No. 4062. 2
the Public Land Law; and SUNBEAM and CORAL BEACH filed a Motion to Dismiss on the following
2) Parcels "A," "A-1," and "A-2," and "A-3" of the same plan Exh. "1-A," grounds:
consisting of 19.4080 hectares, are forest lands or lands of the public domain 1. The Republic of the Philippines should have exhausted all administrative
of the Republic of the Philippines and are therefore inalienable. remedies before filing the case in court;
SO ORDERED. 2. The title issued to SUNBEAM and CORAL BEACH had become indefeasible
Teehankee, C.J., Narvasa, Cruz and Gancayco, JJ., concur. and imprescriptible;
3. The action for reversion was defective, having been initiated by the Solicitor
Sunbeam Convenience Foods, Inc. v. Court of Appeals 181 SCRA 443 General and not by the Director of Lands. 3 The then Court of First Instance of
(1990) Bataan dismissed the complaint in the Order of October 7, 1977, 4adopting
Republic of the Philippines mainly the theory that since the titles sought to be cancelled emanated from
SUPREME COURT the administrative act of the Bureau of Lands Director, the latter, not the
Manila courts, had jurisdiction over the disposition of the land.

54
The Solicitor General received the copy of the Order on October 11, 1977 and moot and academic. Land remains unclassified land until it is released
filed a Notice of Appeal dated October 25, 1977 . 5 The Solicitor General then therefrom and rendered open to disposition. 10
moved for an extension of thirty days within which to file the Record on Our adherence to the Regalian doctrine subjects all agricultural, timber, and
Appeal and to pay the docket fee in order to perfect the appeal. This was to be mineral lands to the dominion of the State. 11 Thus, before any land may be
followed by another motion for extension filed by the Solicitor General, declassified from the forest group and converted into alienable or disposable
resulting in the Court of Appeals granting the petitioner another extension of land for agricultural or other purposes, there must be a positive act from the
fifteen days from December 10, 1977. Finally before this period of extension government. Even rules on the confirmation of imperfect titles do not apply
lapsed, instead of an appeal, a petition for certiorari with the respondent Court unless and until the land classified as forest land is released in an official
of Appeals was filed. proclamation to that effect so that it may form part of the disposable
According to the Solicitor General, the Court of First Instance committed grave agricultural lands of the public domain. 12
abuse of discretion in dismissing the complaint and in The mere fact that a title was issued by the Director of Lands does not confer
a. Not finding that since the lower court acted in a Motion to Dismiss, the any validity on such title if the property covered by the title or patent is part of
correctness of its decision must be decided in the assumed truth and accuracy the public forest. 13
of the allegations of the complaint. The complaint alleges that the lands in The only way to resolve this question of fact as to the classification of the land
question are forest lands; hence, inalienable. is by remanding the case to the lower court for a full- dress trial on the issues
b. Finding that Lots I and 2 are alienable and disposable lands of the public involved.
domain under the jurisdiction of the Director of Lands despite clear and Generally, the rules of procedure must be observed so that the efficient
positive evidence to the contrary. administration of justice is ensured. However, the rules of procedure should be
c. Concluding that the complaint for reversion is defective as it was not viewed as mere tools designed to facilitate the attainment of justice. 14 They
initiated by the Director of Lands. must lead to the proper and just determination of litigation, without tying the
d. Finding that the complaint for reversion states no cause of action for hands of the law or making it indifferent to realities.
alleged failure of petitioner to exhaust administrative remedies. 6 Certiorari is one such remedy. Considered extraordinary, it is made available
The Court of Appeals gave due course to the petition for certiorari, set aside only when there is no appeal, nor any plain, speedy or adequate remedy in
the Order of Dismissal rendered by the Court of First Instance in Civil Case No. the ordinary course of the law. 15 The long line of decisions denying the
4062, and ordered the presiding judge Hon. Pedro T. Santiago to receive the petition for certiorari, either before appeal was availed of or specially in
answers of the private respondents SUNBEAM and CORAL BEACH in the action instances where the appeal period has lapsed, far outnumbers the instances
for reversion. when certiorari was given due course. The few significant exceptions were:
Hence Sunbeam and Coral Beach filed this petition for review. when public welfare and the advancement of public policy dictate; or when
A review is not a matter of right but of sound judicial discretion, and is granted the broader interests of justice so require, or when the writs issued are null, 16
only when there are special and important reasons therefore. The following, or when the questioned order amounts to an oppressive exercise of judicial
while neither controlling nor fully measuring the Court's discretion, authority. 17
enumerates the premises for granting a review: We find nothing disagreeable with the action of the Court of Appeals to give
(a) When the Court of Appeals has decided a question of substance, not due course to the petition considering that the issue affected a matter of
theretofore determined by the Supreme Court or has decided it in a way public concern which is the disposition of the lands of our matrimony No less
probably not in accord with law or the applicable decisions of the Supreme than the Constitution protects its policy.
Court; and We therefore find no compelling reason to disturb the findings of the appellate
(b) When the Court of Appeals has so far departed from the accepted and court, in the absence of a clear showing that the Court of Appeals has decided
usual course of judicial proceedings or so far sanctioned such departure by a a question of substance in a manner inconsistent with jurisprudence, or that
lower court as to call for supervision . 7 the respondent Court has departed from the accepted and usual course of
We agree with the Court of Appeals' granting of the petition filed by the judicial proceedings. In sum, no reversible error has been committed by the
Republic of the Philippines charging the then Court of First Instance with grave respondent court. 18
abuse of discretion. The filing of the Motion to Dismiss the complaint for WHEREFORE, the petition is DENIED and the decision of the Court of Appeals
reversion by SUNBEAM and CORAL BEACH on the ground of lack of cause of is affirmed. Costs against the petitioners.
action, necessarily carried with it the admission, for purposes of the motion, of SO ORDERED.
the truth of all material facts pleaded in the complaint instituted by the
Republic. Republic v. Court of Appeals, 160 SCRA 228 (1998)
An important factual issue raised in the complaint was the classification of the Republic of the Philippines
lands as forest lands. This material allegation stated in the Republic's SUPREME COURT
complaint' was never denied specifically 9 by the defendants (petitioners Manila
herein) SUNBEAM and CORAL BEACH. FIRST DIVISION
If it is true that the lands are forest lands, then all these proceedings become G.R. No. L-43938 April 15, 1988
55
REPUBLIC OF THE PHILIPPINES (DIRECTOR OF FOREST evidenced by its construction of adits, its affidavits of annual assessment, its
DEVELOPMENT), petitioner, geological mappings, geological samplings and trench side cuts, and its
vs. payment of taxes on the land. 8
HON. COURT OF APPEALS (THIRD DIVISION) and JOSE Y. DE LA ROSA, For its part, Atok alleged that a portion of Lots 1-5 and all of Lots 6-9 were
respondents. covered by the Emma and Fredia mineral claims located by Harrison and
G.R. No. L-44081 April 15, 1988 Reynolds on December 25, 1930, and recorded on January 2, 1931, in the
BENGUET CONSOLIDATED, INC., petitioner, office of the mining recorder of Baguio. These claims were purchased from
vs. these locators on November 2, 1931, by Atok, which has since then been in
HON. COURT OF APPEALS, JOSE Y. DE LA ROSA, VICTORIA, BENJAMIN open, continuous and exclusive possession of the said lots as evidenced by its
and EDUARDO, all surnamed DE LA ROSA, represented by their father annual assessment work on the claims, such as the boring of tunnels, and its
JOSE Y. DE LA ROSA, respondents. payment of annual taxes thereon. 9
G.R. No. L-44092 April 15, 1988 The location of the mineral claims was made in accordance with Section 21 of
ATOK-BIG WEDGE MINING COMPANY, petitioner, the Philippine Bill of 1902 which provided that:
vs. SEC. 21. All valuable mineral deposits in public lands in the philippine Islands
HON. COURT OF APPEALS, JOSE Y. DE LA ROSA, VICTORlA, BENJAMIN both surveyed and unsurveyed are hereby declared to be free and open to
and EDUARDO, all surnamed DE LA ROSA, represented by their father, exploration, occupation and purchase and the land in which they are found to
JOSE Y. DE LA ROSA, respondents. occupation and purchase by the citizens of the United States, or of said
islands.
CRUZ, J.: The Bureau of Forestry Development also interposed its objection, arguing
The Regalian doctrine reserves to the State all natural wealth that may be that the land sought to be registered was covered by the Central Cordillera
found in the bowels of the earth even if the land where the discovery is made Forest Reserve under Proclamation No. 217 dated February 16, 1929.
be private. 1 In the cases at bar, which have been consolidated because they Moreover, by reason of its nature, it was not subject to alienation under the
pose a common issue, this doctrine was not correctly applied. Constitutions of 1935 and 1973. 10
These cases arose from the application for registration of a parcel of land filed The trial court * denied the application, holding that the applicants had failed
on February 11, 1965, by Jose de la Rosa on his own behalf and on behalf of to prove their claim of possession and ownership of the land sought to be
his three children, Victoria, Benjamin and Eduardo. The land, situated in registered. 11 The applicants appealed to the respondent court, * which
Tuding, Itogon, Benguet Province, was divided into 9 lots and covered by plan reversed the trial court and recognized the claims of the applicant, but subject
Psu-225009. According to the application, Lots 1-5 were sold to Jose de la Rosa to the rights of Benguet and Atok respecting their mining claims. 12 In other
and Lots 6-9 to his children by Mamaya Balbalio and Jaime Alberto, words, the Court of Appeals affirmed the surface rights of the de la Rosas over
respectively, in 1964. 2 the land while at the same time reserving the sub-surface rights of Benguet
The application was separately opposed by Benguet Consolidated, Inc. as to and Atok by virtue of their mining claims.
Lots 1-5, Atok Big Wedge Corporation, as to Portions of Lots 1-5 and all of Lots Both Benguet and Atok have appealed to this Court, invoking their superior
6-9, and by the Republic of the Philippines, through the Bureau of Forestry right of ownership. The Republic has filed its own petition for review and
Development, as to lots 1-9. 3 reiterates its argument that neither the private respondents nor the two
In support of the application, both Balbalio and Alberto testified that they had mining companies have any valid claim to the land because it is not alienable
acquired the subject land by virtue of prescription Balbalio claimed to have and registerable.
received Lots 1-5 from her father shortly after the Liberation. She testified she It is true that the subject property was considered forest land and included in
was born in the land, which was possessed by her parents under claim of the Central Cordillera Forest Reserve, but this did not impair the rights already
ownership. 4 Alberto said he received Lots 6-9 in 1961 from his mother, Bella vested in Benguet and Atok at that time. The Court of Appeals correctly
Alberto, who declared that the land was planted by Jaime and his declared that:
predecessors-in-interest to bananas, avocado, nangka and camote, and was There is no question that the 9 lots applied for are within the June Bug mineral
enclosed with a barbed-wire fence. She was corroborated by Felix Marcos, 67 claims of Benguet and the "Fredia and Emma" mineral claims of Atok. The
years old at the time, who recalled the earlier possession of the land by June Bug mineral claim of plaintiff Benguet was one of the 16 mining claims of
Alberto's father. 5 Balbalio presented her tax declaration in 1956 and the James E. Kelly, American and mining locator. He filed his declaration of the
realty tax receipts from that year to 1964, 6 Alberto his tax declaration in 1961 location of the June Bug mineral and the same was recorded in the Mining
and the realty tax receipts from that year to 1964. 7 Recorder's Office on October 14, 1909. All of the Kelly claims ha subsequently
Benguet opposed on the ground that the June Bug mineral claim covering Lots been acquired by Benguet Consolidated, Inc. Benguet's evidence is that it had
1-5 was sold to it on September 22, 1934, by the successors-in-interest of made improvements on the June Bug mineral claim consisting of mine tunnels
James Kelly, who located the claim in September 1909 and recorded it on prior to 1935. It had submitted the required affidavit of annual assessment.
October 14, 1909. From the date of its purchase, Benguet had been in actual, After World War II, Benguet introduced improvements on mineral claim June
continuous and exclusive possession of the land in concept of owner, as Bug, and also conducted geological mappings, geological sampling and trench

56
side cuts. In 1948, Benguet redeclared the "June Bug" for taxation and had We agree likewise with the oppositors that having complied with all the
religiously paid the taxes. requirements of the mining laws, the claims were removed from the public
The Emma and Fredia claims were two of the several claims of Harrison domain, and not even the government of the Philippines can take away this
registered in 1931, and which Atok representatives acquired. Portions of Lots 1 right from them. The reason is obvious. Having become the private properties
to 5 and all of Lots 6 to 9 are within the Emma and Fredia mineral claims of of the oppositors, they cannot be deprived thereof without due process of law.
Atok Big Wedge Mining Company. 13

The June Bug mineral claim of Benguet and the Fredia and Emma mineral Such rights were not affected either by the stricture in the Commonwealth
claims of Atok having been perfected prior to the approval of the Constitution Constitution against the alienation of all lands of the public domain except
of the Philippines of 1935, they were removed from the public domain and had those agricultural in nature for this was made subject to existing rights. Thus,
become private properties of Benguet and Atok. in its Article XIII, Section 1, it was categorically provided that:
It is not disputed that the location of the mining claim under consideration SEC. 1. All agricultural, timber and mineral lands of the public domain, waters,
was perfected prior to November 15, 1935, when the Government of the minerals, coal, petroleum and other mineral oils, all forces of potential energy
Commonwealth was inaugurated; and according to the laws existing at that and other natural resources of the Philipppines belong to the State, and their
time, as construed and applied by this court in McDaniel v. Apacible and disposition, exploitation, development, or utilization shall be limited to citizens
Cuisia (42 Phil. 749), a valid location of a mining claim segregated the area of the Philippines or to corporations or associations at least 60% of the capital
from the public domain. Said the court in that case: The moment the locator of which is owned by such citizens, subject to any existing right, grant, lease
discovered a valuable mineral deposit on the lands located, and perfected his or concession at the time of the inauguration of the government established
location in accordance with law, the power of the United States Government to under this Constitution. Natural resources with the exception of public
deprive him of the exclusive right to the possession and enjoyment of the agricultural lands, shall not be alienated, and no license, concession, or lease
located claim was gone, the lands had become mineral lands and they were for the exploitation, development or utilization of any of the natural resources
exempted from lands that could be granted to any other person. The shall be granted for a period exceeding 25 years, except as to water rights for
reservations of public lands cannot be made so as to include prior mineral irrigation, water supply, fisheries, or industrial uses other than the
perfected locations; and, of course, if a valid mining location is made upon development of water power, in which case beneficial use may be the
public lands afterwards included in a reservation, such inclusion or reservation measure and the limit of the grant.
does not affect the validity of the former location. By such location and Implementing this provision, Act No. 4268, approved on November 8, 1935,
perfection, the land located is segregated from the public domain even as declared:
against the Government. (Union Oil Co. v. Smith, 249 U.S. 337; Van Mess v. Any provision of existing laws, executive order, proclamation to the contrary
Roonet, 160 Cal. 131; 27 Cyc. 546). notwithstanding, all locations of mining claim made prior to February 8, 1935
"The legal effect of a valid location of a mining claim is not only to segregate within lands set apart as forest reserve under Sec. 1826 of the Revised
the area from the public domain, but to grant to the locator the beneficial Administrative Code which would be valid and subsisting location except to
ownership of the claim and the right to a patent therefor upon compliance the existence of said reserve are hereby declared to be valid and subsisting
with the terms and conditions prescribed by law. Where there is a valid locations as of the date of their respective locations.
location of a mining claim, the area becomes segregated from the public The perfection of the mining claim converted the property to mineral land and
domain and the property of the locator." (St. Louis Mining & Milling Co. v. under the laws then in force removed it from the public domain. 14 By such
Montana Mining Co., 171 U.S. 650; 655; 43 Law ed., 320, 322.) "When a act, the locators acquired exclusive rights over the land, against even the
location of a mining claim is perfected it has the effect of a grant by the government, without need of any further act such as the purchase of the land
United States of the right of present and exclusive possession, with the right or the obtention of a patent over it. 15 As the land had become the private
to the exclusive enjoyment of all the surface ground as well as of all the property of the locators, they had the right to transfer the same, as they did,
minerals within the lines of the claim, except as limited by the extralateral to Benguet and Atok.
right of adjoining locators; and this is the locator's right before as well as after It is true, as the Court of Appeals observed, that such private property was
the issuance of the patent. While a lode locator acquires a vested property subject to the "vicissitudes of ownership," or even to forfeiture by non-user or
right by virtue of his location made in compliance with the mining laws, the abandonment or, as the private respondents aver, by acquisitive prescription.
fee remains in the government until patent issues."(18 R.C.L. 1152) (Gold However, the method invoked by the de la Rosas is not available in the case
Creek Mining Corporation v. Hon. Eulogio Rodriguez, Sec. of Agriculture and at bar, for two reasons.
Commerce, and Quirico Abadilla, Director of the Bureau of Mines, 66 Phil. 259, First, the trial court found that the evidence of open, continuous, adverse and
265-266) exclusive possession submitted by the applicants was insufficient to support
It is of no importance whether Benguet and Atok had secured a patent for as their claim of ownership. They themselves had acquired the land only in 1964
held in the Gold Creek Mining Corp. Case, for all physical purposes of and applied for its registration in 1965, relying on the earlier alleged
ownership, the owner is not required to secure a patent as long as he possession of their predecessors-in-interest. 16 The trial judge, who had the
complies with the provisions of the mining laws; his possessory right, for all opportunity to consider the evidence first-hand and observe the demeanor of
practical purposes of ownership, is as good as though secured by patent. the witnesses and test their credibility was not convinced. We defer to his

57
judgment in the absence of a showing that it was reached with grave abuse of SEC. 6. The ownership of, and the right to extract and utilize, the minerals
discretion or without sufficient basis. 17 included within all areas for which Torrens titles are granted are excluded and
Second, even if it be assumed that the predecessors-in-interest of the de la excepted from all such titles.
Rosas had really been in possession of the subject property, their possession This is an application of the Regalian doctrine which, as its name implies, is
was not in the concept of owner of the mining claim but of the property as intended for the benefit of the State, not of private persons. The rule simply
agricultural land, which it was not. The property was mineral land, and they reserves to the State all minerals that may be found in public and even private
were claiming it as agricultural land. They were not disputing the lights of the land devoted to "agricultural, industrial, commercial, residential or (for) any
mining locators nor were they seeking to oust them as such and to replace purpose other than mining." Thus, if a person is the owner of agricultural land
them in the mining of the land. In fact, Balbalio testified that she was aware of in which minerals are discovered, his ownership of such land does not give
the diggings being undertaken "down below" 18 but she did not mind, much him the right to extract or utilize the said minerals without the permission of
less protest, the same although she claimed to be the owner of the said land. the State to which such minerals belong.
The Court of Appeals justified this by saying there is "no conflict of interest" The flaw in the reasoning of the respondent court is in supposing that the
between the owners of the surface rights and the owners of the sub-surface rights over the land could be used for both mining and non-mining purposes
rights. This is rather doctrine, for it is a well-known principle that the owner of simultaneously. The correct interpretation is that once minerals are discovered
piece of land has rights not only to its surface but also to everything in the land, whatever the use to which it is being devoted at the time, such
underneath and the airspace above it up to a reasonable height. 19 Under the use may be discontinued by the State to enable it to extract the minerals
aforesaid ruling, the land is classified as mineral underneath and agricultural therein in the exercise of its sovereign prerogative. The land is thus converted
on the surface, subject to separate claims of title. This is also difficult to to mineral land and may not be used by any private party, including the
understand, especially in its practical application. registered owner thereof, for any other purpose that will impede the mining
Under the theory of the respondent court, the surface owner will be planting operations to be undertaken therein, For the loss sustained by such owner, he
on the land while the mining locator will be boring tunnels underneath. The is of course entitled to just compensation under the Mining Laws or in
farmer cannot dig a well because he may interfere with the operations below appropriate expropriation proceedings. 21
and the miner cannot blast a tunnel lest he destroy the crops above. How Our holding is that Benguet and Atok have exclusive rights to the property in
deep can the farmer, and how high can the miner, go without encroaching on question by virtue of their respective mining claims which they validly
each other's rights? Where is the dividing line between the surface and the acquired before the Constitution of 1935 prohibited the alienation of all lands
sub-surface rights? of the public domain except agricultural lands, subject to vested rights
The Court feels that the rights over the land are indivisible and that the land existing at the time of its adoption. The land was not and could not have been
itself cannot be half agricultural and half mineral. The classification must be transferred to the private respondents by virtue of acquisitive prescription, nor
categorical; the land must be either completely mineral or completely could its use be shared simultaneously by them and the mining companies for
agricultural. In the instant case, as already observed, the land which was agricultural and mineral purposes.
originally classified as forest land ceased to be so and became mineral and WHEREFORE, the decision of the respondent court dated April 30, 1976, is SET
completely mineral once the mining claims were perfected. 20 As long as ASIDE and that of the trial court dated March 11, 1969, is REINSTATED,
mining operations were being undertaken thereon, or underneath, it did not without any pronouncement as to costs.
cease to be so and become agricultural, even if only partly so, because it was SO ORDERED.
enclosed with a fence and was cultivated by those who were unlawfully Teehankee, C.J., Narvasa, Gancayco and Grio-Aquino, JJ., concur.
occupying the surface.
What must have misled the respondent court is Commonwealth Act No. 137, Edna Collado v. Court of Appeals G.R. No. 107764, 390 SCRA 124
providing as follows: (2002)
Sec. 3. All mineral lands of the public domain and minerals belong to the Republic of the Philippines
State, and their disposition, exploitation, development or utilization, shall be SUPREME COURT
limited to citizens of the Philippines, or to corporations, or associations, at Manila
least 60% of the capital of which is owned by such citizens, subject to any FIRST DIVISION
existing right, grant, lease or concession at the time of the inauguration of G. R. No. 107764 October 4, 2002
government established under the Constitution. EDNA COLLADO, BERNARDINA TAWAS, JORETO C. TORRES, JOSE AMO,
SEC. 4. The ownership of, and the right to the use of land for agricultural, SERGIO L. MONTEALEGRE, VICENTE C. TORRES, JOSEPH L. NUEZ,
industrial, commercial, residential, or for any purpose other than mining does GLORIA SERRANO, DANILO FABREGAS, FERNANDO T. TORRES, LUZ G.
not include the ownership of, nor the right to extract or utilize, the minerals TUBUNGBANUA, CARIDAD T. TUTANA, JOSE C. TORRES, JR., IMELDA
which may be found on or under the surface. CAYLALUAD, ROSALIE TUTANA, NORMA ASTORIAS, MYRNA M.
SEC. 5. The ownership of, and the right to extract and utilize, the minerals LANCION, NORBERTO CAMILOTE, CECILIA MACARANAS, PEDRO
included within all areas for which public agricultural land patents are granted BRIONES, REMEDIOS BANTIGUE, DANTE L. MONTEALEGRE, AIDA T.
are excluded and excepted from all such patents. GADON, ARMANDO T. TORRES and FIDELITO ECO, petitioners,
58
vs. the latest of which was under Tax Declaration No. 7182 issued on 3 February
COURT OF APPEALS and REPUBLIC OF THE PHILIPPINES, thru the 1957 (Exhibit "I" and testimony of Mariano Leyva, supra).
Director of Lands, respondents, 4. ANGELINA REYNOSO, bought the property from Gregorio Camantique by
BOCKASANJO ISF AWARDEES ASSOCIATION, INC., LITA MENDOZA, virtue of a Deed of Sale on 3 February 1958 (Exhibit "H"). During the
MORADO PREFIDIGNO, TERESITA CRUZ and CALOMA MOISES, ownership of the property by Angelina Reynoso, Mariano Leyva the grandson
respondents/intervernors. of Sesinando Leyva, the previous owner, attended to the farm. (Testimony of
DECISION Mariano Leyva, supra). Angelina Reynoso declared the property in her name
CARPIO, J.: under Tax Declaration No. 7189 in 4 February 1958, under Tax Declaration No.
The Case 8775 on 3 August 1965, under Tax Declaration No. 16945 on 15 December
This Petition1 seeks to set aside the Decision of the Court of Appeals, 2 dated 1975, and under Tax Declaration No. 03-06145 on 25 June 1978.
June 22, 1992, in CA-G.R. SP No. 25597, which declared null and void the 5. MYRNA TORRES bought the property from Angelina Reynoso on 16 October
Decision3 dated January 30, 1991 of the Regional Trial Court of Antipolo, Rizal, 1982 through a Deed of Sale (Exhibit "G").
Branch 71, in LRC No. 269-A, LRC Rec. No. N-59179, confirming the imperfect 6. EDNA COLLADO bought the property from Myrna Torres in a Deed of Sale
title of petitioners over a parcel of land. dated 28 April 1984 (Exhibit "P-1" to "P-3").
The Facts 7. Additional owners BERNARDINA TAWAS, JORETO TORRES, JOSE AMO,
On April 25, 1985, petitioner Edna T. Collado filed with the land registration VICENTE TORRES and SERGIO MONTEALEGRE who bought portions of the
court an application for registration of a parcel of land with an approximate property from Edna Collado through a Deed of Sale on 6 November 1985
area of 1,200,766 square meters or 120.0766 hectares ("Lot" for brevity). The (Exhibit "Q" to "Q-3").
Lot is situated in Barangay San Isidro (formerly known as Boso-boso), Antipolo, 8. And more additional Owners JOSEPH NUNEZ, DIOSDADO ARENOS, DANILO
Rizal, and covered by Survey Plan Psu-162620. Attached to the application FABREGAS, FERNANDO TORRES, LUZ TUBUNGBANUA, CARIDAD TUTANA, JOSE
was the technical description of the Lot as Lot Psu-162620 signed by Robert C. TORRES JR., RODRIGO TUTANA, ROSALIE TUTANA, NORMA ASTORIAS, MYRNA
Pangyarihan, Officer-in-Charge of the Survey Division, Bureau of Lands, which LANCION, CHONA MARCIANO, CECILIA MACARANAS, PEDRO BRIONES,
stated, "[t]his survey is inside IN-12 Mariquina Watershed." On March 24, REMEDIOS BANTIQUE, DANTE MONTEALEGRE, ARMANDO TORRES, AIDA
1986, petitioner Edna T. Collado filed an Amended Application to include GADON and AMELIA M. MALAPAD bought portions of the property in a Deed of
additional co-applicants.4 Subsequently, more applicants joined (collectively Sale on 12 May 1986 (Exhibit "S" to "S-3").
referred to as "petitioners" for brevity).5 9. Co-owners DIOSDADO ARENOS, RODRIGO TUTANA, CHONA MARCIANO and
The Republic of the Philippines, through the Solicitor General, and the AMELIA MALAPAD jointly sold their shares to new OWNERS GLORIA R.
Municipality of Antipolo, through its Municipal Attorney and the Provincial SERRANO, IMELDA CAYLALUAD, NORBERTO CAMILOTE and FIDELITO ECO
Fiscal of Rizal, filed oppositions to petitioners application. In due course, the through a Deed of Sale dated 18 January 1987 (Exhibit "T" to "T-9")." 6
land registration court issued an order of general default against the whole During the hearing on January 9, 1991, only the assistant provincial
world with the exception of the oppositors. prosecutor appeared without the Solicitor General. For failure of the oppositors
Petitioners alleged that they have occupied the Lot since time immemorial. to present their evidence, the land registration court issued an order
Their possession has been open, public, notorious and in the concept of considering the case submitted for decision based on the evidence of the
owners. The Lot was surveyed in the name of Sesinando Leyva, one of their petitioners. The court later set aside the order and reset the hearing to
predecessors-in-interest, as early as March 22, 1902. Petitioners declared the January 14, 1991 for the presentation of the evidence of the oppositors. On
Lot for taxation purposes and paid all the corresponding real estate taxes. this date, counsel for oppositors failed to appear again despite due notice.
According to them, there are now twenty-five co-owners in pro-indiviso shares Hence, the court again issued an order submitting the case for decision based
of five hectares each. During the hearings, petitioners submitted evidence to on the evidence of the petitioners.
prove that there have been nine transfers of rights among them and their The Trial Courts Ruling
predecessors-in-interest, as follows: After appraisal of the evidence submitted by petitioners, the land registration
"1. SESINANDO LEYVA was the earliest known predecessor-in-interest of the court held that petitioners had adduced sufficient evidence to establish their
Applicants who was in actual, open, notorious and continuous possession of registrable rights over the Lot. Accordingly, the court rendered a decision
the property in the concept of owner. He had the property surveyed in his confirming the imperfect title of petitioners. We quote the pertinent portions
name on 22 March 1902 (Exhibit "W" and "W-1" testimonies of J. Torres on 16 of the courts decision, as follows:
December 1987 and Mariano Leyva on 29 December 1987). "From the evidence presented, the Court finds that from the testimony of the
2. DIOSDADO LEYVA, is the son of Sesinando Leyva, who inherited the witnesses presented by the Applicants, the property applied for is in actual,
property. He had the property resurveyed in his name on May 21-28, 1928 open, public and notorious possession by the applicants and their
(Exhibit "X" and "X-1"; testimony of Mariano Leyva, a son of Diosdado Leyva). predecessor-in-interest since time immemorial and said possession had been
3. GREGORIO CAMANTIQUE bought the property from Diosdado Leyva before testified to by witnesses Jimmy Torres, Mariano Leyva, Sergio Montealegre,
the Japanese Occupation of the Philippines during World War II. He owned and Jose Amo and one Chona who were all cross-examined by Counsel for
possessed the property until 1958. He declared the property for tax purposes, Oppositor Republic of the Philippines.

59
Evidence was likewise presented that said property was declared for taxation appropriation.
purposes in the names of the previous owners and the corresponding taxes On November 29, 1991, Bockasanjo ISF Awardees Association, Inc., an
were paid by the Applicants and the previous owners and said property was association of holders of certificates of stewardship issued by the Department
planted to fruit bearing trees; portions to palay and portions used for grazing of Environment and Natural Resources ("DENR" for brevity) under its
purposes. Integrated Social Forestry Program ("ISF" for brevity), filed with the Court of
To the mind of the Court, Applicants have presented sufficient evidence to Appeals a Motion for Leave to Intervene and to Admit Petition-In-Intervention.
establish registrable title over said property applied for by them. They likewise opposed the registration and asserted that the Lot, which is
On the claim that the property applied for is within the Marikina Watershed, situated inside the Marikina Watershed Reservation, is inalienable. They
the Court can only add that all Presidential Proclamations like the claimed that they are the actual occupants of the Lot pursuant to the
Proclamation setting aside the Marikina Watershed are subject to "private certificates of stewardship issued by the DENR under the ISF for tree planting
rights." purposes.
In the case of Municipality of Santiago vs. Court of Appeals, 120 SCRA 734, The Court of Appeals granted the motion to intervene verbally during the
1983 "private rights" is proof of acquisition through (sic) among means of preliminary conference held on April 6, 1992. During the preliminary
acquisition of public lands. conference, all the parties as represented by their respective counsels agreed
In the case of Director of Lands vs. Reyes, 68 SCRA 193-195, by "private that the only issue for resolution was whether the Lot in question is part of the
rights" means that applicant should show clear and convincing evidence that public domain.8
the property in question was acquired by applicants or their ancestors either The Court of Appeals Ruling
by composition title from the Spanish government or by Possessory In a decision dated June 22, 1992, the Court of Appeals granted the petition
Information title, or any other means for the acquisition of public lands xxx" and declared null and void the decision dated January 30, 1991 of the land
(underscoring supplied). registration court. The Court of Appeals explained thus:
The Court believes that from the evidence presented as above stated, "Under the Regalian Doctrine, which is enshrined in the 1935 (Art. XIII, Sec. 1),
Applicants have acquired private rights to which the Presidential Proclamation 1973 (Art. XIV, Sec. 8), and 1987 Constitution (Art. XII, Sec. 2), all lands of the
setting aside the Marikina Watershed should be subject to such private rights. public domain belong to the State. An applicant, like the private respondents
At any rate, the Court notes that evidence was presented by the applicants herein, for registration of a parcel of land bears the burden of overcoming the
that as per Certification issued by the Bureau of Forest Development dated presumption that the land sought to be registered forms part of the public
March 18, 1980, the area applied for was verified to be within the area domain (Director of Lands vs. Aquino, 192 SCRA 296).
excluded from the operation of the Marikina Watershed Lands Executive Order A positive Act of government is needed to declassify a public land and to
No. 33 dated July 26, 1904 per Proclamation No. 1283 promulgated on June convert it into alienable or disposable land for agricultural or other purposes
21, 1974 which established the Boso-boso Town Site Reservation, amended by (Republic vs. Bacas, 176 SCRA 376).
Proclamation No. 1637 dated April 18, 1977 known as the Lungsod Silangan In the case at bar, the private respondents failed to present any evidence
Townsite Reservation. (Exhibit "K")."7 whatsoever that the land applied for as described in Psu-162620 has been
In a motion dated April 5, 1991, received by the Solicitor General on April 6, segregated from the bulk of the public domain and declared by competent
1991, petitioners alleged that the decision dated January 30, 1991 confirming authority to be alienable and disposable. Worse, the technical description of
their title had become final after the Solicitor General received a copy of the Psu-162620 signed by Robert C. Pangyarihan, Officer-in-Charge, Survey
decision on February 18, 1991. Petitioners prayed that the land registration Division, Bureau of Lands, which was attached to the application of private
court order the Land Registration Authority to issue the necessary decree in respondents, categorically stated that "This survey is inside IN-12 Mariquina
their favor over the Lot. Watershed.""
On April 11, 1991, the Solicitor General inquired from the Provincial Prosecutor That the land in question is within the Marikina Watershed Reservation is
of Rizal whether the land registration court had already rendered a decision confirmed by the Administrator of the National Land Titles and Deeds in a
and if so, whether the Provincial Prosecutor would recommend an appeal. Report, dated March 2, 1988, submitted to the respondent Court in LR Case
However, the Provincial Prosecutor failed to answer the query. No. 269-A. These documents readily and effectively negate the allegation in
According to the Solicitor General, he received on April 23, 1991 a copy of the private respondent Collados application that "said parcel of land known as
land registration courts decision dated January 30, 1991, and not on February Psu-162620 is not covered by any form of title, nor any public land application
18, 1991 as alleged by petitioners in their motion. and are not within any government reservation (Par. 8, Application; Emphasis
In the meantime, on May 7, 1991, the land registration court issued an order supplied). The respondent court could not have missed the import of these
directing the Land Regulation Authority to issue the corresponding decree of vital documents which are binding upon the courts inasmuch as it is the
registration in favor of the petitioners. exclusive prerogative of the Executive Department to classify public lands.
On August 6, 1991, the Solicitor General filed with the Court of Appeals a They should have forewarned the respondent judge from assuming jurisdiction
Petition for Annulment of Judgment pursuant to Section 9(2) of BP Blg. 129 on over the case.
the ground that there had been no clear showing that the Lot had been "x x x inasmuch as the said properties applied for by petitioners are part of
previously classified as alienable and disposable making it subject to private the public domain, it is the Director of Lands who has jurisdiction in the

60
disposition of the same (subject to the approval of the Secretary of Natural Petitioners arguments find no basis in law.
Resources and Environment), and not the courts. x x x Even assuming that The Regalian Doctrine: An Overview
petitioners did have the said properties surveyed even before the same was Under the Regalian Doctrine, all lands not otherwise appearing to be clearly
declared to be part of the Busol Forest Reservation, the fact remains that it within private ownership are presumed to belong to the State. 11 The Spaniards
was so converted into a forest reservation, thus it is with more reason that this first introduced the doctrine to the Philippines through the Laws of the Indies
action must fail. Forest lands are inalienable and possession thereof, no and the Royal Cedulas, specifically, Law 14, Title 12, Book 4 of the Novisima
matter how long, cannot convert the same into private property. And courts Recopilacion de Leyes de las Indias 12 which laid the foundation that "all lands
are without jurisdiction to adjudicate lands within the forest zone. (Heirs of that were not acquired from the Government, either by purchase or by grant,
Gumangan vs. Court of Appeals. 172 SCRA 563; Emphasis supplied). belong to the public domain." 13 Upon the Spanish conquest of the Philippines,
Needless to say, a final judgment may be annulled on the ground of lack of ownership of all "lands, territories and possessions" in the Philippines passed
jurisdiction, fraud or that it is contrary to law (Panlilio vs. Garcia, 119 SCRA to the Spanish Crown.14
387, 391) and a decision rendered without jurisdiction is a total nullity and The Laws of the Indies were followed by the Ley Hipotecaria or the Mortgage
may be struck down at any time (Suarez vs. Court of Appeals, 186 SCRA Law of 1893. The Spanish Mortgage Law provided for the systematic
339)."9 registration of titles and deeds as well as possessory claims. The Royal Decree
Hence, the instant petition. of 1894 or the "Maura Law" partly amended the Mortgage Law as well as the
The Issues Law of the Indies. The Maura Law was the last Spanish land law promulgated
The issues raised by petitioners are restated as follows: in the Philippines. It required the "adjustment" or registration of all agricultural
I lands, otherwise the lands would revert to the state. 15
WHETHER THE COURT OF APPEALS ERRED OR GRAVELY ABUSED ITS Four years later, Spain ceded to the government of the United States all
DISCRETION IN REVERSING THE DECISION OF THE TRIAL COURT GRANTING rights, interests and claims over the national territory of the Philippine Islands
THE APPLICATION OF THE PETITIONERS FOR CONFIRMATION OF TITLE; through the Treaty of Paris of December 10, 1898. In 1903, the United States
II colonial government, through the Philippine Commission, passed Act No. 926,
WHETHER THE COURT OF APPEALS ERRED OR GRAVELY ABUSED ITS the first Public Land Act, which was described as follows:
DISCRETION IN GIVING DUE COURSE TO THE PETITION FOR ANNULMENT OF "Act No. 926, the first Public Land Act, was passed in pursuance of the
JUDGMENT FILED BY THE REPUBLIC LONG AFTER THE DECISION OF THE TRIAL provisions of the Philippine Bill of 1902. The law governed the disposition of
COURT HAD BECOME FINAL; lands of the public domain. It prescribed rules and regulations for the
III homesteading, selling and leasing of portions of the public domain of the
WHETHER THE COURT OF APPEALS ERRED OR GRAVELY ABUSED ITS Philippine Islands, and prescribed the terms and conditions to enable persons
DISCRETION IN GIVING DUE COURSE TO THE INTERVENORS PETITION FOR to perfect their titles to public lands in the Islands. It also provided for the
INTERVENTION WHICH WAS FILED OUT OF TIME OR LONG AFTER THE "issuance of patents to certain native settlers upon public lands," for the
DECISION OF THE TRIAL COURT HAD BECOME FINAL. establishment of town sites and sale of lots therein, for the completion of
The Courts Ruling imperfect titles, and for the cancellation or confirmation of Spanish
The petition is bereft of merit. concessions and grants in the Islands." In short, the Public Land Act operated
First Issue: whether petitioners have registrable title over the Lot. on the assumption that title to public lands in the Philippine Islands remained
There is no dispute that Executive Order No. 33 ("EO 33" for brevity) dated in the government; and that the governments title to public land sprung from
July 26, 190410 established the Marikina Watershed Reservation ("MWR" for the Treaty of Paris and other subsequent treaties between Spain and the
brevity) situated in the Municipality of Antipolo, Rizal. Petitioners even United States. The term "public land" referred to all lands of the public domain
concede that the Lot, described as Lot Psu-162620, is inside the technical, whose title still remained in the government and are thrown open to private
literal description of the MWR. However, the main thrust of petitioners claim appropriation and settlement, and excluded the patrimonial property of the
over the Lot is that "all Presidential proclamations like the proclamation government and the friar lands."16
setting aside the Marikina Watershed Reservation are subject to private Thus, it is plain error for petitioners to argue that under the Philippine Bill of
rights." They point out that EO 33 contains a saving clause that the 1902 and Public Land Act No. 926, mere possession by private individuals of
reservations are "subject to existing private rights, if any there be." Petitioners lands creates the legal presumption that the lands are alienable and
contend that their claim of ownership goes all the way back to 1902, when disposable.
their known predecessor-in-interest, Sesinando Leyva, laid claim and Act 2874, the second Public Land Act, superseded Act No. 926 in 1919. After
ownership over the Lot. They claim that the presumption of law then the passage of the 1935 Constitution, Commonwealth Act No. 141 ("CA 141"
prevailing under the Philippine Bill of 1902 and Public Land Act No. 926 was for brevity) amended Act 2874 in 1936. CA 141, as amended, remains to this
that the land possessed and claimed by individuals as their own are day as the existing general law governing the classification and disposition of
agricultural lands and therefore alienable and disposable. They conclude that lands of the public domain other than timber and mineral lands. 17
private rights were vested on Sesinando Leyva before the issuance of EO 33, In the meantime, in order to establish a system of registration by which
thus excluding the Lot from the Marikina Watershed Reservation. recorded title becomes absolute, indefeasible and imprescriptible, the

61
legislature passed Act 496, otherwise known as the Land Registration Act, brevity).28 The Court defined watershed as "an area drained by a river and its
which took effect on February 1, 1903. Act 496 placed all registered lands in tributaries and enclosed by a boundary or divide which separates it from
the Philippines under the Torrens system.18 The Torrens system requires the adjacent watersheds." However, the Court also recognized that:
government to issue a certificate of title stating that the person named in the "The definition does not exactly depict the complexities of a watershed. The
title is the owner of the property described therein, subject to liens and most important product of a watershed is water which is one of the most
encumbrances annotated on the title or reserved by law. The certificate of title important human necessit(ies). The protection of watershed ensures an
is indefeasible and imprescriptible and all claims to the parcel of land are adequate supply of water for future generations and the control of flashfloods
quieted upon issuance of the certificate. 19 PD 1529, known as the Property that not only damage property but also cause loss of lives. Protection of
Registration Decree enacted on June 11, 1978, 20 amended and updated Act watersheds is an "intergenerational" responsibility that needs to be answered
496. now."
The 1935, 1973, 1987 Philippine Constitutions Article 67 of the Water Code of the Philippines (PD 1067) provides:
The 1935, 1973 and 1987 Constitutions adopted the Regalian doctrine "Art. 67. Any watershed or any area of land adjacent to any surface water or
substituting, however, the state, in lieu of the King, as the owner of all lands overlying any ground water may be declared by the Department of Natural
and waters of the public domain.21 Justice Reynato S. Puno, in his separate Resources as a protected area. Rules and Regulations may be promulgated by
opinion in Cruz vs. Secretary of Environment and Natural Resources, 22 such Department to prohibit or control such activities by the owners or
explained thus: occupants thereof within the protected area which may damage or cause the
"One of the fixed and dominating objectives of the 1935 Constitutional deterioration of the surface water or ground water or interfere with the
Convention was the nationalization and conservation of the natural resources investigation, use, control, protection, management or administration of such
of the country. There was an overwhelming sentiment in the Convention in waters."
favor of the principle of state ownership of natural resources and the adoption The Court in Sta. Rosa Realty also recognized the need to protect watershed
of the Regalian doctrine. State ownership of natural resources was seen as a areas and took note of the report of the Ecosystems Research and
necessary starting point to secure recognition of the states power to control Development Bureau (ERDB), a research arm of the DENR, regarding the
their disposition, exploitation, development, or utilization. The delegates to environmental assessment of the Casile and Kabanga-an river watersheds
the Constitutional Convention very well knew that the concept of State involved in that case. That report concluded as follows:
ownership of land and natural resources was introduced by the Spaniards, "The Casile barangay covered by CLOA in question is situated in the heartland
however, they were not certain whether it was continued and applied by the of both watersheds. Considering the barangays proximity to the Matangtubig
Americans. To remove all doubts, the Convention approved the provision in waterworks, the activities of the farmers which are in conflict with proper soil
the Constitution affirming the Regalian doctrine." and water conservation practices jeopardize and endanger the vital
Thus, Section 1, Article XIII 23 of the 1935 Constitution, on "Conservation and waterworks. Degradation of the land would have double edge detrimental
Utilization of Natural Resources" barred the alienation of all natural resources effects. On the Casile side this would mean direct siltation of the Mangumit
except public agricultural lands, which were the only natural resources the river which drains to the water impounding reservoir below. On the Kabanga-
State could alienate. The 1973 Constitution reiterated the Regalian doctrine in an side, this would mean destruction of forest covers which acts as recharged
Section 8, Article XIV24 on the "National Economy and the Patrimony of the areas of the Matangtubig springs. Considering that the people have little if no
Nation". The 1987 Constitution reaffirmed the Regalian doctrine in Section 2 of direct interest in the protection of the Matangtubig structures they couldnt
Article XII25 on "National Economy and Patrimony". care less even if it would be destroyed.
Both the 1935 and 1973 Constitutions prohibited the alienation of all natural The Casile and Kabanga-an watersheds can be considered a most vital life
resources except agricultural lands of the public domain. The 1987 support system to thousands of inhabitants directly and indirectly affected by
Constitution readopted this policy. Indeed, all lands of the public domain as it. From these watersheds come the natural God-given precious resource
well as all natural resources enumerated in the Philippine Constitution belong water. x x x
to the State. Clearing and tilling of the lands are totally inconsistent with sound watershed
Watershed Reservation is a Natural Resource management. More so, the introduction of earth disturbing activities like road
The term "natural resource" includes "not only timber, gas, oil coal, minerals, building and erection of permanent infrastructures. Unless the pernicious
lakes, and submerged lands, but also, features which supply a human need agricultural activities of the Casile farmers are immediately stopped, it would
and contribute to the health, welfare, and benefit of a community, and are not be long before these watersheds would cease to be of value. The impact
essential to the well-being thereof and proper enjoyment of property devoted of watershed degradation threatens the livelihood of thousands of people
to park and recreational purposes."26 dependent upon it. Toward this, we hope that an acceptable comprehensive
In Sta. Rosa Realty Development Corp. vs. Court of Appeals, et al., 27 the Court watershed development policy and program be immediately formulated and
had occasion to discourse on watershed areas. The Court resolved the issue of implemented before the irreversible damage finally happens."
whether the parcel of land which the Department of Environment and Natural The Court remanded the case to the Department of Agriculture and
Resources had assessed to be a watershed area is exempt from the coverage Adjudication Board or DARAB to re-evaluate and determine the nature of the
of RA No. 6657 or the Comprehensive Agrarian Reform Law ("CARL" for parcels of land involved in order to resolve the issue of its coverage by the

62
CARL. title because of the failure to complete the required period of possession,
Sta. Rosa Realty gives us a glimpse of the dangers posed by the misuse of whether under the original Section 48 (b) of CA 141 prior to the issuance of EO
natural resources such as watershed reservations which are akin to forest 33, or under the amendment by RA 1942 and PD 1073.
zones. Population growth and industrialization have taken a heavy toll on the There is no proof that prior to the issuance of EO 33 in 1904, petitioners had
environment. Environmental degradation from unchecked human activities acquired ownership or title to the Lot either by deed or by any other mode of
could wreak havoc on the lives of present and future generations. Hence, by acquisition from the State, as for instance by acquisitive prescription. As of
constitutional fiat, natural resources remain to this day inalienable properties 1904, Sesinando Leyva had only been in possession for two years. Verily,
of the State. petitioners have not possessed the parcel of land in the manner and for the
Viewed under this legal and factual backdrop, did petitioners acquire, as they number of years required by law for the confirmation of imperfect title.
vigorously argue, private rights over the parcel of land prior to the issuance of Second, assuming that the Lot was alienable and disposable land prior to the
EO 33 segregating the same as a watershed reservation? issuance of EO 33 in 1904, EO 33 reserved the Lot as a watershed. Since then,
The answer is in the negative. the Lot became non-disposable and inalienable public land. At the time
First. An applicant for confirmation of imperfect title bears the burden of petitioners filed their application on April 25, 1985, the Lot has been reserved
proving that he meets the requirements of Section 48 of CA 141, as amended. as a watershed under EO 33 for 81 years prior to the filing of petitioners
He must overcome the presumption that the land he is applying for is part of application.
the public domain and that he has an interest therein sufficient to warrant The period of occupancy after the issuance of EO 33 in 1904 could no longer
registration in his name arising from an imperfect title. An imperfect title may be counted because as a watershed reservation, the Lot was no longer
have been derived from old Spanish grants such as a titulo real or royal grant, susceptible of occupancy, disposition, conveyance or alienation. Section 48 (b)
a concession especial or special grant, a composicion con el estado or of CA 141, as amended, applies exclusively to alienable and disposable public
adjustment title, or a titulo de compra or title through purchase. 29 Or, that he agricultural land. Forest lands, including watershed reservations, are excluded.
has had continuous, open and notorious possession and occupation of It is axiomatic that the possession of forest lands or other inalienable public
agricultural lands of the public domain under a bona fide claim of ownership lands cannot ripen into private ownership. In Municipality of Santiago, Isabela
for at least thirty years preceding the filing of his application as provided by vs. Court of Appeals,32 the Court declared that inalienable public lands -
Section 48 (b) CA 141. "x x x cannot be acquired by acquisitive prescription. Prescription, both
Originally, Section 48(b) of CA 141 provided for possession and occupation of acquisitive and extinctive, does not run against the State.
lands of the public domain since July 26, 1894. This was superseded by RA The possession of public land, however long the period may have extended,
1942 which provided for a simple thirty-year prescriptive period of occupation never confers title thereto upon the possessor because the statute of
by an applicant for judicial confirmation of an imperfect title. The same, limitations with regard to public land does not operate against the State,
however, has already been amended by Presidential Decree No. 1073, unless the occupant can prove possession and occupation of the same under
approved on January 25, 1977, the law prevailing at the time petitioners claim of ownership for the required number of years to constitute a grant from
application for registration was filed on April 25, 1985. 30 As amended, Section the State. "
48 (b) now reads: Third, Gordula vs. Court of Appeals 33 is in point. In Gordula, petitioners did not
"(b) Those who by themselves or through their predecessors-in-interest have contest the nature of the land. They admitted that the land lies in the heart of
been in open, continuous, exclusive and notorious possession and occupation the Caliraya-Lumot River Forest Reserve, which Proclamation No. 573
of agricultural lands of the public domain, under a bona fide claim of classified as inalienable. The petitioners in Gordula contended, however, that
acquisition or ownership, for at least thirty years immediately preceding the Proclamation No. 573 itself recognizes private rights of landowners prior to the
filing of the application for confirmation of title, except when prevented by reservation. They claim to have established their private rights to the subject
wars or force majeure. Those shall be conclusively presumed to have land. The Court ruled:
performed all the conditions essential to a Government grant and shall be "We do not agree. No public land can be acquired by private persons without
entitled to a certificate of title under the provisions of this chapter." any grant, express or implied from the government; it is indispensable that
Interpreting Section 48 (b) of CA 141, the Court stated that the Public Land Act there be a showing of a title from the state. The facts show that petitioner
requires that the applicant must prove the following: Gordula did not acquire title to the subject land prior to its reservation under
"(a) that the land is alienable public land and (b) that his open, continuous, Proclamation No. 573. He filed his application for free patent only in January,
exclusive and notorious possession and occupation of the same must either be 1973, more than three (3) years after the issuance of Proclamation No. 573 in
since time immemorial or for the period prescribed in the Public Land Act. June, 1969. At that time, the land, as part of the Caliraya-Lumot River Forest
When the conditions set by law are complied with, the possessor of the land, Reserve, was no longer open to private ownership as it has been classified as
by operation of law, acquires a right to a grant, a government grant, without public forest reserve for the public good.
the necessity of a certificate of title being issued." 31 Nonetheless, petitioners insist that the term, "private rights," in Proclamation
Petitioners do not claim to have documentary title over the Lot. Their right to No. 573, should not be interpreted as requiring a title. They opine that it
register the Lot is predicated mainly upon continuous possession since 1902. suffices if the claimant "had occupied and cultivated the property for so many
Clearly, petitioners were unable to acquire a valid and enforceable right or number of years, declared the land for taxation purposes, [paid] the

63
corresponding real estate taxes [which are] accepted by the government, and private rights, if any there be, and to future subdivision survey in accordance
[his] occupancy and possession [is] continuous, open and unmolested and with the development plan to be prepared and approved by the Department of
recognized by the government. Prescinding from this premise, petitioners urge Local Government and Community Development, which parcels are more
that the 25-year possession by petitioner Gordula from 1944 to 1969, albeit particularly described as follows:
five (5) years short of the 30-year possession required under Commonwealth Lot A (Part of Watershed Reservation)
Act (C.A.) No. 141, as amended, is enough to vest upon petitioner Gordula the A parcel of land (Lot A of Proposed Poor Mans Baguio, being a portion of the
"private rights" recognized and respected in Proclamation No. 573. Marikina Watershed, IN-2), situated in the municipality of Antipolo, Province of
The case law does not support this submission. In Director of Lands vs. Reyes, Rizal, Island of Luzon, beginning at a point marked "1" on sketch plan, being
we held that a settler claiming the protection of "private rights" to exclude his N-74-30 E, 8480.00 meters more or less, from BLLM 1, Antipolo, Rizal; thence
land from a military or forest reservation must show "x x x by clear and N 33 28 W 1575.00 m. to point 2; thence N 40 26 W 1538.50 m. to point 3;
convincing evidence that the property in question was acquired by [any] x x x thence N 30 50W 503.17 m. to point 4; thence N 75 02 W 704.33 m. to point
means for the acquisition of public lands." 5; thence N 14 18 W 1399.39 m. to point 6; thence N 43 25 W 477.04 m. to
In fine, one claiming "private rights" must prove that he has complied with point 7; thence N 71 38 W 458.36 m. to point 8; thence N 31 05 W 1025.00
C.A. No. 141, as amended, otherwise known as the Public Land Act, which m. to point 9; thence Due North 490.38 m. to point 10; thence Due North
prescribes the substantive as well as the procedural requirements for 1075.00 m. to point 11; thence Due East 1000.00 m. to point 12; thence Due
acquisition of public lands. This law requires at least thirty (30) years of open, East 1000.00 m. to point 13; thence Due East 1000.00 m. to point 14; thence
continuous, exclusive and notorious possession and possession of agricultural Due East 1000.00 m. to point 15; thence Due East 1000.00 m. to point 16;
lands of the public domain, under a bona fide claim of acquisition, thence Due East 1000.00 m. to point 17; thence Due East 1075.00 m. to point
immediately preceding the filing of the application for free patent. The 18; thence Due South 1000.00 m. to point 19; thence Due South 1000.00 m.
rationale for the 30-year period lies in the presumption that the land applied to point 20; thence Due South 1000.00 m. to point 21; thence Due South
for pertains to the State, and that the occupants and/or possessors claim an 1000.00 m. to point 22; thence Due South 1000.00 m. to point 23; thence Due
interest therein only by virtue of their imperfect title or continuous, open and South 1000.00 m. to point 24; thence Due South 1075.00 m. to point 25;
notorious possession." thence Due West 1000.00 m. to point 26; thence Due West 1000.00 m. to
Next, petitioners argue that assuming no private rights had attached to the point 27; thence Due West 636.56 m. to point of beginning. Containing an
Lot prior to EO 33 in 1904, the President of the Philippines had subsequently area of three thousand seven hundred eighty (3,780) Hectares, more or less.
segregated the Lot from the public domain and made the Lot alienable and Lot B (Alienable and Disposable Land)
disposable when he issued Proclamation No. 1283 on June 21, 1974. A parcel of land (Lot B of Proposed Poor Mans Baguio, being a portion of
Petitioners contend that Proclamation No. 1283 expressly excluded an area of alienable and disposable portion of public domain) situated in the municipality
3,780 hectares from the MWR and made the area part of the Boso-boso of Antipolo, Province of Rizal, Island of Luzon. Beginning at a point marked "1"
Townsite Reservation. Petitioners assert that Lot Psu-162620 is a small part of on sketch plan being N 74 30 E., 8430.00 m., more or less, from BLLM 1.
this excluded town site area. Petitioners further contend that town sites are Antipolo, Rizal; thence Due West 363.44 m. to point 2; thence Due West
considered alienable and disposable under CA 141. 1000.00 m. to point 3; thence Due West 100.00 m. to point 4; thence Due
Proclamation No. 1283 reads thus: West 1000.00 m. to point 5; thence Due West 1075.00 m. to point 6; thence
"PROCLAMATION NO. 1283 Due North 1000.00 m. to point 7; thence Due North 1000.00 m. to point 8;
EXCLUDING FROM THE OPERATION EXECUTIVE ORDER NO. 33, DATED JULY 26, thence Due North 1000.00 m. to point 9; thence Due North 1000.00 m. to
1904, AS AMENDED BY EXECUTIVE ORDERS NOS. 14 AND 16, BOTH SERIES OF point 10; thence Due North 1000.00 m. to point 11; thence Due North 509.62
1915, WHICH ESTABLISHED THE WATERSHED RESERVATION SITUATED IN THE m. to point 12; thence S. 31 05 E 1025.00 m. to point 13; thence S 71 38 E
MUNICIPALITY OF ANTIPOLO, PROVINCE OF RIZAL, ISLAND OF LUZON, A 458.36 m. to point 14; thence S 43 25 E 477.04 m. to point 15; thence S 14
CERTAIN PORTION OF THE LAND EMBRACED THEREIN AND RESERVING THE 18 E 1399.39 m. to point 16; thence S 75 02 E 704.33 m. to point 17; thence
SAME, TOGETHER WITH THE ADJACENT PARCEL OF LAND OF THE PUBLIC S. 30 50 E 503.17 m. to point 18; thence S 40 26 E 1538.50 m. to point 19;
DOMAIN, FOR TOWNSITE PURPOSES UNDER THE PROVISIONS OF CHAPTER XI thence s 33 23 e 1575.00 m to point of beginning. Containing an area of one
OF THE PUBLIC LAND ACT. thousand two hundred twenty five (1,225) Hectares, more or less.
Upon recommendation of the Secretary of Agriculture and Natural Resources Note: All data are approximate and subject to change based on future survey.
and pursuant to the authority vested in me by law, I, FERDINAND E. MARCOS, IN WITNESS WHEREOF, I Have hereunto set my hand and caused the seal of
President of the Philippines, do hereby, exclude from the operation of the Republic of the Philippines to be affixed.
Executive Order No. 33 dated July 26, 1904, as amended by Executive Orders Done in the City of Manila, this 21st day of June, in the year of Our Lord,
Nos. 14 and 16, both series of 1915, which established the Watershed nineteen hundred and seventy-four.
Reservation situated in the Municipality of Antipolo, Province of Rizal, Island of (Sgd.) FERDINAND E. MARCOS
Luzon, certain portions of land embraced therein and reserve the same, President
together with the adjacent parcel of land of the public domain, for townsite Republic of the Philippines"
purposes under the provisions of Chapter XI of the Public Land Act, subject to Proclamation No. 1283 has since been amended by Proclamation No. 1637

64
issued on April 18, 1977. Proclamation No. 1637 revised the area and location The principal document presented by petitioners to prove the private
of the proposed townsite. According to then DENR Secretary Victor O. Ramos, character of the Lot is the Certification of the Bureau of Forest Development
Proclamation No. 1637 excluded Lot A (of which the Lot claimed by petitioners dated March 18, 1986 that the Lot is excluded from the Marikina Watershed
is part) for townsite purposes and reverted it to MWR coverage. 34 Proclamation (Exh. R). The Certification reads:
No. 1637 reads: "Republic of the Philippines
"PROCLAMATION NO. 1637 Ministry of Natural Resources
AMENDING PROCLAMATION NO. 1283, DATED JUNE 21, 1974, WHICH BUREAU OF FOREST DEVELOPMENT
ESTABLISHED THE TOWNSITE RESERVATION IN THE MUNICIPALITIES OF REGION IV
ANTIPOLO AND SAN MATEO, PROVINCE OF RIZAL, ISLAND OF LUZON BY EL AL Building
INCREASING THE AREA AND REVISING THE TECHNICAL DESCRIPTION OF THE 100 Quezon Avenue, Quezon City
LAND EMBRACED THEREIN, AND REVOKING PROCLAMATION NO. 765 DATED MAR 18 1986
OCTOBER 26, 1970 THAT RESERVED PORTIONS OF THE AREA AS VERIFICATION ON THE STATUS OF LAND:
RESETTLEMENT SITE. TO WHOM IT MAY CONCERN:
Upon recommendation of the Secretary of Natural Resources and pursuant to This is to certify that the tract of land situated in Barangay San Isidro,
the authority vested in me by law, I, FERDINAND E. MARCOS, President of the Antipolo, Rizal, containing an area of 1,269,766 square meters, as shown and
Philippines, do hereby amend Proclamation No. 1283, dated June 21, 1974 described on the reverse side hereof, surveyed by Geodetic Engineer Telesforo
which established the townsite reservation in the municipalities of Antipolo Cabading for Angelina C. Reynoso, is verified to be within the area excluded
and San Mateo, Province of Rizal, Island of Luzon, by increasing the area and from the operation of Marikina Watershed Reservation established under
revising the technical descriptions of the land embraced therein, subject to Executive Order No. 33 dated July 26, 1904 per Proclamation No. 1283,
private rights, if any there be, which parcel of land is more particularly promulgated on June 21, 1974, which established the Boso-Boso Townsite
described as follows: Reservation, amended by proclamation No. 1637 dated April 18, 1977 known
(Proposed Lungsod Silangan Townsite) as Lungsod Silangan Townsite Reservation.
A PARCEL OF LAND (Proposed Lungsod Silangan Townsite Reservation Subject area also falls within the bounds of Bagong Lipunan Site under P.D.
amending the area under SWO-41762 establishing the Bagong Silangan 1396 dated June 2, 1978 under the sole jurisdiction of the Ministry of Human
Townsite Reservation) situated in the Municipalities of Antipolo, San Mateo, Settlements, to the exclusion of any other government agencies.
and Montalban, Province of Rizal, Island of Luzon. Bounded on the E., along This verification is made upon the request of the Chief, Legal Staff, R-4 as
lines 1-2-3-4-5-6-7-8-9-10-11-12-13-14-15-16-17-18-19-20-21-22-23 by the contained in his internal memorandum dated March 18, 1986.
Marikina Watershed Reservation (IN-12); on the S., along lines 23-24-25 by the Verified by:
portion of Antipolo; on the W., along lines 25-26-27-28-29-30 by the (Sgd) ROMEO C. PASCUBILLO
Municipalities of Montalban, San Mateo; and on the N., along lines 30-31-32- Cartographer II
33-34-35-36-37-38-39-40-41-42-43-44 by the Angat Watershed Reservation. Checked by:
Beginning at a point marked "1" on the Topographic Maps with the Scale of (Sgd) ARMENDO R. CRUZ
1:50,000 which is the identical corner 38 IN-12, Marikina Watershed Supervising Cartographer
Reservation. ATTESTED:
xxx xxx xxx (Sgd) LUIS G. DACANAY
NOTE: All data are approximate and subject to change based on future survey. Chief, Forest Engineering & Infrastructure Section"
Proclamation No. 765 dated October 26, 1970, which covered areas entirely The above certification on which petitioners rely that a reclassification had
within the herein Lungsod Silangan Townsite, is hereby revoked accordingly. occurred, and that the Lot is covered by the reclassification, is contradicted by
IN WITNESS WHEREOF, I have hereunto set my hand and caused the seal of several documents submitted by the Solicitor General before the land
the Republic of the Philippines to be affixed. registration court.
Done in the City of Manila, this 18th day of April, in the year of Our Lord, The Solicitor General submitted to the land registration court a Report 37 dated
nineteen hundred and seventy-seven. March 2, 1988, signed by Administrator Teodoro G. Bonifacio of the then
(Sgd.) FERDINAND E. MARCOS National Land Titles and Deeds Registration Administration, confirming that
President of the Philippines" the Lot described in Psu-162620 forms part of the MWR. He thus
A positive act (e.g., an official proclamation) of the Executive Department is recommended the dismissal of the application for registration. The Report
needed to declassify land which had been earlier classified as a watershed states:
reservation and to convert it into alienable or disposable land for agricultural "COMES NOW the Administrator of the National Land Titles and Deeds
or other purposes.35 Unless and until the land classified as such is released in Registration Commission and to this Honorable Court respectfully reports that:
an official proclamation so that it may form part of the disposable agricultural 1. A parcel of land described in plan Psu-162620 situated in the Barrio of San
lands of the public domain, the rules on confirmation of imperfect title do not Isidro, Municipality of Antipolo, Province of Rizal, is applied for registration of
apply.36 title in the case at bar.

65
2. After plotting plan Psu-162620 in our Municipal Index Map it was found that issuance of EO 33 in 1904. Petitioners case falters even more because of the
a portion of the SW, described as Lot 3 in plan Psu-173790 was previously the issuance of Proclamation No. 1637 on April 18, 1977. According to then DENR
subject of registration in Land Reg. Case No. N-9578, LRC Record No. N-55948 Secretary Victor Ramos, Proclamation No. 1637 reverted Lot A or the townsite
and was issued Decree No. N-191242 on April 4, 1986 in the name of Apolonia reservation, where petitioners' Lot is supposedly situated, back to the MWR.
Garcia, et al., pursuant to the Decision and Order for Issuance of the Decree Finally, it is of no moment if the areas of the MWR are now fairly populated
dated February 8, 1984 and March 6, 1984, respectively, and the remaining and vibrant communities as claimed by petitioners. The following ruling may
portion of plan Psu-162620 is inside IN-12, Marikina Watershed. x x x be applied to this case by analogy:
"WHEREFORE, this matter is respectfully submitted to the Honorable Court for "A forested area classified as forest land of the public domain does not lose
its information and guidance with the recommendation that the application in such classification simply because loggers or settlers may have stripped it of
the instant proceedings be dismissed, after due hearing (Underlining its forest cover. Parcels of land classified as forest land may actually be
supplied)." covered with grass or planted to crops by kaingin cultivators or other farmers.
Likewise, in a letter38 dated November 11, 1991, the Deputy Land Inspector, "Forest lands" do not have to be on mountains or in out of the way places.
DENR, Region IV, Community Environment and Natural Resources Office, Swampy areas covered by mangrove trees, nipa palms and other trees
Antipolo, Rizal, similarly confirmed that the Lot is within the MWR. The letter growing in brackish or sea water may also be classified as forest land. The
states: classification is descriptive of its legal nature or status and does not have to
"That the land sought to be registered is situated at San Isidro (Boso-boso), be descriptive of what the land actually looks like. Unless and until the land
Antipolo, Rizal, with an area of ONE HUNDRED TWENTY SIX POINT ZERO classified as "forest" is released in an official proclamation to that effect so
SEVEN SIXTY SIX (126.0766) hectares, more particularly described in Psu- that it may form part of the disposable agricultural lands of the public domain,
162620, which is within the Marikina Watershed Reservation under Executive the rules on confirmation of imperfect title do not apply." 40
Order No. 33 dated July 2, 1904 which established the Marikina Watershed Second Issue: Whether the petition for annulment of judgment
Reservation (IN-12) x x x. should have been given due course.
"x x x Petitioners fault the Court of Appeals for giving due course to the Republics
"That the land sought to be registered is not a private property of the petition for annulment of judgment which was filed long after the decision of
Registration Applicant but part of the public domain, not subjected to the land registration court had allegedly become final and executory. The land
disposition and is covered by Proclamation No. 585 for Integrated Social registration court rendered its decision on January 30, 1991 and the Solicitor
Forestry Program hence, L.R.C. No. 269-A is recommended for rejection General received a copy of the decision on April 23, 1991. 41 Petitioners point
(Underlining supplied)." Copy of the letter is attached herewith as Annex "3" out that the Solicitor General filed with the Court of Appeals the petition for
and made an integral part hereof." annulment of judgment invoking Section 9(2) of BP Blg. 129 42 only on August
Lastly, the Solicitor General pointed out that attached to petitioner Edna T. 6, 1991, after the decision had supposedly become final and executory.
Collados [as original applicant] application is the technical description 39 of the Moreover, petitioners further point out that the Solicitor General filed the
Lot signed by Robert C. Pangyarihan, Officer-in-Charge of the Survey Division petition for annulment after the land registration court issued its order of May
of the Bureau of Lands. This technical description categorically stated that the 6, 1991 directing the Land Registration Authority to issue the corresponding
Lot "is inside IN-12 Mariquina Watershed." decree of registration.
The evidence of record thus appears unsatisfactory and insufficient to show The Solicitor General sought the annulment of the decision on the ground that
clearly and positively that the Lot had been officially released from the the land registration court had no jurisdiction over the case, specifically, over
Marikina Watershed Reservation to form part of the alienable and disposable the Lot which was not alienable and disposable. The Solicitor General
lands of the public domain. We hold that once a parcel of land is included maintained that the decision was null and void.
within a watershed reservation duly established by Executive Proclamation, as Petitioners argue that the remedy of annulment of judgment is no longer
in the instant case, a presumption arises that the land continues to be part of available because it is barred by the principle of res judicata. They insist that
such Reservation until clear and convincing evidence of subsequent the land registration court had jurisdiction over the case which involves
declassification is shown. private land. They also argue that the Republic is estopped from questioning
It is obvious, based on the facts on record that neither petitioners nor their the land registration courts jurisdiction considering that the Republic
predecessors-in-interest have been in open, continuous, exclusive and participated in the proceedings before the court.
notorious possession and occupation of the Lot for at least thirty years It is now established that the Lot, being a watershed reservation, is not
immediately preceding the filing of the application for confirmation of title. alienable and disposable public land. The evidence of the petitioners do not
Even if they submitted sufficient proof that the Lot had been excluded from clearly and convincingly show that the Lot, described as Lot Psu-162620,
the MWR upon the issuance of Proclamation No. 1283 on June 21, 1974, ceased to be a portion of the area classified as a watershed reservation of the
petitioners possession as of the filing of their application on April 25, 1985 public domain. Any title to the Lot is void ab initio. In view of this, the alleged
would have been only eleven years counted from the issuance of the procedural infirmities attending the filing of the petition for annulment of
proclamation in 1974. The result will not change even if we tack in the two judgment are immaterial since the land registration court never acquired
years Sesinando Leyva allegedly possessed the Lot from 1902 until the jurisdiction over the Lot. All proceedings of the land registration court

66
involving the Lot are therefore null and void. Reservation, certain parcel of land of the public domain embraced therein
We apply our ruling in Martinez vs. Court of Appeals, 43 as follows: situated in Sitios Bosoboso, Veterans, Kilingan and Barangay San Joseph and
"The Land Registration Court has no jurisdiction over non-registrable Paenaan, Municipality of Antipolo, Province of Rizal and place the same under
properties, such as public navigable rivers which are parts of the public the Integrated Social Forestry Program of the Department of Environment and
domain, and cannot validly adjudge the registration of title in favor of private Natural Resources in accordance with existing laws, rules and regulations,
applicant. Hence, the judgment of the Court of First Instance of Pampanga as which parcel of land is more particularly described as follows:
regards the Lot No. 2 of certificate of Title No. 15856 in the name of "A PARCEL OF LAND, within the Marikina Watershed Reservation situated in
petitioners may be attacked at any time, either directly or collaterally, by the the Municipality of Antipolo, Province of Rizal, beginning at point "1" on plan,
State which is not bound by any prescriptive period provided for by the being identical to corner 1 of Marikina Watershed Reservation; thence
Statute of Limitations." xxx xxx xxx
We also hold that environmental consequences in this case override concerns Containing an area of One Thousand Four Hundred Thirty (1,430) Hectares.
over technicalities and rules of procedure. All other lands covered and embraced under Executive Order No. 33 as
In Republic vs. De los Angeles, 44 which involved the registration of public amended, not otherwise affected by this Proclamation, shall remain in force
lands, specifically parts of the sea, the Court rejected the principle of res and effect.
judicata and estoppel to silence the Republics claim over public lands. The IN WITNESS WHEREOF, I have hereunto set my hand and caused the seal of
Court said: the Republic of the Philippines to be affixed.
"It should be noted further that the doctrine of estoppel or laches does not Done in the City of Manila, this 5th day of June, in the year of Our Lord,
apply when the Government sues as a sovereign or asserts governmental nineteen hundred and ninety.
rights, nor does estoppel or laches validate an act that contravenes law or (Sgd.) CORAZON C. AQUINO
public policy, and that res judicata is to be disregarded if its application would President of the Philippines"
involve the sacrifice of justice to technicality." Pursuant to Proclamation No. 585, the chief of the ISF Unit, acting through the
The Court further held that "the right of reversion or reconveyance to the Regional Executive Director of the DENR (Region IV), issued sometime
State of the public properties registered and which are not capable of private between the years 1989 to 1991 certificates of stewardship contracts to bona
appropriation or private acquisition does not prescribe." fide residents of the barangays mentioned in the proclamation as qualified
Third issue: Whether the petition-in-intervention is proper. recipients of the ISF programs. Among those awarded were intervenors. The
The Bockasanjo ISF Awardees Association, Inc., an association of holders of certificates of stewardship are actually contracts of lease granted by the DENR
certificates of stewardship issued by the DENR under its Integrated Social to actual occupants of parcels of land under its ISF programs for a period of
Forestry Program, filed with the Court of Appeals on November 29, 1991 a twenty-five (25) years, renewable for another twenty-five (25) years. 45 The
Motion for Leave to Intervene and to Admit Petition-In-Intervention. DENR awarded contracts of stewardship to ISF participants in Barangay San
According to intervenors, they are the actual occupants of the Lot which Isidro (or Boso-boso) and the other barangays based on the Inventory of
petitioners sought to register. Aware that the parcels of land which their Forest Occupants the DENR had conducted.46
forefathers had occupied, developed and tilled belong to the Government, According to intervenors, they learned only on July 31, 1991 about the
they filed a petition with then President Corazon C. Aquino and then DENR pendency of LRC Case No. 269-A before the Regional Trial Court of Antipolo,
Secretary Fulgencio S. Factoran, to award the parcels of land to them. Rizal. On August 8, 1991, they filed a Motion for Leave to Intervene and to
Secretary Factoran directed the Director of Forest Management Bureau to take Admit Opposition in Intervention before the land registration court to assert
steps for the segregation of the aforementioned area from the MWR for their rights and to protect their interests.
development under the DENRs ISF Programs. Subsequently, then President However, shortly after the filing of their opposition, intervenors learned that
Aquino issued Proclamation No. 585 dated June 5, 1990 excluding 1,430 the land registration court had already rendered a decision on January 30,
hectares from the operation of EO 33 and placed the same under the DENRs 1991 confirming petitioners imperfect title. Intervenors counsel received a
Integrated Social Forestry Program. Proclamation No. 585 reads: copy of the decision on August 9, 1991.
PROCLAMATION NO. 585 On August 14, 1991, intervenors filed a motion to vacate judgment and for
AMENDING FURTHER EXECUTIVE ORDER NO. 33, DATED JULY 26, 1904 WHICH new trial before the land registration court. According to intervenors, the land
ESTABLISHED THE MARIKINA WATERSHED RESERVATION (IN-12) AS AMENDED, registration court could not act on its motions due to the restraining order
BY EXCLUDING CERTAIN PORTIONS OF LANDS EMBRACED THEREIN SITUATED issued by the Court of Appeals on August 8, 1991, enjoining the land
AT SITIOS BOSOBOSO, KILINGAN, VETERANS, BARANGAYS SAN JOSEPH AND registration court from executing its decision, as prayed for by the Solicitor
PAENAAN, MUNICIPALITY OF ANTIPOLO, PROVINCE OF RIZAL, ISLAND OF General in its petition for annulment of judgment. The intervenors were thus
LUZON. constrained to file a petition for intervention before the Court of Appeals which
Upon recommendation of the Secretary of Environment and Natural Resources allowed the same.
and pursuant to the authority vested in me by law, I, CORAZON C. AQUINO, Rule 19 of the 1997 Rules of Civil Procedure47 provides in pertinent parts:
President of the Philippines, do hereby exclude from the operation of Section 1. Who may intervene. A person who has a legal interest in the
Executive Order No. 33, which established the Marikina Watershed matter in litigation, or in the success of either of the parties, or an interest

67
against both, or is so situated as to be adversely affected by a distribution or Vitug, and Ynares-Santiago, JJ., concur.Davide, Jr., C.J., (Chairman), on official
other disposition of property in the custody of the court, or an officer thereof leave.
may, with leave of court, be allowed to intervene in the action. The Court shall
consider whether or not the intervention will unduly delay or prejudice the Cruz v. Secretary of Environment 347 SCRA 128 (2000)
adjudication of the rights of the original parties, and whether or not the (focuses on Separate Opinions discussing Regalian Doctrine)
inertvenors rights may be fully protected in a separate proceeding.
Sec. 2. Time to intervene. The motion to intervene may be filed at any time Chavez v. Public Estates Authority 384 SCRA 152 (2002)
before rendition of judgment by the trial court. A copy of the pleading-in- Republic of the Philippines
intervention shall be attached to the motion and served on the original SUPREME COURT
parties. Manila
As a rule, intervention is allowed "before rendition of judgment by the trial EN BANC
court," as Section 2, Rule 19 expressly provides. However, the Court has G.R. No. 133250 July 9, 2002
recognized exceptions to this rule in the interest of substantial justice. Mago FRANCISCO I. CHAVEZ, petitioner,
vs. Court of Appeals48 reiterated the ruling in Director of Lands vs. Court of vs.
Appeals, where the Court allowed the motions for intervention even when the PUBLIC ESTATES AUTHORITY and AMARI COASTAL BAY DEVELOPMENT
case had already reached this Court. Thus, in Mago the Court held that: CORPORATION, respondents.
"It is quite clear and patent that the motions for intervention filed by the CARPIO, J.:
movants at this stage of the proceedings where trial had already been This is an original Petition for Mandamus with prayer for a writ of preliminary
concluded x x x and on appeal x x x the same affirmed by the Court of injunction and a temporary restraining order. The petition seeks to compel the
Appeals and the instant petition for certiorari to review said judgment is Public Estates Authority ("PEA" for brevity) to disclose all facts on PEA's then
already submitted for decision by the Supreme Court, are obviously and, on-going renegotiations with Amari Coastal Bay and Development Corporation
manifestly late, beyond the period prescribed under x x x Section 2, Rule 12 of ("AMARI" for brevity) to reclaim portions of Manila Bay. The petition further
the rules of Court. seeks to enjoin PEA from signing a new agreement with AMARI involving such
But Rule 12 of the Rules of Court, like all other Rules therein promulgated, is reclamation.
simply a rule of procedure, the whole purpose and object of which is to make The Facts
the powers of the Court fully and completely available for justice. The purpose On November 20, 1973, the government, through the Commissioner of Public
of procedure is not to thwart justice. Its proper aim is to facilitate the Highways, signed a contract with the Construction and Development
application of justice to the rival claims of contending parties. It was created Corporation of the Philippines ("CDCP" for brevity) to reclaim certain foreshore
not to hinder and delay but to facilitate and promote the administration of and offshore areas of Manila Bay. The contract also included the construction
justice. It does not constitute the thing itself which courts are always striving of Phases I and II of the Manila-Cavite Coastal Road. CDCP obligated itself to
to secure to litigants. It is designed as the means best adopted to obtain that carry out all the works in consideration of fifty percent of the total reclaimed
thing. In other words, it is a means to an end." land.
To be sure, the Court of Appeals did not pass upon the actual status of On February 4, 1977, then President Ferdinand E. Marcos issued Presidential
intervenors in relation to the Lot as this was not in issue. Neither was the Decree No. 1084 creating PEA. PD No. 1084 tasked PEA "to reclaim land,
validity of the certificates of stewardship contracts which intervenors allegedly including foreshore and submerged areas," and "to develop, improve, acquire,
possessed inquired into considering this too was not in issue. In fact, x x x lease and sell any and all kinds of lands." 1 On the same date, then
intervenors did not specifically seek any relief apart from a declaration that President Marcos issued Presidential Decree No. 1085 transferring to PEA the
the Lot in question remains inalienable land of the public domain. We cannot "lands reclaimed in the foreshore and offshore of the Manila Bay" 2 under the
fault the Court of Appeals for allowing the intervention, if only to provide the Manila-Cavite Coastal Road and Reclamation Project (MCCRRP).
rival groups a peaceful venue for ventilating their sides. This case has already On December 29, 1981, then President Marcos issued a memorandum
claimed at least five lives due to the raging dispute between the rival camps directing PEA to amend its contract with CDCP, so that "[A]ll future works in
of the petitioners on one side and those of the DENR awardees on the other. It MCCRRP x x x shall be funded and owned by PEA." Accordingly, PEA and CDCP
also spawned a number of criminal cases between the two rival groups executed a Memorandum of Agreement dated December 29, 1981, which
including malicious mischief, robbery and arson. A strict application of the stated:
rules would blur this bigger, far more important picture. "(i) CDCP shall undertake all reclamation, construction, and such other works
WHEREFORE, the Petition is DENIED. The Decision of the Court of Appeals in the MCCRRP as may be agreed upon by the parties, to be paid according to
dated June 22, 1992 declaring null and void the Decision dated January 30, progress of works on a unit price/lump sum basis for items of work to be
1991 of Branch 71, Regional Trial Court of Antipolo, Rizal, in LRC No. 269-A, agreed upon, subject to price escalation, retention and other terms and
LRC Rec. No. N-59179 is AFFIRMED. conditions provided for in Presidential Decree No. 1594. All the financing
SO ORDERED. required for such works shall be provided by PEA.
xxx
68
(iii) x x x CDCP shall give up all its development rights and hereby agrees to reports that there were on-going renegotiations between PEA and AMARI
cede and transfer in favor of PEA, all of the rights, title, interest and under an order issued by then President Fidel V. Ramos. According to these
participation of CDCP in and to all the areas of land reclaimed by CDCP in the reports, PEA Director Nestor Kalaw, PEA Chairman Arsenio Yulo and retired
MCCRRP as of December 30, 1981 which have not yet been sold, transferred Navy Officer Sergio Cruz composed the negotiating panel of PEA.
or otherwise disposed of by CDCP as of said date, which areas consist of On April 13, 1998, Antonio M. Zulueta filed before the Court a Petition for
approximately Ninety-Nine Thousand Four Hundred Seventy Three (99,473) Prohibition with Application for the Issuance of a Temporary Restraining Order
square meters in the Financial Center Area covered by land pledge No. 5 and and Preliminary Injunction docketed as G.R. No. 132994 seeking to nullify the
approximately Three Million Three Hundred Eighty Two Thousand Eight JVA. The Court dismissed the petition "for unwarranted disregard of judicial
Hundred Eighty Eight (3,382,888) square meters of reclaimed areas at varying hierarchy, without prejudice to the refiling of the case before the proper
elevations above Mean Low Water Level located outside the Financial Center court."12
Area and the First Neighborhood Unit."3 On April 27, 1998, petitioner Frank I. Chavez ("Petitioner" for brevity) as a
On January 19, 1988, then President Corazon C. Aquino issued Special Patent taxpayer, filed the instant Petition for Mandamus with Prayer for the Issuance
No. 3517, granting and transferring to PEA "the parcels of land so reclaimed of a Writ of Preliminary Injunction and Temporary Restraining Order. Petitioner
under the Manila-Cavite Coastal Road and Reclamation Project (MCCRRP) contends the government stands to lose billions of pesos in the sale by PEA of
containing a total area of one million nine hundred fifteen thousand eight the reclaimed lands to AMARI. Petitioner prays that PEA publicly disclose the
hundred ninety four (1,915,894) square meters." Subsequently, on April 9, terms of any renegotiation of the JVA, invoking Section 28, Article II, and
1988, the Register of Deeds of the Municipality of Paraaque issued Transfer Section 7, Article III, of the 1987 Constitution on the right of the people to
Certificates of Title Nos. 7309, 7311, and 7312, in the name of PEA, covering information on matters of public concern. Petitioner assails the sale to AMARI
the three reclaimed islands known as the "Freedom Islands" located at the of lands of the public domain as a blatant violation of Section 3, Article XII of
southern portion of the Manila-Cavite Coastal Road, Paraaque City. The the 1987 Constitution prohibiting the sale of alienable lands of the public
Freedom Islands have a total land area of One Million Five Hundred Seventy domain to private corporations. Finally, petitioner asserts that he seeks to
Eight Thousand Four Hundred and Forty One (1,578,441) square meters or enjoin the loss of billions of pesos in properties of the State that are of public
157.841 hectares. dominion.
On April 25, 1995, PEA entered into a Joint Venture Agreement ("JVA" for After several motions for extension of time,13 PEA and AMARI filed their
brevity) with AMARI, a private corporation, to develop the Freedom Islands. Comments on October 19, 1998 and June 25, 1998, respectively. Meanwhile,
The JVA also required the reclamation of an additional 250 hectares of on December 28, 1998, petitioner filed an Omnibus Motion: (a) to require PEA
submerged areas surrounding these islands to complete the configuration in to submit the terms of the renegotiated PEA-AMARI contract; (b) for issuance
the Master Development Plan of the Southern Reclamation Project-MCCRRP. of a temporary restraining order; and (c) to set the case for hearing on oral
PEA and AMARI entered into the JVA through negotiation without public argument. Petitioner filed a Reiterative Motion for Issuance of a TRO dated
bidding.4 On April 28, 1995, the Board of Directors of PEA, in its Resolution No. May 26, 1999, which the Court denied in a Resolution dated June 22, 1999.
1245, confirmed the JVA.5 On June 8, 1995, then President Fidel V. Ramos, In a Resolution dated March 23, 1999, the Court gave due course to the
through then Executive Secretary Ruben Torres, approved the JVA. 6 petition and required the parties to file their respective memoranda.
On November 29, 1996, then Senate President Ernesto Maceda delivered a On March 30, 1999, PEA and AMARI signed the Amended Joint Venture
privilege speech in the Senate and denounced the JVA as the "grandmother of Agreement ("Amended JVA," for brevity). On May 28, 1999, the Office of the
all scams." As a result, the Senate Committee on Government Corporations President under the administration of then President Joseph E. Estrada
and Public Enterprises, and the Committee on Accountability of Public Officers approved the Amended JVA.
and Investigations, conducted a joint investigation. The Senate Committees Due to the approval of the Amended JVA by the Office of the President,
reported the results of their investigation in Senate Committee Report No. 560 petitioner now prays that on "constitutional and statutory grounds the
dated September 16, 1997.7 Among the conclusions of their report are: (1) the renegotiated contract be declared null and void."14
reclaimed lands PEA seeks to transfer to AMARI under the JVA are lands of the The Issues
public domain which the government has not classified as alienable lands and The issues raised by petitioner, PEA15 and AMARI16 are as follows:
therefore PEA cannot alienate these lands; (2) the certificates of title covering I. WHETHER THE PRINCIPAL RELIEFS PRAYED FOR IN THE PETITION ARE MOOT
the Freedom Islands are thus void, and (3) the JVA itself is illegal. AND ACADEMIC BECAUSE OF SUBSEQUENT EVENTS;
On December 5, 1997, then President Fidel V. Ramos issued Presidential II. WHETHER THE PETITION MERITS DISMISSAL FOR FAILING TO OBSERVE THE
Administrative Order No. 365 creating a Legal Task Force to conduct a study PRINCIPLE GOVERNING THE HIERARCHY OF COURTS;
on the legality of the JVA in view of Senate Committee Report No. 560. The III. WHETHER THE PETITION MERITS DISMISSAL FOR NON-EXHAUSTION OF
members of the Legal Task Force were the Secretary of Justice, 8 the Chief ADMINISTRATIVE REMEDIES;
Presidential Legal Counsel,9 and the Government Corporate Counsel. 10 The IV. WHETHER PETITIONER HAS LOCUS STANDI TO BRING THIS SUIT;
Legal Task Force upheld the legality of the JVA, contrary to the conclusions V. WHETHER THE CONSTITUTIONAL RIGHT TO INFORMATION INCLUDES
reached by the Senate Committees.11 OFFICIAL INFORMATION ON ON-GOING NEGOTIATIONS BEFORE A FINAL
On April 4 and 5, 1998, the Philippine Daily Inquirer and Today published AGREEMENT;

69
VI. WHETHER THE STIPULATIONS IN THE AMENDED JOINT VENTURE the Court involving Section 3, Article XII of the 1987 Constitution, or its
AGREEMENT FOR THE TRANSFER TO AMARI OF CERTAIN LANDS, RECLAIMED counterpart provision in the 1973 Constitution, 18 covered agricultural lands
AND STILL TO BE RECLAIMED, VIOLATE THE 1987 CONSTITUTION; AND sold to private corporations which acquired the lands from private parties. The
VII. WHETHER THE COURT IS THE PROPER FORUM FOR RAISING THE ISSUE OF transferors of the private corporations claimed or could claim the right to
WHETHER THE AMENDED JOINT VENTURE AGREEMENT IS GROSSLY judicial confirmation of their imperfect titles 19 under Title II of
DISADVANTAGEOUS TO THE GOVERNMENT. Commonwealth Act. 141 ("CA No. 141" for brevity). In the instant case, AMARI
The Court's Ruling seeks to acquire from PEA, a public corporation, reclaimed lands and
First issue: whether the principal reliefs prayed for in the petition are submerged areas for non-agricultural purposes by purchase under PD No.
moot and academic because of subsequent events. 1084 (charter of PEA) and Title III of CA No. 141. Certain undertakings by
The petition prays that PEA publicly disclose the "terms and conditions of the AMARI under the Amended JVA constitute the consideration for the purchase.
on-going negotiations for a new agreement." The petition also prays that the Neither AMARI nor PEA can claim judicial confirmation of their titles because
Court enjoin PEA from "privately entering into, perfecting and/or executing any the lands covered by the Amended JVA are newly reclaimed or still to be
new agreement with AMARI." reclaimed. Judicial confirmation of imperfect title requires open, continuous,
PEA and AMARI claim the petition is now moot and academic because AMARI exclusive and notorious occupation of agricultural lands of the public domain
furnished petitioner on June 21, 1999 a copy of the signed Amended JVA for at least thirty years since June 12, 1945 or earlier. Besides, the deadline for
containing the terms and conditions agreed upon in the renegotiations. Thus, filing applications for judicial confirmation of imperfect title expired on
PEA has satisfied petitioner's prayer for a public disclosure of the December 31, 1987.20
renegotiations. Likewise, petitioner's prayer to enjoin the signing of the Lastly, there is a need to resolve immediately the constitutional issue raised in
Amended JVA is now moot because PEA and AMARI have already signed the this petition because of the possible transfer at any time by PEA to AMARI of
Amended JVA on March 30, 1999. Moreover, the Office of the President has title and ownership to portions of the reclaimed lands. Under the Amended
approved the Amended JVA on May 28, 1999. JVA, PEA is obligated to transfer to AMARI the latter's seventy percent
Petitioner counters that PEA and AMARI cannot avoid the constitutional issue proportionate share in the reclaimed areas as the reclamation progresses. The
by simply fast-tracking the signing and approval of the Amended JVA before Amended JVA even allows AMARI to mortgage at any time the entire
the Court could act on the issue. Presidential approval does not resolve the reclaimed area to raise financing for the reclamation project. 21
constitutional issue or remove it from the ambit of judicial review. Second issue: whether the petition merits dismissal for failing to
We rule that the signing of the Amended JVA by PEA and AMARI and its observe the principle governing the hierarchy of courts.
approval by the President cannot operate to moot the petition and divest the PEA and AMARI claim petitioner ignored the judicial hierarchy by seeking relief
Court of its jurisdiction. PEA and AMARI have still to implement the Amended directly from the Court. The principle of hierarchy of courts applies generally
JVA. The prayer to enjoin the signing of the Amended JVA on constitutional to cases involving factual questions. As it is not a trier of facts, the Court
grounds necessarily includes preventing its implementation if in the meantime cannot entertain cases involving factual issues. The instant case, however,
PEA and AMARI have signed one in violation of the Constitution. Petitioner's raises constitutional issues of transcendental importance to the public. 22 The
principal basis in assailing the renegotiation of the JVA is its violation of Court can resolve this case without determining any factual issue related to
Section 3, Article XII of the Constitution, which prohibits the government from the case. Also, the instant case is a petition for mandamus which falls under
alienating lands of the public domain to private corporations. If the Amended the original jurisdiction of the Court under Section 5, Article VIII of the
JVA indeed violates the Constitution, it is the duty of the Court to enjoin its Constitution. We resolve to exercise primary jurisdiction over the instant case.
implementation, and if already implemented, to annul the effects of such Third issue: whether the petition merits dismissal for non-exhaustion
unconstitutional contract. of administrative remedies.
The Amended JVA is not an ordinary commercial contract but one which seeks PEA faults petitioner for seeking judicial intervention in compelling PEA to
to transfer title and ownership to 367.5 hectares of reclaimed lands disclose publicly certain information without first asking PEA the needed
and submerged areas of Manila Bay to a single private corporation. It information. PEA claims petitioner's direct resort to the Court violates the
now becomes more compelling for the Court to resolve the issue to insure the principle of exhaustion of administrative remedies. It also violates the rule
government itself does not violate a provision of the Constitution intended to that mandamus may issue only if there is no other plain, speedy and adequate
safeguard the national patrimony. Supervening events, whether intended or remedy in the ordinary course of law.
accidental, cannot prevent the Court from rendering a decision if there is a PEA distinguishes the instant case from Taada v. Tuvera 23 where the Court
grave violation of the Constitution. In the instant case, if the Amended JVA granted the petition for mandamus even if the petitioners there did not
runs counter to the Constitution, the Court can still prevent the transfer of title initially demand from the Office of the President the publication of the
and ownership of alienable lands of the public domain in the name of AMARI. presidential decrees. PEA points out that in Taada, the Executive Department
Even in cases where supervening events had made the cases moot, the Court had an affirmative statutory duty under Article 2 of the Civil Code 24 and
did not hesitate to resolve the legal or constitutional issues raised to formulate Section 1 of Commonwealth Act No. 638 25 to publish the presidential decrees.
controlling principles to guide the bench, bar, and the public. 17 There was, therefore, no need for the petitioners in Taada to make an initial
Also, the instant petition is a case of first impression. All previous decisions of demand from the Office of the President. In the instant case, PEA claims it has

70
no affirmative statutory duty to disclose publicly information about its they 'immediately affect the social, economic and moral well being of the
renegotiation of the JVA. Thus, PEA asserts that the Court must apply the people.'
principle of exhaustion of administrative remedies to the instant case in view Moreover, the mere fact that he is a citizen satisfies the requirement of
of the failure of petitioner here to demand initially from PEA the needed personal interest, when the proceeding involves the assertion of a public right,
information. such as in this case. He invokes several decisions of this Court which have set
The original JVA sought to dispose to AMARI public lands held by PEA, a aside the procedural matter of locus standi, when the subject of the case
government corporation. Under Section 79 of the Government Auditing involved public interest.
Code,26 the disposition of government lands to private parties requires public xxx
bidding. PEA was under a positive legal duty to disclose to the public In Taada v. Tuvera, the Court asserted that when the issue concerns a public
the terms and conditions for the sale of its lands. The law obligated PEA right and the object of mandamus is to obtain the enforcement of a public
to make this public disclosure even without demand from petitioner or from duty, the people are regarded as the real parties in interest; and because it is
anyone. PEA failed to make this public disclosure because the original JVA, like sufficient that petitioner is a citizen and as such is interested in the execution
the Amended JVA, was the result of a negotiated contract, not of a public of the laws, he need not show that he has any legal or special interest in the
bidding. Considering that PEA had an affirmative statutory duty to make the result of the action. In the aforesaid case, the petitioners sought to enforce
public disclosure, and was even in breach of this legal duty, petitioner had the their right to be informed on matters of public concern, a right then
right to seek direct judicial intervention. recognized in Section 6, Article IV of the 1973 Constitution, in connection with
Moreover, and this alone is determinative of this issue, the principle of the rule that laws in order to be valid and enforceable must be published in
exhaustion of administrative remedies does not apply when the issue involved the Official Gazette or otherwise effectively promulgated. In ruling for the
is a purely legal or constitutional question. 27 The principal issue in the instant petitioners' legal standing, the Court declared that the right they sought to be
case is the capacity of AMARI to acquire lands held by PEA in view of the enforced 'is a public right recognized by no less than the fundamental law of
constitutional ban prohibiting the alienation of lands of the public domain to the land.'
private corporations. We rule that the principle of exhaustion of administrative Legaspi v. Civil Service Commission, while reiterating Taada, further declared
remedies does not apply in the instant case. that 'when a mandamus proceeding involves the assertion of a public right,
Fourth issue: whether petitioner has locus standi to bring this suit the requirement of personal interest is satisfied by the mere fact that
PEA argues that petitioner has no standing to institute mandamus proceedings petitioner is a citizen and, therefore, part of the general 'public' which
to enforce his constitutional right to information without a showing that PEA possesses the right.'
refused to perform an affirmative duty imposed on PEA by the Constitution. Further, in Albano v. Reyes, we said that while expenditure of public funds
PEA also claims that petitioner has not shown that he will suffer any concrete may not have been involved under the questioned contract for the
injury because of the signing or implementation of the Amended JVA. Thus, development, management and operation of the Manila International
there is no actual controversy requiring the exercise of the power of judicial Container Terminal, 'public interest [was] definitely involved considering the
review. important role [of the subject contract] . . . in the economic development of
The petitioner has standing to bring this taxpayer's suit because the petition the country and the magnitude of the financial consideration involved.' We
seeks to compel PEA to comply with its constitutional duties. There are two concluded that, as a consequence, the disclosure provision in the Constitution
constitutional issues involved here. First is the right of citizens to information would constitute sufficient authority for upholding the petitioner's standing.
on matters of public concern. Second is the application of a constitutional Similarly, the instant petition is anchored on the right of the people to
provision intended to insure the equitable distribution of alienable lands of the information and access to official records, documents and papers a right
public domain among Filipino citizens. The thrust of the first issue is to compel guaranteed under Section 7, Article III of the 1987 Constitution. Petitioner, a
PEA to disclose publicly information on the sale of government lands worth former solicitor general, is a Filipino citizen. Because of the satisfaction of the
billions of pesos, information which the Constitution and statutory law two basic requisites laid down by decisional law to sustain petitioner's legal
mandate PEA to disclose. The thrust of the second issue is to prevent PEA standing, i.e. (1) the enforcement of a public right (2) espoused by a Filipino
from alienating hundreds of hectares of alienable lands of the public domain in citizen, we rule that the petition at bar should be allowed."
violation of the Constitution, compelling PEA to comply with a constitutional We rule that since the instant petition, brought by a citizen, involves the
duty to the nation. enforcement of constitutional rights - to information and to the equitable
Moreover, the petition raises matters of transcendental importance to the diffusion of natural resources - matters of transcendental public importance,
public. In Chavez v. PCGG,28 the Court upheld the right of a citizen to bring a the petitioner has the requisite locus standi.
taxpayer's suit on matters of transcendental importance to the public, thus - Fifth issue: whether the constitutional right to information includes
"Besides, petitioner emphasizes, the matter of recovering the ill-gotten wealth official information on on-going negotiations before a final
of the Marcoses is an issue of 'transcendental importance to the public.' He agreement.
asserts that ordinary taxpayers have a right to initiate and prosecute actions Section 7, Article III of the Constitution explains the people's right to
questioning the validity of acts or orders of government agencies or information on matters of public concern in this manner:
instrumentalities, if the issues raised are of 'paramount public interest,' and if "Sec. 7. The right of the people to information on matters of public concern

71
shall be recognized. Access to official records, and to documents, and Mr. Suarez: This contemplates inclusion of negotiations leading to
papers pertaining to official acts, transactions, or decisions, as well as the consummation of the transaction.
to government research data used as basis for policy development, shall be Mr. Ople: Yes, subject only to reasonable safeguards on the national
afforded the citizen, subject to such limitations as may be provided by law." interest.
(Emphasis supplied) Mr. Suarez: Thank you."32 (Emphasis supplied)
The State policy of full transparency in all transactions involving public AMARI argues there must first be a consummated contract before petitioner
interest reinforces the people's right to information on matters of public can invoke the right. Requiring government officials to reveal their
concern. This State policy is expressed in Section 28, Article II of the deliberations at the pre-decisional stage will degrade the quality of decision-
Constitution, thus: making in government agencies. Government officials will hesitate to express
"Sec. 28. Subject to reasonable conditions prescribed by law, the State adopts their real sentiments during deliberations if there is immediate public
and implements a policy of full public disclosure of all its transactions dissemination of their discussions, putting them under all kinds of pressure
involving public interest." (Emphasis supplied) before they decide.
These twin provisions of the Constitution seek to promote transparency in We must first distinguish between information the law on public bidding
policy-making and in the operations of the government, as well as provide the requires PEA to disclose publicly, and information the constitutional right to
people sufficient information to exercise effectively other constitutional rights. information requires PEA to release to the public. Before the consummation of
These twin provisions are essential to the exercise of freedom of expression. If the contract, PEA must, on its own and without demand from anyone, disclose
the government does not disclose its official acts, transactions and decisions to the public matters relating to the disposition of its property. These include
to citizens, whatever citizens say, even if expressed without any restraint, will the size, location, technical description and nature of the property being
be speculative and amount to nothing. These twin provisions are also disposed of, the terms and conditions of the disposition, the parties qualified
essential to hold public officials "at all times x x x accountable to the to bid, the minimum price and similar information. PEA must prepare all these
people,"29 for unless citizens have the proper information, they cannot hold data and disclose them to the public at the start of the disposition process,
public officials accountable for anything. Armed with the right information, long before the consummation of the contract, because the Government
citizens can participate in public discussions leading to the formulation of Auditing Code requires public bidding. If PEA fails to make this disclosure,
government policies and their effective implementation. An informed citizenry any citizen can demand from PEA this information at any time during the
is essential to the existence and proper functioning of any democracy. As bidding process.
explained by the Court in Valmonte v. Belmonte, Jr.30 Information, however, on on-going evaluation or review of bids or
"An essential element of these freedoms is to keep open a continuing dialogue proposals being undertaken by the bidding or review committee is not
or process of communication between the government and the people. It is in immediately accessible under the right to information. While the evaluation or
the interest of the State that the channels for free political discussion be review is still on-going, there are no "official acts, transactions, or decisions"
maintained to the end that the government may perceive and be responsive on the bids or proposals. However, once the committee makes its official
to the people's will. Yet, this open dialogue can be effective only to the extent recommendation, there arises a "definite proposition" on the part of the
that the citizenry is informed and thus able to formulate its will intelligently. government. From this moment, the public's right to information attaches, and
Only when the participants in the discussion are aware of the issues and have any citizen can access all the non-proprietary information leading to such
access to information relating thereto can such bear fruit." definite proposition. In Chavez v. PCGG,33 the Court ruled as follows:
PEA asserts, citing Chavez v. PCGG,31 that in cases of on-going negotiations "Considering the intent of the framers of the Constitution, we believe that it is
the right to information is limited to "definite propositions of the government." incumbent upon the PCGG and its officers, as well as other government
PEA maintains the right does not include access to "intra-agency or inter- representatives, to disclose sufficient public information on any proposed
agency recommendations or communications during the stage when common settlement they have decided to take up with the ostensible owners and
assertions are still in the process of being formulated or are in the 'exploratory holders of ill-gotten wealth. Such information, though, must pertain to
stage'." definite propositions of the government, not necessarily to intra-agency
Also, AMARI contends that petitioner cannot invoke the right at the pre- or inter-agency recommendations or communications during the stage when
decisional stage or before the closing of the transaction. To support its common assertions are still in the process of being formulated or are in the
contention, AMARI cites the following discussion in the 1986 Constitutional "exploratory" stage. There is need, of course, to observe the same restrictions
Commission: on disclosure of information in general, as discussed earlier such as on
"Mr. Suarez. And when we say 'transactions' which should be distinguished matters involving national security, diplomatic or foreign relations, intelligence
from contracts, agreements, or treaties or whatever, does the Gentleman refer and other classified information." (Emphasis supplied)
to the steps leading to the consummation of the contract, or does he refer to Contrary to AMARI's contention, the commissioners of the 1986 Constitutional
the contract itself? Commission understood that the right to information "contemplates
Mr. Ople: The 'transactions' used here, I suppose is generic and inclusion of negotiations leading to the consummation of the
therefore, it can cover both steps leading to a contract and already a transaction." Certainly, a consummated contract is not a requirement for the
consummated contract, Mr. Presiding Officer. exercise of the right to information. Otherwise, the people can never exercise

72
the right if no contract is consummated, and if one is consummated, it may be Judicial power.39 This is not the situation in the instant case.
too late for the public to expose its defects.1wphi1.nt We rule, therefore, that the constitutional right to information includes official
Requiring a consummated contract will keep the public in the dark until the information on on-going negotiations before a final contract. The
contract, which may be grossly disadvantageous to the government or even information, however, must constitute definite propositions by the
illegal, becomes a fait accompli. This negates the State policy of full government and should not cover recognized exceptions like privileged
transparency on matters of public concern, a situation which the framers of information, military and diplomatic secrets and similar matters affecting
the Constitution could not have intended. Such a requirement will prevent the national security and public order.40 Congress has also prescribed other
citizenry from participating in the public discussion of any proposed contract, limitations on the right to information in several legislations. 41
effectively truncating a basic right enshrined in the Bill of Rights. We can allow Sixth issue: whether stipulations in the Amended JVA for the transfer
neither an emasculation of a constitutional right, nor a retreat by the State of to AMARI of lands, reclaimed or to be reclaimed, violate the
its avowed "policy of full disclosure of all its transactions involving public Constitution.
interest." The Regalian Doctrine
The right covers three categories of information which are "matters of public The ownership of lands reclaimed from foreshore and submerged areas is
concern," namely: (1) official records; (2) documents and papers pertaining to rooted in the Regalian doctrine which holds that the State owns all lands and
official acts, transactions and decisions; and (3) government research data waters of the public domain. Upon the Spanish conquest of the Philippines,
used in formulating policies. The first category refers to any document that is ownership of all "lands, territories and possessions" in the Philippines passed
part of the public records in the custody of government agencies or officials. to the Spanish Crown. 42 The King, as the sovereign ruler and representative of
The second category refers to documents and papers recording, evidencing, the people, acquired and owned all lands and territories in the Philippines
establishing, confirming, supporting, justifying or explaining official acts, except those he disposed of by grant or sale to private individuals.
transactions or decisions of government agencies or officials. The third The 1935, 1973 and 1987 Constitutions adopted the Regalian doctrine
category refers to research data, whether raw, collated or processed, owned substituting, however, the State, in lieu of the King, as the owner of all lands
by the government and used in formulating government policies. and waters of the public domain. The Regalian doctrine is the foundation of
The information that petitioner may access on the renegotiation of the JVA the time-honored principle of land ownership that "all lands that were not
includes evaluation reports, recommendations, legal and expert opinions, acquired from the Government, either by purchase or by grant, belong to the
minutes of meetings, terms of reference and other documents attached to public domain."43 Article 339 of the Civil Code of 1889, which is now Article
such reports or minutes, all relating to the JVA. However, the right to 420 of the Civil Code of 1950, incorporated the Regalian doctrine.
information does not compel PEA to prepare lists, abstracts, summaries and Ownership and Disposition of Reclaimed Lands
the like relating to the renegotiation of the JVA. 34 The right only affords access The Spanish Law of Waters of 1866 was the first statutory law governing the
to records, documents and papers, which means the opportunity to inspect ownership and disposition of reclaimed lands in the Philippines. On May 18,
and copy them. One who exercises the right must copy the records, 1907, the Philippine Commission enacted Act No. 1654 which provided for
documents and papers at his expense. The exercise of the right is also subject the lease, but not the sale, of reclaimed lands of the government to
to reasonable regulations to protect the integrity of the public records and to corporations and individuals. Later, on November 29, 1919, the Philippine
minimize disruption to government operations, like rules specifying when and Legislature approved Act No. 2874, the Public Land Act, which authorized the
how to conduct the inspection and copying.35 lease, but not the sale, of reclaimed lands of the government to
The right to information, however, does not extend to matters recognized as corporations and individuals. On November 7, 1936, the National
privileged information under the separation of powers. 36 The right does not Assembly passed Commonwealth Act No. 141, also known as the Public Land
also apply to information on military and diplomatic secrets, information Act, which authorized the lease, but not the sale, of reclaimed lands of
affecting national security, and information on investigations of crimes by law the government to corporations and individuals. CA No. 141 continues
enforcement agencies before the prosecution of the accused, which courts to this day as the general law governing the classification and disposition of
have long recognized as confidential. 37 The right may also be subject to other lands of the public domain.
limitations that Congress may impose by law. The Spanish Law of Waters of 1866 and the Civil Code of 1889
There is no claim by PEA that the information demanded by petitioner is Under the Spanish Law of Waters of 1866, the shores, bays, coves, inlets and
privileged information rooted in the separation of powers. The information all waters within the maritime zone of the Spanish territory belonged to the
does not cover Presidential conversations, correspondences, or discussions public domain for public use. 44 The Spanish Law of Waters of 1866 allowed the
during closed-door Cabinet meetings which, like internal deliberations of the reclamation of the sea under Article 5, which provided as follows:
Supreme Court and other collegiate courts, or executive sessions of either "Article 5. Lands reclaimed from the sea in consequence of works constructed
house of Congress,38 are recognized as confidential. This kind of information by the State, or by the provinces, pueblos or private persons, with proper
cannot be pried open by a co-equal branch of government. A frank exchange permission, shall become the property of the party constructing such works,
of exploratory ideas and assessments, free from the glare of publicity and unless otherwise provided by the terms of the grant of authority."
pressure by interested parties, is essential to protect the independence of Under the Spanish Law of Waters, land reclaimed from the sea belonged to the
decision-making of those tasked to exercise Presidential, Legislative and party undertaking the reclamation, provided the government issued the

73
necessary permit and did not reserve ownership of the reclaimed land to the and best bidder therefore, subject to such regulations and safeguards as the
State. Governor-General may by executive order prescribe." (Emphasis supplied)
Article 339 of the Civil Code of 1889 defined property of public dominion as Act No. 1654 mandated that the government should retain title to all
follows: lands reclaimed by the government. The Act also vested in the
"Art. 339. Property of public dominion is government control and disposition of foreshore lands. Private parties could
1. That devoted to public use, such as roads, canals, rivers, torrents, ports and lease lands reclaimed by the government only if these lands were no longer
bridges constructed by the State, riverbanks, shores, roadsteads, and that of a needed for public purpose. Act No. 1654 mandated public bidding in the
similar character; lease of government reclaimed lands. Act No. 1654 made government
2. That belonging exclusively to the State which, without being of general reclaimed lands sui generis in that unlike other public lands which the
public use, is employed in some public service, or in the development of the government could sell to private parties, these reclaimed lands were available
national wealth, such as walls, fortresses, and other works for the defense of only for lease to private parties.
the territory, and mines, until granted to private individuals." Act No. 1654, however, did not repeal Section 5 of the Spanish Law of Waters
Property devoted to public use referred to property open for use by the public. of 1866. Act No. 1654 did not prohibit private parties from reclaiming parts of
In contrast, property devoted to public service referred to property used for the sea under Section 5 of the Spanish Law of Waters. Lands reclaimed from
some specific public service and open only to those authorized to use the the sea by private parties with government permission remained private
property. lands.
Property of public dominion referred not only to property devoted to public Act No. 2874 of the Philippine Legislature
use, but also to property not so used but employed to develop the national On November 29, 1919, the Philippine Legislature enacted Act No. 2874, the
wealth. This class of property constituted property of public dominion Public Land Act.46 The salient provisions of Act No. 2874, on reclaimed lands,
although employed for some economic or commercial activity to increase the were as follows:
national wealth. "Sec. 6. The Governor-General, upon the recommendation of the
Article 341 of the Civil Code of 1889 governed the re-classification of property Secretary of Agriculture and Natural Resources, shall from time to
of public dominion into private property, to wit: time classify the lands of the public domain into
"Art. 341. Property of public dominion, when no longer devoted to public use (a) Alienable or disposable,
or to the defense of the territory, shall become a part of the private property (b) Timber, and
of the State." (c) Mineral lands, x x x.
This provision, however, was not self-executing. The legislature, or the Sec. 7. For the purposes of the government and disposition of alienable or
executive department pursuant to law, must declare the property no longer disposable public lands, the Governor-General, upon recommendation
needed for public use or territorial defense before the government could lease by the Secretary of Agriculture and Natural Resources, shall from
or alienate the property to private parties.45 time to time declare what lands are open to disposition or concession
Act No. 1654 of the Philippine Commission under this Act."
On May 8, 1907, the Philippine Commission enacted Act No. 1654 which Sec. 8. Only those lands shall be declared open to disposition or
regulated the lease of reclaimed and foreshore lands. The salient provisions of concession which have been officially delimited or classified x x x.
this law were as follows: xxx
"Section 1. The control and disposition of the foreshore as defined in Sec. 55. Any tract of land of the public domain which, being neither timber nor
existing law, and the title to all Government or public lands made or mineral land, shall be classified as suitable for residential purposes or for
reclaimed by the Government by dredging or filling or otherwise commercial, industrial, or other productive purposes other than
throughout the Philippine Islands, shall be retained by the Government agricultural purposes, and shall be open to disposition or concession, shall
without prejudice to vested rights and without prejudice to rights conceded to be disposed of under the provisions of this chapter, and not otherwise.
the City of Manila in the Luneta Extension. Sec. 56. The lands disposable under this title shall be classified as
Section 2. (a) The Secretary of the Interior shall cause all Government or follows:
public lands made or reclaimed by the Government by dredging or filling or (a) Lands reclaimed by the Government by dredging, filling, or other
otherwise to be divided into lots or blocks, with the necessary streets and means;
alleyways located thereon, and shall cause plats and plans of such surveys to (b) Foreshore;
be prepared and filed with the Bureau of Lands. (c) Marshy lands or lands covered with water bordering upon the shores or
(b) Upon completion of such plats and plans the Governor-General shall banks of navigable lakes or rivers;
give notice to the public that such parts of the lands so made or (d) Lands not included in any of the foregoing classes.
reclaimed as are not needed for public purposes will be leased for x x x.
commercial and business purposes, x x x. Sec. 58. The lands comprised in classes (a), (b), and (c) of section
xxx fifty-six shall be disposed of to private parties by lease only and not
(e) The leases above provided for shall be disposed of to the highest otherwise, as soon as the Governor-General, upon recommendation by

74
the Secretary of Agriculture and Natural Resources, shall declare "Section 1. All agricultural, timber, and mineral lands of the public domain,
that the same are not necessary for the public service and are open waters, minerals, coal, petroleum, and other mineral oils, all forces of potential
to disposition under this chapter. The lands included in class (d) may be energy and other natural resources of the Philippines belong to the State, and
disposed of by sale or lease under the provisions of this Act." their disposition, exploitation, development, or utilization shall be limited to
(Emphasis supplied) citizens of the Philippines or to corporations or associations at least sixty per
Section 6 of Act No. 2874 authorized the Governor-General to "classify lands centum of the capital of which is owned by such citizens, subject to any
of the public domain into x x x alienable or disposable"47 lands. Section 7 of existing right, grant, lease, or concession at the time of the inauguration of
the Act empowered the Governor-General to "declare what lands are open to the Government established under this Constitution. Natural resources,
disposition or concession." Section 8 of the Act limited alienable or disposable with the exception of public agricultural land, shall not be alienated,
lands only to those lands which have been "officially delimited and classified." and no license, concession, or lease for the exploitation, development, or
Section 56 of Act No. 2874 stated that lands "disposable under this title 48 shall utilization of any of the natural resources shall be granted for a period
be classified" as government reclaimed, foreshore and marshy lands, as well exceeding twenty-five years, renewable for another twenty-five years, except
as other lands. All these lands, however, must be suitable for residential, as to water rights for irrigation, water supply, fisheries, or industrial uses other
commercial, industrial or other productive non-agricultural purposes. These than the development of water power, in which cases beneficial use may be
provisions vested upon the Governor-General the power to classify inalienable the measure and limit of the grant." (Emphasis supplied)
lands of the public domain into disposable lands of the public domain. These The 1935 Constitution barred the alienation of all natural resources except
provisions also empowered the Governor-General to classify further such public agricultural lands, which were the only natural resources the State
disposable lands of the public domain into government reclaimed, foreshore or could alienate. Thus, foreshore lands, considered part of the State's natural
marshy lands of the public domain, as well as other non-agricultural lands. resources, became inalienable by constitutional fiat, available only for lease
Section 58 of Act No. 2874 categorically mandated that disposable lands of for 25 years, renewable for another 25 years. The government could alienate
the public domain classified as government reclaimed, foreshore and marshy foreshore lands only after these lands were reclaimed and classified as
lands "shall be disposed of to private parties by lease only and not alienable agricultural lands of the public domain. Government reclaimed and
otherwise." The Governor-General, before allowing the lease of these lands marshy lands of the public domain, being neither timber nor mineral lands, fell
to private parties, must formally declare that the lands were "not necessary under the classification of public agricultural lands. 50 However, government
for the public service." Act No. 2874 reiterated the State policy to lease and reclaimed and marshy lands, although subject to classification as disposable
not to sell government reclaimed, foreshore and marshy lands of the public public agricultural lands, could only be leased and not sold to private parties
domain, a policy first enunciated in 1907 in Act No. 1654. Government because of Act No. 2874.
reclaimed, foreshore and marshy lands remained sui generis, as the only The prohibition on private parties from acquiring ownership of government
alienable or disposable lands of the public domain that the government could reclaimed and marshy lands of the public domain was only a statutory
not sell to private parties. prohibition and the legislature could therefore remove such prohibition. The
The rationale behind this State policy is obvious. Government reclaimed, 1935 Constitution did not prohibit individuals and corporations from acquiring
foreshore and marshy public lands for non-agricultural purposes retain their government reclaimed and marshy lands of the public domain that were
inherent potential as areas for public service. This is the reason the classified as agricultural lands under existing public land laws. Section 2,
government prohibited the sale, and only allowed the lease, of these lands to Article XIII of the 1935 Constitution provided as follows:
private parties. The State always reserved these lands for some future public "Section 2. No private corporation or association may acquire, lease,
service. or hold public agricultural lands in excess of one thousand and
Act No. 2874 did not authorize the reclassification of government reclaimed, twenty four hectares, nor may any individual acquire such lands by
foreshore and marshy lands into other non-agricultural lands under Section 56 purchase in excess of one hundred and forty hectares, or by lease in
(d). Lands falling under Section 56 (d) were the only lands for non-agricultural excess of one thousand and twenty-four hectares, or by homestead in
purposes the government could sell to private parties. Thus, under Act No. excess of twenty-four hectares. Lands adapted to grazing, not exceeding two
2874, the government could not sell government reclaimed, foreshore and thousand hectares, may be leased to an individual, private corporation, or
marshy lands to private parties, unless the legislature passed a law association." (Emphasis supplied)
allowing their sale.49 Still, after the effectivity of the 1935 Constitution, the legislature did not
Act No. 2874 did not prohibit private parties from reclaiming parts of the sea repeal Section 58 of Act No. 2874 to open for sale to private parties
pursuant to Section 5 of the Spanish Law of Waters of 1866. Lands reclaimed government reclaimed and marshy lands of the public domain. On the
from the sea by private parties with government permission remained private contrary, the legislature continued the long established State policy of
lands. retaining for the government title and ownership of government reclaimed
Dispositions under the 1935 Constitution and marshy lands of the public domain.
On May 14, 1935, the 1935 Constitution took effect upon its ratification by the Commonwealth Act No. 141 of the Philippine National Assembly
Filipino people. The 1935 Constitution, in adopting the Regalian doctrine, On November 7, 1936, the National Assembly approved Commonwealth Act
declared in Section 1, Article XIII, that No. 141, also known as the Public Land Act, which compiled the then existing

75
laws on lands of the public domain. CA No. 141, as amended, remains to this Sec. 60. Any tract of land comprised under this title may be leased or sold, as
day the existing general law governing the classification and disposition of the case may be, to any person, corporation, or association authorized to
lands of the public domain other than timber and mineral lands. 51 purchase or lease public lands for agricultural purposes. x x x.
Section 6 of CA No. 141 empowers the President to classify lands of the public Sec. 61. The lands comprised in classes (a), (b), and (c) of section
domain into "alienable or disposable" 52 lands of the public domain, which prior fifty-nine shall be disposed of to private parties by lease only and not
to such classification are inalienable and outside the commerce of man. otherwise, as soon as the President, upon recommendation by the
Section 7 of CA No. 141 authorizes the President to "declare what lands are Secretary of Agriculture, shall declare that the same are not necessary
open to disposition or concession." Section 8 of CA No. 141 states that the for the public service and are open to disposition under this chapter. The
government can declare open for disposition or concession only lands that are lands included in class (d) may be disposed of by sale or lease under
"officially delimited and classified." Sections 6, 7 and 8 of CA No. 141 read as the provisions of this Act." (Emphasis supplied)
follows: Section 61 of CA No. 141 readopted, after the effectivity of the 1935
"Sec. 6. The President, upon the recommendation of the Secretary of Constitution, Section 58 of Act No. 2874 prohibiting the sale of government
Agriculture and Commerce, shall from time to time classify the lands reclaimed, foreshore and marshy disposable lands of the public domain. All
of the public domain into these lands are intended for residential, commercial, industrial or other non-
(a) Alienable or disposable, agricultural purposes. As before, Section 61 allowed only the lease of such
(b) Timber, and lands to private parties. The government could sell to private parties only
(c) Mineral lands, lands falling under Section 59 (d) of CA No. 141, or those lands for non-
and may at any time and in like manner transfer such lands from one class to agricultural purposes not classified as government reclaimed, foreshore and
another,53 for the purpose of their administration and disposition. marshy disposable lands of the public domain. Foreshore lands, however,
Sec. 7. For the purposes of the administration and disposition of alienable or became inalienable under the 1935 Constitution which only allowed the lease
disposable public lands, the President, upon recommendation by the of these lands to qualified private parties.
Secretary of Agriculture and Commerce, shall from time to time Section 58 of CA No. 141 expressly states that disposable lands of the public
declare what lands are open to disposition or concession under this domain intended for residential, commercial, industrial or other productive
Act. purposes other than agricultural "shall be disposed of under the
Sec. 8. Only those lands shall be declared open to disposition or provisions of this chapter and not otherwise." Under Section 10 of CA
concession which have been officially delimited and classified and, No. 141, the term "disposition" includes lease of the land. Any disposition of
when practicable, surveyed, and which have not been reserved for government reclaimed, foreshore and marshy disposable lands for non-
public or quasi-public uses, nor appropriated by the Government, nor in agricultural purposes must comply with Chapter IX, Title III of CA No. 141, 54
any manner become private property, nor those on which a private right unless a subsequent law amended or repealed these provisions.
authorized and recognized by this Act or any other valid law may be claimed, In his concurring opinion in the landmark case of Republic Real Estate
or which, having been reserved or appropriated, have ceased to be so. x x x." Corporation v. Court of Appeals,55 Justice Reynato S. Puno summarized
Thus, before the government could alienate or dispose of lands of the public succinctly the law on this matter, as follows:
domain, the President must first officially classify these lands as alienable or "Foreshore lands are lands of public dominion intended for public use. So too
disposable, and then declare them open to disposition or concession. There are lands reclaimed by the government by dredging, filling, or other means.
must be no law reserving these lands for public or quasi-public uses. Act 1654 mandated that the control and disposition of the foreshore and lands
The salient provisions of CA No. 141, on government reclaimed, foreshore and under water remained in the national government. Said law allowed only the
marshy lands of the public domain, are as follows: 'leasing' of reclaimed land. The Public Land Acts of 1919 and 1936 also
"Sec. 58. Any tract of land of the public domain which, being neither declared that the foreshore and lands reclaimed by the government were to
timber nor mineral land, is intended to be used for residential be "disposed of to private parties by lease only and not otherwise." Before
purposes or for commercial, industrial, or other productive purposes leasing, however, the Governor-General, upon recommendation of the
other than agricultural, and is open to disposition or concession, Secretary of Agriculture and Natural Resources, had first to determine that the
shall be disposed of under the provisions of this chapter and not land reclaimed was not necessary for the public service. This requisite must
otherwise. have been met before the land could be disposed of. But even then, the
Sec. 59. The lands disposable under this title shall be classified as foreshore and lands under water were not to be alienated and sold to
follows: private parties. The disposition of the reclaimed land was only by
(a) Lands reclaimed by the Government by dredging, filling, or other lease. The land remained property of the State." (Emphasis supplied)
means; As observed by Justice Puno in his concurring opinion, "Commonwealth Act No.
(b) Foreshore; 141 has remained in effect at present."
(c) Marshy lands or lands covered with water bordering upon the shores or The State policy prohibiting the sale to private parties of government
banks of navigable lakes or rivers; reclaimed, foreshore and marshy alienable lands of the public domain, first
(d) Lands not included in any of the foregoing classes. implemented in 1907 was thus reaffirmed in CA No. 141 after the 1935

76
Constitution took effect. The prohibition on the sale of foreshore lands, needed for public purposes, the Director of Lands shall ask the Secretary of
however, became a constitutional edict under the 1935 Constitution. Agriculture and Commerce (now the Secretary of Natural Resources) for
Foreshore lands became inalienable as natural resources of the State, unless authority to dispose of the same. Upon receipt of such authority, the Director
reclaimed by the government and classified as agricultural lands of the public of Lands shall give notice by public advertisement in the same manner as in
domain, in which case they would fall under the classification of government the case of leases or sales of agricultural public land, x x x.
reclaimed lands. Sec. 67. The lease or sale shall be made by oral bidding; and
After the effectivity of the 1935 Constitution, government reclaimed and adjudication shall be made to the highest bidder. x x x." (Emphasis
marshy disposable lands of the public domain continued to be only leased and supplied)
not sold to private parties. 56 These lands remained sui generis, as the only Thus, CA No. 141 mandates the Government to put to public auction all leases
alienable or disposable lands of the public domain the government could not or sales of alienable or disposable lands of the public domain. 58
sell to private parties. Like Act No. 1654 and Act No. 2874 before it, CA No. 141 did not repeal
Since then and until now, the only way the government can sell to private Section 5 of the Spanish Law of Waters of 1866. Private parties could still
parties government reclaimed and marshy disposable lands of the public reclaim portions of the sea with government permission. However, the
domain is for the legislature to pass a law authorizing such sale. CA No. 141 reclaimed land could become private land only if classified as
does not authorize the President to reclassify government reclaimed and alienable agricultural land of the public domain open to disposition
marshy lands into other non-agricultural lands under Section 59 (d). Lands under CA No. 141. The 1935 Constitution prohibited the alienation of all
classified under Section 59 (d) are the only alienable or disposable lands for natural resources except public agricultural lands.
non-agricultural purposes that the government could sell to private parties. The Civil Code of 1950
Moreover, Section 60 of CA No. 141 expressly requires congressional The Civil Code of 1950 readopted substantially the definition of property of
authority before lands under Section 59 that the government previously public dominion found in the Civil Code of 1889. Articles 420 and 422 of the
transferred to government units or entities could be sold to private parties. Civil Code of 1950 state that
Section 60 of CA No. 141 declares that "Art. 420. The following things are property of public dominion:
"Sec. 60. x x x The area so leased or sold shall be such as shall, in the (1) Those intended for public use, such as roads, canals, rivers, torrents, ports
judgment of the Secretary of Agriculture and Natural Resources, be reasonably and bridges constructed by the State, banks, shores, roadsteads, and others
necessary for the purposes for which such sale or lease is requested, and shall of similar character;
not exceed one hundred and forty-four hectares: Provided, however, That this (2) Those which belong to the State, without being for public use, and are
limitation shall not apply to grants, donations, or transfers made to a province, intended for some public service or for the development of the national
municipality or branch or subdivision of the Government for the purposes wealth.
deemed by said entities conducive to the public interest; but the land so x x x.
granted, donated, or transferred to a province, municipality or Art. 422. Property of public dominion, when no longer intended for public use
branch or subdivision of the Government shall not be alienated, or for public service, shall form part of the patrimonial property of the State."
encumbered, or otherwise disposed of in a manner affecting its title, Again, the government must formally declare that the property of public
except when authorized by Congress: x x x." (Emphasis supplied) dominion is no longer needed for public use or public service, before the same
The congressional authority required in Section 60 of CA No. 141 mirrors the could be classified as patrimonial property of the State. 59 In the case of
legislative authority required in Section 56 of Act No. 2874. government reclaimed and marshy lands of the public domain, the declaration
One reason for the congressional authority is that Section 60 of CA No. 141 of their being disposable, as well as the manner of their disposition, is
exempted government units and entities from the maximum area of public governed by the applicable provisions of CA No. 141.
lands that could be acquired from the State. These government units and Like the Civil Code of 1889, the Civil Code of 1950 included as property of
entities should not just turn around and sell these lands to private parties in public dominion those properties of the State which, without being for public
violation of constitutional or statutory limitations. Otherwise, the transfer of use, are intended for public service or the "development of the national
lands for non-agricultural purposes to government units and entities could be wealth." Thus, government reclaimed and marshy lands of the State, even if
used to circumvent constitutional limitations on ownership of alienable or not employed for public use or public service, if developed to enhance the
disposable lands of the public domain. In the same manner, such transfers national wealth, are classified as property of public dominion.
could also be used to evade the statutory prohibition in CA No. 141 on the sale Dispositions under the 1973 Constitution
of government reclaimed and marshy lands of the public domain to private The 1973 Constitution, which took effect on January 17, 1973, likewise
parties. Section 60 of CA No. 141 constitutes by operation of law a lien on adopted the Regalian doctrine. Section 8, Article XIV of the 1973 Constitution
these lands.57 stated that
In case of sale or lease of disposable lands of the public domain falling under "Sec. 8. All lands of the public domain, waters, minerals, coal, petroleum and
Section 59 of CA No. 141, Sections 63 and 67 require a public bidding. other mineral oils, all forces of potential energy, fisheries, wildlife, and other
Sections 63 and 67 of CA No. 141 provide as follows: natural resources of the Philippines belong to the State. With the exception
"Sec. 63. Whenever it is decided that lands covered by this chapter are not of agricultural, industrial or commercial, residential, and

77
resettlement lands of the public domain, natural resources shall not "Sec. 4. Purpose. The Authority is hereby created for the following purposes:
be alienated, and no license, concession, or lease for the exploration, (a) To reclaim land, including foreshore and submerged areas, by
development, exploitation, or utilization of any of the natural resources shall dredging, filling or other means, or to acquire reclaimed land;
be granted for a period exceeding twenty-five years, renewable for not more (b) To develop, improve, acquire, administer, deal in, subdivide, dispose,
than twenty-five years, except as to water rights for irrigation, water supply, lease and sell any and all kinds of lands, buildings, estates and other
fisheries, or industrial uses other than the development of water power, in forms of real property, owned, managed, controlled and/or operated by the
which cases, beneficial use may be the measure and the limit of the grant." government;
(Emphasis supplied) (c) To provide for, operate or administer such service as may be necessary for
The 1973 Constitution prohibited the alienation of all natural resources with the efficient, economical and beneficial utilization of the above properties.
the exception of "agricultural, industrial or commercial, residential, and Sec. 5. Powers and functions of the Authority. The Authority shall, in carrying
resettlement lands of the public domain." In contrast, the 1935 Constitution out the purposes for which it is created, have the following powers and
barred the alienation of all natural resources except "public agricultural lands." functions:
However, the term "public agricultural lands" in the 1935 Constitution (a)To prescribe its by-laws.
encompassed industrial, commercial, residential and resettlement lands of the xxx
public domain.60 If the land of public domain were neither timber nor mineral (i) To hold lands of the public domain in excess of the area permitted to
land, it would fall under the classification of agricultural land of the public private corporations by statute.
domain. Both the 1935 and 1973 Constitutions, therefore, prohibited (j) To reclaim lands and to construct work across, or otherwise, any stream,
the alienation of all natural resources except agricultural lands of the watercourse, canal, ditch, flume x x x.
public domain. xxx
The 1973 Constitution, however, limited the alienation of lands of the public (o) To perform such acts and exercise such functions as may be necessary for
domain to individuals who were citizens of the Philippines. Private the attainment of the purposes and objectives herein specified." (Emphasis
corporations, even if wholly owned by Philippine citizens, were no longer supplied)
allowed to acquire alienable lands of the public domain unlike in the 1935 PD No. 1084 authorizes PEA to reclaim both foreshore and submerged areas of
Constitution. Section 11, Article XIV of the 1973 Constitution declared that the public domain. Foreshore areas are those covered and uncovered by the
"Sec. 11. The Batasang Pambansa, taking into account conservation, ebb and flow of the tide.61 Submerged areas are those permanently under
ecological, and development requirements of the natural resources, shall water regardless of the ebb and flow of the tide. 62 Foreshore and submerged
determine by law the size of land of the public domain which may be areas indisputably belong to the public domain 63 and are inalienable unless
developed, held or acquired by, or leased to, any qualified individual, reclaimed, classified as alienable lands open to disposition, and further
corporation, or association, and the conditions therefor. No private declared no longer needed for public service.
corporation or association may hold alienable lands of the public The ban in the 1973 Constitution on private corporations from acquiring
domain except by lease not to exceed one thousand hectares in area nor alienable lands of the public domain did not apply to PEA since it was then,
may any citizen hold such lands by lease in excess of five hundred hectares or and until today, a fully owned government corporation. The constitutional ban
acquire by purchase, homestead or grant, in excess of twenty-four hectares. applied then, as it still applies now, only to "private corporations and
No private corporation or association may hold by lease, concession, license associations." PD No. 1084 expressly empowers PEA "to hold lands of the
or permit, timber or forest lands and other timber or forest resources in excess public domain" even "in excess of the area permitted to private corporations
of one hundred thousand hectares. However, such area may be increased by by statute." Thus, PEA can hold title to private lands, as well as title to
the Batasang Pambansa upon recommendation of the National Economic and lands of the public domain.
Development Authority." (Emphasis supplied) In order for PEA to sell its reclaimed foreshore and submerged alienable lands
Thus, under the 1973 Constitution, private corporations could hold alienable of the public domain, there must be legislative authority empowering PEA to
lands of the public domain only through lease. Only individuals could now sell these lands. This legislative authority is necessary in view of Section 60 of
acquire alienable lands of the public domain, and private corporations CA No.141, which states
became absolutely barred from acquiring any kind of alienable land "Sec. 60. x x x; but the land so granted, donated or transferred to a province,
of the public domain. The constitutional ban extended to all kinds of municipality, or branch or subdivision of the Government shall not be
alienable lands of the public domain, while the statutory ban under CA No. 141 alienated, encumbered or otherwise disposed of in a manner affecting its title,
applied only to government reclaimed, foreshore and marshy alienable lands except when authorized by Congress; x x x." (Emphasis supplied)
of the public domain. Without such legislative authority, PEA could not sell but only lease its
PD No. 1084 Creating the Public Estates Authority reclaimed foreshore and submerged alienable lands of the public domain.
On February 4, 1977, then President Ferdinand Marcos issued Presidential Nevertheless, any legislative authority granted to PEA to sell its reclaimed
Decree No. 1084 creating PEA, a wholly government owned and controlled alienable lands of the public domain would be subject to the constitutional ban
corporation with a special charter. Sections 4 and 8 of PD No. 1084, vests PEA on private corporations from acquiring alienable lands of the public domain.
with the following purposes and powers: Hence, such legislative authority could only benefit private individuals.

78
Dispositions under the 1987 Constitution MR. VILLEGAS: I think that is the spirit of the provision.
The 1987 Constitution, like the 1935 and 1973 Constitutions before it, has FR. BERNAS: In existing decisions involving the Iglesia ni Cristo, there were
adopted the Regalian doctrine. The 1987 Constitution declares that all natural instances where the Iglesia ni Cristo was not allowed to acquire a mere 313-
resources are "owned by the State," and except for alienable agricultural square meter land where a chapel stood because the Supreme Court said it
lands of the public domain, natural resources cannot be alienated. Sections 2 would be in violation of this." (Emphasis supplied)
and 3, Article XII of the 1987 Constitution state that In Ayog v. Cusi,64 the Court explained the rationale behind this constitutional
"Section 2. All lands of the public domain, waters, minerals, coal, petroleum ban in this way:
and other mineral oils, all forces of potential energy, fisheries, forests or "Indeed, one purpose of the constitutional prohibition against purchases of
timber, wildlife, flora and fauna, and other natural resources are owned by public agricultural lands by private corporations is to equitably diffuse land
the State. With the exception of agricultural lands, all other natural ownership or to encourage 'owner-cultivatorship and the economic family-size
resources shall not be alienated. The exploration, development, and farm' and to prevent a recurrence of cases like the instant case. Huge
utilization of natural resources shall be under the full control and supervision landholdings by corporations or private persons had spawned social unrest."
of the State. x x x. However, if the constitutional intent is to prevent huge landholdings, the
Section 3. Lands of the public domain are classified into agricultural, forest or Constitution could have simply limited the size of alienable lands of the public
timber, mineral lands, and national parks. Agricultural lands of the public domain that corporations could acquire. The Constitution could have followed
domain may be further classified by law according to the uses which they may the limitations on individuals, who could acquire not more than 24 hectares of
be devoted. Alienable lands of the public domain shall be limited to alienable lands of the public domain under the 1973 Constitution, and not
agricultural lands. Private corporations or associations may not hold more than 12 hectares under the 1987 Constitution.
such alienable lands of the public domain except by lease, for a If the constitutional intent is to encourage economic family-size farms, placing
period not exceeding twenty-five years, renewable for not more than the land in the name of a corporation would be more effective in preventing
twenty-five years, and not to exceed one thousand hectares in area. the break-up of farmlands. If the farmland is registered in the name of a
Citizens of the Philippines may lease not more than five hundred hectares, or corporation, upon the death of the owner, his heirs would inherit shares in the
acquire not more than twelve hectares thereof by purchase, homestead, or corporation instead of subdivided parcels of the farmland. This would prevent
grant. the continuing break-up of farmlands into smaller and smaller plots from one
Taking into account the requirements of conservation, ecology, and generation to the next.
development, and subject to the requirements of agrarian reform, the In actual practice, the constitutional ban strengthens the constitutional
Congress shall determine, by law, the size of lands of the public domain which limitation on individuals from acquiring more than the allowed area of
may be acquired, developed, held, or leased and the conditions therefor." alienable lands of the public domain. Without the constitutional ban,
(Emphasis supplied) individuals who already acquired the maximum area of alienable lands of the
The 1987 Constitution continues the State policy in the 1973 Constitution public domain could easily set up corporations to acquire more alienable
banning private corporations from acquiring any kind of alienable land of public lands. An individual could own as many corporations as his means
the public domain. Like the 1973 Constitution, the 1987 Constitution allows would allow him. An individual could even hide his ownership of a corporation
private corporations to hold alienable lands of the public domain only by putting his nominees as stockholders of the corporation. The corporation is
through lease. As in the 1935 and 1973 Constitutions, the general law a convenient vehicle to circumvent the constitutional limitation on acquisition
governing the lease to private corporations of reclaimed, foreshore and by individuals of alienable lands of the public domain.
marshy alienable lands of the public domain is still CA No. 141. The constitutional intent, under the 1973 and 1987 Constitutions, is to
The Rationale behind the Constitutional Ban transfer ownership of only a limited area of alienable land of the public
The rationale behind the constitutional ban on corporations from acquiring, domain to a qualified individual. This constitutional intent is safeguarded by
except through lease, alienable lands of the public domain is not well the provision prohibiting corporations from acquiring alienable lands of the
understood. During the deliberations of the 1986 Constitutional Commission, public domain, since the vehicle to circumvent the constitutional intent is
the commissioners probed the rationale behind this ban, thus: removed. The available alienable public lands are gradually decreasing in the
"FR. BERNAS: Mr. Vice-President, my questions have reference to page 3, line face of an ever-growing population. The most effective way to insure faithful
5 which says: adherence to this constitutional intent is to grant or sell alienable lands of the
`No private corporation or association may hold alienable lands of the public public domain only to individuals. This, it would seem, is the practical benefit
domain except by lease, not to exceed one thousand hectares in area.' arising from the constitutional ban.
If we recall, this provision did not exist under the 1935 Constitution, but this The Amended Joint Venture Agreement
was introduced in the 1973 Constitution. In effect, it prohibits private The subject matter of the Amended JVA, as stated in its second Whereas
corporations from acquiring alienable public lands. But it has not been very clause, consists of three properties, namely:
clear in jurisprudence what the reason for this is. In some of the cases 1. "[T]hree partially reclaimed and substantially eroded islands along Emilio
decided in 1982 and 1983, it was indicated that the purpose of this is to Aguinaldo Boulevard in Paranaque and Las Pinas, Metro Manila, with a
prevent large landholdings. Is that the intent of this provision? combined titled area of 1,578,441 square meters;"

79
2. "[A]nother area of 2,421,559 square meters contiguous to the three and other mineral oils, all forces of potential energy, fisheries, forests or
islands;" and timber, wildlife, flora and fauna, and other natural resources are owned by the
3. "[A]t AMARI's option as approved by PEA, an additional 350 hectares more State. With the exception of agricultural lands, all other natural
or less to regularize the configuration of the reclaimed area." 65 resources shall not be alienated. x x x.
PEA confirms that the Amended JVA involves "the development of the Freedom xxx
Islands and further reclamation of about 250 hectares x x x," plus an option Section 3. x x x Alienable lands of the public domain shall be limited to
"granted to AMARI to subsequently reclaim another 350 hectares x x x."66 agricultural lands. Private corporations or associations may not hold
In short, the Amended JVA covers a reclamation area of 750 hectares. Only such alienable lands of the public domain except by lease, x x
157.84 hectares of the 750-hectare reclamation project have been x."(Emphasis supplied)
reclaimed, and the rest of the 592.15 hectares are still submerged Classification of Reclaimed Foreshore and Submerged Areas
areas forming part of Manila Bay. PEA readily concedes that lands reclaimed from foreshore or submerged areas
Under the Amended JVA, AMARI will reimburse PEA the sum of of Manila Bay are alienable or disposable lands of the public domain. In its
P1,894,129,200.00 for PEA's "actual cost" in partially reclaiming the Freedom Memorandum,67 PEA admits that
Islands. AMARI will also complete, at its own expense, the reclamation of the "Under the Public Land Act (CA 141, as amended), reclaimed lands are
Freedom Islands. AMARI will further shoulder all the reclamation costs of all classified as alienable and disposable lands of the public domain:
the other areas, totaling 592.15 hectares, still to be reclaimed. AMARI and PEA 'Sec. 59. The lands disposable under this title shall be classified as follows:
will share, in the proportion of 70 percent and 30 percent, respectively, the (a) Lands reclaimed by the government by dredging, filling, or other means;
total net usable area which is defined in the Amended JVA as the total x x x.'" (Emphasis supplied)
reclaimed area less 30 percent earmarked for common areas. Title to AMARI's Likewise, the Legal Task Force68 constituted under Presidential Administrative
share in the net usable area, totaling 367.5 hectares, will be issued in the Order No. 365 admitted in its Report and Recommendation to then President
name of AMARI. Section 5.2 (c) of the Amended JVA provides that Fidel V. Ramos, "[R]eclaimed lands are classified as alienable and
"x x x, PEA shall have the duty to execute without delay the necessary deed of disposable lands of the public domain."69 The Legal Task Force concluded
transfer or conveyance of the title pertaining to AMARI's Land share based on that
the Land Allocation Plan. PEA, when requested in writing by AMARI, "D. Conclusion
shall then cause the issuance and delivery of the proper certificates Reclaimed lands are lands of the public domain. However, by statutory
of title covering AMARI's Land Share in the name of AMARI, x x x; authority, the rights of ownership and disposition over reclaimed lands have
provided, that if more than seventy percent (70%) of the titled area at any been transferred to PEA, by virtue of which PEA, as owner, may validly convey
given time pertains to AMARI, PEA shall deliver to AMARI only seventy percent the same to any qualified person without violating the Constitution or any
(70%) of the titles pertaining to AMARI, until such time when a corresponding statute.
proportionate area of additional land pertaining to PEA has been titled." The constitutional provision prohibiting private corporations from holding
(Emphasis supplied) public land, except by lease (Sec. 3, Art. XVII,70 1987 Constitution), does not
Indisputably, under the Amended JVA AMARI will acquire and own a apply to reclaimed lands whose ownership has passed on to PEA by statutory
maximum of 367.5 hectares of reclaimed land which will be titled in grant."
its name. Under Section 2, Article XII of the 1987 Constitution, the foreshore and
To implement the Amended JVA, PEA delegated to the unincorporated PEA- submerged areas of Manila Bay are part of the "lands of the public domain,
AMARI joint venture PEA's statutory authority, rights and privileges to reclaim waters x x x and other natural resources" and consequently "owned by the
foreshore and submerged areas in Manila Bay. Section 3.2.a of the Amended State." As such, foreshore and submerged areas "shall not be alienated,"
JVA states that unless they are classified as "agricultural lands" of the public domain. The
"PEA hereby contributes to the joint venture its rights and privileges to mere reclamation of these areas by PEA does not convert these inalienable
perform Rawland Reclamation and Horizontal Development as well as own the natural resources of the State into alienable or disposable lands of the public
Reclamation Area, thereby granting the Joint Venture the full and exclusive domain. There must be a law or presidential proclamation officially classifying
right, authority and privilege to undertake the Project in accordance with the these reclaimed lands as alienable or disposable and open to disposition or
Master Development Plan." concession. Moreover, these reclaimed lands cannot be classified as alienable
The Amended JVA is the product of a renegotiation of the original JVA dated or disposable if the law has reserved them for some public or quasi-public
April 25, 1995 and its supplemental agreement dated August 9, 1995. use.71
The Threshold Issue Section 8 of CA No. 141 provides that "only those lands shall be declared open
The threshold issue is whether AMARI, a private corporation, can acquire and to disposition or concession which have been officially delimited and
own under the Amended JVA 367.5 hectares of reclaimed foreshore and classified."72 The President has the authority to classify inalienable lands of
submerged areas in Manila Bay in view of Sections 2 and 3, Article XII of the the public domain into alienable or disposable lands of the public domain,
1987 Constitution which state that: pursuant to Section 6 of CA No. 141. In Laurel vs. Garcia, 73 the Executive
"Section 2. All lands of the public domain, waters, minerals, coal, petroleum, Department attempted to sell the Roppongi property in Tokyo, Japan, which

80
was acquired by the Philippine Government for use as the Chancery of the "Article 5. Lands reclaimed from the sea in consequence of works constructed
Philippine Embassy. Although the Chancery had transferred to another location by the State, or by the provinces, pueblos or private persons, with proper
thirteen years earlier, the Court still ruled that, under Article 422 74 of the Civil permission, shall become the property of the party constructing such works,
Code, a property of public dominion retains such character until formally unless otherwise provided by the terms of the grant of authority."
declared otherwise. The Court ruled that (Emphasis supplied)
"The fact that the Roppongi site has not been used for a long time for actual Under Article 5 of the Spanish Law of Waters of 1866, private parties could
Embassy service does not automatically convert it to patrimonial property. reclaim from the sea only with "proper permission" from the State. Private
Any such conversion happens only if the property is withdrawn from public use parties could own the reclaimed land only if not "otherwise provided by the
(Cebu Oxygen and Acetylene Co. v. Bercilles, 66 SCRA 481 [1975]. A terms of the grant of authority." This clearly meant that no one could reclaim
property continues to be part of the public domain, not available for from the sea without permission from the State because the sea is property of
private appropriation or ownership 'until there is a formal public dominion. It also meant that the State could grant or withhold
declaration on the part of the government to withdraw it from being ownership of the reclaimed land because any reclaimed land, like the sea from
such' (Ignacio v. Director of Lands, 108 Phil. 335 [1960]." (Emphasis supplied) which it emerged, belonged to the State. Thus, a private person reclaiming
PD No. 1085, issued on February 4, 1977, authorized the issuance of special from the sea without permission from the State could not acquire ownership of
land patents for lands reclaimed by PEA from the foreshore or submerged the reclaimed land which would remain property of public dominion like the
areas of Manila Bay. On January 19, 1988 then President Corazon C. Aquino sea it replaced.76 Article 5 of the Spanish Law of Waters of 1866 adopted the
issued Special Patent No. 3517 in the name of PEA for the 157.84 hectares time-honored principle of land ownership that "all lands that were not
comprising the partially reclaimed Freedom Islands. Subsequently, on April 9, acquired from the government, either by purchase or by grant, belong to the
1999 the Register of Deeds of the Municipality of Paranaque issued TCT Nos. public domain."77
7309, 7311 and 7312 in the name of PEA pursuant to Section 103 of PD No. Article 5 of the Spanish Law of Waters must be read together with laws
1529 authorizing the issuance of certificates of title corresponding to land subsequently enacted on the disposition of public lands. In particular, CA No.
patents. To this day, these certificates of title are still in the name of PEA. 141 requires that lands of the public domain must first be classified as
PD No. 1085, coupled with President Aquino's actual issuance of a special alienable or disposable before the government can alienate them. These lands
patent covering the Freedom Islands, is equivalent to an official proclamation must not be reserved for public or quasi-public purposes. 78 Moreover, the
classifying the Freedom Islands as alienable or disposable lands of the public contract between CDCP and the government was executed after the
domain. PD No. 1085 and President Aquino's issuance of a land patent also effectivity of the 1973 Constitution which barred private corporations from
constitute a declaration that the Freedom Islands are no longer needed for acquiring any kind of alienable land of the public domain. This contract could
public service. The Freedom Islands are thus alienable or disposable not have converted the Freedom Islands into private lands of a private
lands of the public domain, open to disposition or concession to corporation.
qualified parties. Presidential Decree No. 3-A, issued on January 11, 1973, revoked all laws
At the time then President Aquino issued Special Patent No. 3517, PEA had authorizing the reclamation of areas under water and revested solely in the
already reclaimed the Freedom Islands although subsequently there were National Government the power to reclaim lands. Section 1 of PD No. 3-A
partial erosions on some areas. The government had also completed the declared that
necessary surveys on these islands. Thus, the Freedom Islands were no longer "The provisions of any law to the contrary notwithstanding, the
part of Manila Bay but part of the land mass. Section 3, Article XII of the 1987 reclamation of areas under water, whether foreshore or inland, shall be
Constitution classifies lands of the public domain into "agricultural, forest or limited to the National Government or any person authorized by it
timber, mineral lands, and national parks." Being neither timber, mineral, nor under a proper contract. (Emphasis supplied)
national park lands, the reclaimed Freedom Islands necessarily fall under the x x x."
classification of agricultural lands of the public domain. Under the 1987 PD No. 3-A repealed Section 5 of the Spanish Law of Waters of 1866 because
Constitution, agricultural lands of the public domain are the only natural reclamation of areas under water could now be undertaken only by the
resources that the State may alienate to qualified private parties. All other National Government or by a person contracted by the National Government.
natural resources, such as the seas or bays, are "waters x x x owned by the Private parties may reclaim from the sea only under a contract with the
State" forming part of the public domain, and are inalienable pursuant to National Government, and no longer by grant or permission as provided in
Section 2, Article XII of the 1987 Constitution. Section 5 of the Spanish Law of Waters of 1866.
AMARI claims that the Freedom Islands are private lands because CDCP, then Executive Order No. 525, issued on February 14, 1979, designated PEA as the
a private corporation, reclaimed the islands under a contract dated November National Government's implementing arm to undertake "all reclamation
20, 1973 with the Commissioner of Public Highways. AMARI, citing Article 5 of projects of the government," which "shall be undertaken by the PEA or
the Spanish Law of Waters of 1866, argues that "if the ownership of reclaimed through a proper contract executed by it with any person or entity."
lands may be given to the party constructing the works, then it cannot be said Under such contract, a private party receives compensation for reclamation
that reclaimed lands are lands of the public domain which the State may not services rendered to PEA. Payment to the contractor may be in cash, or in kind
alienate."75 Article 5 of the Spanish Law of Waters reads as follows: consisting of portions of the reclaimed land, subject to the constitutional ban

81
on private corporations from acquiring alienable lands of the public domain. still needed for public service.1wphi1.nt
The reclaimed land can be used as payment in kind only if the reclaimed land Section 3 of EO No. 525, by declaring that all lands reclaimed by PEA "shall
is first classified as alienable or disposable land open to disposition, and then belong to or be owned by the PEA," could not automatically operate to classify
declared no longer needed for public service. inalienable lands into alienable or disposable lands of the public domain.
The Amended JVA covers not only the Freedom Islands, but also an additional Otherwise, reclaimed foreshore and submerged lands of the public domain
592.15 hectares which are still submerged and forming part of Manila Bay. would automatically become alienable once reclaimed by PEA, whether or not
There is no legislative or Presidential act classifying these classified as alienable or disposable.
submerged areas as alienable or disposable lands of the public The Revised Administrative Code of 1987, a later law than either PD No. 1084
domain open to disposition. These submerged areas are not covered by or EO No. 525, vests in the Department of Environment and Natural Resources
any patent or certificate of title. There can be no dispute that these ("DENR" for brevity) the following powers and functions:
submerged areas form part of the public domain, and in their present state "Sec. 4. Powers and Functions. The Department shall:
are inalienable and outside the commerce of man. Until reclaimed from (1) x x x
the sea, these submerged areas are, under the Constitution, "waters x x x xxx
owned by the State," forming part of the public domain and consequently (4) Exercise supervision and control over forest lands, alienable and
inalienable. Only when actually reclaimed from the sea can these submerged disposable public lands, mineral resources and, in the process of exercising
areas be classified as public agricultural lands, which under the Constitution such control, impose appropriate taxes, fees, charges, rentals and any such
are the only natural resources that the State may alienate. Once reclaimed form of levy and collect such revenues for the exploration, development,
and transformed into public agricultural lands, the government may then utilization or gathering of such resources;
officially classify these lands as alienable or disposable lands open to xxx
disposition. Thereafter, the government may declare these lands no longer (14) Promulgate rules, regulations and guidelines on the issuance of
needed for public service. Only then can these reclaimed lands be considered licenses, permits, concessions, lease agreements and such other
alienable or disposable lands of the public domain and within the commerce of privileges concerning the development, exploration and utilization of
man. the country's marine, freshwater, and brackish water and over all
The classification of PEA's reclaimed foreshore and submerged lands into aquatic resources of the country and shall continue to oversee,
alienable or disposable lands open to disposition is necessary because PEA is supervise and police our natural resources; cancel or cause to cancel
tasked under its charter to undertake public services that require the use of such privileges upon failure, non-compliance or violations of any regulation,
lands of the public domain. Under Section 5 of PD No. 1084, the functions of order, and for all other causes which are in furtherance of the conservation of
PEA include the following: "[T]o own or operate railroads, tramways and other natural resources and supportive of the national interest;
kinds of land transportation, x x x; [T]o construct, maintain and operate such (15) Exercise exclusive jurisdiction on the management and
systems of sanitary sewers as may be necessary; [T]o construct, maintain and disposition of all lands of the public domain and serve as the sole
operate such storm drains as may be necessary." PEA is empowered to issue agency responsible for classification, sub-classification, surveying and
"rules and regulations as may be necessary for the proper use by private titling of lands in consultation with appropriate agencies." 80 (Emphasis
parties of any or all of the highways, roads, utilities, buildings and/or supplied)
any of its properties and to impose or collect fees or tolls for their use." As manager, conservator and overseer of the natural resources of the State,
Thus, part of the reclaimed foreshore and submerged lands held by the PEA DENR exercises "supervision and control over alienable and disposable public
would actually be needed for public use or service since many of the functions lands." DENR also exercises "exclusive jurisdiction on the management and
imposed on PEA by its charter constitute essential public services. disposition of all lands of the public domain." Thus, DENR decides whether
Moreover, Section 1 of Executive Order No. 525 provides that PEA "shall be areas under water, like foreshore or submerged areas of Manila Bay, should be
primarily responsible for integrating, directing, and coordinating all reclaimed or not. This means that PEA needs authorization from DENR before
reclamation projects for and on behalf of the National Government." The same PEA can undertake reclamation projects in Manila Bay, or in any part of the
section also states that "[A]ll reclamation projects shall be approved by the country.
President upon recommendation of the PEA, and shall be undertaken by the DENR also exercises exclusive jurisdiction over the disposition of all lands of
PEA or through a proper contract executed by it with any person or entity; x x the public domain. Hence, DENR decides whether reclaimed lands of PEA
x." Thus, under EO No. 525, in relation to PD No. 3-A and PD No.1084, PEA should be classified as alienable under Sections 6 81 and 782 of CA No. 141.
became the primary implementing agency of the National Government to Once DENR decides that the reclaimed lands should be so classified, it then
reclaim foreshore and submerged lands of the public domain. EO No. 525 recommends to the President the issuance of a proclamation classifying the
recognized PEA as the government entity "to undertake the reclamation of lands as alienable or disposable lands of the public domain open to
lands and ensure their maximum utilization in promoting public welfare disposition. We note that then DENR Secretary Fulgencio S. Factoran, Jr.
and interests."79 Since large portions of these reclaimed lands would countersigned Special Patent No. 3517 in compliance with the Revised
obviously be needed for public service, there must be a formal declaration Administrative Code and Sections 6 and 7 of CA No. 141.
segregating reclaimed lands no longer needed for public service from those In short, DENR is vested with the power to authorize the reclamation of areas

82
under water, while PEA is vested with the power to undertake the physical aforesaid contract shall be recognized and respected.
reclamation of areas under water, whether directly or through private Henceforth, the Public Estates Authority shall exercise the rights and assume
contractors. DENR is also empowered to classify lands of the public domain the obligations of the Republic of the Philippines (Department of Public
into alienable or disposable lands subject to the approval of the President. On Highways) arising from, or incident to, the aforesaid contract between the
the other hand, PEA is tasked to develop, sell or lease the reclaimed alienable Republic of the Philippines and the Construction and Development Corporation
lands of the public domain. of the Philippines.
Clearly, the mere physical act of reclamation by PEA of foreshore or In consideration of the foregoing transfer and assignment, the Public Estates
submerged areas does not make the reclaimed lands alienable or disposable Authority shall issue in favor of the Republic of the Philippines the
lands of the public domain, much less patrimonial lands of PEA. Likewise, the corresponding shares of stock in said entity with an issued value of said
mere transfer by the National Government of lands of the public domain to shares of stock (which) shall be deemed fully paid and non-assessable.
PEA does not make the lands alienable or disposable lands of the public The Secretary of Public Highways and the General Manager of the Public
domain, much less patrimonial lands of PEA. Estates Authority shall execute such contracts or agreements, including
Absent two official acts a classification that these lands are alienable or appropriate agreements with the Construction and Development Corporation
disposable and open to disposition and a declaration that these lands are not of the Philippines, as may be necessary to implement the above.
needed for public service, lands reclaimed by PEA remain inalienable lands of Special land patent/patents shall be issued by the Secretary of
the public domain. Only such an official classification and formal declaration Natural Resources in favor of the Public Estates Authority without
can convert reclaimed lands into alienable or disposable lands of the public prejudice to the subsequent transfer to the contractor or his
domain, open to disposition under the Constitution, Title I and Title III 83 of CA assignees of such portion or portions of the land reclaimed or to be
No. 141 and other applicable laws.84 reclaimed as provided for in the above-mentioned contract. On the
PEA's Authority to Sell Reclaimed Lands basis of such patents, the Land Registration Commission shall issue
PEA, like the Legal Task Force, argues that as alienable or disposable lands of the corresponding certificate of title." (Emphasis supplied)
the public domain, the reclaimed lands shall be disposed of in accordance with On the other hand, Section 3 of EO No. 525, issued on February 14, 1979,
CA No. 141, the Public Land Act. PEA, citing Section 60 of CA No. 141, admits provides that -
that reclaimed lands transferred to a branch or subdivision of the government "Sec. 3. All lands reclaimed by PEA shall belong to or be owned by the
"shall not be alienated, encumbered, or otherwise disposed of in a manner PEA which shall be responsible for its administration, development, utilization
affecting its title, except when authorized by Congress: x x x."85 or disposition in accordance with the provisions of Presidential Decree No.
(Emphasis by PEA) 1084. Any and all income that the PEA may derive from the sale, lease or use
In Laurel vs. Garcia,86 the Court cited Section 48 of the Revised of reclaimed lands shall be used in accordance with the provisions of
Administrative Code of 1987, which states that Presidential Decree No. 1084."
"Sec. 48. Official Authorized to Convey Real Property. Whenever real property There is no express authority under either PD No. 1085 or EO No. 525 for PEA
of the Government is authorized by law to be conveyed, the deed of to sell its reclaimed lands. PD No. 1085 merely transferred "ownership and
conveyance shall be executed in behalf of the government by the following: x administration" of lands reclaimed from Manila Bay to PEA, while EO No. 525
x x." declared that lands reclaimed by PEA "shall belong to or be owned by PEA."
Thus, the Court concluded that a law is needed to convey any real property EO No. 525 expressly states that PEA should dispose of its reclaimed lands "in
belonging to the Government. The Court declared that - accordance with the provisions of Presidential Decree No. 1084," the charter
"It is not for the President to convey real property of the government on his or of PEA.
her own sole will. Any such conveyance must be authorized and PEA's charter, however, expressly tasks PEA "to develop, improve, acquire,
approved by a law enacted by the Congress. It requires executive and administer, deal in, subdivide, dispose, lease and sell any and all kinds of
legislative concurrence." (Emphasis supplied) lands x x x owned, managed, controlled and/or operated by the
PEA contends that PD No. 1085 and EO No. 525 constitute the legislative government."87 (Emphasis supplied) There is, therefore, legislative
authority allowing PEA to sell its reclaimed lands. PD No. 1085, issued on authority granted to PEA to sell its lands, whether patrimonial or
February 4, 1977, provides that alienable lands of the public domain. PEA may sell to private parties its
"The land reclaimed in the foreshore and offshore area of Manila Bay patrimonial properties in accordance with the PEA charter free from
pursuant to the contract for the reclamation and construction of the Manila- constitutional limitations. The constitutional ban on private corporations from
Cavite Coastal Road Project between the Republic of the Philippines and the acquiring alienable lands of the public domain does not apply to the sale of
Construction and Development Corporation of the Philippines dated November PEA's patrimonial lands.
20, 1973 and/or any other contract or reclamation covering the same area is PEA may also sell its alienable or disposable lands of the public domain
hereby transferred, conveyed and assigned to the ownership and to private individuals since, with the legislative authority, there is no longer
administration of the Public Estates Authority established pursuant to any statutory prohibition against such sales and the constitutional ban does
PD No. 1084; Provided, however, That the rights and interests of the not apply to individuals. PEA, however, cannot sell any of its alienable or
Construction and Development Corporation of the Philippines pursuant to the disposable lands of the public domain to private corporations since Section 3,

83
Article XII of the 1987 Constitution expressly prohibits such sales. The public auction."
legislative authority benefits only individuals. Private corporations remain At the public auction sale, only Philippine citizens are qualified to bid for PEA's
barred from acquiring any kind of alienable land of the public domain, reclaimed foreshore and submerged alienable lands of the public domain.
including government reclaimed lands. Private corporations are barred from bidding at the auction sale of any kind of
The provision in PD No. 1085 stating that portions of the reclaimed lands could alienable land of the public domain.
be transferred by PEA to the "contractor or his assignees" (Emphasis supplied) PEA originally scheduled a public bidding for the Freedom Islands on
would not apply to private corporations but only to individuals because of the December 10, 1991. PEA imposed a condition that the winning bidder should
constitutional ban. Otherwise, the provisions of PD No. 1085 would violate reclaim another 250 hectares of submerged areas to regularize the shape of
both the 1973 and 1987 Constitutions. the Freedom Islands, under a 60-40 sharing of the additional reclaimed areas
The requirement of public auction in the sale of reclaimed lands in favor of the winning bidder. 92 No one, however, submitted a bid. On
Assuming the reclaimed lands of PEA are classified as alienable or disposable December 23, 1994, the Government Corporate Counsel advised PEA it could
lands open to disposition, and further declared no longer needed for public sell the Freedom Islands through negotiation, without need of another public
service, PEA would have to conduct a public bidding in selling or leasing these bidding, because of the failure of the public bidding on December 10, 1991. 93
lands. PEA must observe the provisions of Sections 63 and 67 of CA No. 141 However, the original JVA dated April 25, 1995 covered not only the Freedom
requiring public auction, in the absence of a law exempting PEA from holding a Islands and the additional 250 hectares still to be reclaimed, it also granted an
public auction.88 Special Patent No. 3517 expressly states that the patent is option to AMARI to reclaim another 350 hectares. The original JVA, a
issued by authority of the Constitution and PD No. 1084, "supplemented by negotiated contract, enlarged the reclamation area to 750 hectares.94 The
Commonwealth Act No. 141, as amended." This is an acknowledgment that failure of public bidding on December 10, 1991, involving only 407.84
the provisions of CA No. 141 apply to the disposition of reclaimed alienable hectares,95 is not a valid justification for a negotiated sale of 750 hectares,
lands of the public domain unless otherwise provided by law. Executive Order almost double the area publicly auctioned. Besides, the failure of public
No. 654,89 which authorizes PEA "to determine the kind and manner of bidding happened on December 10, 1991, more than three years before the
payment for the transfer" of its assets and properties, does not exempt PEA signing of the original JVA on April 25, 1995. The economic situation in the
from the requirement of public auction. EO No. 654 merely authorizes PEA to country had greatly improved during the intervening period.
decide the mode of payment, whether in kind and in installment, but does not Reclamation under the BOT Law and the Local Government Code
authorize PEA to dispense with public auction. The constitutional prohibition in Section 3, Article XII of the 1987 Constitution
Moreover, under Section 79 of PD No. 1445, otherwise known as the is absolute and clear: "Private corporations or associations may not hold such
Government Auditing Code, the government is required to sell valuable alienable lands of the public domain except by lease, x x x." Even Republic Act
government property through public bidding. Section 79 of PD No. 1445 No. 6957 ("BOT Law," for brevity), cited by PEA and AMARI as legislative
mandates that authority to sell reclaimed lands to private parties, recognizes the
"Section 79. When government property has become unserviceable for any constitutional ban. Section 6 of RA No. 6957 states
cause, or is no longer needed, it shall, upon application of the officer "Sec. 6. Repayment Scheme. - For the financing, construction, operation and
accountable therefor, be inspected by the head of the agency or his duly maintenance of any infrastructure projects undertaken through the build-
authorized representative in the presence of the auditor concerned and, if operate-and-transfer arrangement or any of its variations pursuant to the
found to be valueless or unsaleable, it may be destroyed in their presence. If provisions of this Act, the project proponent x x x may likewise be repaid in
found to be valuable, it may be sold at public auction to the highest the form of a share in the revenue of the project or other non-monetary
bidder under the supervision of the proper committee on award or similar payments, such as, but not limited to, the grant of a portion or percentage of
body in the presence of the auditor concerned or other authorized the reclaimed land, subject to the constitutional requirements with
representative of the Commission, after advertising by printed notice in respect to the ownership of the land: x x x." (Emphasis supplied)
the Official Gazette, or for not less than three consecutive days in A private corporation, even one that undertakes the physical reclamation of a
any newspaper of general circulation, or where the value of the property government BOT project, cannot acquire reclaimed alienable lands of the
does not warrant the expense of publication, by notices posted for a like public domain in view of the constitutional ban.
period in at least three public places in the locality where the property is to be Section 302 of the Local Government Code, also mentioned by PEA and
sold. In the event that the public auction fails, the property may be AMARI, authorizes local governments in land reclamation projects to pay the
sold at a private sale at such price as may be fixed by the same contractor or developer in kind consisting of a percentage of the reclaimed
committee or body concerned and approved by the Commission." land, to wit:
It is only when the public auction fails that a negotiated sale is allowed, in "Section 302. Financing, Construction, Maintenance, Operation, and
which case the Commission on Audit must approve the selling price. 90 The Management of Infrastructure Projects by the Private Sector. x x x
Commission on Audit implements Section 79 of the Government Auditing Code xxx
through Circular No. 89-29691 dated January 27, 1989. This circular In case of land reclamation or construction of industrial estates, the
emphasizes that government assets must be disposed of only through public repayment plan may consist of the grant of a portion or percentage of the
auction, and a negotiated sale can be resorted to only in case of "failure of reclaimed land or the industrial estate constructed."

84
Although Section 302 of the Local Government Code does not contain a or patents involving public lands, provides that 'Whenever public lands in the
proviso similar to that of the BOT Law, the constitutional restrictions on land Philippine Islands belonging to the Government of the United States or to the
ownership automatically apply even though not expressly mentioned in the Government of the Philippines are alienated, granted or conveyed to persons
Local Government Code. or to public or private corporations, the same shall be brought forthwith under
Thus, under either the BOT Law or the Local Government Code, the contractor the operation of this Act (Land Registration Act, Act 496) and shall become
or developer, if a corporate entity, can only be paid with leaseholds on registered lands.'"
portions of the reclaimed land. If the contractor or developer is an individual, The first four cases cited involve petitions to cancel the land patents and the
portions of the reclaimed land, not exceeding 12 hectares 96 of non-agricultural corresponding certificates of titles issued to private parties. These four
lands, may be conveyed to him in ownership in view of the legislative cases uniformly hold that the Director of Lands has no jurisdiction over private
authority allowing such conveyance. This is the only way these provisions of lands or that upon issuance of the certificate of title the land automatically
the BOT Law and the Local Government Code can avoid a direct collision with comes under the Torrens System. The fifth case cited involves the registration
Section 3, Article XII of the 1987 Constitution. under the Torrens System of a 12.8-hectare public land granted by the
Registration of lands of the public domain National Government to Mindanao Medical Center, a government unit under
Finally, PEA theorizes that the "act of conveying the ownership of the the Department of Health. The National Government transferred the 12.8-
reclaimed lands to public respondent PEA transformed such lands of the public hectare public land to serve as the site for the hospital buildings and other
domain to private lands." This theory is echoed by AMARI which maintains facilities of Mindanao Medical Center, which performed a public service. The
that the "issuance of the special patent leading to the eventual issuance of Court affirmed the registration of the 12.8-hectare public land in the name of
title takes the subject land away from the land of public domain and converts Mindanao Medical Center under Section 122 of Act No. 496. This fifth case is
the property into patrimonial or private property." In short, PEA and AMARI an example of a public land being registered under Act No. 496 without the
contend that with the issuance of Special Patent No. 3517 and the land losing its character as a property of public dominion.
corresponding certificates of titles, the 157.84 hectares comprising the In the instant case, the only patent and certificates of title issued are those in
Freedom Islands have become private lands of PEA. In support of their theory, the name of PEA, a wholly government owned corporation performing public
PEA and AMARI cite the following rulings of the Court: as well as proprietary functions. No patent or certificate of title has been
1. Sumail v. Judge of CFI of Cotabato,97 where the Court held issued to any private party. No one is asking the Director of Lands to cancel
"Once the patent was granted and the corresponding certificate of title was PEA's patent or certificates of title. In fact, the thrust of the instant petition is
issued, the land ceased to be part of the public domain and became private that PEA's certificates of title should remain with PEA, and the land covered by
property over which the Director of Lands has neither control nor jurisdiction." these certificates, being alienable lands of the public domain, should not be
2. Lee Hong Hok v. David,98 where the Court declared - sold to a private corporation.
"After the registration and issuance of the certificate and duplicate certificate Registration of land under Act No. 496 or PD No. 1529 does not vest in the
of title based on a public land patent, the land covered thereby automatically registrant private or public ownership of the land. Registration is not a mode
comes under the operation of Republic Act 496 subject to all the safeguards of acquiring ownership but is merely evidence of ownership previously
provided therein."3. Heirs of Gregorio Tengco v. Heirs of Jose Aliwalas,99 where conferred by any of the recognized modes of acquiring ownership. Registration
the Court ruled - does not give the registrant a better right than what the registrant had prior to
"While the Director of Lands has the power to review homestead patents, he the registration.102 The registration of lands of the public domain under the
may do so only so long as the land remains part of the public domain and Torrens system, by itself, cannot convert public lands into private lands. 103
continues to be under his exclusive control; but once the patent is registered Jurisprudence holding that upon the grant of the patent or issuance of the
and a certificate of title is issued, the land ceases to be part of the public certificate of title the alienable land of the public domain automatically
domain and becomes private property over which the Director of Lands has becomes private land cannot apply to government units and entities like PEA.
neither control nor jurisdiction." The transfer of the Freedom Islands to PEA was made subject to the provisions
4. Manalo v. Intermediate Appellate Court,100 where the Court held of CA No. 141 as expressly stated in Special Patent No. 3517 issued by then
"When the lots in dispute were certified as disposable on May 19, 1971, and President Aquino, to wit:
free patents were issued covering the same in favor of the private "NOW, THEREFORE, KNOW YE, that by authority of the Constitution of the
respondents, the said lots ceased to be part of the public domain and, Philippines and in conformity with the provisions of Presidential Decree No.
therefore, the Director of Lands lost jurisdiction over the same." 1084, supplemented by Commonwealth Act No. 141, as amended,
5.Republic v. Court of Appeals,101 where the Court stated there are hereby granted and conveyed unto the Public Estates Authority the
"Proclamation No. 350, dated October 9, 1956, of President Magsaysay legally aforesaid tracts of land containing a total area of one million nine hundred
effected a land grant to the Mindanao Medical Center, Bureau of Medical fifteen thousand eight hundred ninety four (1,915,894) square meters; the
Services, Department of Health, of the whole lot, validly sufficient for initial technical description of which are hereto attached and made an integral part
registration under the Land Registration Act. Such land grant is constitutive of hereof." (Emphasis supplied)
a 'fee simple' title or absolute title in favor of petitioner Mindanao Medical Thus, the provisions of CA No. 141 apply to the Freedom Islands on matters
Center. Thus, Section 122 of the Act, which governs the registration of grants not covered by PD No. 1084. Section 60 of CA No. 141 prohibits, "except when

85
authorized by Congress," the sale of alienable lands of the public domain that reclamation projects for and on behalf of the National Government.
are transferred to government units or entities. Section 60 of CA No. 141 All reclamation projects shall be approved by the President upon
constitutes, under Section 44 of PD No. 1529, a "statutory lien affecting title" recommendation of the PEA, and shall be undertaken by the PEA or through a
of the registered land even if not annotated on the certificate of title. 104 proper contract executed by it with any person or entity; Provided, that,
Alienable lands of the public domain held by government entities under reclamation projects of any national government agency or entity authorized
Section 60 of CA No. 141 remain public lands because they cannot be under its charter shall be undertaken in consultation with the PEA upon
alienated or encumbered unless Congress passes a law authorizing their approval of the President.
disposition. Congress, however, cannot authorize the sale to private x x x ."
corporations of reclaimed alienable lands of the public domain because of the As the central implementing agency tasked to undertake reclamation projects
constitutional ban. Only individuals can benefit from such law. nationwide, with authority to sell reclaimed lands, PEA took the place of DENR
The grant of legislative authority to sell public lands in accordance with as the government agency charged with leasing or selling reclaimed lands of
Section 60 of CA No. 141 does not automatically convert alienable lands of the the public domain. The reclaimed lands being leased or sold by PEA are not
public domain into private or patrimonial lands. The alienable lands of the private lands, in the same manner that DENR, when it disposes of other
public domain must be transferred to qualified private parties, or to alienable lands, does not dispose of private lands but alienable lands of the
government entities not tasked to dispose of public lands, before these lands public domain. Only when qualified private parties acquire these lands will the
can become private or patrimonial lands. Otherwise, the constitutional ban will lands become private lands. In the hands of the government agency
become illusory if Congress can declare lands of the public domain as private tasked and authorized to dispose of alienable of disposable lands of
or patrimonial lands in the hands of a government agency tasked to dispose of the public domain, these lands are still public, not private lands.
public lands. This will allow private corporations to acquire directly from Furthermore, PEA's charter expressly states that PEA "shall hold lands of
government agencies limitless areas of lands which, prior to such law, are the public domain" as well as "any and all kinds of lands." PEA can hold both
concededly public lands. lands of the public domain and private lands. Thus, the mere fact that
Under EO No. 525, PEA became the central implementing agency of the alienable lands of the public domain like the Freedom Islands are transferred
National Government to reclaim foreshore and submerged areas of the public to PEA and issued land patents or certificates of title in PEA's name does not
domain. Thus, EO No. 525 declares that automatically make such lands private.
"EXECUTIVE ORDER NO. 525 To allow vast areas of reclaimed lands of the public domain to be transferred
Designating the Public Estates Authority as the Agency Primarily Responsible to PEA as private lands will sanction a gross violation of the constitutional ban
for all Reclamation Projects on private corporations from acquiring any kind of alienable land of the public
Whereas, there are several reclamation projects which are ongoing or being domain. PEA will simply turn around, as PEA has now done under the
proposed to be undertaken in various parts of the country which need to be Amended JVA, and transfer several hundreds of hectares of these reclaimed
evaluated for consistency with national programs; and still to be reclaimed lands to a single private corporation in only one
Whereas, there is a need to give further institutional support to the transaction. This scheme will effectively nullify the constitutional ban in
Government's declared policy to provide for a coordinated, economical and Section 3, Article XII of the 1987 Constitution which was intended to diffuse
efficient reclamation of lands; equitably the ownership of alienable lands of the public domain among
Whereas, Presidential Decree No. 3-A requires that all reclamation of areas Filipinos, now numbering over 80 million strong.
shall be limited to the National Government or any person authorized by it This scheme, if allowed, can even be applied to alienable agricultural lands of
under proper contract; the public domain since PEA can "acquire x x x any and all kinds of lands."
Whereas, a central authority is needed to act on behalf of the This will open the floodgates to corporations and even individuals acquiring
National Government which shall ensure a coordinated and hundreds of hectares of alienable lands of the public domain under the guise
integrated approach in the reclamation of lands; that in the hands of PEA these lands are private lands. This will result in
Whereas, Presidential Decree No. 1084 creates the Public Estates corporations amassing huge landholdings never before seen in this country -
Authority as a government corporation to undertake reclamation of creating the very evil that the constitutional ban was designed to prevent.
lands and ensure their maximum utilization in promoting public This will completely reverse the clear direction of constitutional development
welfare and interests; and in this country. The 1935 Constitution allowed private corporations to acquire
Whereas, Presidential Decree No. 1416 provides the President with continuing not more than 1,024 hectares of public lands. 105 The 1973 Constitution
authority to reorganize the national government including the transfer, prohibited private corporations from acquiring any kind of public land, and the
abolition, or merger of functions and offices. 1987 Constitution has unequivocally reiterated this prohibition.
NOW, THEREFORE, I, FERDINAND E. MARCOS, President of the Philippines, by The contention of PEA and AMARI that public lands, once registered under Act
virtue of the powers vested in me by the Constitution and pursuant to No. 496 or PD No. 1529, automatically become private lands is contrary to
Presidential Decree No. 1416, do hereby order and direct the following: existing laws. Several laws authorize lands of the public domain to be
Section 1. The Public Estates Authority (PEA) shall be primarily registered under the Torrens System or Act No. 496, now PD No. 1529, without
responsible for integrating, directing, and coordinating all losing their character as public lands. Section 122 of Act No. 496, and Section

86
103 of PD No. 1529, respectively, provide as follows: covering such expropriated lands. Section 85 of PD No. 1529 states
Act No. 496 "Sec. 85. Land taken by eminent domain. Whenever any registered land, or
"Sec. 122. Whenever public lands in the Philippine Islands belonging to the x x interest therein, is expropriated or taken by eminent domain, the National
x Government of the Philippine Islands are alienated, granted, or conveyed to Government, province, city or municipality, or any other agency or
persons or the public or private corporations, the same shall be brought instrumentality exercising such right shall file for registration in the proper
forthwith under the operation of this Act and shall become registered lands." Registry a certified copy of the judgment which shall state definitely by an
PD No. 1529 adequate description, the particular property or interest expropriated, the
"Sec. 103. Certificate of Title to Patents. Whenever public land is by the number of the certificate of title, and the nature of the public use. A
Government alienated, granted or conveyed to any person, the same shall memorandum of the right or interest taken shall be made on each certificate
be brought forthwith under the operation of this Decree." (Emphasis supplied) of title by the Register of Deeds, and where the fee simple is taken, a new
Based on its legislative history, the phrase "conveyed to any person" in certificate shall be issued in favor of the National Government,
Section 103 of PD No. 1529 includes conveyances of public lands to public province, city, municipality, or any other agency or instrumentality
corporations. exercising such right for the land so taken. The legal expenses incident to the
Alienable lands of the public domain "granted, donated, or transferred to a memorandum of registration or issuance of a new certificate of title shall be
province, municipality, or branch or subdivision of the Government," as for the account of the authority taking the land or interest therein." (Emphasis
provided in Section 60 of CA No. 141, may be registered under the Torrens supplied)
System pursuant to Section 103 of PD No. 1529. Such registration, however, is Consequently, lands registered under Act No. 496 or PD No. 1529 are not
expressly subject to the condition in Section 60 of CA No. 141 that the land exclusively private or patrimonial lands. Lands of the public domain may also
"shall not be alienated, encumbered or otherwise disposed of in a manner be registered pursuant to existing laws.
affecting its title, except when authorized by Congress." This provision AMARI makes a parting shot that the Amended JVA is not a sale to AMARI of
refers to government reclaimed, foreshore and marshy lands of the public the Freedom Islands or of the lands to be reclaimed from submerged areas of
domain that have been titled but still cannot be alienated or encumbered Manila Bay. In the words of AMARI, the Amended JVA "is not a sale but a joint
unless expressly authorized by Congress. The need for legislative authority venture with a stipulation for reimbursement of the original cost incurred by
prevents the registered land of the public domain from becoming private land PEA for the earlier reclamation and construction works performed by the CDCP
that can be disposed of to qualified private parties. under its 1973 contract with the Republic." Whether the Amended JVA is a sale
The Revised Administrative Code of 1987 also recognizes that lands of the or a joint venture, the fact remains that the Amended JVA requires PEA to
public domain may be registered under the Torrens System. Section 48, "cause the issuance and delivery of the certificates of title conveying AMARI's
Chapter 12, Book I of the Code states Land Share in the name of AMARI."107
"Sec. 48. Official Authorized to Convey Real Property. Whenever real property This stipulation still contravenes Section 3, Article XII of the 1987 Constitution
of the Government is authorized by law to be conveyed, the deed of which provides that private corporations "shall not hold such alienable lands
conveyance shall be executed in behalf of the government by the following: of the public domain except by lease." The transfer of title and ownership to
(1) x x x AMARI clearly means that AMARI will "hold" the reclaimed lands other than by
(2) For property belonging to the Republic of the Philippines, but lease. The transfer of title and ownership is a "disposition" of the reclaimed
titled in the name of any political subdivision or of any corporate lands, a transaction considered a sale or alienation under CA No. 141, 108 the
agency or instrumentality, by the executive head of the agency or Government Auditing Code,109 and Section 3, Article XII of the 1987
instrumentality." (Emphasis supplied) Constitution.
Thus, private property purchased by the National Government for expansion The Regalian doctrine is deeply implanted in our legal system. Foreshore and
of a public wharf may be titled in the name of a government corporation submerged areas form part of the public domain and are inalienable. Lands
regulating port operations in the country. Private property purchased by the reclaimed from foreshore and submerged areas also form part of the public
National Government for expansion of an airport may also be titled in the domain and are also inalienable, unless converted pursuant to law into
name of the government agency tasked to administer the airport. Private alienable or disposable lands of the public domain. Historically, lands
property donated to a municipality for use as a town plaza or public school reclaimed by the government are sui generis, not available for sale to
site may likewise be titled in the name of the municipality. 106 All these private parties unlike other alienable public lands. Reclaimed lands retain their
properties become properties of the public domain, and if already registered inherent potential as areas for public use or public service. Alienable lands of
under Act No. 496 or PD No. 1529, remain registered land. There is no the public domain, increasingly becoming scarce natural resources, are to be
requirement or provision in any existing law for the de-registration of land distributed equitably among our ever-growing population. To insure such
from the Torrens System. equitable distribution, the 1973 and 1987 Constitutions have barred private
Private lands taken by the Government for public use under its power of corporations from acquiring any kind of alienable land of the public domain.
eminent domain become unquestionably part of the public domain. Those who attempt to dispose of inalienable natural resources of the State, or
Nevertheless, Section 85 of PD No. 1529 authorizes the Register of Deeds to seek to circumvent the constitutional ban on alienation of lands of the public
issue in the name of the National Government new certificates of title domain to private corporations, do so at their own risk.

87
We can now summarize our conclusions as follows:
1. The 157.84 hectares of reclaimed lands comprising the Freedom Islands, Republic v. Heirs of Felipe Alejaga, Sr. 393 SCRA 361 (2002)
now covered by certificates of title in the name of PEA, are alienable lands THIRD DIVISION
of the public domain. PEA may lease these lands to private corporations but [G.R. No. 146030. December 3, 2002]
may not sell or transfer ownership of these lands to private corporations. PEA REPUBLIC OF THE PHILIPPINES, represented by the Department of
may only sell these lands to Philippine citizens, subject to the ownership Environment and Natural Resources, petitioner, vs. HEIRS OF FELIPE
limitations in the 1987 Constitution and existing laws. ALEJAGA SR., represented by ROQUETA ALEJAGA, FELIPE ALEJAGA JR.,
2. The 592.15 hectares of submerged areas of Manila Bay remain inalienable MARIA DULLA ALEJAGA, FELIPE ALEJAGA III, ROQUETA ALEJAGA,
natural resources of the public domain until classified as alienable or JENNIFER ALEJAGA, EVERETTE CAPUNDAN, AND LYNETTE ALEJAGA;
disposable lands open to disposition and declared no longer needed for public THE PHILIPPINE NATIONAL BANK and THE REGISTER OF DEEDS OF
service. The government can make such classification and declaration only ROXAS CITY, respondents.
after PEA has reclaimed these submerged areas. Only then can these lands DECISION
qualify as agricultural lands of the public domain, which are the only natural PANGANIBAN, J.:
resources the government can alienate. In their present state, the 592.15 We reiterate the familiar doctrine that a free patent obtained through fraud or
hectares of submerged areas are inalienable and outside the commerce misrepresentation is void. Furthermore, the one-year prescriptive period
of man. provided in the Public Land Act does not bar the State from asking for the
3. Since the Amended JVA seeks to transfer to AMARI, a private corporation, reversion of property acquired through such means.
ownership of 77.34 hectares110 of the Freedom Islands, such transfer is void for Statement of the Case
being contrary to Section 3, Article XII of the 1987 Constitution which prohibits Before us is a Petition for Review on Certiorari under Rule 45 of the Rules of
private corporations from acquiring any kind of alienable land of the public Court, assailing the November 15, 2000 Decision[if !supportFootnotes][1][endif] of the
domain. Court of Appeals (CA) in CA-GR CV No. 44568. The decretal portion of the
4. Since the Amended JVA also seeks to transfer to AMARI ownership of challenged Decision reads as follows:
290.156 hectares111 of still submerged areas of Manila Bay, such transfer is WHEREFORE, the appealed decision is hereby REVERSED, SET ASIDE and
void for being contrary to Section 2, Article XII of the 1987 Constitution which RECALLED.[if !supportFootnotes][2][endif]
prohibits the alienation of natural resources other than agricultural lands of The Facts
the public domain. PEA may reclaim these submerged areas. Thereafter, the The factual antecedents of the case are summarized by the CA thus:
government can classify the reclaimed lands as alienable or disposable, and On December 28, 1978, [Respondent] Felipe Alejaga, Sr. x x x filed with the
further declare them no longer needed for public service. Still, the transfer of District Land Office, Roxas City, Free Patent Application No. (VI-2) 8442
such reclaimed alienable lands of the public domain to AMARI will be void in covering a parcel of land identified as Lot 1, Mli-06-000020-D, with an area of .
view of Section 3, Article XII of the 1987 Constitution which prohibits private 3899 hectares, more or less located at Dumolog, Roxas City (Exh. A; Exh 9). It
corporations from acquiring any kind of alienable land of the public domain. appears that on December 27, 1978, when the application was executed
Clearly, the Amended JVA violates glaringly Sections 2 and 3, Article XII of the under oath, Efren L. Recio, Land Inspector, submitted a report of his
1987 Constitution. Under Article 1409 112 of the Civil Code, contracts whose investigation and verification of the land to the District Land Office, Bureau of
"object or purpose is contrary to law," or whose "object is outside the Lands, City of Roxas. On March 14, 1979, the District Land Officer of Roxas
commerce of men," are "inexistent and void from the beginning." The Court City approved the application and the issuance of [a] Free Patent to the
must perform its duty to defend and uphold the Constitution, and therefore applicant. On March 16, 1979, the patent was also ordered to be issued and
declares the Amended JVA null and void ab initio. the patent was forwarded to defendant Register of Deeds, City of Roxas, for
Seventh issue: whether the Court is the proper forum to raise the registration and issuance of the corresponding Certificate of Title. Thereafter,
issue of whether the Amended JVA is grossly disadvantageous to the Original Certificate of Title No. P-15 Free Patent No. (VI-2) 3358 was issued to
government. [respondent] by defendant Register of Deeds.
Considering that the Amended JVA is null and void ab initio, there is no On April 4, 1979, the heirs of Ignacio Arrobang, through counsel in a letter-
necessity to rule on this last issue. Besides, the Court is not a trier of facts, complaint requested the Director of Lands, Manila, for an investigation of the
and this last issue involves a determination of factual matters. District Land Officer, Roxas City, and the Regional Office, Region VI, Iloilo City,
WHEREFORE, the petition is GRANTED. The Public Estates Authority and for irregularities in the issuance of the title of a foreshore land in favor of
Amari Coastal Bay Development Corporation are PERMANENTLY ENJOINED [respondent]. Isagani Cartagena, Supervising Special Investigator, Legal
from implementing the Amended Joint Venture Agreement which is hereby Division, Land Management Bureau (formerly Bureau of Lands) submitted his
declared NULL and VOID ab initio. Report dated April 17, 1989. The Chief, Legal Division, Land Management
SO ORDERED. Bureau, Manila, recommended to the Director of Lands appropriate civil
Davide, Jr., C.J., Bellosillo, Puno, Vitug, Kapunan, Mendoza, Panganiban, proceeding for the cancellation of Free Patent Title No. (VI-2) 3358 and the
Quisumbing, Ynares-Santiago, Sandoval-Gutierrez, Austria-Martinez, and corresponding Original Certificate of Title No. P-15 in the name of
Corona, JJ., concur. [respondent].

88
In the meantime, [respondent] obtained a NACIDA loan under the Cottage Issues
Industry Guarantee and Loan Fund by the defendant Philippine National Bank Petitioner raises the following issues for this Courts consideration:
(hereinafter referred to as PNB) executed in Cebu City in the amount of I
P100,000.00 on August 18, 1981. The loan was secured by a real estate The Honorable Court of Appeals erred in not finding that the case is already
mortgage in favor of defendant PNB. The promissory note of appellant was final and executory as against respondent PNB.
annotated at the back of the title. II
On April 18, 1990, the government through the Solicitor General instituted an The Court of Appeals erred in not considering that petitioner has proven the
action for Annulment/Cancellation of Patent and Title and Reversion against allegations to the Complaint.
[respondent], the PNB of Roxas City and defendant Register of Deeds of Roxas III
City covering Free Patent Application (VI-2) 8442 of the parcel of land with an The Honorable Court of Appeals erred in declaring that the action for reversion
area of .3899 hectares more or less located at Dumolog, Roxas City. is unavailing.[if !supportFootnotes][9][endif]
On November 17, 1990, while the case is pending hearing, [respondent] died. Simply stated, the issues can be summed up into two: (1) the efficacy of the
He was substituted by his wife Roqueta Alejaga and his children, namely: grant of the free patent and (2) the indefeasibility of the Certificate of Title
Everette Alejaga, Lynnette Alejaga, Felipe Alejaga, Jr., Maria Dulla Alejaga. issued in consequence thereof.
Roqueta Alejaga, Jennifer Alejaga and Felipe Alejaga III. This Courts Ruling
xxxxxxxxx The Petition is meritorious.
After hearing, the [trial] court in its dispositive portion decreed as follows: First Issue:
WHEREFORE, judgment is rendered declaring that the approval of Free Patent Efficacy of the Grant
Application No. 3358 and issuance of Original Certificate of Title No. P-15 in Petitioner argues that it has proven fraud in the issuance of Respondent
the name of Felipe Alejaga is by means of fraud hence, null and void ab initio Alejagas free patent and Certificate of Title. [if !supportFootnotes][10][endif] It also avers
and the court orders: that Respondent PNB has failed to file a timely Notice of Appeal.
a) the cancellation of the approval of the application No. (VI-2) 8442 covering On the other hand, the Alejagas contend that they have acquired a vested
Lot No. 1, Mli-06-000020-D with an area of .3899 hectares, more or less, right over the parcel of land covered by OCT No. P-15 by virtue of their proven
located at Dumulog, Roxas City; open, actual, exclusive and undisputed possession of the land for more than
b) the cancellation of Original Certificate of Title No. P-15, Free Patent No. (VI- 30 years.[if !supportFootnotes][11][endif]
2) 3358 in the name of Felipe Alejaga; At the outset, we must immediately clarify that the records show receipt by
c) the land covered thereby as above described is reverted to the mass of the Respondent PNB of a copy of the Decision on October 27, not on October 3,
public domain; 1993 as alleged by petitioner. [if !supportFootnotes][12][endif] Further, the bank filed its
d) the defendants, Heirs of Felipe Alejaga Sr. or defendant, Philippine National Notice of Appeal on November 9, 1993, within the 15-day reglementary
Bank, Roxas City Branch, to surrender the owners duplicate copy of above period.
described Original Certificate of Title No. P-15 to the Register of Deeds (now In addition, we must point out that the essential issue raised in this Petition --
Registries of Land Titles and Deeds), Roxas City; the presence of fraud -- is factual. As a general rule, this Court does not review
e) the defendant, Register of Deeds, Roxas City, to cancel Original Certificate factual matters.[if !supportFootnotes][13][endif] However, the instant case falls under one
of Title No. P-15 and the owners duplicate copy of said title surrendered by of the exceptions, because the findings of the CA conflict with those of the RTC
above stated defendants; and with the evidence on record.[if !supportFootnotes][14][endif]
f) defendants, Philippine National Bank, cross-claim is dismissed. We begin our resolution of this issue with the well-settled rule that the party
Costs against the defendants Heirs of Felipe, Alejaga, Sr. [if !supportFootnotes][3][endif] alleging fraud or mistake in a transaction bears the burden of proof. [if !
Ruling of the Court of Appeals supportFootnotes][15][endif]
The circumstances evidencing fraud are as varied as the
In reversing the RTC, the CA ruled that petitioner failed to prove its allegation people who perpetrate it in each case. [if !supportFootnotes][16][endif] It may assume
that respondents had obtained the free patent and the Certificate of Title different shapes and forms; it may be committed in as many different ways. [if !
through fraud and misrepresentation. [if !supportFootnotes][4][endif] The appellate court supportFootnotes][17][endif]
Thus, the law requires that it be established by clear and
likewise held that, assuming there was misrepresentation or fraud as claimed convincing evidence.[if !supportFootnotes][18][endif]
by petitioner, the action for reversion should have been brought within one (1) In the case before us, we find that petitioner has adduced a preponderance of
year from the registration of the patent with the Registry of Deeds. [if ! evidence before the trial court, showing manifest fraud in procuring the
supportFootnotes][5][endif]
patent.[if !supportFootnotes][19][endif] This Court agrees with the RTC that in obtaining a
Further, the CA brushed aside as hearsay Isagani Cartagenas testimony that free patent over the lot under scrutiny, petitioner had resorted to
Land Inspector Efren L. Recio had not conducted an investigation on the free misrepresentation or fraud, signs of which were [if !supportFootnotes][20][endif] ignored by
patent application of Felipe Alejaga Sr.[if !supportFootnotes][6][endif] The CA added that the Court of Appeals.[if !supportFootnotes][21][endif]
petitioner had failed to support its claim that the lot covered by respondents First, the issuance of the free patent was not made in accordance with
free patent and title was foreshore land.[if !supportFootnotes][7][endif] the procedure laid down by Commonwealth Act No. 141, otherwise known as
Hence, this Petition.[if !supportFootnotes][8][endif] the Public Land Act.[if !supportFootnotes][22][endif] Under Section 91 thereof, an

89
investigation should be conducted for the purpose of ascertaining whether the On the other hand, the part referring to the statement made by Recio may be
material facts set out in the application are true.[if !supportFootnotes][23][endif] considered as independently relevant.[if !supportFootnotes][35][endif]
Further, after the filing of the application, the law requires sufficient The doctrine on independently relevant statements holds that
notice to the municipality and the barrio where the land is located, in order to conversations communicated to a witness by a third person may be admitted
give adverse claimants the opportunity to present their claims. [if !supportFootnotes][24] as proof that, regardless of their truth or falsity, they were actually made.
[endif]
Note that this notice and the verification and investigation of the parcel of Evidence as to the making of such statements is not secondary but primary,
land are to be conducted after an application for free patent has been filed for in itself it (a) constitutes a fact in issue [if !supportFootnotes][36][endif] or (b) is
with the Bureau of Lands. circumstantially relevant to the existence of such fact. [if !supportFootnotes][37][endif]
In this case, however, Felipe Alejaga Sr.s Application for Free Patent [if ! Since Cartagenas testimony was based on the report of the
supportFootnotes][25][endif]
was dated and filed on December 28, 1978. On the other investigation he had conducted, his testimony was not hearsay and was,
hand, the Investigation & Verification Report[if !supportFootnotes][26][endif] prepared by hence, properly admitted by the trial court.[if !supportFootnotes][38][endif]
Land Inspector Elfren L. Recio of the District Land Office of the Bureau of Based on the foregoing badges of fraud, we sustain petitioners
Lands of Roxas City was dated December 27, 1978. In that Report, he stated contention that the free patent granted to Felipe Alejaga Sr. is void. [if !
that he had conducted the necessary investigation and verification in the supportFootnotes][39][endif]
Such fraud is a ground for impugning the validity of the
presence of the applicant. Even if we accept this statement as gospel truth, Certificate of Title.[if !supportFootnotes][40][endif] The invalidity of the patent is sufficient
the violation of the rule cannot be condoned because, obviously, the required basis for nullifying the Certificate of Title issued in consequence thereof, since
notice to adverse claimants was not served. the latter is merely evidence of the former. [if !supportFootnotes][41][endif] Verily, we must
Evidently, the filing of the application and the verification and uphold petitioners claim that the issuance of the Alejagas patent and title was
investigation allegedly conducted by Recio were precipitate and beyond the tainted with fraud.[if !supportFootnotes][42][endif]
pale of the Public Land Act.[if !supportFootnotes][27][endif] As correctly pointed out by the Second Issue:
trial court, investigation and verification should have been done only after the Indefeasibility of Title
filing of the application. Hence, it would have been highly anomalous for Recio Petitioner contends that the State has an imprescriptible right to cause the
to conduct his own investigation and verification on December 27, 1998, a day reversion of a piece of property belonging to the public domain. [if !supportFootnotes][43]
before Felipe Alejaga Sr. filed the Application for Free Patent. [if !supportFootnotes][28] [endif]
On the other hand, the Alejagas claim that, pursuant to Section 32 of PD
[endif]
It must also be noted that while the Alejagas insist that an investigation 1529[if !supportFootnotes][44][endif] -- otherwise known as the Property Registration
was conducted, they do not dispute the fact that it preceded the filing of the Decree -- the one-year period for reversion has already lapsed. [if !supportFootnotes][45]
application.[if !supportFootnotes][29][endif] [endif]
Thus, the States Complaint for reversion should be dismissed.
Second, the claim of the Alejagas that an actual investigation was We agree with petitioner.
conducted is not sustained by the Verification & Investigation Report itself, True, once a patent is registered and the corresponding certificate of title
which bears no signature. [if !supportFootnotes][30][endif] Their reliance on the issued, the land covered by them ceases to be part of the public domain and
presumption of regularity in the performance of official duty [if !supportFootnotes][31][endif] becomes private property. Further, the Torrens Title issued pursuant to the
is thus misplaced. Since Recios signature does not appear on the December patent becomes indefeasible a year after the issuance of the latter. [if !
27, 1978 Report, there can be no presumption that an investigation and supportFootnotes][46][endif]
However, this indefeasibility of a title does not attach to
verification of the parcel of land was actually conducted. Strangely, titles secured by fraud and misrepresentation.[if !supportFootnotes][47][endif] Well-settled
respondents do not proffer any explanation why the Verification & is the doctrine that the registration of a patent under the Torrens System does
Investigation Report was not signed by Recio. Even more important and as will not by itself vest title; it merely confirms the registrants already existing one.
later on be explained, this alleged presumption of regularity -- assuming it Verily, registration under the Torrens System is not a mode of acquiring
ever existed -- is overcome by the evidence presented by petitioner. ownership.[if !supportFootnotes][48][endif]
Third, the report of Special Investigator Isagani P. Cartagena has not Therefore, under Section 101 of Commonwealth Act No. 141, [if !supportFootnotes][49]
been successfully rebutted. In that report, Recio supposedly admitted that he [endif]
the State -- even after the lapse of one year -- may still bring an action for
had not actually conducted an investigation and ocular inspection of the the reversion to the public domain of land that has been fraudulently granted
parcel of land. Cartagenas statement on Recios alleged admission may be to private individuals.[if !supportFootnotes][50][endif] Further, this indefeasibility cannot be
considered as independently relevant. A witness may testify as to the state of a bar to an investigation by the State as to how the title has been acquired, if
mind of another person -- the latters knowledge, belief, or good or bad faith -- the purpose of the investigation is to determine whether fraud has in fact
and the formers statements may then be regarded as independently relevant been committed in securing the title.[if !supportFootnotes][51][endif]
without violating the hearsay rule.[if !supportFootnotes][32][endif] In the case before us, the indefeasibility of a certificate of title cannot be
Thus, because Cartagena took the witness stand and opened himself to invoked by the Alejagas, whose forebear obtained the title by means of fraud.
cross-examination, the Investigation Report[if !supportFootnotes][33][endif] he had [if !supportFootnotes][52][endif]
Public policy demands that those who have done so should
submitted to the director of the Bureau of Lands constitutes part of his not be allowed to benefit from their misdeed. [if !supportFootnotes][53][endif] Thus,
testimony. Those portions of the report that consisted of his personal prescription and laches will not bar actions filed by the State to recover its
knowledge, perceptions and conclusions are not hearsay. [if !supportFootnotes][34][endif] own property acquired through fraud by private individuals. [if !supportFootnotes][54][endif]

90
This is settled law.[if !supportFootnotes][55][endif] disposable agricultural lots of the State to land-destitute citizens for their
Prohibition Against Alienation home and cultivation. Pursuant to such benevolent intention the State
or Encumbrance prohibits the sale or encumbrance of the homestead (Section 116) within five
Assuming arguendo that the Alejagas title was validly issued, there is another years after the grant of the patent.
basis for the cancellation of the grant and the reversion of the land to the Further, an encumbrance on a parcel of land acquired through free patent
public domain. Section 118 of Commonwealth Act No. 141 [if !supportFootnotes][56][endif] constitutes sufficient ground for the nullification of such grant, as provided
proscribes the encumbrance of a parcel of land acquired under a free patent under Commonwealth Act No. 141, which we quote:
or homestead within five years from its grant. [if !supportFootnotes][57][endif] The SEC. 124. Any acquisition, conveyance, alienation, transfer, or other contract
prohibition against any alienation or encumbrance of the land grant is a made or executed in violation of any of the provisions of sections one hundred
proviso attached to the approval of every application.[if !supportFootnotes][58][endif] and eighteen, one hundred and twenty, one hundred and twenty-one, one
Further, corporations are expressly forbidden by law to have any right or title hundred and twenty-two, and one hundred and twenty-three of this Act shall
to, or interest in, lands that are granted under free or homestead patents; or be unlawful and null and void from its execution and shall produce the effect
any improvements thereon. They are forbidden from enjoying such right, title of annulling and canceling the grant, title, patent, or permit originally issued,
or interest, if they have not secured the consent of the grantee and the recognized or confirmed, actually or presumptively, and cause the reversion
approval of the secretary of the Department of Agriculture and Natural of the property and its improvements to the State.
Resources; and if such lands are to be devoted to purposes other than Mortgage over a parcel of land acquired through a free patent grant nullifies
education, charity, or easement of way.[if !supportFootnotes][59][endif] the award and constitutes a cause for the reversion of the property to the
In the case at bar, Free Patent No. (VI-2) 3358 [if !supportFootnotes][60][endif] was state, as we held in Republic v. Court of Appeals:[if !supportFootnotes][68][endif]
approved and issued on March 14, 1979. Corresponding Original Certificate of The foregoing legal provisions clearly proscribe the encumbrance of a parcel
Title No. P-15[if !supportFootnotes][61][endif] was issued on the same date. On August 18, of land acquired under a free patent or homestead within five years from the
1981, or two (2) years after the grant of the free patent, Felipe Alejaga Sr. grant of such patent. Furthermore, such encumbrance results in the
obtained from Respondent PNB a loan [if !supportFootnotes][62][endif] in the amount of cancellation of the grant and the reversion of the land to the public domain. [if !
P100,000. Despite the statement on the title certificate itself that the land supportFootnotes][69][endif]

granted under the free patent shall be inalienable for five (5) years from the To comply with the condition for the grant of the free patent, within five years
grant, a real estate mortgage was nonetheless constituted on the parcel of from its issuance, Felipe Alejaga Sr. should not have encumbered the parcel
land covered by OCT No. P-15.[if !supportFootnotes][63][endif] In his testimony, Gabriel D. land granted to him. The mortgage he made over the land violated that
Aranas Jr., then Cashier III of respondent bank, even admitted that the PNB condition.[if !supportFootnotes][70][endif] Hence, the property must necessarily revert to
was aware of such restriction. the public domain, pursuant to Section 124 of the Public Land Act.
COURT You testified Mr. Aranas that you inspected the title also when you WHEREFORE, the Petition is GRANTED and the assailed Decision SET
credit investigated the loan applicant Felipe Alejaga and you have personally ASIDE. The Decision of the RTC of Roxas City (Branch 15) dated October 27,
examined this? 1993 is REINSTATED. No costs.
A Yes, your Honor. SO ORDERED.
COURT Do you conclude that this Original Certificate of Title is a [free] patent? Sandoval-Gutierrez, Corona, and Carpio-Morales, JJ., concur.
A Yes, your Honor. Puno, (Chairman), J., abroad on official business.
COURT And this [free] patent was granted on March 19, 1979.
A Yes, your honor. Republic v. Southside Homeowners Association, Inc., 502 SCRA 587
COURT And as such [free] patent it cannot be alienated except [to] the (2006)
government or within five years from its issuance? Republic of the Philippines
A Yes, your honor. SUPREME COURT
COURT Why did you recommend the loan? Manila
A Because it is just a mortgage.[if !supportFootnotes][64][endif] SECOND DIVISION
Thus, the mortgage executed by Respondent Felipe Alejaga Sr. falls squarely G.R. No. 156951 September 22, 2006
within the term encumbrance proscribed by Section 118 of the Public Land REPUBLIC OF THE PHILIPPINES, petitioner,
Act.[if !supportFootnotes][65][endif] A mortgage constitutes a legal limitation on the estate, vs.
and the foreclosure of the mortgage would necessarily result in the auction of SOUTHSIDE HOMEOWNERS ASSOCIATION, INC. and the REGISTER OF
the property.[if !supportFootnotes][66][endif] DEEDS OF PASIG, RIZAL, respondents.
As early as Pascua v. Talens,[if !supportFootnotes][67][endif] we have explained the x-------------------------------------------x
rationale for the prohibition against the encumbrance of a homestead -- its BASES CONVERSION DEVELOPMENT AUTHORITY, intervenor
lease and mortgage included -- an encumbrance which, by analogy, applies to x-------------------------------------------x
a free patent. We ruled as follows: DEPARTMENT OF NATIONAL DEFENSE, represented by HON.
It is well-known that the homestead laws were designed to distribute SECRETARY ANGELO T. REYES, and the ARMED FORCES OF THE
91
PHILIPPINES, represented by CHIEF OF STAFF, AFP, GENERAL square meters are described in the proclamation as situated inside Fort
NARCISO L. ABAYA, intervenors McKinley, Rizal. Specifically mentioned as excluded from Parcel No. 4 albeit
x-------------------------------------------x within its boundaries are the American Battle Monument Cemetery (606,985
G.R. No. 173408 September 22, 2006 sq. m.), the Traffic Circle (7,093 sq. m.) and the Diplomatic and Consular area
RENE A.V. SAGUISAG, MGEN. MARCIANO ILAGAN (Ret.), MGEN. (100,000 sq.m.).
PONCIANO MILLENA (Ret.), BGEN. JUANITO MALTO (Ret.), BGEN. Several presidential proclamations would later issue excluding certain defined
RAYMUNDO JARQUE (Ret.) and COL. DOMINADOR P. AMADOR (Ret.), areas from the operation of Proclamation No. 423 and declaring them open for
petitioners, disposition. These are Proclamation No. 4616 and Proclamation No. 462,7 both
vs. series of 1965, excluding portions of the reservation and declaring them the
L/T. GEN. HERMOGENES C. ESPERON, JR., respondent. AFP Officers Village and the AFP EMs Village, respectively, to be disposed of
x-------------------------------------------x under Republic Act (R.A.) 2748 and R.A. 7309 in relation to the Public Land Act
DECISION (C.A. 141, as amended). Excluded, too, under Proclamation No. 172 dated
GARCIA, J.: October 16, 1987 and to be disposed pursuant to the same laws
Before the Court are these two petitions having, as common denominator, the aforementioned, save those used or earmarked for public/quasi-public
issue of ownership of a large tract of land. purposes, are portions of the reservation known as Lower and Upper Bicutan,
In the first, a petition for review under Rule 45 of the Rules of Court and Western Bicutan and the Signal Village, all in Taguig, Metro Manila.
docketed as G.R. No. 156951, the petitioner Republic of the Philippines In 1992, Congress enacted the Bases Conversion and Development Act (R.A.
seeks to nullify and set aside the Decision 1 dated January 28, 2003 of the 7227, as amended), investing the BCDA the power to own, hold and
Court of Appeals (CA) in CA-G.R. CV No. 59454, affirming the dismissal by the administer portions of Metro Manila military camps that may be transferred to
Regional Trial Court (RTC) of Pasig City, Branch 71, of the Republics complaint it by the President10 and to dispose, after the lapse of a number of months,
for declaration of nullity and cancellation of a land title against the herein portions of Fort Bonifacio.11
private respondent, the Southside Homeowners Association, Inc. (SHAI). At the core of the instant proceedings for declaration of nullity of title are
In the second, docketed as G.R. No. 173408, petitioners Rene Saguisag and parcels of land with a total area of 39.99 hectares, more or less, known as or
five (5) retired military officers pray that Lt. Gen. Hermogenes C. Esperon, Jr., are situated in what is referred to as the JUSMAG housing area in Fort
the present Chief of Staff of the Armed Forces of the Philippines (AFP), be Bonifacio. As may be gathered from the pleadings, military officers, both in
asked to show cause why he should not be cited for contempt for having the active and retired services, and their respective families, have been
announced time and again that the military officers and their families in the occupying housing units and facilities originally constructed by the AFP on the
contempt action would be ousted and evicted from the property subject of the JUSMAG area.
main petition even before the issue of ownership thereof is finally resolved by Private respondent SHAI is a non-stock corporation organized mostly by wives
the Court. of AFP military officers. Records show that SHAI was able to secure from the
After the private respondent SHAI had filed its Comment2 to the petition in Registry of Deeds of the Province of Rizal a title Transfer Certificate of Title
G.R. No. 156951, the Bases Conversion Development Authority (BCDA), (TCT) No. 1508412 - in its name to the bulk of, if not the entire, JUSMAG area.
followed by the Department of National Defense (DND) and the AFP, joined TCT No. 15084 particularly describes the property covered thereby as follows:
causes with the petitioner Republic and thus sought leave to intervene. The A parcel of land (Lot 3-Y-1, Psd-76057, being a portion of Parcel 3 of plan Psu-
Court, per its Resolutions dated September 3, 2003, 3 and September 29, 2031) situated in Jusmang (sic) Area, Fort Bonifacio, Province of Rizal.
2003,4 respectively, allowed the intervention and admitted the corresponding containing an area of (398,602) SQUARE METERS. xxx.
petitions-for-intervention. A parcel of land (Lot 3-Y-2, Psd-76057 as shown on subdivision Plan Psd 76057,
Per Resolution of the Court dated August 09, 2006, both petitions were being a portion of parcel 3 of plan Psu-2031, LRC Rec. No.) situated in Jusmang
ordered consolidated. (sic) Area, Fort Bonifacio, Province of Rizal. containing an area of (1,320)
The Republics recourse in G.R. No. 156951 is cast against the following SQUARE METERS xxx.. (Underscoring added.)
backdrop: The Rizal Registry issued TCT No. 15084 on October 30, 1991 on the basis of a
On July 12, 1957, then President Carlos P. Garcia issued Proclamation No. 423 5 notarized Deed of Sale13 purportedly executed on the same date by then
establishing a military reservation known as Fort William McKinley later Director Abelardo G. Palad, Jr. (Palad, for brevity) of the Lands Management
renamed Fort Andres Bonifacio Military Reservation (FBMR). The Bureau (LMB) in favor of SHAI. The total purchase price as written in the
proclamation "withdr[ew] from sale or settlement and reserve[d] for military conveying deed was P11,997,660.00 or P30.00 per square meter.
purposes, under the administration of the Chief of Staff of the [AFP] the It appears that in the process of the investigation conducted by the
[certain] parcels of the public domain [indicated in plan Psu-2031]" situated in Department of Justice on reported land scams at the FBMR, a copy of the
the several towns and a city of what was once the Province of Rizal. On its aforesaid October 30, 1991 deed of sale surfaced and eventually referred to
face, the proclamation covers three (3) large parcels of land, to wit: Parcel No. the National Bureau of Investigation (NBI) for examination. The results of the
2 (portion), Parcel No. 3 (or 3-A) and Parcel No. 4 (or 4-A). Parcel No. 3 with an examination undertaken by NBI Document Examiner Eliodoro Constantino are
area of 15,912,684 square meters and Parcel No. 4 with an area of 7,660,128 embodied in his Questioned Documents Report (QDR) No. 815-1093. 14 Its

92
highlights: estopped to deny.18 SHAI also alleged paying in full the purchase price
QUESTIONED SPECIMENS: indicated in the deed as evidenced by Official Receipt No. 6030203-C dated
1. Original copy of the Deed of Sale issued in favor of the Navy Officers October 29, 1991.
Village Association (NOVA) containing the signature of "ABELARDO G. On October 19, 1994, the case was heard on pre-trial in the course of which
PALAD, JR." designated as "Q-961" . the Republic, as plaintiff therein, marked (and later offered in evidence) the
2. Original copy of the Deed of Sale issued in favor of SHAI containing the Deed of Sale dated October 30, 1991 as its Exhibit "A," and TCT No. 15084
signature of "ABELARDO G. PALAD, JR." ... designated as "Q-962. as Exhibit "B." Respondent, then defendant SHAI adopted Exhibits "A" and
xxx xxx xxx "B" as its Exhibits "1" and "2," respectively. As the pre-trial order was
PURPOSE OF EXAMINATION: written, it would appear that the parties agreed to limit the issue to the due
To determine whether or not the questioned and sample/specimen signatures execution and genuineness of Exhs. "A" and "B."19
"ABELARDO G. PALAD, JR." were written by one and the same person. During the trial, the Republic presented as expert witness NBI Document
FINDINGS: Examiner Eliodoro Constantino who testified on NBI QDR No. 815-1093 and
Scientific comparative examination and analysis of the specimens, submitted, asserted that the signature of Palad in Exhibit "A" is a forgery. For his part,
under stereoscopic microscope and magnifying lens, with the aid of Palad dismissed as forged his signature appearing in the same document and
photographic enlargement reveals that there exist fundamental, significant denied ever signing the same, let alone in front of a notary public holding
differences in writing characteristics between the questioned and the office outside of the LMB premises. Pressing the point, Palad stated that he
standard/sample signatures "ABELARDO G. PALAD, JR." such as in: could not have had signed the conveying deed involving as it did a reservation
- The questioned signatures show slow, drawn, painstaking laborious manner area which, apart from its being outside of the LMBs jurisdiction, is inalienable
in execution of strokes; that of the standard/sample signatures show free, in the first place. The testimony of other witnesses revolved around the
rapid coordinated and spontaneous strokes in the manner of execution of absence of bureau records respecting SHAIs application to acquire, payment
letters/elements. of the purchase price and Psd-76057, the plan described in TCT No. 15084. 20
xxx xxx xxx For its part, then defendant SHAI presented an opposing expert witness in the
Furthermore, the questioned signature "ABELARDO G. PALAD, JR." marked "Q- person of Police Inspector Redencion Caimbon who brought with him PNP QDR
961" is a product of TRACING PROCESS by CARBON-OUTLINE METHOD. No. 001-96 and testified that Palads signature in Exhibit "A" (same as Exh.
CONCLUSION: "1") is genuine. Mrs. Virginia Santos, then SHAI president, likewise testified,
Based on the above FINDINGS, the questioned and the standard/sample saying that applications to purchase were signed and then filed with the LMB
signatures "ABELARDO G. PALAD, JR." were not written by one and the same by one Engr. Eugenia Balis,21 followed by the payment in full of the contract
person. price. Atty. Vicente Garcia, the then Register of Deeds of Rizal, also testified
The questioned signature "ABELARDO G. PALAD, JR." marked "Q-961" is a about his having endorsed to Palad a letter-inquiry he received from SHAI
TRACED FORGERY by carbon process. respecting the authenticity of TCT No. 15084. Palads response-letter dated
REMARKS: January 23, 1992 (Exh. "10"), according to Atty. Garcia, is to the effect that
The other questioned Deeds of Sale containing the signatures of "ABELARDO TCT No. 15084 must be genuine as it emanated from the Registrys office on
G. PALAD, JR." are still in the process of examination. 15 the basis of the October 30, 1991 Deed of Sale. 22
On October 16, 1993, then President Fidel V. Ramos issued Memorandum On rebuttal, Palad would deny authorship of Exhibit "10" and an LMB official
Order No. 17316 directing the Office of the Solicitor General (OSG) to institute would disclaim transmitting the same to Atty. Garcia.
action towards the cancellation of TCT No. 15084 and the title acquired by the Eventually, in a decision23 dated October 7, 1997, the trial court rendered
Navy Officers Village Association (NOVA) over a bigger parcel within the judgment dismissing the Republics complaint, to wit:
reservation. A month later, the OSG, in behalf of the petitioner Republic, filed WHEREFORE, in view of the foregoing, the Complaint dated November 15,
with the RTC of Pasig City the corresponding nullification and cancellation of 1991 is hereby DISMISSED without pronouncement as to costs.
title suit against the private respondent SHAI. In its complaint, docketed as The counterclaims are also DISMISSED.
Civil Case No. 63883 and eventually raffled to Branch 71 of the court, the SO ORDERED.
Republic alleged that fraud attended SHAIs procurement of TCT No. 15084. In In not so many words, the trial court considered the parcels covered by the
paragraph No. 5 of the complaint, the Republic alleged that TCT No. 15084 is deed in question as no longer part of the FBMR.
void owing, inter alia, to the following circumstances: a) the conveying deed is Therefrom, the Republic went on appeal to the CA whereat its appellate
spurious as the purported signature thereon of Palad is a forgery; b) there are recourse was docketed as CA-G.R. CV No. 59454.
no records with the LMB of (i) the application to purchase and (ii) the alleged In the herein assailed Decision24 dated January 28, 2003, the appellate court
payment of the purchase price; and c) the property in question is inalienable, affirmed in toto that of the trial court.
being part of a military reservation established under Proclamation No. 423. 17 Hence, this petition of the Republic on the threshold abstract submission that
In its ANSWER with counterclaim, respondent SHAI denied the material the CA "completely ignored, overlooked and/or grossly misappreciated facts
allegations of the complaint and countered that the impugned title as well as of substance which, if duly considered, will materially affect the outcome of
the October 30, 1991 Deed of Sale are valid documents which the Republic is this case."

93
In its COMMENT To Petition, private respondent SHAI parlays the "what-can- government lands that has been assigned to other government agencies that
be-raised" line. It urges the dismissal of the petition on the ground that the is no longer within my jurisdiction. Meaning to say I have no more say on that
issues raised therein, particularly those bearing on the authenticity of Exhibit because the proclamation to the effect was reserving this for particular
"A"/"1," are mainly questions of fact, adding that the matter of the purpose under the DND .32 (Words in bracket added.)
inalienability of the area purportedly sold is outside the issue agreed upon At any rate, Palads testimony drew nary an objection from private respondent
during the pre-trial stage. SHAI. It even cross-examined said witness. 33 The rule obtains that the
The desired dismissal cannot be granted on the bases of the reasons proffered introduction of evidence bearing on an issue not otherwise included in the pre-
above. trial order amounts to implied consent conferring jurisdiction on the court to
While the Court, in a petition for review of CA decisions under Rule 45 of the try such issue.34
Rules of Court, usually limits its inquiry only to questions of law, this rule is far Digressing from the procedural aspects of this case, we now consider the
from absolute. Reyes v. Court of Appeals,25 citing Floro v. Llenado,26 for one, clashing assertions regarding the JUSMAG area. Was it, during the period
suggests as much. In Floro, we wrote: material, alienable or inalienable, as the case may be, and, therefore, can or
xxx There are, however, exceptional circumstances that would compel the cannot be subject of a lawful private conveyance?
Court to review the finding of facts of the [CA], summarized in and Petitioner Republic, as do the intervenors, asserts the inalienable character of
subsequent cases as follows: 1) when the inference made is manifestly the JUSMAG area, the same having not effectively been separated from the
mistaken, absurd or impossible; 2) when there is grave abuse of discretion; 3) military reservation and declared as alienable and disposable.
when the finding is grounded entirely on speculations, surmises or The Republics and the intervenors parallel assertions are correct.
conjectures; 4) when the judgment of the [CA] are based on misapprehension The President, upon the recommendation of the Secretary of Environment and
of facts; 5) when the findings of facts are conflicting; 6) ; 7) ; 8) ; 9) Natural Resources, may designate by proclamation any tract or tracts of land
when the [CA] manifestly overlooked certain relevant facts not disputed by of the public domain as reservations for the use of the Republic or any of its
the parties and which if properly considered would justify a different branches, or for quasi-public uses or purposes. 35 Such tract or tracts of land
conclusion; and 10) when the findings of facts are premised on the absence thus reserved shall be non-alienable and shall not be subject to sale or other
of evidence and are contradicted by the evidence on record. (Words in disposition until again declared alienable.36 Consistent with the foregoing
bracket, added.) postulates, jurisprudence teaches that a military reservation, like the FBMR, or
To the mind of the Court, the instant case is within the purview of at least a part thereof is not open to private appropriation or disposition and,
three of the exceptions listed above, foremost of which is item #9. therefore, not registrable,37 unless it is in the meantime reclassified and
Private respondent SHAIs stance about the petitioner Republic being barred declared as disposable and alienable public land. 38 And until a given parcel of
from raising the issue of inalienability since it failed to plead or assert the land is released from its classification as part of the military reservation zone
same at the pre-trial proceedings is, to a degree, correct. For the general rule, and reclassified by law or by presidential proclamation as disposable and
as articulated in Permanent Concrete Products, Inc. v. Teodoro, 27 is that the alienable, its status as part of a military reservation remains, 39 even if
determination of issues at a pre-trial conference bars the consideration of incidentally it is devoted for a purpose other than as a military camp or for
others on appeal. It should be pointed out, however, that the rationale for defense. So it must be here.
such preliminary, albeit mandatory, conference is to isolate as far as possible There can be no quibbling that the JUSMAG area subject of the questioned
the trial out of the realm of surprises and back-handed maneuverings. And October 30, 1991 sale formed part of the FBMR as originally established under
lest it be overlooked, the adverted rule on the procedure to be observed in Proclamation No. 423. And while private respondent SHAI would categorically
pre-trials is, as Bergano v. Court of Appeals28 teaches, citing Gicano v. say that the petitioner Republic had not presented evidence that "subject land
Gegato,29 subject to exceptions. And without meaning to diminish the is within military reservation,"40 and even dared to state that the JUSMAG area
importance of the same rule, the Court is possessed with inherent power to is the private property of the government and therefore removed from the
suspend its own rules or to except a particular case from its operations concept of public domain per se,41 its own evidence themselves belie its
whenever the demands of justice so require.30 posture. We start with its Exhibit "2" (petitioners Exh. "B"), a copy of TCT No.
Given the foregoing considerations, the rule to be generally observed in pre- 15084, which described the area covered thereby measuring 399,922 square
trial conferences hardly poses an insurmountable obstacle to tackling the meters as a "portion of Parcel 3 of plan Psu-2031 situated in Jusmang (sic)
question of inalienability which, under the premises, is an issue more legal area Fort Bonifacio." Complementing its Exhibit "2" is its Exhibit "1" - the deed
than factual. As it were, the element of surprise is not really present here. For of sale - which technically described the property purportedly being conveyed
the issue of inalienability, which is central to the Republics cause of action, to private respondent SHAI as follows:
was raised in its basic complaint, passed upon by the CA and, before it, by the A PARCEL OF LAND (Lot 3-Y-1, Psd-76067, being a portion of Parcel 3 of plan
trial court31 and of which at least one witness (Palad) was examined as follows: Psu-2031) situated in Jusmag (sic) area, Fort Bonifacio, Province of Rizal. Xxx
Q: Mr. Witness you stated that the parcel of land in question at the time of the (Emphasis added)
land alleged sale was part of the [FBMR]. Now as part of the [FBRM] do As the Court distinctly notes, the disputed property, as described in private
you know whether the said parcel of land can be the subject of disposition? respondents Exhibits "1" and "2," formed part of that wide expanse under
A: If it is part of the reservation it cannot be sold and it is already part of those Proclamation No. 423 which lists, as earlier stated, three (3) parcels of land of

94
the public domain as falling within its coverage. These include, inter alia, the that what SHAIs TCT No. 15084 covers is part of the military reservation. In
entire 15,912,684-square meter area constituting Parcel No. 3 of Plan Psu the process, private respondent SHAI is deemed to admit the reality of such
2031 located inside the now renamed Fort Mckinley which, to a redundant averment.
point, was declared a military reservation. To be sure, the petitioner Republic, as plaintiff below, had more than
The Court has, on the issue of inalienability, taken stock of the Compilation sufficiently established its claim on the inalienability of the parcels of land
Map of Approved Surveys Plan inside Parcels 1, 2, 3 and 4, of plan Psu 2031 42 covered by TCT No. 15084. In fine, it had discharged the burden of proof on
prepared in September 1995 and certified by the Department of Environment the issue of inalienability. Be that as it may, the burden of evidence to
and Natural Resources (DENR). It indicates in colored ink the outlines of disprove inalienability or, to be precise, that said parcels of land had, for
Parcels 2, 3 and 4 covered by Proclamation No. 423. As there also shown, the settlement purposes, effectively been withdrawn from the reservation or
399,992-square meter area embraced by SHAIs TCT No. 15084, excluded from the coverage of Proclamation No. 423, devolves upon the
defined in the legend by red-colored stripes, is within the violet-colored private respondent. This is as it should be for the cogency of SHAIs claim
borders of Parcel No. 3 and Parcel No. 4 of Proclamation No. 423. respecting the validity of both the underlying deed of sale (Exh. "A"/"1") and
Indubitably, the area covered by SHAIs TCT No. 15084 was and is still part of its TCT No. 15084 (Exh. "B"/"2") rests on the postulate that what it purportedly
the FBMR, more particularly within the 15,912,684- square meter Parcel No. 3 bought from the LMB had ceased to be part of the reserved lands of the public
of the reservation. The petitioner Republic, joined by the intervenors BCDA, domain. Elsewise put, SHAI must prove that the JUSMAG area had been
DND and AFP in this appellate proceedings, has maintained all along this withdrawn from the reservation and declared open for disposition, failing
thesis. Towards discharging its burden of proving that the disputed property is which it has no enforceable right over the area as against the State.
part of the reservation, the petitioner Republic need only to demonstrate that Private respondent SHAI has definitely not met its burden by reason of lack of
all of the 15,912,684 square meters of Parcel No. 3 of Plan Psu 2031 have evidence. To be sure, it has not, because it cannot even if it wanted to,
been reserved for military purposes. The evidence, however, of the fact of pointed to any presidential act specifically withdrawing the disputed parcels
reservation is the law or, to be more precise, Proclamation No. 423 itself, the from the coverage of Proclamation No. 423. Worse still, its own Exhibit "5,"47
contents and issuance of which courts can and should take judicial notice of a letter dated March 19, 1991 of then PA Commanding General, M/Gen
under Section 1, Rule 129 of the Rules of Court. 43 Lisandro Abadia, to one Mrs. Gabon, then President of the SHAI, cannot but be
The Republic has, since the filing of its underlying complaint, invoked viewed as a partys judicial admission that the disputed land has yet to be
Proclamation No. 423. In the process, it has invariably invited attention to the excluded from the military reservation. The Abadia letter, with its feature dis-
proclamations specific area coverage to prove the nullity of TCT No. 15084, serving to private respondent SHAI, reads in part as follows:
inasmuch as the title embraced a reserved area considered inalienable, and Dear Mrs. Gabon:
hence, beyond the commerce of man. In this regard, the appellate court This is in connection with your move to make a petition to President Aquino
seemed to have glossed over, if not entirely turned a blind eye on, certain regarding the possible exclusion of Southside Housing Area from the military
admissions made by the private respondent, the most basic being those made reservation and for its eventual allotment to the military officers presently
in its answer to the Republics allegations in paragraph 5 (e) and (g) of its residing thereat. Allow me to state that I interpose no objection . I find it
complaint. To the Republics allegations that the property covered by TCT No. helpful to our officers to be provided a portion of the Fort Bonifacio military
15084 was and remains part the FBMR, SHAIs answer thereto reads: reservation . (Underscoring added.)
2. It specifically denies the allegations in paragraphs 5 of the Owing to the foregoing considerations, the Court is hard put to understand
complaint, the truth of the matter being that in the Deed of Sale , the how the CA could still have found for SHAI.. The appellate court, apparently
Director of Lands Certificate (sic) that he is "authorized under the law to sell" swayed by what SHAI said in its Brief for the Appellees48 that:
the subject property and that the "lots were duly awarded by the [LBM] to the Appellant [petitioner Republic] is probably unaware that , then President
vendee.44 ( Emphasis and word in bracket added.) Diosdado Macapagal issued Proclamation 461 when he excluded from the
In net effect, private respondent SHAI admitted what the petitioner Republic operation of Proclamation No. 423 an area of 2,455,810 square meters
alleged in par. 5 (e) and (g) of the complaint, the formers denial to such more or less. Likewise on October 16, 1987, then President Corazon Aquino
allegations on the inalienable nature of the property covered by TCT No. issued Proclamation No. 172 excluding five (5) parcels of land from the
15084 being in the nature of a general denial. Under the rules on pleadings, a operation of Proclamation No. 423 also located at Fort Bonifacio containing an
specific, not a general, denial is required; a denial is not specific because it is area of 4,436, 478 . So if we deduct the 6,892,288 [2,455,810 + 4,436,478
so qualified or termed "specific" by the pleader. 45 The defendant must specify = 6,892,288] square meters covered by Proclamation Nos. 461 and 172 of the
each material factual allegation the truth of which he absolutely denies and, areas reserved for military purposes of 7,053,143 square meters, what is only
whenever practicable, shall set forth the substance of the matters upon which left is 160,857 square meters or more or less 16 hectares . 49
he will rely to support his denial. 46 Else, the denial will be regarded as general justified its holding on the alienability of the disputed land with the following
and will, therefore, be regarded as an admission of a given material fact/s disquisition:
stated in the complaint. The foregoing admission aside, appellants [now petitioners] reliance on
What private respondent SHAI did under the premises was to enter what, Proclamation No. 493 [should be 423] in insisting that the land in litigation is
under the Rules, is tantamount to a general denial of the Republics averments inalienable because it is part of the [FBMR] is too general to merit serous

95
consideration. While it is true that, under the said July 12, 1957 Proclamation, of what appears to be Palads signature thereon.
then President Carlos P. Garcia reserved the area now known as Fort Bonifacio With the view we take of the case, the interplay of compelling circumstances
for military purposes, appellee [now respondent] correctly calls our attention and inferences deducible therefrom, would, as a package, cast doubt on the
to the fact, among other matters, that numerous exceptions thereto had authenticity of such deed, if not support a conclusion that the deed is
already been declared through the years. The excluded areas under spurious. Consider:
Proclamation No. 461, dated September 29, 1965 and Proclamation No. 172, 1. Palad categorically declared that his said signature on the deed is a forgery.
dated October 16, 1987 alone already total 6,892,338 square meters. (Figures The Court perceives no reason why he should lie, albeit respondent states,
in bracket added.) without elaboration, that Palads declaration is aimed at avoiding "criminal
The CAs justifying line does not commend itself for concurrence. prosecution".54 The NBI signature expert corroborated Palads allegation on
For one, it utilizes SHAIs misleading assertion as a springboard to justify forgery.55 Respondent SHAIs expert witness from the PNP, however, disputes
speculative inferences. Per our count, Proclamation 423 reserved for military the NBIs findings. In net effect, both experts from the NBI and the PNP cancel
purposes roughly a total area of 25,875,000 square meters, not 7,053,143. each other out.
On the other hand, Proclamation Nos. 461 and 172 excluded a combined area 2. Palad signed the supposed deed of sale in Manila, possibly at the LMB
of 6,892,338 square meters. Now then, the jump from an acknowledgment of office at Plaza Cervantes, Binondo. Even if he acted in an official capacity,
the disputed parcels of land having been reserved for military purposes to a Palad nonetheless proceeded on the same day to Pasig City to appear before
rationalization that they must have been excluded from the reservation the notarizing officer. The deed was then brought to the Rizal Registry and
because 6,892,338 square meters had already been withdrawn from there stamped "Received" by the entry clerk. That same afternoon, or at 3:14
Proclamation 423 is simply speculative. Needless to stress, factual p.m. of October 30, 1991 to be precise, TCT No. 15084 was issued. In other
speculations do not make for proof. words, the whole conveyance and registration process was done in less than a
Corollary to the first reason is the fact that private respondent SHAI - and day. The very unusual dispatch is quite surprising. Stranger still is why a
quite understandably, the appellate court - had not pointed to any bureau head, while in the exercise of his functions as the bureaus authorized
proclamation, or legislative act for that matter, segregating the property contracting officer, has to repair to another city just to have a deed notarized.
covered by TCT No. 15084 from the reservation and classifying the same as 3. There is absolutely no record of the requisite public land application to
alienable and disposable lands of the public domain. To reiterate what we purchase required under Section 89 of the Public Land Act. 56 There is also no
earlier said, lands of the public domain classified as a military reservation record of the deed of sale and of documents usually accompanying an
remains as such until, by presidential fiat or congressional act, the same is application to purchase, inclusive of the investigation report and the property
released from such classification and declared open to disposition. 50 The valuation. The Certification under the seal of the LMB bearing date November
October 30, 1991 Deed of Sale purportedly executed by Palad, assuming for 24, 1994 and issued/signed by Alberto Recalde, OIC, Records Management
the nonce its authenticity, could not plausibly be the requisite classifying Division of the LMB pursuant to a subpoena issued by the trial court 57 attest to
medium converting the JUSMAG area into a disposable parcel. And private this fact of absence of records. Atty. Alice B. Dayrit, then Chief, Land Utilization
respondent SHAIs unyielding stance that would have the Republic in estoppel and Disposition Division, LMB, testified having personally looked at the bureau
to question the transfer to it by the LMB Director of the JUSMAG area is record book, but found no entry pertaining to SHAI.58
unavailing. It should have realized that the Republic is not usually estopped by 4. In its Answer as defendant a quo, respondent SHAI states that the "deed of
the mistake or error on the part of its officials or agents. 51 sale specifically meritorious Official Receipt No. 6030203C dated 29 October
Since the parcels of land in question allegedly sold to the private respondent 1991, (sic) as evidence of full payment of the agreed purchase price.." An
are, or at least at the time of the supposed transaction were, still part of the official receipt (O.R.) is doubtless the best evidence to prove payment. While it
FBMR, the purported sale is necessarily void ab initio. kept referring to O.R. No. 6030203 as its evidence of the required payment, 59 it
The Court can hypothetically concede, as a matter of fact, the withdrawal of failed to present and offer the receipt in evidence. A Certification under date
the JUSMAG area from the ambit of Proclamation No. 423 and its September 15, 1993 of the OIC Cash Division, LMB, states that "OR #
reclassification as alienable and disposable lands of the public domain. Still, 6030203 in the amount of P11,977,000.00 supposedly paid by [SHAI] is not
such hypothesis would not carry the day for private respondent SHAI. The among the series of [ORs] issued at any time by the National Printing Office to
reason therefor is basic: Article XII, Section 352 of the 1987 Constitution forbids the Cashier, LMB, Central Office." 60 A copy of the OR receipt is not appended
private corporations from acquiring any kind of alienable land of the public to any of the pleadings filed before the Court. We can thus validly presume
domain, except through lease for a limited period. While Fr. Bernas had stated that no such OR exists or, if it does, that its presentation would be adverse to
the observation that the reason for the ban is not very clear under existing SHAI.
jurisprudence,53 the fact remains that private corporations, like SHAI, are A contract of sale is void where the price, which appears in the document as
prohibited from purchasing or otherwise acquiring alienable public lands. paid has, in fact, never been paid.61
Even if on the foregoing score alone, the Court could write finis to this 5. The purchase price was, according to the witnesses for SHAI, paid in full in
disposition. An appropriate closure to this case could not be had, however, cash to the cashier of the LMB the corresponding amount apparently coming
without delving to an extent on the issue of the validity of the October 30, in a mix of P500 and P100 denominations. Albeit plausible, SHAIs witnesses
1991 Deed of Sale which necessarily involves the question of the authenticity account taxes credulity to the limit.

96
A final consideration in G.R. No. 156951. This case could not have come to has, in the interim, been duly excluded by law or proclamation from such
pass without the participation of a cabal of cheats out to make a dishonest reservation. Private respondent SHAI, its members, representatives and/or
buck at the expense of the government and most likely the members of SHAI. their assigns shall vacate the subject parcels of land immediately upon the
No less than its former president (Ms. Virginia Santos) testified that a finality of this decision, subject to the provisions of Republic Act No. 7227,
"facilitator" did, for a fee, the necessary paper and leg work before the LMB otherwise known as the Bases Conversion and Development Act.
and the Registry of Deeds that led to the execution of the Deed of Sale and Cost against the private respondent SHAI.
issuance of the certificate of title in question. 62 Ms. Santos identified Eugenia Having said our piece in G.R. No. 173408, we need not speak any further
Balis, a geodetic engineer, as the "facilitator"63 who "facilitated all these thereon other than to deny as we hereby similarly DENY the same.
presentation" of documents,64 and most of the time, "directly transacted" with SO ORDERED.
the LMB and the Register of Deeds leading to acquisition of title. 65 Engr. Balis Puno, Chairperson, Sandoval-Gutierrez, Azcuna, Garcia, J.J., concur
was, in the course of Ms. Santos testimony, directly mentioned by name for at
least fifteen (15) times. Not surprisingly, Engr. Balis did not appear in court, Buenaventura v. Republic 517 SCRA 271 (2007)
despite SHAIs stated intention to present her as witness. 66 Republic of the Philippines
The extent of the misappropriation of the Fort Bonifacio land involved in this SUPREME COURT
and the NOVA area litigations is, as described in the Report of the FactFinding Manila
Commission,67 "so epic in scale as to make the overpricing of land THIRD DIVISION
complained of in the two hundred AFP [Retirement and Separation Benefits G.R. No. 166865 March 2, 2007
System] RSBS cases (P703 million) seem like petty shoplifting in ANGELITA F. BUENAVENTURA and PRECIOSA F. BUENAVENTURA,
comparison."68 The members of private respondent SHAI may very well have Petitioners,
paid for what they might have been led to believe as the purchase price of the vs.
JUSMAG housing area. The sad reality, however, is that the over P11 Million REPUBLIC OF THE PHILIPPINES, Respondent.
they paid, if that be the case, for a piece of real estate contextually outside DECISION
the commerce of man apparently fell into the wrong hands and did not enter CHICO-NAZARIO, J.:
the government coffers. Else, there must be some memorials of such The case before this Court is a Petition for Review on Certiorari under Rule 45
payment. of the 1997 Revised Rules of Civil Procedure seeking to annul and set aside
At bottom, this disposition is nothing more than restoring the petitioner the Decision1 and Resolution2 of the Court of Appeals in CA-G.R. CV No. 72925
Republic, and eventually the BCDA, to what rightfully belongs to it in law and entitled, Angelita F. Buenaventura and Preciosa F. Buenaventura vs. Republic
in fact. There is nothing unjust to this approach. of the Philippines, dated 23 August 2004 and 25 January 2005, respectively,
With the foregoing disquisitions, the petition for contempt in G.R. No. which granted the appeal filed by the Republic of the Philippines (Republic)
173408 need not detain us long. As it were, the question raised by the and declared the parcel of land subject matter of this Petition as public land,
petitioners therein respecting the ownership of the JUSMAG area and, thus, reversing the Order 3 of the Regional Trial Court (RTC) of Paraaque City
accordingly, of the right of the petitioning retired military officers to remain in dated 29 October 2001, which recognized and confirmed the rights of herein
the housing units each may be occupying is now moot and academic. petitioners Angelita F. Buenaventura (Angelita) and Preciosa F. Buenaventura
However, contempt petitioners expressed revulsion over the efforts of the (Preciosa), over the subject property, and issued a decree of registration of the
military establishment, particularly the AFP Chief of Staff, to oust them from same in their favor.
their respective dwellings, if that really be the case, even before G.R. No. The antecedent facts of the case are as follows:
156951 could be resolved, is understandable as it is justified. We thus end this Petitioners Angelita and Preciosa are the applicants for registration of title
ponencia with a reminder to all and sundry that might is not always right; that over the subject property. They are the heirs of spouses Amado Buenaventura
ours is still a government of laws and not of men, be they in the civilian or and Irene Flores (spouses Buenaventura) from whom they acquired the
military sector. Accordingly, the Court will not treat lightly any attempt to subject property.
trifle, intended or otherwise, with its processes and proceedings. A becoming The facts reveal that the subject property was acquired by the spouses
respect to the majesty of the law and the prerogatives of the Court is a must Buenaventura from the Heirs of Lazaro de Leon, namely: Aurelio de Leon and
for the orderly administration of justice to triumph. his sister Rodencia Sta. Agueda even before World War II. However, it was only
WHEREFORE, the petition in G.R. No. 156951 is GRANTED and the on 30 January 1948 that the corresponding Deed of Sale 4 was executed in
appealed CA Decision is REVERSED and SET ASIDE. Accordingly, the Deed of favor of the spouses Buenaventura. After the execution of the said Deed of
Sale dated October 30, 1991 (Exh. "A"/"1") purportedly executed in favor of Sale, the spouses Buenaventura transferred the tax declaration in their name.
private respondent SHAI and TCT No. 15084 (Exh. "B"/"2") of the Registry of Consequently, Tax Declaration (T.D.) No. 5492 covering the subject property in
Deeds of Rizal issued on the basis of such deed are declared VOID. The the names of Aurelio and Rodencia was cancelled and T.D. No. 6103 5 was
Register of Deeds of Pasig or Taguig, as the case may be, is hereby ordered to issued in the name of spouses Buenaventura.
CANCEL TCT No. 15084 in the name of SHAI and the area covered thereby is In 1978, the spouses Buenaventura transferred, by way of Deed of Sale, 6 the
DECLARED part of the Fort Bonifacio Military Reservation, unless the same subject property, together with the adjacent property, which they previously
97
acquired from Mariano Pascual, to their children, among whom are herein surnamed Buenaventura, free from all liens and encumbrances.
petitioners. As a result thereof, a new tax declaration (T.D. No. A-004-05698) 7 ONCE THIS DECISION has become final, let another one issue directing the
was issued in the name of the spouses Buenaventuras children. Land Registration Authority to issue the corresponding decree.
Petitioners then filed an Application for Registration of Title on 5 June 2000 Let copies of this [D]ecision be furnished to the adjoining owners, Land
before the RTC of Paraaque City of the subject property, more particularly Registration Authority, Land Management Bureau, Office of the Solicitor
described as Cadastral Lot No. 5001-B, Csd-007604-000176-D, Paraaque General, Sec. of Public Works and Highways, Department of Agrarian Reform,
Cadastre, located in San Dionisio, Paraaque City, with an area of 3,520.92 the Director, Forest Management Bureau, Chairman Metropolitan Manila
square meters, more or less. Petitioners alleged that "they and their Development Authority, DENR [Department of Environment and Natural
predecessors-in-interest acquired title to the said parcel of land thru Resources], South CENRO, Land Management Sector, City Mayor of Paraaque
inheritance, transfer, and possession as owners of the same since time and Registry of Deeds, Paraaque City.11
immemorial and/or within the period provided for by law." 8 Feeling aggrieved with the aforementioned Order of the trial court, the
As the trial court found the application to be sufficient in form and substance, Republic appealed to the Court of Appeals. According to the Republic,
it thereby set the case for hearing, and directed the service and publication of petitioners failed to prove continuous, open, exclusive and notorious
the notice thereof pursuant to Section 23 9 of the Property Registration Decree possession by their predecessors-in-interest and by themselves; hence, the
(Presidential Decree No. 1529). trial court erred in granting petitioners application for registration of the
On 27 September 2001, when the case was called for hearing, no interested subject property. The Republic prayed for the reversal of the Order of the trial
party appeared before the trial court other than the petitioners. Consequently, court and for the dismissal of the application for registration filed by
petitioners proceeded to present several documents in order to establish petitioners.
compliance with the jurisdictional requirements. The same were marked and On 23 August 2004, the Court of Appeals rendered a Decision in favor of the
offered in evidence before the court a quo. Republic, thus, overturning the Order of the court a quo. The dispositive
No formal opposition had been filed and no oppositor appeared in any of the portion of the Decision reads as:
previously set hearings of the case; hence, petitioners counsel moved for the WHEREFORE, the appeal is GRANTED and the Decision of the Regional Trial
declaration of general default except for the Republic. The same was granted Court, Branch 274, Paraaque City dated October 29, 2001 is REVERSED and
by the court a quo. The case was then referred to a commissioner, who SET ASIDE and the parcel of land subject matter of the application is declared
directly received petitioners evidence in chief. public land.12
Petitioners presented five witnesses, namely: Aniceta C. Capiral, Engr. Teofilo Petitioners filed a Motion for Reconsideration of the aforesaid Decision on 20
R. La Guardia, Atty. Reginald L. Hernandez, Ricardo H. Lopez, and herein September 2004. In a Resolution dated 25 January 2005 rendered by the
petitioner Angelita, in order to establish the fact that petitioners and their appellate court, said Motion for Reconsideration was forthwith denied for lack
predecessors have acquired vested right over the subject property by their of merit.
open, continuous, and exclusive possession under a bona fide claim of Hence, this Petition.
ownership for over 50 years completely unmolested by any adverse claim, Petitioners raise the following issues for the resolution of this Court:
meaning, their possession of the subject property was in the manner and for I. Whether or not the Court of Appeals erred in nullifying the Decision of the
the period required by law; likewise, to prove the alienable and disposable trial court confirming petitioners title over the subject property for not being
character of the subject property. allegedly supported by substantial evidence as required by law.
Other than the respective testimonies of the above-named witnesses, they II. Whether or not the Court of Appeals gravely erred in declaring the subject
also presented and identified several documents 10 offered in evidence, which property as pubic land and ignoring petitioners evidence of over 50 year
tend to establish further the following: (1) petitioners fee simple title over the possession in the concept of an owner and completely unmolested by any
subject property; (2) the nature of the possession and occupation of the adverse claim.
property; (3) its classification as part of the alienable and disposable zone of In the Memorandum13 of the petitioners, they allege that the appellate court
the government; and (4) the improvements introduced thereon and the taxes committed grave error when it nullified the trial courts Order dated 29
paid on the subject property. Said documents were duly admitted by the trial October 2001, which confirmed their title to the subject property. Petitioners
court. claim that contrary to the findings of the Court of Appeals that the above-
On 29 October 2001, based on the pieces of evidence presented by mentioned Order was not supported by evidence, the records of the case
petitioners, the court a quo issued an Order granting the application for clearly speak of the existence, not absence, of sufficient evidence to sustain
registration of title of the subject property, the decretal portion of which reads the findings of the court a quo that petitioners have established possession of
as follows: the subject property in the manner and for the period required by law, that is
WHEREFORE, finding the application of registration of title to the subject by open, continuous, exclusive, and notorious possession in the concept of an
parcel of land, known as Lot 5001-B Cad 299, Paraaque Cadastre, and more owner since 12 June 1945 or earlier, to warrant the registration of their title to
particularly described in approved Survey Plan Csd 007604-000176 is hereby the subject property.
confirmed and ordered registered in the names of [petitioners] Preciosa, Petitioners likewise argue that the appellate court gravely erred when it
Angelita, [and in the names of their other siblings] Crisostomo, and Alfredo, all declared as public land the subject property despite the fact that they were

98
able to prove by clear and convincing evidence that their possession of the (3) that such possession is under a bona fide claim of ownership since 12 June
subject property was indeed in the manner and within the period required by 1945 or earlier.17 The second classification relates to the acquisition of private
law. Having been in possession of the subject property for more than 30 years, lands by prescription.
they have already acquired vested right or title over the subject property by In the case at bar, the Republic argues, through the Office of the Solicitor
operation of law based on the period provided for under the prevailing land General, that petitioners own evidence tends to show that the subject
registration and property laws; hence, the Decision of the Court of Appeals is property is not alienable and disposable because it was a salt bed and a
inconsistent with the facts and the law. fishpond and under Section 2, Article XII of the Constitution, except for
The Petition is meritorious. agricultural lands, all other natural resources shall not be alienated. Likewise,
In resolving the issues involved in the present case, there is a need for this under the Regalian Doctrine, all lands not otherwise appearing to be clearly
Court to re-examine the facts of the case for the proper determination of the within private ownership are presumed to belong to the State.
issues raised herein. It is true that under the Regalian Doctrine all lands of the public domain
As a rule, in the exercise of the Supreme Courts power of review, the Court is belong to the State and all lands not otherwise appearing to be clearly within
not a trier of facts and does not normally undertake the re-examination of the private ownership are presumed to belong to the State. 18 However, such
evidence presented by the contending parties during the trial of the case presumption is not conclusive. It can be rebutted by the applicants
considering that the findings of fact of the Court of Appeals are conclusive and presentation of incontrovertible evidence showing that the land subject of the
binding on the Court.14 However, the rule is not without exceptions. There are application for registration is alienable and disposable. 19
several recognized exceptions15 in which factual issues may be resolved by After a thorough examination of the records of this case, this Court found out
this Court and two of these exceptions find application in this present case, to that petitioners offered in evidence a certification 20 from the Department of
wit: (1) when the findings of the appellate court are contrary to those of the Environment and Natural Resources, National Capital Region dated 29 October
trial court; and (2) when the findings of fact of the appellate court are 2001, to prove that the subject property was alienable and disposable land of
premised on the supposed absence of evidence but contradicted by the the public domain. The said certification contains the following statements:
evidence on record. This is to certify that the parcel of land as shown and described on the reverse
The issues presented by petitioners will be discussed concurrently, since they side of this plan- Lot 5001-B, Cad-299, Paraaque Cadastre situated at San
are interrelated. Dionisio, Paraaque City, Metro Manila containing an area of 3,520.92 square
In the assailed Decision of the Court of Appeals, it ruled that petitioners failed meters as prepared by Geodetic Engineer Mariano V. Flotildes for Amado
to show possession and occupation of the subject property under a bona fide Buenaventura, et al., was verified to be within the Alienable and Disposable
claim of ownership since 12 June 1945 or earlier as provided for in Section Land per L.C. Map 2623, Project No. 25 of Paraaque per Forestry
14(1) of the Property Registration Decree. It further said that the testimonial Administrative Order No. 4-1141 dated January 3, 1968.21 (Emphasis supplied.)
evidence presented by petitioners was not sufficient to prove petitioners To our minds, the said certification is sufficient to establish the true nature or
possession in the manner and within the period required by the aforesaid law character of the subject property. The certification enjoys a presumption of
because petitioners witnesses merely testified on their familiarity with the regularity in the absence of contradictory evidence. 22 As it is, the said
subject property. certification remains uncontested and even the Republic itself did not present
Section 14 of the Property Registration Decree speaks of who may apply for any evidence to refute the contents of the said certification. Therefore, the
registration of land. The said provision of law refers to an original registration alienable and disposable character of the questioned parcel of land has been
through ordinary registration proceedings.16 It specifically provides: clearly established by the evidence of the petitioners, by 3 January 1968, at
SEC. 14. Who may apply. The following persons may file in the proper Court the latest.
of First Instance [now Regional Trial Court] an application for registration of Now, going to the requisites of open, continuous, exclusive and notorious
title to land, whether personally or through their duly authorized possession and occupation under a bona fide claim of ownership since 12 June
representatives: 1945 or earlier, Republic alleges that no sufficient evidence was adduced by
(1) Those who by themselves or through their predecessors-in-interest have petitioners to show that they and their predecessors-in-interest have been in
been in open, continuous, exclusive and notorious possession and occupation exclusive possession of the subject property since 12 June 1945 or earlier in
of alienable and disposable lands of the public domain under a bona fide claim the concept of an owner, to which the Court of Appeals agreed. The Court of
of ownership since June 12, 1945, or earlier. Appeals in its decision said that:
(2) Those who have acquired ownership of private lands by prescription under Although they were able to show possession by their parents, their
the provisions of existing laws. predecessors-in-interest, since 1948, they failed to prove the fact of
From the aforesaid provisions of the Property Registration Decree, we can possession since [12 June 1945] before the filing of the application. 23
deduce that there are three requisites for the filing of an application for Emphasis should be given to the fact that the Court of Appeals, in its Decision,
registration of title under the first category, to wit: (1) that the property in did not question petitioners possession of the subject property since 1948.
question is alienable and disposable land of the public domain; (2) that the Verily, it even stated in the said Decision that petitioners possession may be
applicants by themselves or through their predecessors-in-interest have been reckoned from 1948, the year of the execution of the Deed of Sale. The only
in open, continuous, exclusive and notorious possession and occupation; and reason posited by the appellate court in denying the Order of the trial court

99
which granted the application for registration of title of the petitioners was the June 1945, application for registration of the said property is still possible by
fact that petitioners evidence was not sufficient to prove that their possession virtue of Section 14(2) of the Property Registration Decree which speaks of
of the subject property was since 12 June 1945 or earlier. prescription.
We agree with the findings of the Court of Appeals that the evidence Under the Civil Code, prescription is one of the modes of acquiring
presented by petitioners was not enough to prove that their possession of the ownership.27 Article 1106 of the Civil Code provides:
subject property started since 12 June 1945 or earlier because the evidence By prescription, one acquires ownership and other real rights through the
established that the questioned parcel of land was acquired by petitioners lapse of time in the manner and under the conditions laid down by law.
parents only on 30 January 1948, the date of the execution of the Deed of Also in Article 1113 of the Civil Code, it is provided that:
Absolute Sale by its previous owners. They can neither tack their possession All things which are within the commerce of men are susceptible of
to that of the previous owners because they failed to present any evidence of prescription, unless otherwise provided. Property of the State or any of its
possession by those prior owners. Moreover, petitioners possession of the subdivision not patrimonial in character shall not be the object of prescription.
subject property could only ripen into ownership on 3 January 1968, when the Likewise, Article 1137 of the Civil Code states that:
same became alienable and disposable. "Any period of possession prior to the Ownership and other real rights over immovables also prescribe through
date when the [s]ubject [property was] classified as alienable and disposable uninterrupted adverse possession thereof for thirty years, without need of title
is inconsequential and should be excluded from the computation of the period or of good faith. (Emphasis supplied.)
of possession; such possession can never ripen into ownership and unless the It is well-settled that properties classified as alienable and disposable land
land had been classified as alienable and disposable, the rules on confirmation may be converted into private property by reason of open, continuous and
of imperfect title shall not apply thereto."24 exclusive possession of at least 30 years. 28 Such property now falls within the
Be that as it may, this will not be an insurmountable bar to the petitioners to contemplation of "private lands" under Section 14(2), over which title by
have the title to the subject property registered in their names. prescription can be acquired. Hence, because of Section 14(2) of Presidential
In the case of Republic v. Court of Appeals,25 this Court closely examined the Decree No. 1529, those who are in possession of alienable and disposable
land registration laws governing land registration proceedings in the land, and whose possession has been characterized as open, continuous and
Philippines. In the aforesaid case, the Court made the following exclusive for 30 years or more, may have the right to register their title to
pronouncements: such land despite the fact that their possession of the land commenced only
When the Public Land Act was first promulgated in 1936, the period of after 12 June 1945.29
possession deemed necessary to vest the right to register their title to The aforesaid jurisprudential rule truly demonstrates that, in the present case,
agricultural lands of the public domain commenced from July 26, 1894. while petitioners possession over the subject property can be reckoned only
However, this period was amended by R.A. [Republic Act] No. 1942, which on 3 January 1968, the date when according to evidence, the subject property
provided that the bona fide claim of ownership must have been for at least became alienable and disposable, they can still have the subject property
thirty (30) years. Then in 1977, Section 48(b) of the Public Land Act was again registered in their names by virtue of Section 14(2) of the Property
amended, this time by P.D. No. 1073, which pegged the reckoning date at June Registration Decree.
12, 1945. This new starting point is concordant with Section 14(1) of the The records, indeed, reveal that petitioners were in possession of the subject
Property Registration Decree. property for more than 30 years, 32 years to be exact, reckoned from the year
Indeed, there are no material differences between Section 14(1) of the 1968, when the subject property was finally declared alienable and disposable
Property Registration Decree and Section 48(b) of the Public Land Act, as by the DENR to the time they filed an application for registration of title over
amended. True, the Public Land Act does refer to "agricultural lands of the the subject property on 5 June 2000. Petitioners possession of the subject
public domain," while the Property Registration Decree uses the term property since 1968 has been characterized as open, continuous, exclusive
"alienable and disposable lands of the public domain." It must be noted and notorious possession and occupation in the concept of an owner.
though that the Constitution declares that "alienable lands of the public Petitioners presented as evidence their tax declarations covering the years
domain shall be limited to agricultural lands." Clearly the subject lands under from 1948 until the third quarter of 2001. They also offered in evidence a
Section 48(b) of the Public Land Act and Section 14(1) of the Property certification30 from the Office of the Treasurer of the City of Paraaque to
Registration Decree are of the same type. prove that realty taxes over the subject property had been duly paid by
Did the enactment of the Property Registration Decree and the amendatory petitioners. As a rule, tax declarations or realty tax payments of property are
P.D. No. 1073 preclude the application for registration of alienable lands of the not conclusive evidence of ownership, nevertheless, they are good indicia of
public domain, possession over which commenced only after June 12, 1945? It possession in the concept of owner, for no one in his right mind would be
did not, considering Section 14(2) of the Property Registration Decree, which paying taxes for a property that is not in his actual or constructive possession.
governs and authorizes the application of "those who have acquired They constitute at least proof that the holder has a claim of title over the
ownership of private lands by prescription under the provisions of existing property. The voluntary declaration of a piece of property for taxation
laws."26 (Emphasis supplied.) purposes manifests not only ones sincere and honest desire to obtain title to
It becomes crystal clear from the aforesaid ruling of the Court that even if the the property and announces his adverse claim against the State and all other
possession of alienable lands of the public domain commenced only after 12 interested parties, but also the intention to contribute needed revenues to the

100
Government. Such an act strengthens ones bona fide claim of acquisition of Decision reversing the Order of the trial court being the insufficiency of the
ownership.31 evidence presented by petitioners to establish their possession of the subject
In the same breath, it cannot be gainsaid that petitioners have been in actual property prior to 12 June 1945.
possession of the subject property since 1968, at the latest. According to the IN ALL, petitioners were able to prove sufficiently that they have been in
testimony of their witnesses, parts of the subject property are planted with possession of the subject property for more than 30 years, which possession is
bananas and some vegetables, and a bamboo grove. The other parts of the characterized as open, continuous, exclusive, and notorious, in the concept of
subject property were used as a fishpond, as well as devoted to salt making an owner. By this, the subject alienable and disposable public land had been
until 1990.32 However, when the property was no longer suitable for effectively converted into private property over which petitioners have
agricultural purposes, for fishpond, and for salt making because of its acquired ownership through prescription to which they are entitled to have
conversion to non-agricultural purposes consistent with the zonal title through registration proceedings. Petitioners right to have their title to
development of the area, the petitioners backfilled the subject property with the subject property registered cannot be defeated simply because the
gravel and sand, for which they paid their farm helpers just compensation. possession of petitioners commenced on a date later than 12 June 1945, for
Thereafter, they enclosed the property with perimeter fence, installed guards the law and supplementing jurisprudence amply, justly and rightfully provides
and a caretaker to prevent potential squatters from penetrating the area. 33 the necessary remedy to what would otherwise result in an unjust and
When tax declarations and receipts are coupled with actual possession, they unwarranted situation. It would be the height of injustice if petitioners
constitute evidence of great weight and can be the basis of a claim of registration of title over the said property will de denied solely on that ground.
ownership through prescription.34 WHEREFORE, premises considered, the instant Petition is hereby GRANTED.
Conspicuously, the petitioners witnesses are one in pointing out that The Decision and Resolution of the Court of Appeals dated 23 August 2004
petitioners and their predecessors-in-interest are the sole claimants of the and 25 January 2005, respectively, are hereby REVERSED and SET ASIDE. The
subject property. Order of the trial court dated 29 October 2001 which granted petitioners
It bears stressing that the pieces of evidence submitted by petitioners are application for registration of the subject property and directing the issuance
incontrovertible. No one, not even the Republic, presented any evidence to of a decree of registration in petitioners favor once the judgment has become
contradict the claims of the petitioners that they are in possession of the final and executory is hereby REINSTATED. No costs.
subject property and their possession of the same is open, continuous and SO ORDERED.
exclusive in the concept of an owner for over 30 years. Verily, even the MINITA V. CHICO-NAZARIO
appellate court mentioned in its Decision that petitioners were able to show Associate Justice
possession of the subject property as early as 1948, the only basis for its

101

Potrebbero piacerti anche